You are on page 1of 550

1

1.1 (a) Thermostatic bath imposes its temperature T on the system.


(b) Container imposes constraint of constant volume. Thermal isolation implies
that heat flow must be zero, while mechanical isolation (and constant volume)
implies there is no work flow. Consequently there is no mechanism for adding
or removing energy from the system. Thus, system volume and energy are
constant.
(c) Thermally isolated ⇒ adiabatic
Frictionless piston ⇒ pressure of system equals ambient pressure (or ambient
pressure + wg/A if piston-cylinder in vertical position. Here
w = w eight of piston , A = its area and g is the force of gravity.)
(d) Thermostatic bath ⇒ constant temperature T.
Frictionless piston ⇒ constant pressure (see part c above).
(e) Since pressure difference induces a mass flow, pressure equilibrates rapidly.
Temperature equilibration, which is a result of heat conduction, occurs much
more slowly. Therefore, if valve between tanks is opened for only a short time
and then shut, the pressure in the two tanks will be the same, but not the
temperatures.
1

1.2 (a) Water is inappropriate as a thermometric fluid between 0°C and 10°C, since
the volume is not a unique function of temperature in this range, i.e., two
temperatures will correspond to the same specific volume,
V (T = 1° C) ~ V (T = 7° C); V (T = 2° C) ~ V (T = 6° C); etc.

1.0002
1.000191

1.0001
VH2O
1

1 0.9999
0 5 10
0 T 9

T in o C and V in cc / g
Consequently, while T uniquely determines, V , V does not uniquely determine
T.
(b) Assuming that a mercury thermometer is calibrated at 0°C and 100°C, and that the specific
volume of mercury varies linearly between these two temperatures yields

c h V cT = 100 Ch − V cT = 0 Ch
 o o
V (T ) = V 0o C +
100 C - 0 C
cT − 0 Ch
o o s
o

= 0.0735560 + 0.000013421 Ts (*)

where T is the actual temperature, and Ts is the temperature read on the thermometer scale. At
10°C, Vexp (T = 10° C) = 0.0736893 cc g . However, the scale temperature for this specific
volume is, from eqn. (*) above

Vexp (T ) − 0.0735560 0.0736893 − 0.0735560


Ts = −5
= = 9.932° C
13421
. × 10 13421
. × 10−5

Thus, T − Ts at 10°C = −0.068° C . Repeating calculation at other temperatures yields figure


below.
0
0

∆T i 0.1

0.153491 0.2
0 50 100
0 T 100
i

The temperature error plotted here results from the nonlinear dependence of the volume of
mercury on temperature. In a real thermometer there will also be an error associated with the
imperfect bore of the capillary tube.

(c) When we use a fluid-filled thermometer to measure ∆T we really measure ∆L , where

∆L =
∆V
=
c
M ∂V ∂T ∆T h
A A

A small area A and a large mass of fluid M magnifies ∆L obtained for a


given ∆T . Thus, we use a capillary tube (small A) and bulb (large M) to get an accurate
c h
thermometer, since ∂V ∂T is so small.
2
d (C ⋅ V ) dM 
2.1 = =M IN -M OUT
dt dt
a) at steady state
g
M IN =1 = M OUT = C ⋅ volumetric flow rate out
min
g 10m3 1 g g
1 = C× C= = 0.1 3
min min 10 m3 m
dC m3 dC m3
V = M − C × 10 V = M − C − 10
b) dt min dt min
V = 6 × 6 × 3 = 108 m 3
V = 6 × 6 × 3 = 108 m3
3
dC g 10m
108m3 =1 −C×
dt min min
dC 1 g 1 0 −1
= 3
−C m
dt 108 m min 108
or for C in g 3
m
dC dC
108 = − (10C − 1) 10.8 = − dt
dt ( − 0.1)
C
( C-0.1)
10.8 ln = −t
( -0.1)
⎛ -0.005 ⎞
For C = 0.95 * 0.1 = 0.095 10.8 ln ⎜ ⎟ = −t
⎝ -0.1 ⎠
108 ln 20 = t t = 32.35 minutes
2
dC Cf
2.2 = − kC ln = − kt
dt Ci
ln 2
If Cf = 0.5 Ci ln 0.5 = -kt or k =
t
for a half life of seven years k = 0.099 years-1
so that
C
ln f = − k 25 = −0.099 × 25 = −2.475
22
Cf = 22 exp (-2.475) = 1.85 ppm
2
m3
2.3 Basis: 2-liter vessel = 2000 cm3 = 2×103 ×10-6 = 2 × 10-3 m3
cm3
kmol mol
Initial moles of phosphine = 5 ×103 ×2×10-5 m3 = 10 mol
m3 kmol
a) Mass balance from stoichiometry
Species Initial Final
PH3 10 10 – 4X = 7
P4 0 X
H2 0 6X
Since 10 - 4X = 7 3 = 4X X=¾
⇒ P4 = 0.75 moles
H2 = 6 moles
(10-4X ) moles moles
b) CPH3 at any time = -3 3
= ( 5-2X ) ×10-3
2×10 m m3
Therefore
dCPH
=3.715×10-6 CPH
3

dt 3

dX
-2×10-3 =-3.715×10-6 ( 5-2X ) ×10-3
dt
dX
=3.715×10-6 ( 2.5-X )
dt
dX d(2.5-X )
- = =-3.715×10-6 dt
2.5 - X 2.5-X
ln ( 2.5-X )t -ln ( 2.5-X )0 =-3.715×10-6 t
2.5-X -2.715×10 -6
t
=e
2.5
2.5-X =2.5 e2.715×10
-6
t

(
X =2.5 1-e-2.715×10
-6
t
)
Therefore
Moles PH 3 =10-4X =10-10 1-e-2.715×10( -6
t
)= 10 e-2.715×10
-6
t

(
Moles PH 4 =2.5 1-e-2.715×10
-6
t
)
(
Moles H 2 = 15 1- e-2.715×10
-6
t
)
Time required for 3 moles to react = time for X = 0.75
(
X =2.5 1-e-2.715×10
-6
t
) = 0.75
T = 1.314 × 105 sec = 36.49 hours
2
2.4 2 NO + O2 → 2 NO2
a) 0.5 moles NO produces 0.5 moles NO2
b)
Initial Conc. at
Species Conc X

NO 1 1–2X
O2 3 3–X
NO2 0 6X
4–X
dCNO
=-1.4×10-4 CNO
2
CO2
dt
d (1- 2 X ) dX
=-1.4×10-4 (1-2X ) ( 3-X ) =-2
2

dt dt
dX
=0.7×10-4 (1-2X ) ( 3-X )
2

dt
1 1 ( 2X-1)
0.7 × 10-4 t = + ln
5 (1-2X ) 25 ( X-3)
Now for 0.5 mole of NO to react ⇒ 1 – 2X = 0.5 X = 0.25
1 1
0.7×10-4 t = + ln ( 0.5/2.75 ) =0.332
5×0.5 25
0.332
t= ×104 =0.474×104 sec = 1.317 hrs
0.7
3

3.1 (a) By an energy balance, the bicycle stops when final potential energy equals initial kinetic energy.
Therefore

F 20 km × 1000 m × 1 hr I 2

1 2 v2
mvi = mgh f or h f = i =
H hr km 3600 sec K
2 2g m
2 × 9.807
sec2
or h=1.57 m.
(b) The energy balance now is
1 2 1 2
mv f = mvi + mghi or v 2f = vi2 + 2 ghi
2 2
F I
km 2 m F km I
3600 sec 2
H
v 2f = 20
K
hr
+ 2 × 9.807
sec2
× 70 m
H
×
1000 m
×
hr K
v f = 134.88 km/hr. Anyone who has bicycled realizes that this number is much
too high, which demonstrates the importance of air and wind resistance.
3

3.2 The velocity change due to the 55 m fall is

c∆v h = 2 × 9.807 secm F km × 3600 sec I 2


× 55 m ×
H 1000 m hr K
2
2

v f = 118.24 km/hr. Now this velocity component is in the vertical direction. The initial velocity of
8 km/hr was obviously in the horizontal direction. So the final velocity is
km
v = v x2 + v 2y = 11851
.
hr
3

3.3 (a) System: contents of the piston and cylinder


(closed isobaric = constant pressure)
M.B.: M 2 − M1 = ∆M = 0 ⇒ M 2 = M1 = M

E.B.: M 2U 2 − M1U1 = ∆M H


0
d i 0
+Q + Ws − PdV z
c h z z a f
M U 2 − U1 = Q − PdV = Q − P dV = Q − P V2 − V1
M cU− U h = Q − PM cV − V h
2 1 2 1

Q = M cU − U h + M c PV − PV h = M cU + PV h − cU + PV h


2 1 2 1 2 2 1 1

= M c H − H h2 1
P = 1013
. bar ≈ 01
. MPa
V U H
T = 100 1.6958 2506.7 2676.2
T = 150 1.9364 2582.8 2776.4
Linear interpolation
T = 125° C 1.8161 2544.8 2726.3 Initial state
Final state P = 01 
. MPa , V2 = 36322
. 3
m /kg
T = 500° C 3.565 3488.1
T = 600° C 4.028 3704.7
Linear interpolation

36322
. − 3565
. T − 500
= 2 T2 = 514.5° C
4.028 − 3565
. 600 − 500
514.5 − 500 H 2 − 34881
.
= H 2 = 3519.5
600 − 500 3704.7 − 34881 .
Q = 1 kg(3519.5 − 2726.3) kJ kg = 7932 . kJ

z a
Pa 1 kg
f
W = − PdV = −1 bar × V2 − V1 = −1 bar × (36322
. − 18161
.
1J
) m3 kg

= −1 bar × 100,000 × × × 18161


. m3 kg
bar m ⋅ s2 ⋅ Pa m2 ⋅ s2 ⋅ kg
= −1816
. kJ kg

(b) System is closed and constant volume


M.B.: M 2 = M1 = M

E.B.: M 2U 2 − M1U1 = ∆M H


0
d i
0
+Q + Ws – PdV
0
z
c
Q = M U 2 − U1 h
Here final state is P = 2 × 1013
. bar ~ 0.2 MPa ; V2 = V1 = 18161
. m3 kg (since piston-cylinder
volume is fixed)
P = 0.2 MPa ; V2 = 18161
.
T(° C) V U
500 1.7814 3130.8
600 2.013 3301.4

18161
. − 17814
. T − 500 0.0347
= = = 01498
. ~ 015
.
2.013 − 17814
. 600 − 500 0.2316
T = 515° C
U 2 − 31308 .
= 01498
. U 2 = 3156.4 kJ kg
33014. − 31308 .
Q = 1 kg × (3156.4 − 2544.8) kJ kg = 6116 . kJ

(c) Steam as an ideal gas—constant pressure

PV PV PV
N= ⇒ 1 1 = 2 2 but V 2 = 2V 1 ; P1 = P2
RT RT1 RT2
PV P 2V
1 1
= 1 1 ⇒ T2 = 2 × T1
T1 T2
T1 = 27315
. + 125 = 39815. K
T2 = 2 × T1 = 796.3 K = 52315
. °C

1000 g kg 1 kJ
Q = N∆ H = × 34.4 J mol K × (796.3 − 39815
. ) K×
18 g mol 1000 J
= 760.9 kJ

z
W = − PdV = − P∆V = − P
FG NRT − NRT IJ = − NRaT − T f
H P P K
2 1
2 1
1
1000
=− × 8.314 × 39815
. = −1839
. kJ
18

(d) Ideal gas - constant volume

PV PV
1 1
= 2 2 here V 1 = V 2 ; P2 = 2 P1
RT1 RT2

PV 2 P ⋅V
So again 1 1
= 1 1 ; T2 = 2T1 = 796.3 K .
T1 T2

1000 g kg 1
Q = N∆U = × (34.4 − 8.314) × (796.3 − 39815
. )×
18 g mol 1000
CV = CP − R ; Q = 577.0 kJ
3

3.4
M wU w, f − M wU w, i = Ws = M weight × g × 1 m
M w = M weight = 1 kg

a f
1 kg × CP Tf − Ti = 1 kg × 9.807 m s2 × 1 m ×
1J
m2 kg s2
= 9.807 J

1000 g
1 kg × 4184
. J g K× × ∆T = 9.807
kg
9.807
∆T = K = 2.344 × 10−3 K
4184
. × 1000
3

3.5
a f a f
From Illustration 3.2-3 we have that H T1, P1 = H T2 , P2 for a Joule-Thomson expansion. On the
Mollier diagram for steam, Fig. 3.3-1a, the upstream and downstream conditions are connected
by a horizontal line. Thus, graphically, we find that T ~ 383 K . (Alternatively, one could also
use the Steam Tables of Appendix III.)

a f a f
For the ideal gas, enthalpy is a function of temperature only. Thus, H T1, P1 = H T2 , P2 becomes
af a f
H T1 = H T2 , which implies that T1 = T2 = 600° C .
3

3.6 System: Contents of Drum (open


system)
mass balance: M t2 − M t1 = ∆M
steam
energy balance:
MU
t2
− MU
t1 z
= ∆MH in + Q + Ws − PdV

but Q=0 by problem statement,


Ws = 0
and z PdV = P∆V is negligible. (Note V (T = 25° C) = 1003
. × 10−3 m3 kg ,
V (T = 80° C) = 1029
. × 10−3 m3 kg ). Also from the Steam Tables

H in = H (T = 300° C, P = 30
. bar = 300 kPa) = 3069.3 kJ kg

and recognizing that the internal energy of a liquid does not depend on pressure gives

U = U (T = 25° C, 1.013 bar ) = U (sat., T = 25° C) = 104.88 kJ kg


t1

and

U = U (T = 80° C, 1.013 bar ) = U (sat., T = 80° C) = 334.86 kJ kg


t2

Now using mass balance and energy balances with M t1 = 100 kg yields

M t2 × 334.86 kJ − 100 × 104.88 kJ = M t2 − 100 × 3069.3 kJ

Thus

M t2 (3069.3 − 334.86) = 100 × (3069.3 − 104.88)

M t2 = 108.41 kg , and ∆M = M t2 − M t1 = 8.41 kg of steam added.


3

3.7 (a) Consider a change from a given state 1 to a given state 2 in a closed system. Since initial and final
states are fixed, U1 , U 2 , V1 , V2 , P1 , P2 , etc. are all fixed. The energy balance for the closed
system is

z
U 2 − U1 = Q + Ws − PdV = Q + W

z
where W = Ws − PdV = total work. Also, Q = 0 since the change of state is adiabatic. Thus,
U 2 − U1 = W .

Since U1 and U 2 are fixed (that is, the end states are fixed regardless of the path), it follows
that W is the same for all adiabatic paths. This is not in contradiction with Illustration 3.5-6,
which established that the sum Q + W is the same for all paths. If we consider only the subset
of paths for which Q = 0 , it follows, from that illustration that W must be path independent.
(b) Consider two different adiabatic paths between the given initial and final states, and let W * and
W ** be the work obtained along each of these paths, i.e.,
Path 1: U 2 − U1 = W * ; Path 2: U 2 − U1 = W **
Now suppose a cycle is constructed in which path 1 is followed from the initial to the final state,
and path 2, in reverse, from the final state (state 2) back to state 1. The energy balance for this
cycle is

U 2 − U1 = W *
a f
− U 2 − U1 = −W **
0 = W * − W **

Thus if the work along the two paths is different, i.e., W * ≠ W ** , we have created energy!
3

3.8 System = contents of tank at any time


mass balance: M 2 − M1 = ∆M
c h c h
energy balance: MU − MU = ∆MH
2 1 in

(a) Tank is initially evacuated ⇒ M1 = 0


Thus M 2 = ∆M , and U 2 = H in = H (5 bar, 370° C) = 3209.6 kJ kg (by interpolation). Then
U = U ( P = 5 bar, T = ?) = 3209.6 kJ kg . By interpolation, using the Steam Tables (Appendix
2
A.III) T = 548° C

V ( P = 5 bar, T = 548° C) ≅ 0.756 m3 kg

c h
Therefore M = V V = 1 m3 0.756 m3 kg = 13228
. kg .
(b) Tank is initially filled with steam at 1 bar and 150°C
⇒ V1 = V ( P = 1 bar, T = 150° C) = 194
. m3 kg and U1 = 2583 kJ kg ,
M1 = V V = 1 V = 05155. kg . Thus, M 2 = 05155
. + ∆M kg . Energy balance is
M 2U 2 − 05155
. × 2583 = ( M − 05155
. ) × 3209.6 . Solve by guessing value of T2 , using T2 and
 
P = 5 bar to find V and U in the Steam Tables (Appendix A.III). See if energy balance and
2 2 2

M 2 = 1 m3 V2 are satisfied. By trial and error: T2 ~ 425° C and M 2 ≅ 1563


. kg of which
1.323 kg was present in tank intially. Thus, ∆M = M 2 − M1 = 0.24 kg .
3

3.9 a) Use kinetic energy = mv2/2 to find velocity.


v 2 m2 kg
1 kg × = 1000 J = 1000 2 2 so v= 44.72 m/sec
2 sec2 m sec

b) Heat supplied = specific heat capacity × temperature change, so


1 mol J
1000g × × 2510
. × ∆T = 1000 J so ∆T=2.225 K.
55.85g mol ⋅ K
3

3.10 System = resistor


Energy balance: dU dt = Ws + Q
where Ws = E ⋅ I , and since we are interested only in steady state dU dt = 0 . Thus
−Q = W = 1 amp × 10 volts = 0.2 × (T − 25° C) J s
s
and 1 watt = 1 volt × 1 amp = 1 J s .
10 watt × 1 J s ⋅ watt
⇒T = + 25° C = 750
. °C
0.2 J s ⋅ K
3

3.11 System = gas contained in piston and cylinder (closed)


0
Energy balance: U t2 − U t1 = Q + Ws − PdV z
(a) V = constant,
From ideal gas law
z d i a
PdV = 0 , Q = U t2 − U t1 = N U t2 − U t1 = NCV T2 − T1 f

PV 114,367 Pa × 0120
. m3
N= = = 5539
. mol (see note following)
RT 8.314 Pa ⋅ m3 mol ⋅ K × 298 K

Thus

Q 10,500 J
T2 = T1 + = 298 K +
NCV 5.539 mol × 30.1 J mol ⋅ K
= 298 + 630
. = 3610
. K

Since N and V are fixed, we have, from the ideal gas law, that

P2 T2 T 3610
.
= or P2 = 2 P1 = × 114.367 kPa = 1385
. × 105 Pa
P1 T1 T1 298.0

(b) P = constant = 114367


. × 105 Pa ;
CP = CV + R = 301 . + 8.314 = 38.414 J mol ⋅ K
Energy balance U t2 − U t1 = Q − P∆V , since P = constant
a f a f
⇒ NCV T2 − T1 = Q − P V2 − V1 = Q − N RT2 − RT1 a f
⇒ Q = NCP T2 − T1a f
Q 10,500
T2 = T1 + = 298 + = 347.35 K
NCP 5539
. × 38.414
and
NR∆T 5539
. mol × 8.314 Pa ⋅ m3 mol ⋅ K × 49.35 K
∆V = = = 0.01987 m3
P 114,367 Pa
V = 012
. + 0.0199 = 01399
. m3

Note: The initial pressure P = Patm + Pwt of piston


Patm = 1013
. bar = 1013
. × 105 kPa
200 kg 1 Ns2
Pwt piston = × × 9.8 m s2 = 13,067 N m2 = 13,067 Pa
0.15 m2 kg ⋅ m
= 13067
. kPa
Thus, intial pressure = 114.367 kPa .
3

3.12 System = contents of storage tank (open system)


Mass balance: M 2 − M1 = ∆M
c h c h
Energy balance: MU − MU = (∆M ) H in since Q = W = 0 and steam entering is of constant
2 1
properties.
Initially system contains 0.02 m3 of liquid water and (40 − 0.02) = 39.98 m3 of steam.
Since vapor and liquid are in equilibrium at 50°C, from Steam Tables, P = 12.349 Pa . Also from
the Steam Tables V L = 0.001012 m3 kg , V V = 12.03 m3 kg , H V = 25921
. kJ kg ,
H L = 209.33 kJ kg , U L = 209.32 kJ kg , and U V = 24435. kJ kg .

M1L =
0.02 m3
= 19.76 kg;
U|
0.001012 m3 kg |V M = M1L + M1V = 2308
. kg .
M1V =
39.98 m 3
= 332 ||1

12.03 m3 kg
. kg;
W
U1 = 19.76 × 209.32 + 332
. × 24435
. = 12,248.6 kJ

Also

H in = 0.90 × 26761
. + 010
. × 419.04 = 2450.4 kJ kg

Possibilities for final state: 1) vapor-liquid mixture, 2) all vapor, and 3) all liquid. First possibility is
most likely, so we will assume V-L mixture. Since P = 1013 . bar , T must be 100°C. Thus we can
find properties of saturated vapor and saturated liquid in the Steam Tables: V L = 0.001044 m3 kg ,
V V = 16729
. m3 kg , U L = 418.94 kJ / kg , H V = 26761
. kJ kg , and U V = 25065 . kJ / kg.
V2 = x(16729
. ) + (1 − x )0.001044 = 0.001044 + 16719
. x m3 kg , where
x = quality
U 2 = x(25065
. ) + (1 − x)418.94 = 418.94 + 2087.56x kJ kg
Substituting into energy balance

a
M 2 (418.94 + 2087.56 x) − 12,248.6 = M 2 − 2308 f
. ⋅ 2450.4

where

V 40 m3
M2 = =
V2 0.001044 + 16719
. x

Solving by trial and error yields x = 05154


. (quality), M 2 = 46.36 kg , and ∆M = 2328
. kg . Also the
final state is a vapor-liquid mixture, as assumed.
3

3.13 System = tank and its contents (open


system) . .
M1 M2
(a) Steady state mass balance
T1 T2
dM  +M  +M 
=0= M 1 2 3
dt
⇒M  =− M
3 b
 +M
1
 = −10 kg min
2 g .
M3 , T3

Steady state energy balance


dU  H + M  H + M  H
=0= M 1 1 2 2 3 3
dt
H 3 = H exit stream = H at temperature of tank contents
Also T3 = T = temperature of tank contents
0 f P a
Now H = H + C T − T , assuming C is not a function of temperature
0 P

0 = 5mH + C aT − T fr + 5mH + C aT − T fr − 10mH + C aT − T fr


0 P 1 0 0 P 2 0 0 P 0

= aT + T f = 65° C
5T + 5T 1
⇒T = 1 2
1 2
10 2

dM  +M  +M  (no useful information here)


(b) mass balance: =0= M 1 2 3
dt
dU  H + M H + M  H
energy balance: =M 1 1 2 2 3 3
dt
dU
but
dU
dt
=
d
dt
MU = Mc h dt
= MCV
dT
dt
~ MCP
dT
dt
since CP ≈ CV for liquids. Thus
dT
MCP 3 = 5CPT1 + 5CPT2 − 10CPT3 and M = 50 kg .
dt

dT3
10 + 2T3 = (80 + 50) = 130 ⇒ T3 = Ae − t 5 + C (t = minutes)
dt

At t → ∞ , T3 = C = 65° C
At t = 0 , T3 = A + C = 25° C ⇒ A = −40° C
So finally T3 = 65° C − 40° Ce − t 5 (t = minutes)

E.B: M FU F − M iU i = ∆MH in


c M U
F
L
F
L h c h
+ M VFU VF − M LiU Li + M ViU Vi = M LF + M VF 01
. H L, in + 0.9 H V, in
Also known is that V = 60 m = 3
M LFVLF + M VFVVF . ⇒2 equations and 2 unknowns

V − M VFVVF
= M LF
VLF
FG V − M V U + M U IJ − c M U + M U h
F F
F F F i i i i
H V K
V V
F L V V L L V V
L

=M
LV − M V + M OP 01. H + 0.9 H
 F F
V V F

N V L
F
Q V L, in V, in
3

3.14 Thermodynamic properties of steam from the Steam Tables


Initial conditions:
Specific volume of liquid and of vapor:
m3  i m3
VLi = 1061
. × 10−3 ; VV = 08857
.
kg kg
Specific internal energy of liquid and of vapor
kJ kJ
U Li = 3139
. ; U Vi = 24759
.
kg kg
M.B: M f − M i = ∆M i
200 liters
M i = M Li + M Vi ; M Li = = 194.932 kg;
VLi
60 m3 − 200 liters
M Vi = =14.476 kg and so Mi=209.408 kg
VVi

E.B.
M f U f − M iU i = ∆MH in
c h c h
M LfU Lf + M VfU Vf − M LiU Li + M ViU Vi = M Lf + M Vf 01
. H L, in + 0.9 H V, in
Total internal energy of steam + water in the tank
194.932×313.0 + 14.476×2475.9 = 9.686×104 kJ
Properties of steam entering, 90% quality
Specific volume = Vin = 0.1×1.061×10-3+ 0.9×0.8857 = 0.797 m3/kg
Specific enthalpy = H in =0.1×504.70 + 0.9 × 2706.7 = 2.486 ×103 kJ/kg

Also have that V = 60 m3 = M LfVLf + M VfVVf .


This gives two equations, and two unknowns, M Lf and M Vf .
The solution (using MATHCAD) is M Lf = 215.306 kg and M Vf = 67.485 kg.
Therefore, the fraction of the tank contents that is liquid by weight is 0.761.
3

3.15 System = contents of both chambers (closed, adiabatic system of constant volume. Also Ws = 0 ).
af af
Energy balance: U t2 − U t1 = 0 or U t2 = U t1 af af
(a) For the ideal gas u is a function of temperature only. Thus,
af af af af
U t2 = U t1 ⇒ T t2 = T t1 = 500 K . From ideal gas law

1 1 = N1 RT1
PV but N1 = N 2 since system is closed
2 2 = N 2 RT2
PV T1 = T2 see above
and V2 = 2V1 see problem statement.

1
⇒ P2 = P1 = 5 bar = 05
. MPa ⇒ T2 = 500 K, P2 = 05
. MPa
2
af af
(b) For steam the analysis above leads to U t2 = U t1 or, since the system is closed U t2 = U t1 , af af
af af
V t = 2V t . From the Steam Tables, Appendix III,
2 1

U at f = U (T = 500 K, P = 1 MPa ) = U (T = 22685


1 . ° C, P = 1 MPa)
≅ 2669.4 kJ kg
af
V t1 = V (T = 22685
. ° C, P = 1 MPa ) ≅ 0.2204 m3 kg

af af
Therefore U t2 = U t1 = 2669.4 kg kg and
af af
V t2 = 2V t1 = 0.4408 m3 kg . By, essentially, trial and error, find that T ~ 216.3° C ,
. MPa .
P ~ 05
af af af af af
(c) Here U t2 = U t1 , as before, except that U t1 = U I t1 + U II t1 , where superscript denotes
chamber.
af af
Also, M (t ) = M I t1 + M II t1 {mass balance} and
af af
V t = 2V M t = 2V M I t + M II t
2 1 2 1 af 1 af
1
For the ideal gas, using mass balance, we have

a f
P2 2V1 P IV P IIV 2P P I P II
= 1 I 1 + 1 II 1 ⇒ 2 = 1I + 1II (1)
T2 T1 T1 T2 T1 T1

Energy balance: N 2U 2 = N1I U 1I + N1II U 1II


a f
Substitute U = U 0 + NCV T − T0 , and cancel terms, use N = PV RT and get
2 P2 = P1I + P1II (2)
Using Eqns. (1) and (2) get P2 = 7.5 × 10 Pa = 0.75 MPa and T2 = 529.4 K (256.25° C) .
5

(d) For steam, solution is similar to (b). Use Steam Table to get M1I and M1II in terms of V.
Chamber 1: U1I = 2669.4 kJ kg ; V1I = 0.2204 m3 kg ;
M I = V1 V1I = 4.537V1
Chamber 2: a
U1II = U T = 600 K, P = 05 f
. MPa = 28459
. kJ kg ; V1II = 05483
. m3 kg ;
M = 1824
II
. V =V V  II
1 1 1
2V 2V1
Thus, V2 = I 1 II = = 0.3144 m3 kg ;
M +M 4.537V1 + 1824
. V1
 c I I
U 2 = M U1 + M U1II  II
hb g
M1 + M1 = 2720.0 kJ kg
I II

By trial and error: T2 ~ 302° C (575 K) and P ~ 0.76 MPa .


3

3.16 System: contents of the turbine (open, steady state)


(a) adiabatic
dM  +M  ⇒M  = −M 
mass balance: =0= M 1 2 2 1
dt
0
energy balance:
dU
=0= M  H + M H + Q 0 + W − P dV
1 1 2 2 s
dt dt
  c  h 
⇒ Ws = − M1 H1 − H2 = − M1(3450.9 − 28656 . ) kJ kg
= −M1 . c
 5853 × 105 J kgh
But Ws = −7.5 × 10 watt = −7.5 × 105 J s
5

 = −7.50 × 10 J s = 1281
5
M . kg s = 4.613 × 103 kg h
−5853 × 105 J kg
1
.
(b) Energy balance is
0
dU
=0= M  H + M  H + Q + W − P dV
1 1 2 2 s
dt dt
where Q = M a
 −60 kJ kg
1 f H = H (150° C, 0.3 MPa ) = 27610
2 . kJ kg
Thus
−Ws = 1281
. kg s (3450.9 − 27610. − 60) kJ kg = 807 kJ s
= 8.07 × 105 watt = 807 kW
3

3.17 System: 1 kg of water (closed system).


z z
Work of vaporization = PdV = P dV = P∆V since P is constant at 1.013 bar. Also, from Steam
Tables
V L = 0.001044 m3 kg ; V V = 16729
. m3 kg ; ∆V = 16719
. m3 kg
Energy balance for closed system (1 kg):
z
U 2 − U1 = Q − PdV = Q − 1013
. × 105 Pa × 16719
. m3 kg
= Q − 16945
. × 105 J kg
U 2 = 25065
. kJ kg = 2.5065 × 106 J kg
U1 = 418.94 kJ kg = 41894
. × 105 J kg

Thus
Q = U 2 − U1 + W = 2.5065 × 106 − 41894
. × 105 + 16945
. × 105
= 2.2570 × 106 J kg
z
W = − PdV = 16945
. × 105 J kg .

So heat needed to vaporize liquid = 2.2570 × 106 J kg of which 016945


. × 106 is recovered as work
against the atmosphere. The remainder, 2.088 × 106 kJ kg , goes to increase internal energy.
3

3.18 System = Contents of desuperheater (open, steady state)

Superheated steam
T=500ºC Desuper- Saturated steam
P=3 MPa heater
2.25 MPa
Water
25ºC

M = 500 kg hr ; H = 34565 . kJ kg
1 1
 = ?; H = H (sat’d liq., T = 25° C) = 104.89 kJ kg
M 2 2

 +M
Mass B: 0 = M  +M
1 2 3
0
Energy B: 0 = M H + M
1 1
 H + M
2 2
 H + Q 0 +W
3 3
 0 − P dV
s
dt
 b  g  
M 3 = − 500 + M 2 kg hr ; H3 = H (sat’d steam, P = 2.25 MPa) = 28017
. kJ kg
Thus,
0 = 500 × 34565
. +M 2 b
 = 104.89 − 500 + M
2 g
 × 28017
.
 = 1214
⇒M . kg hr
2
3

3.19 The process here is identical to that of Illustration 3.5-3, so that we can use the equation

P2
T2 =
a f
P1 T1 + CV CP P2 − P1 Tin

developed in the illustration. Here, P2 = 2.0 MPa , Tin = 120° C = 39315


. K , CP = 29.3 J mol K ,
CV = CP − R = 20.99 J mol K .
C
Cylinder 1: P1 = 0 , T2 = P Tin = 5488
. K = 27565
. °C
CV
Cylinder 2: P1 = 01
. MPa , T1 = 20° C = 29315
. K

2.0
T2 = = 52587
. K = 252.7° C
01 . + 20.99 29.3 (2.0 − 01
. 29315 . ) 39315
.

Cylinder 3: P1 = 1 MPa , T1 = 20° C = 29315


. K ; ⇒ T2 = 38216
. K = 109.01° C
3

3.20 System: Gas contained in the cylinder (closed system)


M piston g 4000 kg 9.8 m s2
(a) P = 01013
. MPa + = 10133
. × 105 + ×
A 2.5 m2 1 kgm Ns2
= 11701
. × 105 Pa = 0117
. MPa
moles of
PV 11701
. × 105 Pa × 25 m3
N = gas initially = =
RT 8.314 Pa ⋅ m3 mol K × 29315. K
in system
= 1200
. × 103 mol = 1200
. kmol
z
(b) Energy balance: U 2 − U1 = Q − PdV = Q − P∆V since P is constant.
∆V = 3 m × 2.5 m = 7.5 m ; P∆V = 11701
2 3
. × 105 Pa × 7.5 m3 = 8.7758 × 105 J
Final temperature:
PV2 11701
. × 105 Pa × (25 + 7.5)m3
T2 = = = 3812. K = 108.05° C
NR 12 . × 10 mol × 8.314 Pam3 mol K
3

a f a
U 2 − U1 = N U 2 − U 1 = NCV T2 − T1 f
. × 103 mol × (30 − 8.314) J mol K × (3812
= 12 . )K
. − 29315
= 2.291 × 106 J
(c)
Q = ∆U + P∆V = 2.291 × 106 + 8.7758 × 105
∆T of work
gas work = 27.7% of energy absorbed
∆T = 72.3%

= 3169
. × 106 J = 3169
. MJ

(d) System: Gas contained within Piston + Cylinder (open system).


[Note: Students tend to assume dT dt = 0 . This is true, but not obvious!]
dN
mass balance: = N
dt
d 0 dV
energy balance: ( NU ) = N H out + Q −P
dt dt
Here (1) P is constant, (2) Ideal Gas Law V = NRT P , (3) T and P of Gas Leaving Cylinder
= T and P of gas in the system. Thus,
dU dN dN d NRT F I
N
dt
+U
dt
=H
dt
−P
dt PH K
dN dT d
⇒ (H − U ) = NCV + R ( NT )
dt dt dt
RT
dN
dt
= NCV
dT
dt
+ NR
dT
dt
+ RT
dN
dt
a
⇒ N CV + R f
dT
dt
=0

dT
⇒ = 0 Q. E.D.
dt
Thus T3 = T2 = 3812. K
Now going back to

dU dN dN dV dT dU
N +U =H −P and using =0=
dt dt dt dt dt dt
dN dN dV dN P dV
⇒ (H − U ) = RT =P or = (**)
dt dt dt dt RT dt

Since P and T are constants

N 3 V3 25 m3
= = = 0.7692
N 2 V2 25 + 7.5 m3

Thus N 3 = 0.7692 × 1200 mol = 923 mol ;


∆N = −277 mol = −0.277 kmol
3

3.21 (a) System: Gas contained within piston-cylinder (closed system) [neglecting the potential energy
change of gas]
energy balance:

d ( NU ) dU dV dT dh
=N = Q − P ; NCV = Q − PA
dt dt dt dt dt

PV dT P dV F I
PA dh
But T =
NR

dt
=
NR dt H K
=
NR dt
.

Thus

ACV P dh dh FC Idh PACP dh


Q=
R dt
+ AP
dt H
= PA V + 1
R K dt
=
R dt
30 J mol K
= × 11701
. × 105 Pa × 2.5 m2 × 0.2 m s
8.314 J mol K
= 2111
. × 105 J s

(b) System: Gas contained within piston and cylinder (open system). Start from result of Part (d),
Problem 3.20 (see eqn. (**) in that illustration)

dN P dV PA dh
= = with P and T constant
dt RT dt RT dt

(See solution to Problem 3.20)

dN 11701
dt
=
. × 105 Pa × 2.5 m2
8.314 J mol K × 3812. K
a f
× −0.2 m s = −18.46 mol s

= −0.01846 kmol s

[check: −18.46 mol sec × 15 sec = −276.9 mol compare with part d of Problem 3.20]
3

3.22 System: gas contained in the cylinder (open system)


Important observation . . . gas leaving the system (That is, entering the exit valve of the cylinder) has
same properties as gas in the cylinder.
dN  ⎫ Note that these are
mass balance =N ⎪⎪
dt
⎬ Eqns. (d) and (e) of
d ( NU )  ⎪
energy balance = NH Illustration 3.5-5
dt ⎪⎭

mass balance
dN
= N U|
Note that these are
dt
d ( NU )  V|
Eqns. (d) and (e) of
= N H Illustration 2.5- 5
energy balance
dt W
Proceeding as in that illustration we get Eqn. (f)

FG T(t ) IJ CP R
=
FG P(t ) IJ or T (t )
=
320
= 169.05 K
H T(0)K H P(0)K P(t ) R CP 10a8.314 30f
(1)

where we have used a slightly different notation. Now using the mass balance we get

dN d PV F I
V d PT
= N
a f
dt
=
dt RT H K
=
R dt

or

a f
d PT
=

NR
=
a f
− 4.5 28 mol s × 8.314 Pa ⋅ m3 mol K
= −8.908 Pa K ⋅ s
dt V . m3
015

and

P P
= − 8.908 × 10−5 t bar K for P in bar and t in secs. (2)
T t T t =0

Using t = 5 minutes = 300 secs in Eqn. (2) and simultaneously solving Eqns. (1) and (2) yields

T (5 min) = 152.57 K , P(5 min) = 0.6907 bar

Computation of rates of change from mass balance

d PF I F
1 dP Pd ln T 
NR I

d ln P d ln T NRT
H K
dt T
=
T dt

dt H =
V
or
dt

dt
=
K
PV
(3)

From energy balance (using 2 eqns. above and eqn. (f) in Illustration (3.5-5))
=
a f
CV d ln T d ln P T C d ln T d ln P
or P = (4)
R dt dt R dt dt

Now using Eqn. (4) in Eqn. (3). Thus,

CV d ln T CV dT NRT 
= = or
R dt RT dt PV
dT N ( RT )2
= = −1151
. K sec
dt t =5 min PVCV t =5 min
and

dP C P dT
= P = −0.0188 bar s
dt 5 min RT dt 5 min
3

3.23 Consider a fixed mass of gas as the (closed) system for this problem. The energy balance is:

d ( NU ) dU dT dV
=N = NCV = −P
dt dt dt dt

From the ideal gas law we have P = NRT V . Thus

dT − NRT dV C d ln T −d ln V
CV N = ⇒ V =
dt V dt R dt dt

or

CV T2 V T
ln = − ln 2 ⇒ 2
FG IJ CV R
=
FG V IJ
H K HV K
1
(*)
R T1 V1 T1 2

or

V2T2 CV R
= V1T1CV R
= VT CV R
= constant

Substituting the ideal gas law gives PV CP CV = PV γ = constant. Note that the heat capacity must be
independent of temperature to do the integration in Eqn. (*) as indicated.
3

3.24 System: Contents of the tank (at any time)


(a) Final temperature (T = 330 K) and pressure P = 1013
. c h
× 105 Pa are known. Thus, there is no
need to use balance equations.

PV 1013
. × 105 Pa × 0.3 m3
Nf = = = 1108
. mol = 0.01108 kmol
RT 8.314 J mol K ⋅ 330 K

(b) Assume, as usual, that enthalpy of gas leaving the cylinder is the same as gas in the cylinder . . .
See Illustration 3.5-5. From Eqn. (f) of that illustration we have

FG IJ CP R
FG IJ R CP
F 10133
=G
× 10 I5 8.314 29

H 10. × 10 JK
Pf T T P .
= f or f = f = 05187
Pi H K
Ti Ti H K
Pi 6
.

Thus Tf = 05187
. × 330 K = 17119
. K , Pf = 1013
. bar , and N f = 2136
. mol = 0.02136 kmol .
3

3.25 Except for the fact that the two cylinders have different volumes, this problem is just like Illustration
3.5-5. Following that illustration we obtain

2 P1i 2 P1f P2f


= f + f for Eqn. (a')
T1i T1 T2
2 i
2 P1i = 2 P1f + P2f or P f = P1 for Eqn. (c')
3

and again get Eqn. (f)

FG T IJ
f CP R
=
FG P IJ
f

HT K HP K
1 1
i i
1 1

Then we obtain P f = 1333


. bar , T1f = 2234
. K , and T2f = 328.01 K .
3

3.26 From problem statement P1f = P2f = P f and T1f = T2f = T f .


Mass balance on the composite system of two cylinders

2 P1f P2f 3P f 2 P i
N1f + N 2f = N1i or + f = f = i
T1f T2 T T

Energy balance on composite system

2 P i 2 × 200 bar
N1iU 1i = N1f U 1f + N 2f U 2f ⇒ P f = = = 1333
. bar (as before)
3 3

3P f i 3 2 iFI
and T f =
2 P1i
T1 =
HK
2 3
T1 = T1i = 250 K .
3

3.27 Even though the second cylinder is not initially evacuated, this problem still bears many similarities
to Illustration 3.5-5). Proceeding as in that illustration, we obtain

2 P1i P2i 2 P1f P2f


+ i = f + f instead of Eqn (a')
T1i T2 T1 T2
2 P1i + P2i = 2 P1f + P2f = 3 P f instead of Eqn. (c)

[Thus, P f = (2 × 200 + 1 × 20) / 3 = 140 bar ] and again recover Eqn. (f) for Cylinder 1

FG T IJ
f CP R
=
FG P IJ
f

HT K HP K
1 1
i i
Eqn. (f)
1 1

Solution is P1f = P2f = 140 bar , T1f = 226.47 K , T2f = 28651


. K.
3

3.29 In each case we must do work to get the weights on the piston, either by pushing the piston down to
where it can accept the weights, or by lifting the weights to the location of the piston. We will
consider both alternatives here. First, note that choosing the gas contained within piston and
cylinder as the system, ∆U = Q + W . But ∆U = 0 , since the gas is ideal and T = constant. Also

z a f
W = − PdV = − NRT ln Vf Vi , for the same reasons. Thus, in each case, we have that the net heat
and work flows to the gas are

W (work done on gas) = − NRT ln


FG V IJ = −2479 ln 1213
. × 10 −2
= 1622.5 J
HV K
f
−2
i 2.334 × 10
and Q = −W = −1622.5 J (removed from gas)

If more work is delivered to the piston, the piston will oscillate eventually dissipating the addition
work as heat. Thus, more heat will be removed from the gas + piston and cylinder than if only the
minimum work necessary had been used.
Note that in each case the atmosphere will provide

Watm = P∆V = 1013


. . ) × 10−2 m3 = 11356
× 105 kPa × (2.334 − 1213 . J

and the change in potential energy of piston

. ) × 10−2 m3
(2.334 − 1213
mg∆h = 5 kg × 9.8 m s2 × = 54.9 J
1 × 10−2 m

The remainder 1622.5 − 11356 . − 54.9 = 432.0 J must be supplied from other sources, as a minimum.
(a) One 100 kg weight.
An efficient way of returning the system to its original state is to slowly (i.e., at zero velocity)
force the piston down by supplying 432.0 J of energy. When the piston is down to its original
location, the 100 kg is slid sideways, onto the piston, with no energy expenditure.

An inefficient process would be to lift the 100 kg weight up to the present location of the piston
and then put the weight on the piston. In this case we would supply

Mg∆h = Mg
∆V b
−2
m 2.334 × 10 − 1213
= 100 kg × 9.8 2 ×
. × 10−2 m3 g
A s 1 × 10−2 m2
= 1098.6 kg m s = 1098.6 J
2 2

This energy would be transmitted to the gas as the piston moved down. Thus

W (on gas) = 11356


. J + 54.9 J + 1098.6 J = 22891
.
(atmosphere) ( PE of piston ) ( PE of weight )
W (J ) = −Q(J ) Wcycle = −Qcycle
Efficient 1622.5 1622.5 − 11905
. = 432.0
Inefficient 2289.1 22891. − 11905
. = 1098.6

(b) Two 50 kg weights


In this case we also recover the potential energy of the topmost weight.

m (1597
. . ) × 10−2 m3
− 1213
mg∆h = 50 kg × 9.8 × = 188.2 J
s2 0.01 m2

Thus in an efficient process we need supply only

1622.5 − 11356
. − 54.9 − 188.2 = 2438
. J

An efficient process would be to move the lowest weight up to the position of the piston, by
supplying

. ) × 10−2 m3
m (2.334 − 1213
50 kg × 9.8 × = 549.3 J
s2 1 × 10−2 m2

Slide this weight onto the piston and let go. The total work done in this case is

. +
11356 54.9 + 2438
. + 549.3 = 19836
. J
(atmosphere) ∆PE of piston ∆PE of weight supplied by us

Therefore

W (J ) = −Q Wcycle = −Qcycle
Efficient 1622.5 1622.5 − 1378.7 = 2438
. J
Inefficient 1983.6 . − 1378.7 = 604.9 J
19836

(c) Four 25 kg weights.


In this case the recovered potential energy of weights is

25 kg × 9.8 m s2 ×
FG (1897
. . ) + (1597
− 1213 . . ) + (1379
− 1213 . . ) × 10 I
− 1213
JK m
−2

H 1 × 10 −2

= 302.3 J

Thus in an efficient process we need supply only

1622.5 − 11356
. − 54.9 − 302.3 = 129.7 J

An inefficient process would be to raise the lowest weight up to the piston, expending

. ) × 10−2 m3
(2.334 − 1213
25 kg × 9.8 m s2 × = 274.6 J
1 × 10−2 m

Thus the total work done is

. + 54.9 + 302.3 + 274.6 = 1767.4 J


11356
and

W = −Q Wcycle = −Qcycle
Efficient 1622.5 1622.5 − 14930
. = −129.5
Inefficient 1767.4 1767.4 − 14930
. = −274.4

(d) Grains of sand


Same analysis as above, except that since one grain of sand has essentially zero weight
W = 1622.5 J , Q = −1622.5 J , Wcycle = −Qcycle = 0 .
3

3.30 System = Gas contained in the cylinder (closed system)


d ( NU ) dU dT dV − NRT dV
energy balance: =N = NCV = −P = {Using the ideal gas equation
dt dt dt dt V dt
of state}
Since CV and CP are constant

CV 1 dT
=−
1 dV T V
or 2 = 1
FG IJ FG IJ R CV
=
FG L IJ R CV

H K H K HL K
1
R T dt V dt T1 V2 2

⇒ T2 = (25 + 27315
. )×
FG 0.03 m IJ 3 8.314 (30 −8.314 )

H 0.03 + 0.6 × 0.05 m K


3

= 22857
. K = −44.58° C and
F V IF T I
P = P G J G J = 20 × ×
1 22857
.
= 7.666 bar
HV KH T K
1 2
2 1
2 2 29815
1 .

From the difference (change of state) form of energy balance

∆U = Q
0
a f
+ W = NCV T2 − T1 = − PdV z
PV 20 bar × 0.03 m3 kmol ⋅ K
and N = = = 0.0242 kmol
RT 298.15 K × 8.314 × 10−2 bar ⋅ m3
⇒ W = ∆U = −0.0242 kmol × (30 − 8.314) kJ kmol ⋅ K ⋅ (29815 . )K
. − 22857
= −3652. kJ
Where has this work gone?
(a) To increase potential energy of piston
(b) To increase kinetic energy of piston
(c) To push back atmosphere so system can expand
(d) Work done against friction (and converted to heat).
To see this, write Newton’s 2nd Law of Motion for the piston

Patm × A
f Fr

mg Frictional Force f Fr

Pressure of gas (P) × A

a
f = MA ⇒ PA − Patm A − mg − f fr = m f dv
dt
; v = velocity of piston
m dv mg f fr
Thus, P = + Patm + +
A dt A A
− ∆U = 36,520 J = + PdV z
z
= + PatmdV + z
m dv dV
A dt dt
dt +
mg
A
dV +
1
A
f fr
dv
dt
dt z z (1)

Now
1 dV dh
A dt
=
dt
= v ( h = piston height) and v
dv 1 d 2
=
dt 2 dt
v c h
36,520 J = Patm∆V +
3000 J
Work against
mv 2
2
+
since
mg∆h
1760 J
Work used to
z
+ f fr vdt

atmosphere vinitial = 0 increase potential


energy of piston

Thus 36,520 J = 3000 +


mv 2
2 z
+ 1760 + f fr vdt .

(a) If there is no friction f fr = 0 then

(36520 − 3000 − 1760)J × 2


v2 = = 2117
. m2 s2 ⇒ v = 14.55 m s
300 kg

(b) If we assume only sliding friction, f fr = kv

z z
f fr vdv = k v 2dt ⇒ (36520 − 3000 − 1760) =
m 2
2
v + k v 2dt z
In order to determine the velocity now we need to know the coefficient of sliding friction k, and
then would have to solve the integral equation above (or integrate successively over small time
steps). It is clear, however, that

v( with friction) < v(without friction) = 14.55 m s

W = 1622.5 J , Q = −1622.5 J , Wcycle = −Qcycle = 0 .

System for System for


part a part b
25° C,
3.0 × 106 Pa = 3Mpa
125 kg/s
3

. × 106 Pa = 3 MPa
3.31 25°C, 30
125 kg s
(a) mass balance (steady-state)
0= M +M
1 2
⇒M  = −M  = 125 kg s
1 2

Energy balance (neglecting PE terms)

0= M
FG 2
IJ
 H + v1 + M  H + v2 FG
2
IJ
1
H 1
2 K 2 2
2 H K

M = ρvA = mnvA ; ρ = mass density, n = molar density,
v = velocity, A = pipe area, m = molecular weight.

M P
= vA
m RT


125 kg s
=
. × 106 Pa
30
16 kg kmol 298.15 K × 8.314 × 103 Pa ⋅ m3
a f
× v m s × π × 0.09 m2

⇒ v = 22.83 m s
a
mv 2 16 kg kmol × 22.83 m s 2
= = 4170
.
f × 103 J kmol = 417
. kJ kmol
2 2 × 1 kg ⋅ m Ns2
Back to energy balance, now on a molar basis

H1 − H 2 =
mv22 mv12
2

2
a
= C p T1 − T2 f
As a first guess, neglect kinetic energy terms . . .
a f
Cp T1 − T2 = 0 ⇒ T1 = T2 = 25° C
Now check this assumption
nv Pv . × 106 v1
30
v2 = 1 1 = 1 1 = = 34.24 m s
n2 P2 2.0 × 106
Recalculate including the kinetic energy terms
a
Cp T1 − T2 = f
m 2 2 16
2
c
v1 − v2 =
2
h c h
34.242 − 22.832 = 5209 J kmol
5209 J kmol
T2 = T1 − = T1 − 014
. °C
. J mol × 1000 mol kmol
368
Thus the kinetic energy term makes such a small contribution, we can safely ignore it.

(b) Mass balance on compressor (steady-state) 0 = N 1 + N 2

2.0 × 106 Pa 3.0 × 106 Pa


compressor
T1=25° C T2= ?
Energy balance on compressor, which is in steady-state operation

0
0 = N 1 H 1 + N 2 H 2 + Q +Ws ⇒ Ws = N 1Cp (T2 − T1)
= 25°C
adiabatic compressor

Can compute Ws if T2 is known or vice versa. However, can not compute both without further
information.

2.0 × 106 Pa Gas cooler 3.0 × 106 Pa


T2= ? T3=25° C

Analysis as above except that Q ≠ 0 but W = 0 .


|RS0 = N 2
Here we get Q = N C ( T − T )
+ N 3
|T1 p 3 2
= 25° C

Can not compute Q until T2 is known.

See solution to Problem 3.10.


3

3.32 a) Define the system to be the nitrogen gas. Since a Joule-Thomson expansion is isenthalpic,
a f a f
H T1, P1 = H T2 , P2 . Using the pressure enthalpy diagram for nitrogen, Figure 3.3-3, we have
c h
H (135 K,20 MPa) = 153 kJ / kg and then T = T P = 0.4 MPa, H = 153 kJ / kg
2 2
From which we find that T = 90 K, with approximately 55% of the nitrogen as vapor, and 45% as
liquid.
b) Assuming nitrogen to be an ideal gas (poor assumption), then the enthalpy depends only on
temperature. Since a Joule-Thomson expansion is isenthalpic, this implies that the temperature is
unchanged, so that the final state will be all vapor.
3

3.33 Plant produces 13. 6 × 109 kwh of energy per year


⇒ Plant uses 136
. × 109 × 4 = 544. × 109 kwh of heat
1 kwh = 36. × 10 J
6

J year
⇒ Plant uses 36. × 106 × 544
. × 109 kwh = 19.584 × 1015 J year
kwh
a
∆H of rock (total) = M ⋅ Cp Tf − Ti f
= 1012
kg × 1 J g K × 1000 g kg × (110 − 600) K
= −490 × 1015 J
⇒ 19.58 × 1015 J year × x years = 490 × 1015 J
x = 2502
. years
3

3.34
a) Body temperature is 37oC. From Appendix A.III, ice at 0oC has a volume of
0.0010908 m3/kg and an enthalpy (and internal energy) of -333.43 kJ/kg. For
water at 35oC Û = 146.67 kJ/kg and at 40oC Û = 167.56 kJ/kg, so that at 37oC
-3 3 -3 3
Û = 155.03 kJ/kg. Also, 1 L = 10 m , so that 1 L of water 10 m /0.0010908
m3/kg = 0.9168 kg. So that the amount of energy needed to melt 1 L of ice is
0.9168 kg ¯(155.03-(-333.43))kJ/kg = 447.8 kJ.

b) 447.8 kJ/(42 kJ/kg fat) = 10.66 g of fat (= 0.023 lbs of fat)

c) For water at 0oC, Û = 0 kJ/kg, and V̂ = 0.001 m3/kg. Therefore, 1 L of water =


1 kg, and the energy required to warm up the water is
1 kg¯155.03 kJ/kg = 155.03 kJ, and only 155.03/42 = 3.69 g of fat would be
consumed.
3

3.35 
M 2
a)


M 3

Mass balance
 +M
M  =M = 10 kg/s M
 +M
 =M = 10 kg/s M
 +M
 =M = 10 kq/s
1 2 3 1 2 3 1 2 3

 H  ˆ kg ˆ  ˆ  H kq ˆ
1 1 + M 2 H 2 = 10 H 3 M1H1 + M 2 2 = 10
Energy balance M ˆ ˆ H3
s s
Stream 1 20°C (1 atm) Ĥ1 = 83.96 J/g
Stream 2 450°C, 2.5 MPa Ĥ 2 = 3344.0 J/g
Stream 3 90°C (1 atm) Ĥ 3 = 376.92 J/g

MB  = 10-M
M 
EB M (
 × 83.96 + 10-M

1 ) 3344 = 10 • 376.92
 = 9.101 kg/s M
M  = 9.101 kg/s M
 = 9.101 kq/s M
 = 0.899 kq/s
1 1 1 2

10 kg/s, T=90°C 1 10 kg/s, T=20°C


b)
3

2
4 10 kg/s, T=20°C
100 kg/s, quality = 50%

1 1
Ĥ = ( 2676.1) + ( 419.04 ) = 12547.6 J/g
2 2

Steady balance:
 H  ˆ  ˆ  ˆ
1 1 + M 2 H 2 = M3 H3 + M 4 H 4
M ˆ
 =M
M  = 10 kg/s
1 3
But
M =M  = ? kg/s
2 4

10 × 83.96 + M 3344 = 10 × 376.92 + M  × 1547.6


2 2

M (3344 – 1547.6) = 10 × (376.92 – 83.96)
2
 =1.63 kg/s
M 2
3

3.36 ∆vapH at 37°C ~ 2412 J/g


J 1
Amount evaporated = 4184 ×103 = 1.735 × 102 g
hr 2412 J hr
g
= 1.735 kg/hr
1.735
If only 75% evaporates ⇒ = 1.631 kg/hr of sweat produced
0.75
3

3.37
1 2 25°C, 0.8 bar
–50°C
0.1 bar

Mass balance – steady state


 +N
O=N   +N
N 
1 2 2 1

Energy balance – steady state


 +W
 H +Q
O=N  (H − H ) + Q
 =N  +W

1 2 1 1 2

 +W
Q  T2  +W
Q  T2

= H 2 − H1 = ∫ Cp dT = Cp ( T2 − T1 ) = H 2 − H1 = ∫ Cp dT = Cp ( T2 − T1 )

N 
N
1 T1 1 T1

J J J J
= 30 × 75k = 2250 = 30 × 75k = 2250
mol K mol mol K Mol
PV = NRT PV 0.8bar × 100 m3
N= = × 298.15 M = 0.03228 × 105 mol = 3228 mols
100 m3 ⇒ RT −5 bar m
3
8.314 × 10
mol K
⇒ N  = 3228 mols/min
1
 +W
Q  = 7.262 × 106 J/min = 1.210 × 105 J/s Q  +W  = 7.262 × 106 J/min = 1.210 × 105 J/s
= 1.210 × 105 Watts = 121.0 kw = 1.210 × 105 Watts = 121.0 kw
$ $
Cost = 0.2 × 121.0 kw = 24.2 $/hr Cost = 0.2 × 121.0 kw = 24.2 $/hr
kw ⋅ hr kw ⋅ hr
4

4.1 (a) System = Ball (1) + Water (2)


Energy balance: M1U1f + M2U 2f − M1U1i − M 2U 2i = 0

c h c h
⇒ M1CV,1 T1 f − T1i + M 2CV,2 T2f − T2i = 0 ; also T1 f − T2f . Thus
M1CV,1T1i + M 2CV,2T2i 5 × 10 × 05
3
. × 75 + 12 × 103 × 4.2 × 5
Tf = =
M1CV,1 + M 2CV,2 5 × 103 × 05
. + 12 × 103 × 4.2
= 8.31° C
[Note: Since only ∆T s are involved, ° C were used instead of K)].
(b) For solids and liquids we have (eqn. 4.4-6). That ∆S = M CP
dT
T
T
z
= MCP ln 2 for the case in
T1
which CP is a constant. Thus

Ball: ∆S = 5 × 103 g × 05
J
× ln RS
8.31 + 27315
. UV
= −53161
J
.
g⋅K T
75 + 27315
. W .
K
= −53161
. s

Water: ∆S = 12 × 103 g × 4.2


J
× ln RS
8.31 + 27315
. UV
= +596.22
J
g⋅K T
5 + 27315
. W K
and
J J
∆S (Ball + Water ) = 596.22 − 53161
. = 64.61
K K
Note that the system Ball + Water is isolated. Therefore
J
∆S = Sgen = 64.61
K
4
4.2 Energy balance on the combined system of casting and the oil bath

c h c h
M cCV,c T f − Tci + M oCV,o T f − Toi = 0 since there is a common final temperature.

20 kg × 0.5
kJ
kg ⋅ K
dT − 450iK + 150 kg × 2.6 kgkJ⋅ K dT
f f
i
− 450 K = 0
f o
This has the solution T = 60 C = 313.15 K
Since the final temperature is known, the change in entropy of this system can be
F 27315
. + 60 I
+ 150 × 2.6 × lnF
. + 60 I
27315 kJ
calculated from ∆S = 20 × 05
. × ln
H 27315
. + 450 K H . + 50
27315 K = 4135
.
K
4
4.3 Closed system energy and entropy balances
dU dV dS Q 
= Q + Ws − P ; = + Sgen ;
dt dt dt T
dS
Thus, in general Q = T − TSgen and
dt

dU  dV dU dS dV
Ws = −Q+ P = −T + TSgen + P
dt dt dt dt dt

c h
Reversible work: WsRev = WsRev Sgen = 0 =
dU
dt
−T
dS
dt
+P
dV
dt
dU dV
(a) System at constant U & V ⇒ = 0 and =0
dt dt

 S
W c
 h  Rev = −T dS
s gen = 0 = WS
dt

dS dP dV d
(b) System at constant S & P ⇒ = 0 and = 0⇒ P = ( PV )
dt dt dt dt
so that

c h
Ws Sgen = 0 = WSrev =
dU d d
+ ( PV ) = (U + PV ) =
dt dt dt
dH
dt
4
4.4
700 bar, 600oC 10 bar, T = ?

Steady-state balance equations


dM  +M 
=0= M 1 2
dt
0
dU
=0= M  H + M  H + Q 0 + W 0 − P dV  H + M
=M  H
1 1 2 2 s 1 1 2 2
dt dt
or H 1 = H 2
Drawing a line of constant enthalpy on Mollier Diagram we find, at P = 10 bar,
T ≅ 308° C

At 700 bar and 600° C At 10 bar and 308° C


V = 0.003973 m3 kg 
V ≈ 0.2618 m3 kg
H = 3063 kJ kg H ≈ 3063 kJ kg
S = 5522
. kJ kg K S = 7145
. kJ kg K
Also

 0
dS
=0= M  S + Q
 S + M + Sgen = 0
1 1 2 2
dt T

⇒ Sgen =M1 2c 1 h
 S − S or Sgen = S − S = 7145
M 
1
2 1 . − 5522
. = 1623
.
kJ
kg ⋅ K
4
1 2
4.5
System Ws

Energy balance

c h c
∆U = U 2f − U 2i + U1f − U1i = Q h adiabatic
z constant
+ WS − PdV volume

c h c
Ws = MCp T2f − T2i + MCp T1 f − T1i = MCp T2f − T2i + T1 f − T1i h c h c h
Ws
but T1 f = T2f = T f ⇒ = 2T f − T1i − T2i
MCP
Entropy balance
adiabatic

∆S = c S2f − S2i h+c S1f − S1i h= z  0


Q
T
dt +Sgen
0 for maximum work

c h c h Tf
T2
Tf
S2f − S2i + S1f − S1i = 0 = MCP ln 2i + MCP ln 1 i
T1
RS T T UV = 0 ; T T = T T ; but T
or ln
f f
2 1 f f i i f
= T2f = T f
TTT W i i
2 1
1 2 1 2 1

⇒ cT h = cT T h or T = T T and
f 2 i i
1 2
f i i
1 2

Ws
= 2T f − T1i − T2i = 2 T1i T2i − T1i − T2i
MCP
4
4.6

1 bar 10 bar
(a) Entropy change per mole of gas
T P 290 K 575 K
∆S = CP ln 2 − R ln 2 eqn. (4.4-3)
T1 P1
J 575 J 10 J
Thus ∆S = 29.3 ln − 8.314 ln = 0.9118
mol K 290 mol K 1 mol K
(b) System = contents of turbine (steady-state system)
dN
Mass balance = 0 = N 1 + N 2 ⇒ − N 2 = N 1 = N
dt
dU 0 dV 0
Energy balance = 0 = N 1 H 1 + N 2 H 2 + Q + Ws − P
dt dt
a f
Ws = N H 2 − H 1 = NC a
 T −T
P 2 f
1

W
a f
W = s = CP T2 − T1 = 29.3
N
J
mol K
× (575 − 290)K

J
= 83505
.
mol
(c) In Illustration 4.5-1, W = 7834.8 J mol because of irreversiblitities (∆S ≠ 0) ,
more work is done on the gas here. What happens to this additional energy input? It
appears as an increase of the internal energy (temperature) of the gas.
4
4.7 Heat loss from metal block
dU dT
= CP = Q
dt dt

−W =
T − T2 
Q(−1)
RS
Q = heat out of metal
T T
−Q = heat into heat engine
a
dT T − T2 f
= −W ⇒ − Wdt
 =C
2
T
z
t T

z FH I
CP
dt T 0
P
T
1 − 2 dT
T
1
K
a T
f T
−W = CP T2 − T1 − CP ⋅ T2 ln 2 = CP T2 − T1 − T2 ln 2
LMa f OP
T1 T1 N Q
−W = CPT2 1 −
LMFG T1 TIJ
− ln 2
OP
NH T2 T1K Q
z
T2
Q = CPdT = CP T2 − T1 = CPT2 1 − a f FG T1 IJ
T1
H T2 K
Alternate way to solve the problem

T2

System is the metal block + heat engine (closed)


dU dT
E.B.: = CP = Q + W
dt dt
dS Q
S.B.: = 2 +Sgen
dt T
0 for maximum work
dS dU dS  C
Q = T2 ; = T2 + W ; dU = CPdT ; dS = P dT
dt dt dt T
W =
dU dS C T F I
dt
− T2
dt
= CP dT − T2 P dT = CP 1 − 2 dT
T T H K
z
 = C 1 − T2 dT = C
T2
T
z F I I z FH
T2
W = Wdt
T
P
T
P
T
1 − 2 dT
T
1
H K K 1

a
W = CP T2 − T1 − T2CP ln f T2 LF T I T O
= C T MG1 − J − ln P
NH T K T Q
1 2
P 2
T1 2 1
4
4.8 This problem is not well posed since we do not know exactly what is happening. There are several
possibilities:
(1) Water contact is very short so neither stream changes T very much. In this case we have the
Carnot efficiency

−W Thigh − Tlow 22 22
η= = = = = 0.0733 = 7.33%
Q Thigh 27 + 273 300

(2) Both warm surface water (27°C) and cold deep water (5°C) enter work producing device, and
they leave at a common temperature.

TH

TO
TL

M.B.:
dM
dt
=0= M  +M
H
 +M
L
 ⇒M
0
 =− M
0
 +M
H

L b g
dU  H + M H + M  H + W = 0
E.B.: =0= M H H L L 0 0
dt
 H − M
W = − M H H
 H + M
L L
 +M
H
 H
L b0 g

=M H c H − H h + M cH − H h
0 H L 0 L

=M H C aT − T f + M
P 0
 C aT − T f
H L P 0 L
 0 0
S.B.:
dS
=0= M  S + M S + Q
 S + M +Sgen
H H L L 0 0
dt T
M 
 S +M
H H

 S − M
L L
 +M
H

 S =0
L 0 b g
M H Hc
 S − S + M
0 hL L 0 H P c
 C ln TH + M
 S − S = 0 ⇒ M
T0
 C ln TL = 0
L P
T0
h
FG T IJ FG T IJ
M H 
M L
 
= 1 or TH M H TL M L = T0 M H + M L
 
HT K HT K
H L

0 0

T0 =

M
TH H
H Lb M
TL L H
+ M g M b M 
+M L g
From this can calculate T0 . Then

W = M
 C T −T + M
H P 0 H
 C T −T
L P 0 a
L f a f
This can be used for any flow rate ratio.
(3) Suppose very large amount of surface water is contacted with a small amount of deep water, i.e.,
 >> M
M  . Then T ~ T
H L 0 H
W = M a
 C T −T + M
H P H H f a
 C T −T ~ M
L P H L f
 C T −T
L P H L a f
(4) Suppose very large amount of deep water is contacted with a small amount
of surface water, i.e., M H << M L , T0 ~ TL .
W = M a
 C T −T + M
H P L H f a
 C T −T ~ M
L P L L f
 C T −T
H P L H a f
4
4.9 System = contents of the turbine. This is a steady-state, adiabatic, constant volume system.
dM  +M  or M  = −M 
(a) Mass balance =0= M 1 2 2 1
dt
Energy balance
constant
dU
=0= M  H + M  H + Q adiabatic +W − P dV volume
1 1 2 2 s
dt dt
Entropy balance

dS
=0= M  S + M S + Q +Sgen
1 1 2 2
dt T
0, by problem statement
Thus
M = −M = −4500 kg h M.B.
2 1

S c
W = − M H − H
 
1 1 2 h E.B.
S2 = S1 S.B.
State T1 = 500° C Steam H 1 = 3422.2 kJ kg
1 ⎯ ⎯⎯⎯→
P1 = 60 bar Tables S = 68803
. kJ kg
1

State P2 = 10 bar Steam T2 ≅ 240.4° C


⎯ ⎯⎯⎯→
2 Tables
S2 = S1 = 68803
.
kJ H 2 ≈ 29205
. kJ kg
kgK
kg kJ kJ
Ws = 4500 × (29205 . − 3422.2) = −2257650 = −6271 . kW
h kg h
a f
(b) Same exit pressure P2 = 10 bar , and still adiabatic
⇒ W = − Ms c
 H − H .
1 1 2 h
Here, however,
Ws = 08
. Ws (Part a) = 08 . −2.258 × 106 c kJ
h
h c
= 4500 H 2 − 3422.2
kJ
h
h

⇒ H2 = 30208 . kJ kg Steam T2 ≅ 286.7 K
⎯ ⎯⎯⎯→
P = 10 bar Tables S2 ≈ 7.0677 kJ kg K
Thus
Sgen = − M 1 1 c 2
h
h
 S − S = −4500 kg × (68803 . − 7.0677)
kJ
kg K
= 8433
.
kJ
K⋅h
(c) Flow across valve is a Joule-Thompson (isenthalpic expansion) ... See Illustration 3.4-1.
Thus, H into valve = H out of valve , and the inlet conditions to the turbine are

H 1 = H out of valve = H into valve = 3422.2 kJ kg


P1 = 30 bar
Steam T1 ≈ 484.8° C
⎯ ⎯⎯⎯→
Tables S1 ≈ 71874
. kJ kg K

Flow across turbine is isentropic, as in part (a)

S2 = S1 = 71874


. kJ kg K Steam T2 ≅ 3181
. °C
⎯ ⎯⎯⎯→ 
P2 = 10 bar Tables H2 ≈ 3090.4 kJ kg

kg kJ kJ
Ws = 4500 × (3090.4 − 3422.2) = −1493
. × 106 = −414.8 kW
h kg h
4
4.10 Since compression is isentropic, and gas is ideal with constant heat capacity, we have

FG T IJ = FG P IJ R CP

HT K H PK
2 2

1 1

So that T2 = T1
FG P IJ FG 3 × 10 IJ
R CP
= 29815
6 8.314 36.8
= 326.75 K . Now using, from solution
HPK H 2 × 10 K
2
. 6
1
to Problem 3.31, that W = NC aT − T f
 
s P 2 1

kg 1 mol J 1000 g
Ws = 125 × × 368
. × (326.75 − 29815
. )K ×
s 16 g mol K kg
= 8.23 × 106 J s

The load on the gas cooler is, from Problem 3.31,

Q = NC a
 T −T
p 3 2 f
125 kg s × 1000 g kg J
= × 368
. × (29815
. − 326.75) K
16 g mol mol K
= −8.23 × 10+6 J s
4
4.11 (a) This is a Joule-Thomson expansion
⇒ H (70 bar, T = ?) = H (10133
. bar, T = 400° C) ≈ H (1 bar, T = 400° C)
= 3278.2 kJ kg
and T = 447° C , S = 6.619 kJ kg K
c h
(b) If turbine is adiabatic and reversible Sgen = 0 , then Sout = Sin = 6.619 kJ kg K and P = 1013
.
bar. This suggests that a two-phase mixture is leaving the turbine

S V = 7.3594 kJ kg K
Let x = fraction vapor
S L = 13026
. kJ kg K
Then x(7.3594) + (1 − x)(13026
. ) = 6.619 kJ kg K or x = 08778
. . Therefore the enthalpy of
fluid leaving turbine is

kJ
H = 08788
. × 26755
. + (1 − 08778
. )× 417.46 = 2399.6
H V
(sat’d, 1 bar ) H ( sat’d, 1 bar )
L
kg

Energy balance

dV 0
0 = M in Hin + M out Hout + Q 0 +Ws − P
dt

but Min = − M out


Ws kJ
⇒− = 3278.2 − 2399.6 = 878.6
M in kg
(c) Saturated vapor at 1 bar

S = 7.3594 kJ kg K ; H = 26755
. kJ kg
Ws
− = 3278.2 − 26755
. = 602.7 kJ kg
M in Actual
602.7 × 100
Efficiency (% ) = = 68.6%
878.6
Sgen
= 7.3594 − 6.619 = 0.740 kJ Kh
M in
(d) W 0 = M1 + M 2 ⇒ M 2 = − M1

Steam Water c h
0 = M1 H1 − H2 + Ws + Q − P
dV
dt
70 bar 1 bar
0 = M cS − S h + + S
447° C 25° C Q
1 1 2 gen
T
Q

Simplifications to balance equations


dV
Sgen = 0 (for maximum work); P = 0 (constant volume)
dt
Q Q
= where T0 = 25° C (all heat transfer at ambient temperature)
T T0
kJ kJ
H (sat' d liq, 25° C) = 104.89 ; S (sat'd liq, T = 25° C) = 0.3674
kg kg K
−Ws
Q
M
c
= T0 S2 − S1 ; h
M
c h c h c
= H1 − H2 + T0 S2 − S1 = H1 − T0 S1 − H2 − T0 S2 h
max

−Ws
= 3278.2 − 29815
. × 6.619 − 104.89 − 29815
. × 0.3674
M max
= 1304.75 + 4.65 = 1309.4 kJ kg
4
4.12 Take that portion of the methane initially in the tank that is also in the tank finally
to be in the system. This system is isentropic S f = Si .
(a) The ideal gas solution

FG P IJ R Cp
F 35. I 8.314 36
S f = S i ⇒ Tf = Ti
H 70 K
= 300 = 150.2 K
f
HPK
i
PV PV Pf V
N= ⇒ Ni = i = 1964.6 mol; N f = = 196.2 mol
RT RTi RTf
∆N = N f − Ni = −1768.4 mol

(b) Using Figure 2.4-2.


70 bar ≈ 7 MPa, T = 300 K Si = 505
. kJ kg K = S f
3
m 0.7m3
Vi = 0.0195 , so that mi = = 3590
. kg.
kg m3
0.0195
kg
g
35.90 kg × 1000
kg
Ni = = 1282 mol
g
28
mol
At 3.5 bar = 0.35 MPa and S f = 505
. kJ kg K ⇒ T ≈ 138 K. Also,
m3 0.7m3
Vf = 0192
. , so that m f = = 3646
. kg.
kg m3
.
0192
kg
g
.
3646 kg × 1000
kg
Nf = = 130.2 mol
g
28
mol

∆N = N f − Ni = 130.2 − 1282 = −11518


. mol
4
dT dV
4.13 dS = C +R eqn. (4.4-1)
T V

∆S = z LNM
(a − R) + bT + cT 2 + dT 3 + OP
e dT
Q
T2 T
+R
dV
V z
so that

a f a f
S T2 , V 2 − S T1, V 1 = (a − R) ln
T2
T1
f c a hc
+ b T2 − T1 + T22 − T12
2

+ cT − T h − cT − T h + R ln
d 3 e 3 V −2 −2 2
2 1 2 1
3 2 V 1

Now using

V 2 T2 P1
PV = RT ⇒ = ⋅ ⇒
V 1 T1 P2

a f a f
S T2 , P2 − S T1, P1 = a ln
T2
T1
a c
f c
+ b T2 − T1 + T22 − T12
2
h
+
3
c
d 3 e
2
h c P
T2 − T13 − T2−2 − T1−2 − R ln 2
P1
h
Finally, eliminating T2 using T2 = T1 P2 V 2 PV
1 1 yields

a f a
S P2 ,V 2 − S P1,V 1 = a lnf FG P V IJ + b a P V − PV f
2
H PV K R
2
2 2 1 1
1 1

+
c
2R
a P V f − a PV f
2 2 2
2
1 1
2

+
d
3R
a P V f − a PV f
3 2 2
3
1 1
3

eR 2

2
d
P2V 2 −2 − PV
1 1
−2 P
− R ln 2
P1
i d i
4
4.14 System: contents of valve (steady-state, adiabatic, constant volume system)

Mass balance 0 = N 1 + N 2
0 dV 0
Energy balance 0 = N 1 H 1 + N 2 H 2 + Q 0 + Ws −P
dt
⇒ H1 = H 2
Q 0
Entropy balance 0 = N 1 S 1 + N 2 S 2 + Sgen +
T
Sgen
⇒ ∆S = S 2 − S 1 =
N
(a) Using the Mollier Diagram for steam (Fig. 3.3-1a) or the Steam Tables

T1 = 600 K P2 = 7 bar T2 ≈ 293° C


 ⇒ 
P1 = 35 bar H2 = 30453. Jg S2 = 7.277 J g K

H 1 = H 2 = 30453
. J g . Thus S1 = 65598
. J g K ; Texit = 293° C
∆S = S − S = 0.717 J g K
2 1
(b) For the ideal gas, H 1 = H 2 ⇒ T1 = T2 = 600 K

a f a f
∆ S = S T2 , P2 − S T1, P1 = Cp ln
T2
T1
P
− R ln 2
P1
P2
= − R ln = 1338
. J mol K ⇒
P1
∆S = 0.743 J mol K
4
4.15 From the Steam Tables
P = 15538
. MPa
V = 0.001157 m3 / kg VV = 012736
L
. m3 / kg
U L = 850.65 kJ / kg U V = 25953
. kJ / kg
At 200oC,
H L = 852.45 kJ / kg H L = 27932
. kJ / kg
S L = 2.3309 kJ / kg ⋅ K S = 6.4323 kJ / kg ⋅ K
V

∆H vap = 1940.7 kJ / kg ∆S vap = 41014


. kJ / kg ⋅ K
(a) Now assuming that there will be a vapor-liquid mixture in the tank at the end, the properties
of the steam and water will be
P = 0.4578 MPa
V L = 0.001091 m3 / kg VV = 0.3928 m3 / kg

o U L = 63168
. kJ / kg U V = 2559.5 kJ / kg
At 150 C,
H L = 632.20 kJ / kg H V = 27465
. kJ / kg
S L = 18418
. kJ / kg ⋅ K 
S = 68379
V
. kJ / kg ⋅ K
∆H = 2114.3 kJ / kg ∆S vap = 4.9960 kJ / kg ⋅ K
 vap

(b) For simplicity of calculations, assume 1 m3 volume of tank.


Then
0.8 m3
Mass steam initially = = 6.2814 kg
0.12736 m3 / kg
0.2 m3
Mass water initially = = 172.86 kg
0.001157 m3 / kg
6.2814
Weight fraction of steam initially = = 0.03506
179.14
6.2814
Weight fraction of water initially = = 0.96494
179.14
The mass, energy and entropy balances on the liquid in the tank (open system)
at any time yields
L L L L
dM L  L ; dM U = M  L H V ; and dM S = M  L SV
=M
dt dt dt
dU L dM L
dM L
or M L + U L =M  L H V = H V
dt dt dt
 L dM L
ML
dU
dt
=
dt
c h
H V − U L

Also, in a similar fashion, from the entropy balance be obtain


dS L dM L V  L
ML
dt
=
dt
c
S −S =
dt
h
dM L  vap
∆S

There are now several ways to proceed. The most correct is to use the steam
tables, and to use either the energy balance or the entropy balance and do the
integrals numerically (since the internal energy, enthalpy, entropy, and the
changes on vaporization depend on temperature. This is the method we will
use first. Then a simpler method will be considered.
Using the energy balance, we have
dM L dU L
= , or replacing the derivatives by finite differences
ML H V − U L
MiL+1 − MiL U iL+1 − U iL
= V
FG U L − U iL
or finally MiL+1 = MiL 1 + i +V1
IJ
M i
L
iH − U
i
L
H H − U Li i K
So we can start with the known initial mass of water, then using the Steam
Tables and the data at every 5oC do a finite difference calculation to obtain the
results below.

i T (oC) U iL (kJ/kg K) H iV (kJ/kg K) MiL (kg)


1 200 850.65 2793.2 172.86
2 195 828.37 2790.0 170.88
3 190 806.19 2786.4 168.95
4 185 784.10 2782.4 167.06
5 180 762.09 2778.2 165.22
6 175 740.17 2773.6 163.42
7 170 718.33 2768.7 161.67
8 165 696.56 2763.5 159.95
9 160 674.87 2758.1 158.27
10 155 653.24 2752.4 156.63
11 150 631.68 2746.5 155.02

So the final total mass of water is 155.02 kg; using the specific volume of
liquid water at 150oC listed at the beginning of the problem, we have that the
water occupies 0.1691 m3 leaving 0.8309 m3 for the steam. Using its specific
volume, the final mass of steam is found to be 2.12 kg. Using these results, we
find that the final volume fraction of steam is 83.09%, the final volume
fraction of water is 16.91%, and the fraction of the initial steam + water that
has been withdrawn is
(172.86+6.28-155.02-2.12)/(172.86+6.28) = 0.1228 or 12.28%. A total of
22.00 kg of steam has withdrawn, and 87.7% of the original mass of steam
and water remain in the tank.

For comparison, using the entropy balance, we have


dM L dS L
= , or replacing the derivatives by finite differences
ML SV − S L
MiL+1 − MiL SiL+1 − SiL
=
FG S L − S L
of finally MiL+1 = MiL 1 + i +1 vap i
IJ
Mi L
∆Sivap
H ∆Si K
So again we can start with the known initial mass of water, then using the
Steam Tables and the data at every 5oC do a finite difference calculation to
obtain the results below.

i T (oC) SiL (kJ/kg K) SiL (kJ/kg K) MiL (kg)


1 200 2.3309 6.4323 172.86
2 195 2.2835 6.4698 170.86
3 190 2.2359 6.5079 168.92
4 185 2.1879 6.5465 167.02
5 180 2.1396 6.5857 165.17
6 175 2.0909 6.6256 163.36
7 170 2.0419 6.6663 161.60
8 165 1.9925 6.7078 159.87
9 160 1.9427 6.7502 158.18
10 155 1.8925 6.7935 156.53
11 150 1.8418 6.8379 154.91

So the final total mass of water is 154.91 kg; using the specific volume of
liquid water at 150oC listed at the beginning of the problem, we have that the
water occupies 0.1690 m3 leaving 0.8310 m3 for the steam. Using its specific
volume, the final mass of steam is found to be 2.12 kg. Using these results, we
find that the final volume fraction of steam is 83.10%, the final volume
fraction of water is 16.90%, and the fraction of the initial steam + water that
has been withdrawn is
(172.86+6.28-154.91-2.12)/(172.86+6.28) = 0.1234 or 12.34%. A total of
22.11 kg of steam has withdrawn, and 87.7% of the original mass of steam
and water remain in the tank.

These results are similar to that from the energy balance. The differences are
the result of round off errors in the simple finite difference calculation scheme
used here (i.e., more complicated predictor-corrector methods would yield
more accurate results.).

A simpler method of doing the calculation, avoiding numerical integration, is


to assume that the heat capacity and change on vaporization of liquid water
are independent of temperature. Since liquid water is a condensed phase and
the pressure change is small, we can make the following assumptions
U L ≈ H L and H V − H L = ∆H vap
dU L dH L dT L dS L CPL dT L
≈ ≈ CPL ; and ≈
dt dt dt dt T dt
With these substitutions and approximations, we obtain from the energy
balance
dU L dM L  V  L dH L dM L  vap
ML
dt
=
dt
H −U c → ML h
dt
=
dt
∆H

dT dM L  vap
M LCPL = ∆H
dt dt
Now using an average value of CPL and ∆H vap over the temperature range we obtain
CPL dT 1 dM L
= L or
∆H vap
dt M dt
CPL
( − ) =
M fL FG IJ
∆H vap
150 200 ln
M iL H K
and from the entropy balance
dS L dM L  vap C L dT dM L  vap
ML = ∆S → ML P = ∆S
dt dt T dt dt
Now using an average value of CPL and ∆S vap over the temperature range we obtain
CPL dT 1 dM L
= or
T∆S vap dt M L dt
CPL F
150 + 27315
. M fL I FG IJ
∆S vap
ln
H
200 + 27315
.
= ln
M iL K H K
From the Steam Table data listed above, we obtain the following estimates:
U (T = 200o C) − U (T = 150o C) 852.45 − 632.20 kJ
CPL = = = 4.405
o
200 C - 150 Co
50 kg ⋅ K
or using the ln mean value (more appropriate for the entropy calculation) based on
FG T IJ = SaT f − SaT f
CPL ln
HTK
2
2 1
1

S(T = 200o C) − S(T = 150o C) 2.3309 − 18418


. kJ
CPL = = = 4.3793
F200 + 27315. I 47315
. F kg ⋅K I
ln
H150 + 27315. K
ln
42315
. H K
Also, obtaining average values of the property changes on vaporization, yields
∆H vap =
1
2
b g 1
b g
× ∆H vap T = 150o C + ∆H vap T = 200o C = × 2114.3 + 1940.7 = 2027.5
2
kJ
kg

∆S vap
1
b g b
1
g
= × ∆S vap T = 150o C + ∆S vap T = 200o C = × 4.9960 + 41014
2 2
. = 4.5487
kJ
kg ⋅ K
With this information, we can now use either the energy of the entropy
balance to solve the problem. To compare the results, we will use both (with
the linear average Cp in the energy balance and the log mean in the entropy
balance. First using the energy balance
CPL
( − ) =
M fL
=
FG IJ
−4.405 × 50
= −010863
∆H vap
150 200 ln
M iL H K
2027.5
.

M fL
= exp(−010863
. ) = 089706
.
M iL
Now using the entropy balance
FG M IJ = C lnF 150 + 27315
L L
. I 4.3793 F 42315
. I
= 0.9628 lnF
. I
42315
H M K ∆S H 200 + 27315 . K 4.5487 H 47315
=
. K H 47315
. K
f P
ln L vap
ln
i

=F
. I
L 0.9628
M 42315
H 47315
. K
= 089805
f
L
.
M i

Given the approximations, the two results are in quite good agreement. For
what follows, the energy balance result will be used. Therefore, the mass of
water finally present (per m3) is
M L ( final) = 0897
. × M L (initial) = 15506
. kg
L bL
g
occupying V = M ( final) × V 150 C = 15506
o
. × 0.001091 = 01692
. m3
Therefore, the steam occupies 0.8308 m3 , corresponding to
0.8308 m3 0.8308 m3
M V (final) = = = 2115
b
V V 150o C g0.3928
m3
. kg

kg
So the fraction of liquid in the tank by mass at the end is 155.06/(155.06+2.12) = 0.9865,
though the fraction by volume is 0.1692. Similarly the fraction of the tank volume that is
steam is 0.8308, though steam is only 2.12/(155.06+2.12) = 0.0135 of the mass in the tank.

(c) Initially there was 6.28 + 172.86 = 179.14 kg of combined steam and water,
and finally from the simpler calculation above there is 155.06 + 2.12 = 157.18
kg. Therefore, 87.7% of the total amount of steam + water initially in the tank
are there finally, or 12.3% has been withdrawn. This corresponds to 21.96 kg
being withdrawn. This is in excellent agreement with the more rigorous finite
difference calculations done above.
4
dN
4.16 (a) = 0 = N 1 + N 2 ; or N 2 = − N 1
dt

dU dV W
= 0 = N 1 H 1 + N 2 H 2 + WS + Q − P = WS + N 1 H 1 − N 1 H 2 or S = H 2 - H 1
dt dt N 1

dS Q Sgen J
= 0 = N 1 S 1 − N 1 S 2 + + Sgen = S 2 − S1 CP = 37151
.
dt T0 N
1 mol ⋅ K

WS
z
Tf

N 1
= H 2 - H1 = c
CPdT = CP ⋅ Tf − 29815
. K h if the heat capacity is independent of
298.15K
temperature. First consider the reversible case,

z z
Tf 10
CP dP
S 2 − S 1 = 0 gives dT = R The solution is 499.14K. Then
Ti
T 1
P

WSrev J
 = CP ⋅ (49914 . K) = 7467
. − 29815 . The actual work is 25% greater
N1 mol

Wact = 125
. WSrev = 9334
J
mol
c
= CP ⋅ Tf − 29815
. K h
The solultion is Tf = 549.39K

(b) Repeat the calculation with a temperature-dependent heat capacity


CP (T ) = 22.243 + 5977
. ⋅ 10−2 T − 3499
. ⋅ 10−5 T 2 + 7.464 ⋅ 10−9 T 3
Assuming reversibility Tf = 479.44K. Repeating the calculations above with the temperature-
dependent heat capacity we find Wact = 9191 J, and Tf =520.92K.
So there is a significant difference between the results for the constant heat capacity and variable
heat capacity cases.
4
4.17 Ti = 300 K, Tf = 800 K, and Pi = 1.0 bar
J
CP (T ) = 29.088 - 0.192 × 10-2 T + 0.4 × 10-5 T 2 - 0.870 × 10-9 T 3
mol ⋅ K

z z
T f =800K Pf
CP (T ) dP
dT = P
T = 300K
T P =1
P
i i

Calculated final pressure Pf = 3.092 × 106 Pa.

z
T f =800K
J
Wrev = CP (T )dT = 1458
. × 104
Ti = 300K
mol
4
4.18 Stage 1 is as in the previous problem.
Stage 2
Following the same calculation as above.
Stage 2 allowed pressure Pf ,2 = 9.563 × 107 Pa
J
Wrev = 1.458 × 10-4 = Stage 2 work
mol
Stage 3
Following the same calculation method
Pf ,3 = 2.957 × 10-9 Pa = Stage 3 allowed pressure.
J
Wrev = 1.458 × 104 = Stage 3 work
mol
Question for the student: Why is the calculated work the same for each stage?
4
4.19 The mass, energy and entropy balances are
dM  +M  = 0, M  = −M 
=M 1 2 2 1
dt
dU
dt
=0= M  H + M
1 1
 H + Q + W ; M
2 2 s c
1 1 2h
 H − H + W = 0;
s

Ws = + M 1c
 H − H
2 1 h

dS
dt
=0= M S + M
1 1
 S + Q + S = M
2 2
T
gen 1 1 c2 h
 S − S + S = 0
gen

Sgen = M c
 S − S
1 2 1 h
300° C, 5 bar = 05
. MPa H 1 = 3064.2 kJ kg
S = 7.4599 kJ kg K
1

100° C, 1 bar = 01
. MPa H 2 = 2676.2 kJ kg
S = 7.3614 kJ kg K
2

Ws
= 2676.2 − 3064.2 = 388 kJ kg satisfied the energy balance.
M
1
Sgen  
 = S2 − S1 = 7.3614 − 7.4599 = −0.0985 kJ kg K
can not be. Therefore the process is impossible.
M 1
4
4.20 Steam 20 bar = 2 MPa and 300° C H = 30235
. kJ kg

S = 6.7664 kJ kg (from Steam Tables)
U = 2772.6 kJ kg
Final pressure = 1 bar. For reference saturation conditions are
P = 01
. MPa, T = 99.63
U = 417.36
L
H L = 417.46 S L = 13026
.

U = 25061
V
. 
H = 26755
V
. 
S = 7.3594
V

(a) Adiabatic expansion valve W = 0 and Q = 0

dM  +M  =0; M  = −M  ;
M.B.: =M 1 2 2 1
dt
dU  H + M  H = 0 ; H = H
E.B.: =M 1 1 2 2 2 1
dt
From Steam Tables
⇒ H2 = 30235 . T = 250° C H = 2974.3 kJ kg S = 8.0333 kJ kg K
P = 01
. MPa T = 300° C H = 3074.3 kJ/kg S = 8.2158 kJ/kg K
By interpolation T = 275° C gives H = 3023.5 kJ / kg ⇒ all vapor
S = 81245
. kJ kg K
dS  +M  S + S = 0
= MS 1 2 2 gen
dt
Sgen
= S2 − S1 = 8.1254 − 6.7664 = 1359
. kJ kg K
M 
gen
(b) Well designed, adiabatic turbine

E.B.: M1 1 2 2 2 c
 H + W = 0 ; W = H − H
 H + M
1 h
 S + M
S.B.: M  S = 0 ; S = S ; S = 6.7664 kJ kg K
1 1 2 2 2 1 2
⇒ Two-phase mixture. Solve for fraction of liquid using entropy balance.

x ⋅(7.3594) + (1 − x)⋅ 13026


. = 6.7664
x = 0.902 not good for turbine!)
(
H 2 = 0.902 × 26755. + 0.098 × 417.46 = 2454.2 kJ kg
W 
( . ) = −569.3 kJ kg
M = 2454.2 − 30235
W
− = 569.3 kJ kg

M

(c) Isorthermal turbine ⇒ superheated vapor


T = 300° C OP H = 3074.3 kJ kg
P = 01
. MPa Q
final state
S = 8.2158 kJ kg K
 H + M
E.B.: M  H + Q + W = 0
1 1 2 2 s

Q 0
 S + M
S.B.: M  S + +Sgen =0
1 1 2 2
T
Q
T
= −M  S − M
1 1
 S = M
2 2 c1 2 h
 S − S
1

Q
M  c h
= T S2 − S1 = (300 + 27315
. )(8.2158 − 6.7664) kJ kg K

= 830.7 kJ kg
 Q
W
− s =
M M c h
+ H 1 − H 2 = 830.7 + (30235
. − 3074.3) = 779.9 kJ kg

⇒ get more work out than in adiabatic case, but have to put in heat.
4
4.21 System = contents of the compressor (steady-state, constant volume). Also, gas is
ideal.
(a) Mass balance 0 = N1 + N 2 → N 2 = − N1
0 dV 0
Energy balance 0 = N1 H 1 + N 2 H 2 + Q +Ws − P
adiabatic
dt
Entropy balance

reversible

Q 0 compressor
0 = N1 S 1 + N 2 S 2 + +Sgen ⇒ S1 = S 2
T
0

From the energy balance Ws = NCP T2 − T1 or a f Ws


N
= CP T2 − T1 a f
From the entropy balance S 1 = S 2 ⇒ T2 = T1
FG P IJ R CP

HPK
2

1
Thus

LMF P I R CP
OP
MNGH P JK
Ws
= CPT1 2
−1
N 1 PQ
(b) Two stage compression, with intercooling, so that gas is returned to initial temperature, before
entering 2nd compressor
LMF P I ∗ R CP OP
MNGH P JK
work in stage 1 = WsI = NCPT1
1
− 1 ; where P∗ = pressure after 1st compressor.
PQ
LP
work in stage 2 = W = NC T MF I
R CP
− 1P
O
NH P K
II 2
s P 1 ∗
Q
Total work = W + W = NC T MG J
LF P I * R CP

+F I
P R CP
OP
MNH P K HP K − 2 = Ws
I II 2
s s P 1
1

PQ
To find P for minimum work, set d Ws dP∗ = 0 .
*

d
b g R P∗ |RS FG IJ a R CP −1 f 1 R P2 F Ia R CP −1 f F P I |UV
dP∗
Ws = 0 = NC T
P 1
CP P1 |T H K −
P1 CP P∗ H K H P K |W
2
∗2

c h = aP P f
2 R CP
⇒ P∗ 1 2
R CP

or
P∗ = P1 P2
Students should check that this results in minimum, and not maximum work.
4
4.22 System: nitrogen contained in both tanks (closed, adiabatic, constant volume)
Mass balance: M1i = M1f + M2f (1)
Energy balance: M1U1 = M1 U1 + M2 U 2
f f f f
(2)
Final pressure condition: P1 = P2
f f
(3)
For the entropy balance, the nitrogen in the first tank that remains in the tank
will be taken as the system. Then
S1i = S1f (4)
Equation (1) –(4), together with eqn. of state information of the form S = S(T , P) ,
U = U (T , P) and V = V (T , P) which we can get from Fig. 3.3-3 provides 4 eqns.
for the 4 unknowns T1 f , P1 f , T2f and P2f . Procedure to be followed in solution
(i) Guess a final pressure P f
(ii) Use eqn. (4) + Fig. 3.3-3 to compute T1 f , caluculate U1f
(iii) Use Fig. 2.4-3 to get V1 f , compute M1f = V  f
V1
(iv) M 2f = M1 −
, andM1f V2f = V M 2f
(v) Use P and V2f to get T2f and U 2f
f

(vi) See if energy balance, Eqn. (2), is satisfied. If it is, guessed P f is correct.
If it is not, guess new p f , go back to (ii), and repeat calculation.
Some preliminaries
T1i = 250 K Figure H 1i = 368 kJ kg
⎯ ⎯⎯ ⎯→
P1i = 200 bar 2.4 - 3 V1i ≈ 0.0037 m3 kg
Thus M1i = V / V1i = 0.01 m3 / 0.0037 m3 kg = 2.703 kg

As a first guess, use ideal gas solution for pressure. (Also try some neighboring
pressures.) My solution is

. bar (same as ideal gas solution)


P1 f = P2f = 1333
c
T1 f = 226 K ideal gas solution: T1 f = 222.8 K h
T2f = 285 K cideal gas solution: T 2
f
= 330.4 Kh
4
4.23 (a) Set up just as in Problem 3.22 above. Solution after a number of iterations is P1 f = P2f ≈ 5 bar .
T1 f = 2756
. ° C and T2f = 497.7° C .
(b) Since now there is heat exchange between the two chambers we have T1 f = T2f . This equation
is used instead of entropy balance. Solution procedure is to guess a final pressure, and then
compute final temperature using first the mass balance

V1 V V V +V V +V
M1i = M1f + M 2f ⇒ = 1f + 2f = 1 f 2 ⇒ Vf = 1 2 V1i (1)
 i 
V1 V1 V2  V V1
That is, choose T f until eqn (1) is satisfied. Then compute T f from energy balance i.e.

c h
M1iU1i = M1f U1f + M 2f U 2f = M1f + M 2f U 2f ⇒ U1i = U f (2)
When guessed P f is correct, T f computed from eqns. (1) and (2) using the Steam Tables will
be identical. My solution is P f = 5 bar and T f ≅ 366° C .
4
4.24 System = contents of turbine (open, constrant volume, steady-state)
dN
Mass balance: = 0 = N 1 + N 2 ⇒ N 2 − N 1
dt
constant
volume
dN 0 dV 0
Energy balance: = 0 = N 1 H 1 + N 2 H 2 + Q +Ws − P
dt adiabatic
dt
a f a f
⇒ Ws = − N 1 H 1 − H 2 = − N 1CP T1 − T2 for the ideal gas
dN Q 0 
Entropy balance: = 0 = N 1 S 1 + N 2 S 2 + + Sgen
dt T

a f T RS FG IJ
P
⇒ Sgen = − N 1 S 1 − S 2 = N 1 CP ln 2 − R ln 2
FG IJ UV
T1 T H K P1 H KW
or
T2 = T1
FG P IJ Sgen
R CP
LM OP
HPK
2
exp 
1 N1CP N Q
(a) For T2 to be a minimum, since Sgen ≥ 0 and N 1 > 0 , Sgen must be zero. Thus the minimum
outlet temperature occurs in reversible operation.
a f
(b) Ws = + N 1CP T2 − T1 . Since T1 > T2 , the maximum work occurs when T2 is a minimum. Thus,
W is a maximum (in magnitude) for a reversible process.
s
4
4.25 (a) For any system:

dS LM 
 S + Q + S
= M
OP
dt Ni i
T
gen
Q
depending 0n the process Q ≥ 0 or Q ≤ 0 and ∑ M i Si ≥ 0 or ∑ M i Si ≤ 0 . Also, Sgen = 0 or
Sgen > 0 , depending on whether or not the process is reversible. Thus, dS dt for a system can
be greater than, less than, or equal to zero.
Since, by definition, the universe contains everything, it must be a closed system and
adiabatic, since there is nothing for the universe to exchange mass or heat with. Therefore

dS dS 
= 0 + 0 + Sgen ⇒ = Sgen ≥ 0
dt dt

Thus the entropy of the universe can not decrease, and the statement is true.
(b) Consider the change from any state 1 to any state 2 in a closed system. The energy and entropy
balances for this transformation are:

RS
(1) U 2 − U 2 = Q + W = W Since the process is adiabatic
(2) S2 − S1 = Sgen T
If the transformation is possible, then Sgen ≥ 0 now consider the transformation from state 2 to
state 1. Here
(3) U1 − U1 = W ∗

(4) S1 − S2 = Sgen
Comparing eqns. (1) and (3) we have W = −W ∗ (This is ok).

Comparing eqns. (2) and (4) we have Sgen = − Sgen (5)

Separately we have, if the processes are possible, that Sgen ≥ 0 and Sgen ≥ 0 . The only way that
∗ ∗
all these three equations for Sgen and Sgen can be satisfied is if Sgen = Sgen = 0 , that is, both
processes are reversible. Generally, processes are not reversible. However, eqn. (5) requires

that only one of Sgen and Sgen can be greater than zero. Thus,

If Sgen > 0 1 → 2 is possible, but 2 → 1 is not possible.



If Sgen > 0 2 → 1 is possible, but 1 → 2 is not possible.
4
4.26
 = − 1 ⎛⎜ Tc − Tamb ⎞⎟ Q
W 
2 ⎝ TC ⎠
 = q − k ( T − T )
Q c amb

Heat flow into Convective heat loss from collector


collector

 = −1 ⎛⎜ 1 − Tamb ⎞⎟ ( q − k ( T − T ) )
W c amb
2 ⎝ Tc ⎠

1 1 1 Tamb 1 1 T2
= − q + k ( Tc − Tamb ) + q − kTamb + k amb
2 2 2 Tc 2 2 Tc

dW 1 1 Tamb 2
1 Tamb
= k= 
q − k = 0 (for a maximum)
dTc 2 2 Tc2 2 Tc2

kTc2 = qTamb + kTamb


2

 amb
qT q
Tc = + Tamb
2
= Tamb +1
k kTamb
4

4.27 System: contents of the tank at any time (open, adiabatic, constant volume
system).
dN
(a) Mass balance: = − N
dt
PV PV
ideal gas law N = ; N = where V = volumetric flow rate.
RT RT
F I
d PV PV F I
d P P
= − V since V and V are both constant.
Thus,
H K
dt RT
=−
RT
⇒V
H K
dt T T
d F I
P V P P FG IJ

Vt
× 10−3
bar
dt H K
ln
T
=−
V
or
T 5 min
=
T0
exp −
H K
V
= 1082
.
K
or

P(5 min) = 1082


. × 10−3 T (5 min) { P = bar, T = K} (1)

d dN dU dN
Energy balance: ( NU ) = − N H ⇒ U +N =H
dt dt dt dt
dU dN dN dT
or N = ( H − U ) ⇒ RT = NCV
dt dt dt dt

using N = PV RT yields R
dP P
= CP
dT
or T2 = T1 2
P FG IJ
P RC

dt T dt P1 H K
[Note: could have gotten this result from the entropy balance also!]

F P(5 min)I 8.314 39


⇒ T (5 min) = 340
H 1 bar K (2)

simultaneously solving equations (1) and (2) yields

P(5 min) = 0.281 bar and T (5 min) = 259.3 K

Since pump is adiabatic and reversible, S in = S out or T3 = T2


FG P IJ R CP

HPK
3
(b) since
2
P3 = P1 . This equation implies that T3 = T1 = 340 K .
4

4.28 Number of moles of gas in tank initially = N 0 = N (0)

P(0)V 15 bar × 0.2 m3 × 105 J m3 bar


N (0) = = = 122.26 mol
RT (0) 8.314 J mol K × (22 + 27315
. )K

dN
= N = −4.5 mol min ⇒ N (t ) = 122.26 − 4.5t mol (t = min)
dt
(a) Entropy balance on an element of gas that remains in the tank (see Illustration 3.5-2) yields

T (t ) FG
P(t ) IJ R CP
F P(t )I 0.3779
S (t ) − S (0) = 0 ⇒
T (0)
=
H
P(0) K .
H 15 K
⇒ T (t ) = 29515 (1)

From the ideal gas equation of state

T (t ) N (t ) V T (0) N (0)
P(t )V = N (t ) RT (t ) ⇒ = =
P(t ) R P(0)

Thus

T (t ) 29515
. K 122.26 19.68
= × = (2)
P(t ) 15 bar 122.26 − 4.5t 1 − 0.03681t

Now using eqn. (1) in eqn. (2) to solve for P(t ) and T (t ) yields

P(t ) = 15 × (1 − 0.03681t )1.6075


T (t ) = 29515
. × (1 − 0.03681t )0.6075

But T (t ) is temperature in the tank. What about temperature of gas leaving the throttling valve?
Gas going thru valve undergoes a Joule-Thomson expansion ⇒ H in = H out . Since gas is ideal,
this implies Tin = Tout . Thus, T (t ) out of valve = 29515
. × (1 − 0.03681t )0.6075
(b) If tank is isothermal, then, instead of eqn. (2), we have

P(t ) RT P(0)
= = ⇒ P(t ) = 15(1 − 0.03681t )bar
N (t ) V N (0)

and T (t ) = constant = 29515


. K
Summary
T(K) P(bar )
Adiabatic . (1 − 0.03681t )
29515 0.6075
15(1 − 0.03681t )1.6075
Isothermal 29515
. 15(1 − 0.03681t )
4

4.29 This is a tough problem!


Subscript 1 denotes properties in initally filled tank
Subscript 2 denotes properties in initially "evacuated" tank
We will use i and f (superscripts) to denote initial and final properties, and we will
assume negligible mass hold-up in engine.
1) Mass balance on closed system consisting of both tanks

P1iV1 P2iV2 P1 f V1 P2f V2


N1i + N 2i = N1f + N 2f ⇒ + i =
T1i T2 T1 f T2f

but

P1 f = P2 f = P f ⇒ P f
FG 0.3 + 0.75IJ = 14.0 × 0.3 + 0.35 × 0.75
H T T K 97315
1
f
. 2
f
29815
.
(1)
bar m3
−3
= 5196
. × 10
K

2) Entropy balance on gas contained in tank 1 initially and finally. This is a closed, adiabatic,
reversible system

dS Tf
= 0 ⇒ S1i = S1f ⇒ CP ln 1 i
FG IJ = R lnFG P IJ f

H K HPK
1
i
(2)
dt T1 1

Thus

T1 f = T1i
FG P IJf R CP
= 97315
FG P IJf 27

HPK H 14.0K
1 1
i
. (3)
1

Equation (2) implies that T1 and P1 are related as follows

d ln T1 R d ln P1
= (4)
dt1 CP dt

3) Mass and energy balances on tank 1

dN1
dt
= N 1 ≡ − N ;
d
dt
a dN
f
= N1U 1 = − N H 1 = U 1 1 + N1
dt
dU 1
dt
or N1CV
dT1 dN1
dt
=
dt
a dN
dt
f
H 1 − U 1 = RT1 1 = − RT1 N

using eqn. (4) gives

CV d ln P1 dN1
N1 = − N = (5)
CP dt dt

4) balances on the engine: adiabatic, reversible (for maximum work) and since no hold-up of
mass, dN1 dt = 0
Subscript eng refers to gas leaving engine and going into tank 2.
d i
0 = H 1 − H eng N + Ws
0 = d S − S i N
1 eng

⇒ S 1 = S eng ;
Also P2 = Peng

⇒ Teng = T1 Peng P1 c h R CP
a
= T1 P2 P1 f R CP
(6)
(Note that Teng ≠ T2 )
and
−Ws = NC
 c
P T1 − Teng h (7)
5) balances on tank 2 [Note, irreversible mixing occurs unless, fortuitously, Teng = T2 at all times
(can this occur?). Thus, Sgen > 0 , and entropy balance gives no useful information]
Mass balance: dN 2 dt = N

Energy balance:
a
d N 2U 2
=U2
f
dN 2
+ N2
dU 2
= N H eng =
dN 2
H eng But,
dt dt dt dt
a f
H = CP T − T0 UV where T = reference temperature
U = C aT − T f − RT = C T − C T W
0
V 0 0 V P 0

dN 2
dt
d
H eng − U 2 =
dN 2
dt
i m dT
CPTeng − CPT0 − CVT2 + CP T0 = N 2CV 2
dt
r
or

m dT
N CP Teng − CVT2 = N 2CV 2
dt
r (8)

and

N =
d
N2 = a f
d PV PV dT FG
V dP2
= − 2 22 2 + 2
IJ
H K
2 2
dt dt RT2 RT2 dt RT2 dt

dT2 RT 2 N T dP
⇒ =− 2 + 2 2 (9)
dt PV
2 2 P2 dt2

using eqns. (6) and (9) in eqn. (8)


R| F I R CP
U| RT22 
S| GH JK
P
N CP T1 2 V|
− CVT2 = − N 2CV ⋅
T dP
N + N 2CV 2 2
T P1 W PV
2 2 P2 dt2
PV T dP
= −CVT2 N + 2 2 ⋅ CV 2 2
RT2 P2 dt2

 T P2 FG IJ R CP
=
CVV2 dP2
NCP 1
P1 H K R dt

and using eqn. (5)

C P
− N V CP T1 2
FG IJ R CP
d ln P1 V dP
= CV 2 2
CP P1 H K dt R dt

or, finally

−V1
FG P IJ R CP
dP1 dP
= V2 2
HPK
2

1 dt dt
dP2 dP
V2 P2 − R CP = −V1 P1− R CP 1
dt dt

or

z z
P2f 1 Pf
dP2 dP1
−V2 R CP
= V1 R CP
P2i
P2 P
Pi 1 1

{c h
2.5 P f
CV CP
− (0.35)CV CP
} = −{c P h f CV CP
− (14.0)CV CP
}
⇒ P f = 3053
. bar: using Eqn. (3), T1 f = 629.8 K .
Now using eqn. (1), T2f = 6119 . K
Finally, to get the total work, we do an overall energy balance (system = two tanks; adiabatic,
closed, constant volume).
N f U 1f + N 2f U 2f − N1i U 1i − N 2i U i2 = Ws

Ws =
CV
R
m a f
P f V1 + V2 − P1iV1 − P2iV2 r
5
. (0.3 + 0.75) − 14 × 0.3 − 0.35 × 0.75}
= {3053
2
= −3142
. bar m3 = −3142
. × 105 J = −314.2 kJ
4

4.30 Note: be careful about coordinate system. A mass flow in the negative × direct is
negative!
L
Heat exchanger is
+x
in steady-state operation

∆L

Case I: Concurrent flow


Mass balance on shaded volume

mass in mass in mass in mass out


element at − element = at face − at face at
time t + ∆t at time t at L in ∆t L + ∆L in ∆t

0
(steady - state)
= M b
 −M
L

L + ∆L ∆t g
 +M  
M L L +∆L = M
Energy balance on the shaded volume

energy in energy in energy flow in energy flow out


element − element = by mass flow + by mass flow at
at t + ∆t at t at L in ∆t L + ∆L in ∆t
energy flow in
+ by heat flow
in time ∆t
(steady-state)

0= M H ∆t − M  
L L L + ∆L H L + ∆L ∆t + Q∆L∆t
 H
M a f a
L + ∆L − H L = MCP TL + ∆L − TL = Q∆L
 f
dividing by ∆L , taking limit as ∆L → 0 , and using subscript 1 to denote fluid 1

 C dT1 = Q = κ T − T
M1 P ,1
dC
2 a
1 f
Q =heat flow rate per unit length of exchanger.
Similarly, for fluid 2 (other part of exchanger)

 C dT2 = −Q = −κ T − T
M 2 P ,2
dL
2 1 a f
( M 1 and M 2 are both + for concurrent flow)
Adding the 2 equations

 C dT1 = M
M  C dT2 = 0
1 P ,1 2 P ,2
dL dL
From problem statement, M 1 = M 2 and CP,1 = CP,2

dT1 dT2
+
dL dL dL
=
d
a f
T1 + T2 = 0 or T1 + T2 = constant = C
and T1 = C − T2 ; T2 = C − T1
now going back to

dT1
=
dL MCP
κ
T2 − T1 =aκ C − 2T1

MC
f a f
P

and integrating

FG C − 2T IJ = − 2κL
f
=−
L∗
; L0 =

MC
H C − 2T K MC
1 P
ln 
1
i
P L0 2κ

T1 f = 15° C , T1i = 35° C , T2f = 5° C , T2i = −15° C .


Also, C = T1i + T2i = T1 f + T2f = 20° C
[ i = initial conditions, conditions at L = 0 ; f = final conditions, conditions at L *
where L* = length of exchanger]
Using this in equation above gives

L* F
20 − 70 I
L0
= ln
H
20 − 30 K
= ln 5 = 1609
.

And, more generally, at any point in the exchanger

C − 2T1( L)
= exp −
FG IJ
L
⇒ T1( L) =
C
1 − exp −
L FG FG IJ IJ + T expFG − L IJ
H K H H KK H LK
i
1
C − 2T1 i
L0 2 L0 0

F LI
T ( L) = 10 + 25 expG − J ° C
1
H LK0

T2 ( L) = C − T1( L) = 10 − 25 exp −
FG L IJ ° C
H LK0

Now writing an entropy balance


 S ∆t − M  Q
0= M L L L + ∆L S L + ∆L ∆t = ∆L∆t
T

⇒M
 
 dS = Q = κ T2 − T1 = a f
−κ 50 exp − L L0 a f
dL T1 T1 10 + 25 exp − L L0 + 27315
. a f need absolute
T here

dS =
−50κL0 exp − L L0 L a FG IJ f
 . + 25 exp − L L0
M 28315 a fH K
d
L0
25 expa− L L f F LI
= −C dG J
. + 25 expa− L L f H L K
0
p
28315 0 0
. + 25 expa− L L f U
⇒ S( L) = S( L = 0) + C lnRS VW
28315 0
T p
30815
.

Case II Countercurrent flow


 = −M
M 
2 1

 C
M1 P
dT
dL
1
a
= κ T2 − T1 f (1)
35 15° C
 C dT2 = −κ T − T
M 2 P
dL
a
2 1 f (2) 5° C -15

M1 P
dL
a
 C dT2 = κ T − T
2 1 f (3)
Eqn. (3) comes from eqn. (2) using M 2 = − M 1 . Subtracting eqn. (1) from eqn. (3)
gives

d
dL
a f
T1 − T2 = 0 ⇒ T1 − T2 = constant = C = 30° C

T2 = T1 − 30° C

Thus

M 1 P
dL
2 a
 C dT1 = κ T − T = −30κ
1 f
30κL
T1 = − + T1i

MC P

L 30κL L
T2 = 5 − 15 = 35 − = 35 − 15
L0 
MCP L0


MC
where L0 = P
and the entropy balance

 Q −κ 30
 dS1 =
M =
dL T1 T1
dS1 −κ 30
=
dL   + 27315
M (35 − 30κL MC . )
P
absolute
temperature
needed here

dS1 = 
−30κL0 L
=
−15CP FG IJ L FG IJ
b 
. − 30κL MCP
M 30815
d
L0 gH K
. − 15 L L0
30815
d
L0 H K
CP
=− dx
. −x
2054
FG
⇒ S( L) = S(0) + CP ln 1 − 0.048
L IJ
H L0 K
Summary
Concurrent flow
FG L IJ
T1 = 10 + 25 exp −
H LK 0

F LI
T = 10 − 25 expG − J
2
H LK 0

S( L) = S( L = 0) + CP ln RS
. + 25 exp − L L0
28315 a f UV
T 30815
. W
Countercurrent flow
L
T1 = 35 − 15
L0
L
T2 = 5 − 15
L0
FG
S( L) = S(0) + CP ln 1 − 0.048
L IJ
H L0 K
4

4.31 (a)
dU
dt
= WS + Q − P
dV
dt
= WS + Q − P0
dV
dt
− P − P0
dV
dt
a f
and

dS Q
= + Sgen
dt T0

Now let

W = WS − P0
dV
dt
a
− P − P0
dV
dt
f
and

Wu = W + P0
dV
dt
a
= WS − P − P0
dV
dt
⇒fdU
dt
= Wu + Q − P0
dV
dt

or

a
U 2 − U1 = Wu + Q − P0 V2 − V1 f
Q
S2 − S1 = + Sgen ⇒ Q = T0S2 − T0S1 − T0Sgen
T0

and

a
U 2 − U1 = Wu + T0S2 − T0 S1 − T0Sgen − P0 V2 − V1 f
a
Wu = U 2 + PV f a
0 2 − T0 S2 − U1 + PV
0 1 − T0 S1 + T0 Sgen f
since T0Sgen ≥ 0
Wumax = A 2 − A1, where A = U + P0V − T0S

(b)
0 = M H 1 − M H 2 + Q + WS
Q
0 = M S1 − M S 2 + + Sgen
T0

Here WS = Wu
⇒ Wu = M H 2 − M H 1 − Q = M H 2 − M H 1 + MT0 S 1 − MT0 S 2 + T0Sgen
a f a f
⇒ Wu = M H 2 − T0 S 2 − M H 1 − T0 S 1 + T0Sgen
Since T0Sgen ≥ 0
b g
Wumax = M B2 − B1 where B = H − T0 S

(c) Using the Steam Tables we find


i) at 30 bar = 3 MPa and 600°C
U = 32850. kJ kg , S = 7.5085 kJ kg K , V = 013243
. m3 kg

A1 = U + PV
0 − T0 S

= 32850
. + 1013
. bar + 013243
. m3 kg × 102 kJ bar ⋅ m3 − 29815
. × 7.5085
= 1059.76 kJ kg

ii) at 5 bar = 05
. MPa and 300°C
U = 2802.9 kJ kg , S = 7.4599 kJ kg K , V = 05226
. m3 kg

A 2 = 2802.9 + 1013
. × 05226
. × 102 − 29815
. × 7.4599 = 63167
. kJ kg

Wu = A 2 − A1 = (63167
. − 1059.76) kJ kg = −428.09 kJ kg

This is the maximum useful work that can be obtained in the transformation with the
environment at 25°C and 1.013 bar. It is now a problem of clever engineering design to
develop a device which will extract this work from the steam in a nonflow process.
(d) Since the inlet and exit streams are at 25°C and P = 1013
. bar, any component which
passes through the power plant unchanged (i.e., the organic matter, nitrogen and excess
oxygen in the air, etc.) does not contribute to the change in availability, or produce any
useful work. Therefore, for each kilogram of coal the net change is:

0.7 kg of carbon = 58.33 mol of C


+58.33 mol of O2
to produce 58.33 mol CO2

also

. kg of water = 8.33 mol of H 2O undergoes a phase change


015
from liquid to vapor

Therefore
i
b
MBin = ∑ Ni Bi g
in
= 58.33 × 0+ 58.33 × 0+ 8.33 × (−68.317 + 29815
(carbon ) (oxygen)
. × 0.039)
(liquid water )

= −1976 kJ kg coal

i
b
MBout = ∑ Ni Bi g
out
= 58.33 × (−94.052)+ 8.33 × (−57.8 + 29815
(carbon dioxide)
. × 0.0106)
( water vapor )

= −24858 kJ kg coal
Wumax = −24858 − (1976) kJ kg coal = 22882 kJ kg coal
Wuactual = −2.2 kW - hr kg coal = 7920 kJ kg coal
7920 × 100
Efficiency in % = = 34.6%
22882

Thus a coal-fired electrical power generation plants converts slightly more


than 1/3 of the useful work obtainable from the coal it consumes. This
suggests that it would be useful to look for another method of generating
electrical power from coal . . . for example, using an electro-chemical fuel
cell. Considering the amount of coal consumed each year in power
generation, and the consequences (strip mining, acid rain, greenhouse
effect, etc.) the potential economic savings and environmental impact of
using only 1/3 as much coal is enormous.
4
4.32 Three subsystems: unknowns T1 f , P1 f , T2f , P2f , T3f , P3f (6 unknowns)
After process P1 f = P2f = P3f (2 equations)
Subsystem 1 has undergone a reversible adiabatic expansion

⇒ S 1f = S 1i , or T1 f = T1i
FG P IJ f R CP

HPK
1
i
(1 equation) (#1)
1
Subsystem 3 has undergone a reversible adiabatic compression

⇒ S 3f = S i3 , or T3 = f
T3i
FG P IJ
f R CP
= T3i
FG P IJ
f R CP

HPK HPK
3
i i
(1 equation) (#2)
3 3
Mass balance subsystems 1 + 2
P1 f V1 P2f V2 P1iV1 P2iV2i
N1f + N 2f = N1i + N 2i ⇒ + = i + i
T1 f T2f T1 T2
or

Pf
FG 05. + V IJ = 10 × 05. + 1 × 0.25 = 0.017909 (1 equation)
f

H T T K 29315
2
f f
(#3)
1 . 2 29315.

Energy balance on subsystems 1 + 2 + 3

N1f U 1f + N 2f U 2f + N 3f U 3f = N1i U 1i + N 2i U i2 + N 3i U i3
P1 f V1 P2f V2 P3f V3 P1iV1 P2iV2 P3iV3
f
CV T1
f
+ f
CV T2
f
+ f
CV T3
f
= i
CV T1
i
+ i
CV T2
i
+ i
CVT3i
RT1 RT2 RT3 RT1 RT2 RT3
c h
P f V1 + V2f + V3f = P1iV1 + P2iV2 + P3iV3

but V1 + V2f + V3f = V1 + V2i + V3i = (05


. + 0.25 + 0.25) = 1 m3

10 × 05
. + 1 × 0.25 + 1 × 0.25
Pf = = 55
. bar
1

using this result in eqn. (1) → T1 f = 252.45 K


in eqn. (2) → T3f = 448.93 K

P3i T3f 1 448.93


V3f = V3i f
⋅ i = 0.25 × × = 0.06961 m3
P3 T3 55
. 29315
.
V2f = 0.25 × 2 − 0.06961 = 0.4304 m3

Now using Eqn. (#3)


FG 05. + V IJ = 55. FG 05. + 0.4304 IJ = 0.017909 ⇒ T
f
f
= 337.41 K
H T T K H 252.45 T K
2
Pf f f f 2
1 2 2

Thus the state of the system is as follows

Initial Final
T1 293.15 K 252.45 K
P1 10 bar 5.5 bar
T2 293.15 K 337.41 K
P2 1 bar 5.5 bar
V2 0.25 m3 0.4304 m3
T3 293.15 K 448.93 K
P3 1 5.5 bar
3
V3 0.25 m 0.0696 m3

Work done on subsystem 3

Energy balance

N 3f U 3f − N 3i U i3 = W = − PdVz
P3f V3f P3iV3i CV C
CVT3f − CVT3i = P3f V3f − P3iV3i V = W
RT3f RT3i R R
CP − R f f
W=
R
c
P3 V3 − P3iV3i = 3(55 h
. × 0.0696 − 1 × 0.25)

= 0.3984 bar ⋅ m3 = 39.84 kJ


4
4.33 From simple statics the change in atmospheric pressure dP accompanying a change in height dh is
dP = − ρgdh
where ρ is the local mass density and g is the gravitational constant. Assuming a packet of air
undergoes an altitude change relatively rapidly (compared to heat transfer), the entropy change for
this process is
C R
d S = P dT − dP = 0 since both Q and Sgen equal zero.
T P
Combining the two equations above we have
CP R R R N M
dT = dP = − ρgdh = − Mgdh = − gdh
T P P PV T
dT Mg
or =−
dh CP
dT K dT
For dry air ≅ −9.7 . Note that is referred to as the adiabatic lapse rate.
dh km dh
Also, its value will be less than that above as the humidity increases.
In fact, if the humidity is 100%, so water will condense as the pressure decreases, the adiabatic lapse rate
will be almost zero.
4
4.34

W = -10 kW

1
i 100°C
Steady State
300°C 2
1 MPa
300°C
1 kg/s
Sat’d L
Mass balance
0=M  +M
 +M
  − 0.5 = 0 ⇒ M
1+ M  = −0.5 kg/s
i 1 2 1 1

Energy balance
 H
0=M   = −10 kw = − 10 kJ
i i − 0.5 H1 − 0.5 H 2 + W
ˆ ˆ ˆ W
s
 =O
assume no heat loss Q ( )
Ĥ i = 1344.0 kJ/kg Sˆ i = 3.2534 kJ/kg ⋅ K
Ĥ 2 = 1344.0 kJ/kg Sˆ 2 = 3.2534 kJ/kg ⋅ K
kg kJ ˆ − 0.5 × 1344.0 − 10 kJ
0 =1 × 1344.0 − 0.5 × H1
s kg s
ˆ = 1344.0 − 0.5 × 1344.0 − 10 = 662
0.5 H1

Ĥ1 =1324.0 kJ/kg at 100°C

At 100°C HL = 419.04 HV = 2676.1


1324 = x (2676.1) + (1-x) 419.04 where x = quality
1324 – 419.04 = x (2676.1 – 419.04)
x = 0.401 quality

Ŝ1 = 0.401 × 7.3549 + 0.599 × 1.3069 = 3.7322 kJ/kg.K

Entropy balance

 Sˆ + M
0=M  Sˆ + S
 Sˆ + M where S gen ≥ 0
i i 1 1 2 2 gen

= 1× 3.2534 − 0.5 × 3.7322 − 0.5 × 3.2534 + S gen = 0


−0.2394 + S gen = O
kJ
⇒ S gen = 0.2394
kg ⋅ K ⋅ s

so device is possible
4
4.35 a) Basis: 1 mole air

Energy balance
0
U f − U i = W + Q = C v ( Tf − Ti )
Entropy balance
Sf − Si = 0 ⇒ Sf − Si
Tf V
⇒ S ( T f , V f ) − S ( T i , V f ) = C*V ln + R ln f = 0
Tf Vi
Tf 8.314 ⎛ 1 ⎞
ln =− ln ⎜ ⎟ = 1.2743
Ti 21 ⎝ 25 ⎠
Tf = Ti e1.2743 = 303.15e1.2743 = 1084.1 K
J J kJ
W = 8.314 × (1084.1 − 303.15 ) K = 6493 = 6.493 = 6.493 kJ / mol
mol K mol mol
b)
Tf 8.314 ⎛ 1 ⎞
ln =− ln ⎜ ⎟ = 0.9116
Ti 21 ⎝ 10 ⎠
Tf = 303.15 × e0.9116 = 754.33
J kJ
W = 8.314 × ( 754.33 − 303.15 ) = 3751.1 = 3.751
mol mol
4
4.36
1 2

200 kPa, 100°C 2 MPa, 200°C

Find initial number of moles in each compartment


PV=NRT

1 m3
200 kPa × 100 × 103 cm3 ×
PV 106 cm3 = 6.447 moles
N= N1 = 3
RT kPa m
8.314 × 10−3 × 373.15 K
mol K
1 m3
2 × 103 kPa × 2 × 105 cm3 ×
N2 = 106 cm3 = 101.68 moles
3
kPa m
8.314 × 10−3 × 473.15 K
mol K

a) Mass balance: Nf – N1 – N2 = 0 Nf = N1 + N2 = 6.45 + 101.68 Nf = 108.13

Energy balance: Nf Uf – N1 U1 – N2 U2 = 0
6.448 × CV (Tf – 373.15) + 101.68 × CV (Tf – 473.15) = 0
(6.447 + 101.68) Tf = 6.447 × 373.15 + 101.68 × 473.15
6.447 × 373.15 + 101.68 × 473.15
Tf = = 467.19 K
6.447 + 101.68

Final Pressure
kPa m3
108.16 mol × 8.314 × 10−3 × 467.19 K
NRT mol K
P= =
V m3
3 × 105 cm3 × 10−6
cm3
1.0813 × 102 × 8.314 × 10 −3 × 4.6719 × 102
= −1
= 14.0 × 102 kPa = 1.4 MPa
3 × 10

b) Entropy Change
N f Sf − N1S1 − N 2 S2 = Sgen ≥ 0
⎧ T P ⎫ ⎧ T P ⎫
N1 ⎨CP ln f − R ln f ⎬ + N 2 ⎨CP ln f − R ln f ⎬ = Sgen
⎩ T1 P1 ⎭ ⎩ T2 P2 ⎭
⎧ 467.9 1.4 ⎫ J ⎧ 467.9 1.4 ⎫
6.45 mol × ⎨30 ln − 8.314 ln ⎬ + 101.68 ⎨30 ln − 8.314 ln ⎬ + Sgen = 0
⎩ 373.15 0.2 ⎭ mol K ⎩ 473.15 2 ⎭
6.45 × {30 × 0.2263 − 8.314 × 1.9459} + 101.68{30 × ( −0.01116 ) − 8.314 ( −0.35667 )} =
J J J
−60.56 + 267.47 = Sgen = 206.9 ≥ 0! (as it should be)
K K K
4
4.37
Assume steady state operation
1 2
2 bar = 0.2 MPa
20 bar
400°C


W

M +M  =0  = −M
M 
1 2 2 1
2 bar = 0.2 MPa
Entropy balance
0 0 for maximum work
 Sˆ + M Q
 Sˆ + + S
0=M1 1 2 2 gen
T
⇒ Sˆ 2 = Sˆ 1 Sˆ 1 = Sˆ ( 2 MPa, 400°C ) = 7.1271 kJ
kg ⋅ K
Ĥ1 = 3247.6 kJ / kg

Ŝ2 ( 0.2MPa, T = ? ) = 7.1271 kJ / kg ⋅ K


at T = 200°C Sˆ = 7.5066 kJ / kg K H ˆ = 2870.5 kJ / kg

150°C Sˆ = 7.1272 kJ / kg Ĥ = 2768.8 kJ / kg



T2 = 150°C
Energy Balance
0
 H
0=M  ˆ 
1 1 − M1 H 2 + W + Q
ˆ

kg kJ kJ kJ
− W = 5000 ( 9247.6 − 2768.8 ) = 2.394 ×106 = 665 = 665 kW
hr kg hr S
J J J
−60.56 + 267.47 = Sgen = 206.9 ≥ 0! (as it should be)
K K K
4
4.38

20°C T=? Heater 400°C


5 bar
1 bar 5 bar

a) Assume adiabatic and reversible operation of compressor


⇒ Isentropic
S (20°C, 1 bar) – S (T = ?, 5 bar) = 0

T 5
= C*p ln − R ln = 0
293.15 1
R
⎛ 5 ⎞ Cp
*
J
T = 293.15 ⎜ ⎟ = 462.7 K since C*p = C*v + R = 21 + 8.314 = 29.314
⎝1⎠ mol ⋅ K
E.B. H (20°C, 1 bar) – H (462.7 K, 5 bar) + W = 0
W = H (462.7 K, 5 bar) – H (293.15 K, 1 bar)
= Cpx ( 462.7 − 293.15 ) = 4971.2 J / mol

b) Heater
H (462.7 K, 5 bar) – H (673.15 K, 5 bar) + Q = 0
= C px ( 673.15 − 462.7 ) = 6169.1 J / mol
4
4.39 From Fig. 3.3-2
At 160 K Hˆ = 260 kJ / kg ˆ ≅ 0.00295 m3 / kg
V
L L
ŜL = 2.05 kJ / kg K
If adiabatic and reversible, Sgen = 0 Sˆ initial = Sˆ final
Sˆ initial = 2.05 kJ / kg = 0.75 Sˆ L ( T = ?, P = ? ) + 0.25 Sˆ V ( T = ?, P = ? )
but since it is an equilibrium V-L mixture T & P are connected by the saturation line

Guess T = 120 K SL = 1.03 Sˆ V = 5.03 ⇒ Sˆ =


ˆ
Close Enough
Ŝ = 0.75 × 1.03 + 0.25 + 5.03 = 2.03

ˆ = 105 kJ / kg
H ˆ = 595 kJ / kg
H
L V
P = 0.2 MPa
VL = 0.0025 m / kg VV = 0.3 m3 / kg
ˆ 3 ˆ

ˆ ≅H
U ˆ = 105 kJ / kg
L L

ˆ =H
U ˆ = 595 kJ − 0.2 MPa × 106 Pa × 1J × 10−3 kJ
ˆ − PV
V L L
kg MPa m3 Pa J
kJ kJ kJ
= 595 − 60 = 535
kg kg kg
ˆ = 0.75 U
U ˆ + 0.25 U
ˆ = 212.5 kJ / kg
f L V

ˆ ∼H
U ˆ = 260 kJ / kg
i i

Energy balance on piston-cylinder (closed system)


o
Uˆ − Û = Q + W = 212.5 − 260 = −47.5 kJ / kg
f i

(Negative sign ⇒ system does work on surroundings)


4

4.40
a) 1 bar
0.1 MPa
20 bar
550°C

20 bar = 2 MPa = 2,000 kPa


From the superheated steam tables
Ĥ (20 bar, 600°C) = 3690.1 kJ/kg
Ŝ (20 bar, 600°C) = 8.6451 kJ/kg K
Estimate From Fig. 3.3-1b (vertical line down to 1 bar)
T ≅ 172°C Hˆ = 2820 kJ / kg Also all vapor
Steady state energy balance
W
0 = M IN H
ˆ +M H
IN OUT OUT + W ⇒
ˆ =Hˆ
OUT − H IN
ˆ
M IN
W kJ kJ kg kJ
= 3690.16 − 2820 = −870.1 W = −870.1 × 5000 − 1327.2 = −1327.2 kW
M IN kg kg hr s
b) If only 90% of the work generated, W = −0.9 × 870.1 = −783.1 kW
kJ
so Ĥ out = 3690.1 − 783.1 = 2907.0 at 1 bar (0.1 MPa) All vapor
kg
By interpolation using the Superheated Steam Tables
kJ
T ∼ 216o C and S=7.8983
kg ⋅ K

Entropy balance 0
dS Q
= 0 = Min Sˆ in + M out Sˆ out + + Sgen
dt T
Sgen kJ kJ
= 7.8983 − 7.7024 = 0.1959 or Sgen = 979.5
M in kg ⋅ K hr ⋅ K
4

4.41 The gas in the tank undergoes a uniform adiabatic expansion, which is isentropic, so that its
temperature and pressure are related by
R / C*p
⎛P ⎞
TT,f = TT,i ⎜ T,f ⎟
⎜P ⎟
⎝ T,i ⎠
The gas in the cylinder to the right of the piston undergoes a uniform adiabatic compression, which is also
isentropic, so its temperature and pressure are related by
R / C*p
⎛P ⎞
TCR ,f = TCR ,i ⎜ CR ,f ⎟⎟
⎜P
⎝ CD,i ⎠

Also PT,f = Pc,f.

An energy balance on the whole system gives

(UT,f + UCL,f + UCR,f) – (UT,I + UCR,i) = 0


NT,f UT,f + NCL,f UCL,f + NCR,f UCR,f = NT,i UT,i + NCR,i UCR,i
Also, the mass balance
NT,f + NCL,f + NCR,f = NT,i + NCR,i

Now using the ideal gas law


V V V V V
M.B. Pf TT + Pf CL,f + Pf CR ,f = PT,i T + PCR ,i CR ,i
R Tf RTCL,f RTCR ,f RTT,i RTCR ,i
Pf VT PV PV P V P V
E.B. CV TT,f + t CL C v TCL,f + f CR ,f CV TCR ,f = T,i T CV TT,i + CR ,i CR ,i CV TCR ,i
RTT,f kTCL,f RTCR ,i RTT,i RTCR ,i
These equations reduce to

⎡V V V ⎤ P V P V
Pf ⎢ T + CL,f + CR ,f ⎥ = T,i T + CR ,i CR ,i
⎣⎢ TTf TCL,f TCR ,f ⎥⎦ TT,i TCR ,i

and Pf ⎡⎣ VT + VCL,f + VCR ,f ⎤⎦ = PT,i VT + PCR ,i VCR ,i

but VCL,f + VCR ,f = VCR ,i = 0.5 m3

Pf ⎡⎣ VT + 0.5 m3 ⎤⎦ = 0.4 VT + 0.1× 0.5 m3

0.4VT + 0.1× 0.5 0.4 × 0.25 + 0.1× 0.5


Pf = = = 0.2 MPa
VT + 0.5 0.25 + 0.5
8.314
⎛ 0.2 ⎞ 29.3
TT,f = 473.15 ⎜ ⎟ = 388.7 K
⎝ 0.4 ⎠
8.314
⎛ 0.2 ⎞ 29.3
TCR ,f = 288.15 ⎜ ⎟ = 350.8 K
⎝ 0.1 ⎠

Mass balance on region to right of piston


P V P V
N CR ,i = N CR ,f ⇒ CR ,i CR ,i = CR ,f CR ,f
RTCR ,i RTCR ,f
0.1× 0.5 0.2 × VCR ,f 0.1× 0.5 350.8
⇒ = VCR ,f = × = 0.3043 m3
288.15 350.8 288.15 0.2

⇒ VCL,f = 0.5 – 0.3043 = 0.1557 m3


Moles in tank initially
P V 0.4 × 0.25
N T,i = T,i T,i = = 25.42 moles
RTi 8.314 × 10−6 × 473.15
Moles in tank finally
0.2 × 0.25
N T,f = = 15.47 moles
8.314 × 10−6 × 388.7
⇒ N CL,f = 25.42 − 15.47 = 9.95 moles
PCL,f VCL,f 0.2 × 0.1557
TCL,f = = = 376.43
R ⋅ N CL,f 8.314 × 10 −6 × 9.95
0.1× 0.5
N CR = = 20.87 moles
8.314 × 10 −6 × 288.15
Entropy change = ∆S of N2 finally in tank + ∆S of N2 finally left of piston + ∆S of N2 finally right of piston

⎛ 388.7 0.2 ⎞ ⎛ 376.43 0.2 ⎞ ⎛ * 350.8 0.2 ⎞ J


= 15.47 ⎜ C*P ln − R ln ⎟ + 9.95 ⎜ C*P ln − R ln + N CR ⎜ CP ln 288.15 − R ln 0.1 ⎟ = 9.26 K
⎝ 473.15 0.4 ⎠ ⎝ 473.15 0.4 ⎟⎠ ⎝ ⎠

[Note that the first and third terms should be zero, since the gas in the tank under went a uniform, adiabatic
expansion, and the gas to the right of the piston under went a uniform, adiabatic compression. So only the
middle term is important.]
4

4.42 Initial To the left of the piston


TCL,f = 376.43 K VCL,f = 0.1557 P = 0.2 MPa
NCL,f = 9.95 Mols

TCR,f = 350.8 K VCR,f = 0.3043 m3


NCR,f = 20.87 K
Mass balance after conduction of heat
PV PV
N CL,f = 9.95 = f CL N CR = 20.87 = f CR
RTf RTf
but Pf is the same on both sides of piston, and Tf is the same on both sides of piston

9.95 V VCL 20.87
= CL = 0.5 − VCL = VCL
20.87 VCR 0.5 − VCL 9.95
0.5
0.5 – VCL = 2.097 VCL VCL = = 0.1614 m3
3.097
VCR = 0.5 –0.1614 = 0.3386 m3

Energy balance
9.95 UL,2 + 20.87 UR,2 = 9.95 UL,1 + 20.87 UR,1
9.95 (T–376.43) + 20.87 (T–350.8) = 0
30.82 T = 9.95 ×376.43 + 20.87 + 350.8
T = 359.1 K

bar m3
9.95 moles × 8.314 ×10 − 5 × 359.1 K
9.95 × R × T mol K
Pf = =
VCL 0.1614 m3
= 1.840 bar = 184.0 kPa
4
4.43

110 bar
80oC

V=0.1 m3, initial conditions 25oC and 1 bar

PVi 1 bar ⋅ 0.1m3


a) Mass balance: N f = Ni + ∆N with Ni = = = 4.034 mol
-5 bar ⋅ m
3
RT
8.314 ⋅10 ⋅ 298.15 K
mol ⋅ K

Energy balance:

N f U f = N i U i + ∆N H in

For simplicity in the calculation of thermal properties, will use 25oC as Tref so that Uref = 0 and
Href=-298.15R, so that
100 bar ⋅ 0.1 m3 1 bar ⋅ 0.1 m3
R ⋅Tf
(
⋅ CV ⋅ T − Tref = ) R ⋅ Ti
(
⋅ CV ⋅ Tin − Tref )
⎛ 100 bar ⋅ 0.1 m3 1 bar ⋅ 0.1 m3 ⎞
+⎜
⎜ R ⋅Tf

R ⋅ Ti
(
⎟ ⋅ CP ⋅ Tin − CV ⋅ Tref
⎟ )
⎝ ⎠
which reduces to
100
Tf
( 1
)
CV ⋅ T f − CP ⋅ Tin = ⋅ ( CV ⋅ Ti − CP ⋅ Tin )
Ti
So that
100 ⋅ CP ⋅ Tin
Tf = = 489.86 K = 216.71o C
T
100 ⋅ CV − CV + CP ⋅ in
Ti
100 bar ⋅ 0.1 m3
b) Number of moles finally N f = = 245.54 mol
-5 bar ⋅ m
3
8.314 ⋅10 ⋅ 489.86 K
mol ⋅ K
241.51 mol
so that ∆N = 241.51 mol and the fill time is t = = 12.075 sec
20 mol/s
c) Final pressure, since volume and number of moles is constant, can be computer from
Pf ' Pf Pf 100
= so that Pf ' = ⋅Tf ' = ⋅ 298.15 = 60.86 bar
Tf ' Tf Tf 489.86
4
4.44
c
1 bar
20°C
e Liquid, 85°C

10 bar
d 10 bar = 1000 kPa =1 MPa
200°C
⎧1 bar, 20°C ˆ = 83.96
H Sˆ 1 = 0.2966
L⎨ 1

⎩1 bar,85°C H 3 = 355.90
ˆ Sˆ 3 = 1.1343
V {10 bar, 200°C H
ˆ = 2827.9
2 Sˆ = 6.6940
2

 = +1 kg = M
−M  +M
M.B.  +M
M  +M =0 3
s
1 2
1 2 3
 = 1− M
M 
1 2

E.B.
0=M H  ˆ  ˆ
1 1 + M 2 H 2 + M3 H3
ˆ

(
= 1− M 2 )
 83.96 + M  ⋅ 2827.9 − 1 ⋅ 355.90
2

 ( 2827.9 − 83.96 ) − 355.90


= 83.96 + M 2

 = 355.90 − 83.96 = 0.0991


M
2827.9 − 83.96
2


M1 = 1 − 0.0991 = 0.9009
S.B.
 Sˆ + M
0=M  Sˆ + M
 Sˆ + S
1 1 2 2 3 3 gen

0 = 0.9091× 0.2966 + 0.0991× 6.6940 − 1× 1.1343 + S gen = 0


J
S gen = 0.2013
K ⋅s
4
4.45 Reactor is I, initially evacuated tank is II. Mass balance is

Ni = N I + N II , which when using the ideal gas law becomes


200 bar ⋅ V I P I V I P II V II I ⎛V ⎞
I
4V I
= + = P ⎜ +
⎜ T I T II ⎟⎟ since P I = P II
673.15 K TI T II ⎝ ⎠
200 ⎛ 1 4 ⎞
or = P I ⎜ I + II ⎟
673.15 ⎝T T ⎠

The energy balance of the reactor and tank is

NiU i = N I U I + N II U II , which when using the ideal gas law and constant heat capacity becomes
200 bar ⋅ CV ⋅ 673.15 K P I CVT I 4 ⋅ P I CVT II
= +
673.15 K TI T II
200
or 200=P I + 4 P I and therefore P I = = 40 bar
5

The entropy balance on just the contents of the reactor that undergo a uniform expansion leads to

S = constant which leads to


R 8.314
⎛ PI ⎞ CP
⎛ 40 ⎞ 73.2
T = Ti ⎜⎜
I
⎟⎟ = 673.15 ⎜ ⎟ = 560.7 K
⎝ Pi ⎠ ⎝ 200 ⎠

Using this result in the mass balance gives

200 ⎡ 1 4 ⎤
= 40 ⎢ + II ⎥
673.15 ⎣ 560.7 T ⎦
which gives T II = 708.7 K
4
4.46
2 steam
1 bar, 250°C
1
steam
1 bar, 100°C
1 bar, liquid water
e

1 steam 1 bar, 100°C 1 ˆ = 2676.1 kJ


H Sˆ = 7.3549
kJ
kg kg ⋅ K
3 liquid 1 bar, 100°C 3 H
ˆ = 419.04 ˆS = 1.3069

2 steam 1 bar, 250°C 2 Hˆ = 2974.3 Sˆ = 8.0333

M.B.  +M
M  +M  =0  +M
M  = −M
 =1
1 2 3 2 3 1

E.B.  ˆ  ˆ 
M1H1 + M 2 H 2 + M 3 H 3 = 0
ˆ

( +M
− M 2 3 )
 2676.1 + M
 2974.3 + M
2
 419.04 = 0
3

 ( 2974.3 − 2676.1) + M
M  ( 419.04 − 2676.1)
2 3


M 2 2676.1 − 419.04
= = 7.5689

M 2974.3 − 2676.1
3
 = 7.5689 M
⇒ M 
2 3

but  +M
M  = −1 = 7.5689 M
 +M = −1
2 3 3 3

 = −1  = −1 + 0.1167 = −0.8833
M 3 = −0.1167 M 2
8.5689
S.B.  Sˆ + M
M  Sˆ + M  Sˆ + Sˆ = 0
1 1 2 2 3 3 gen

1× 7.3549 − 0.8833 × 8.0333 − 0.1167 × 1.3069 + Sˆ gen = 0


kJ ˆ
0.1066 + Sgen = 0
K
kJ
Ŝgen = −0.1066 ⇒ process is not possible
K
5

5.1 This is like Illustration 5.2-2 except that the Rankine rather than vapor compression refrigeration
cycle is used. Only properties of point 2, and path from 1 → 2 changes.
Point 2: isentropic S = 12622
. kJ kg K ; H = 279.8 kJ kg; 55° C ; T = 5° C
S L (5° C) = 10244
. ; x = fraction vapor; S V (5° C) = 17252
.
1.7252 ⋅ x + 10244
. (1 − x) = 12622
.
x = 0.3393
H L (5° C) = 206.8 H = x(4017. ) + (1 − x)(2068
.)
H (5° C) = 401.7
V
H = 272.9
Q BL = H 3 − H 2 = 402 − 272.9 = 1291
.
  
W = H − H = 432 − 402 = 30
4 3
1291.
C.O.P. = = 4.303
30
If the turbine drives compressor
c h c h
W = H 4 − H 3 − H 1 − H 2 = (432 − 402) − (280 − 272.9) = 22.9
1291
.
C.O.P. = = 564
.
22.9

Vapor compression cycle with change of Tlow and Thigh .


Location State Path T (° C) P (kPa) a
H kJ kg f SakJ kgf
1 Sat’d liq 60 1683 287.9 1.286
isenthalp
2 V-L mix 0 293 287.9
P = const
3 Sat’d vap 0 293 398.8 1.728
isentrope ↓
4 superheated 1683 433.6 1.728
vapor
P = const ↑
5 Sat’d liq 66 1683 287.9 1.286

Interpolation

P = 1600 kPa 60° C S = 17134


. H = 429.3
65° C 
S = 17323
. H = 4356
.
70° C S = 17503
. H = 4417
.
P = 1700 kPa 65° C 
S = 17217
. H = 4333
.
70° C S = 17405
. H = 439.7
at 1700 kPa S = 1728
. at 65.16°C and H = 4335
.

at 1600 kPa S = 1728. 
at 63.84°C and H = 4341.
⇒ at 1683 kPa T = 64.9°C and H = 4336 .
QC = H 3 − H 2 = 3988. − 287.9 = 110.9
  
W = H − H = 4336 . − 3988 . = 34.8
4 3
110.9
C.O.P. = = 318
.
34.8
5
5.2 General analysis of a heat engine

dS Q 
z
T3
dS
= ; Q=T ; Q2 − 3 = TdS = area(a − 1 − 2 − 3 − 4 − b − a )
dt T dt T

z
2

a f
Q4 −1 = TdS = T S1 − S4 = area(a − 1 − 4 − b − a )
Integration around a cycle
0 = Q2 − 3 + Q4 −1 + Wnet
−Wnet = +Q2 − 3 + Q4 −1 = area(a − 1 − 2 − 3 − 4 − b − a ) − area(a − 1 − 4 − b − a )
= area(1 − 2 − 3 − 4 − 1)
−Wnet −Wnet area(1 − 2 − 3 − 4 − 1)
η= = =
Qin Q2 − 3 area(a − 1 − 2 − 3 − 4 − b − a )

Original cycle Increasing evaporator T but not pressure


Original cycle Increasing evaporator T and P

Either way more work is obtained, with only a slight increase in heat rejected
⇒ almost all the additional heat input is converted to work.
⇒ efficiency improves

Decreasing evaporator T

(Note: may get in to 2-phase region)

Again, efficiency will increase due to more work being produced and less heat rejected.
5

5.3 a − 1 − 2 − 3′ − 4′ − b′ − a without superheat


a −1− 2 − 3 − 4 − b − a with superheating.

Clearly greater work output and higher efficiency with superheat.

a −1− 2 − 3 − 4 − b − a without subcooling


a ′ − 1′ − 2′ − 3 − 4 − b − a ′ with subcooling
Probably is greater efficiency with subcooling, but is not as evident as with superheating case.
5
5.4
T P S H V

1 80°C 47.39 kPa 1.0753 334.91 0.00102


isentropic
2 2.5 MPa 1.0753 337.4

3 700 2.5 MPa 7.8435 3914.5

4 130.3 47.39 7.8435 2741.8

1 80°C 47.39

at P = 0.050 MPa( ~ 0.04739)


T = 100 ; H = 2682.5 ; S = 7.6947
T = 150 ; H = 27805
. ; S = 7.9401
⇒ T = 130.3; H = 27418. kJ kg
z
Work1→2 = + VdP = 0.001029 m3 kg × (2500 − 47.39)kPa
= 2.524 m3 kg × 103 Pa = 2.524 × 103 m3 kg × J m3
= 2524 kJ kg
−Wnet
η= +Wnet = (27418
. − 3914.5) + 2.5 = −1170.2
Qin
Qin = 3914.5 − 337.4 = 35771
.
1170.2
η= = 0.327 ⇒ 32.7%
35771.
Turbine is only 85%, but adiabatic
⇒ −Wturbine
ideal
= 3914.5 − 27418
. = 1172.7
− Wturbine
act
= 085
. × 1172.7 = 9968 .
⇒ H 4 = 3914.5 − 9968
. = 2917.7 kJ kg
So state 4 ⇒ P = 47.39 ; H = 2917.7
T = 200° C ; H = 2877.7 ; S = 81580
.
 
T = 250° C ; H = 2976.0 ; S = 8.3556
⇒ T = 220.3° C ; S = 8.2384 kJ kg K
. − 2.5
9968
η= = 27.8%
.
35771

P T H S

1 47.39 kPa 80 334.91 1.0753

2 2.5 MPa 1.0753

3 2.5 MPa 700°C 3462.1 7.8435

4 47.39 kPa ~190 2860.2

If turbine is only 85% efficient


W = 34621 . − 2754.0 = 7081
. in previous case
W = 6019 . ⇒ H4 = 34621 . − 6019
. = 2860.2
η = ~ 085
. × 0.226 = 0192
.

U L = 96373
. kJ kg ; S L = 2.5639 kJ kg K ; U L = 26033
. kJ kg ; S L = 6.2503

P kPa T H S V

1 sat L 47.39 kPa 80 334.91 1.0753 0.001029

2 2.5 MPa 1.0753

3 superheated 2.5 MPa 700°C 3462.1 7.8435


vapor
4 superheated 47.39 ~138°C 2754 7.8435
vapor

Qin = H 3 − H 2 = H 3 − H 1 = 34621
. − 334.91
 
W = H − H = 2754 − 34621 .
4 3
W . − 2754
34621
η= = = 0.226
. − 334.91
Q 34621
5

5.5 (a) At steady state


dU 
= QH + Q M + Q L = 0 since W = 0
dt
dS Q H Q M Q L
= + + = 0 since Sgen = 0
dt TH TM TL
⎛ Q Q ⎞
Q M = −TM ⎜ H + L ⎟ and from energy balance
⎝ TH TL ⎠
⎛ T ⎞ T
Q H ⎜ 1 − M ⎟ = Q L M
⎝ TH ⎠ TL
so that
T
 1− M
Q TH T T T T T ⎛ 1 1 ⎞
C.O.P. =  L = = L − M ⋅ L = L − L = TL ⎜ − ⎟
QH TM TM TH TM TM TH ⎝ TM TH ⎠
TL
TH = 750 + 273.15 = 1023.15 K; TM = 27 + 273.15 = 300.15 K; and TL = −3 + 273.15 = 270.15 K
(b) ⎛ 1 1 ⎞
so that C.O.P.=270.15 ⎜ − ⎟ = 0.636
⎝ 300.15 1023.15 ⎠
5
5.6 (a) ~63% sat vapor
37% liquid produced
Work in stage 1 473 − 290 = 183 kJ kg
Work in stage 2 388 − 269 = 119 kJ kg
302 kJ kg
302
kJ kg liquid N 2 = = 816.2 kJ kg N 2 produced
0.37
 =M
M  +M
3 5′ 6
  
M H =M H +M   H
3 3 5' 5 6 6
. = x ⋅ 278 + (1 − x)29
115
x(278 − 29) + 29
153 − 29 124
x= = = 0.498 = fraction vapor
278 − 29 228
⇒ 0502
. fraction of liquid

Enthalpy of stream before compressor


Stage 1 425 − 272
272
Stage 2 = 579.6 kJ kg N 2
0.4693
(as before)
0.498 H (01 . H (01
. MPa,125 K) + 0502 . MPa,135 K)
= 0.498 × 278 + 0502
. × 290 = 284.02 kJ kg
Stage 1 463 − 284 = 179
Stage 2 119
as before 179 + 119 = 298 kJ kg
298
= 5936 . kJ kg LN 2
0502
.
5

5.7 System = contents of the turbine. This is a steady-state, adiabatic, constant volume system.
dM  +M  or M  = −M 
(a) Mass balance =0= M 1 2 2 1
dt
Energy balance
constant
dU
=0= M  H + M  H + Q adiabatic +W − P dV volume
1 1 2 2 s
dt dt
Entropy balance

dS
=0= M  S + M S + Q +Sgen
1 1 2 2
dt T
0, by problem statement
Thus
M = −M = −4500 kg h M.B.
2 1

S c
W = − M H − H
 
1 1 2 h E.B.
S2 = S1 S.B.
State T1 = 500° C Steam H 1 = 3422.2 kJ kg
1 ⎯ ⎯⎯⎯→
P1 = 60 bar Tables S = 68803
. kJ kg
1

State P2 = 10 bar Steam T2 ≅ 240.4° C


⎯ ⎯⎯⎯→
2 Tables
S2 = S1 = 68803
.
kJ H 2 ≈ 29205
. kJ kg
kgK
kg kJ kJ
Ws = 4500 × (29205 . − 3422.2) = −2257650 = −6271 . kW
h kg h
a f
(b) Same exit pressure P2 = 10 bar , and still adiabatic
⇒ W = − Ms c
 H − H .
1 1 2 h
Here, however,
Ws = 08
. Ws (Part a) = 08 . −2.258 × 106 c kJ
h
h c
= 4500 H 2 − 3422.2
kJ
h
h
⇒ H 2 = 30208 . kJ kg Steam T 2 ≅ 286 .7 K
⎯ ⎯⎯⎯→
P = 10 bar Tables S2 ≈ 7.0677 kJ kg K
Thus
Sgen = − M 1 1 c 2
h
h
 S − S = −4500 kg × (68803 . − 7.0677)
kJ
kg K
= 8433
.
kJ
K⋅h
(c) Flow across valve is a Joule-Thompson (isenthalpic expansion) ... See Illustration 3.4-1.
Thus, H into valve = H out of valve , and the inlet conditions to the turbine are

H 1 = H out of valve = H into valve = 3422.2 kJ kg


P1 = 30 bar
Steam T1 ≈ 484.8° C
⎯ ⎯⎯⎯→
Tables S1 ≈ 71874
. kJ kg K

Flow across turbine is isentropic, as in part (a)

S2 = S1 = 71874


. kJ kg K Steam T2 ≅ 3181
. °C
⎯ ⎯⎯⎯→ 
P2 = 10 bar Tables H2 ≈ 3090.4 kJ kg

kg kJ kJ
Ws = 4500 × (3090.4 − 3422.2) = −1493
. × 106 = −414.8 kW
h kg h
5

5.8 a) For each stage of the compressor, assuming steady-state operation and
reversible adiabatic operation we have from the mass, energy and entropy
balances, respectively
0= M +M  or M = −M  = −M 
in out out in
 
0 = Min Hin + M  H + W or W = M ( H − H )
out out out in
and
 S + M
0= M  S or Sout = Sin
in in out out
So through each compressor (but not intercooler) stage, one follows a line on
constant entropy in Fig. 3.3-2
Therefore, for first compressor stage we have

H in (T = 200 K, P = 1 bar ) = 767 kJ / kg and Sin (T = 200 K, P = 1 bar ) = 65


. kJ / kg K
 
H ( S = 65 . kJ / kg K, P = 5 bar ) = 963 kJ / kg and T = 295 K
out out
Therefore the first stage work per kg. of methane flowing through the compressor is
W ( first stage) = 963 − 767 kJ / kg = 196 kJ / kg

After cooling, the temperature of the methane stream is 200 K, so that for the
second compressor stage we have

H in (T = 200 K, P = 5 bar ) = 760 kJ / kg and Sin (T = 200 K, P = 5 bar) = 565


. kJ / kg K
 
H ( S = 565 . kJ / kg K, P = 25 bar ) = 960 kJ / kg and T = 300 K
out out
Therefore the second stage work per kg. of methane flowing through the compressor is
W (sec ond stage) = 960 − 760 kJ / kg = 200 kJ / kg

Similarly, after intercooling, the third stage compressor work is found from

H in (T = 200 K, P = 25 bar ) = 718 kJ / kg and Sin (T = 200 K, P = 25 bar) = 4.65 kJ / kg K


H ( S = 4.65 kJ / kg K, P = 100 bar ) = 855 kJ / kg and T = 288 K
out out
Therefore the third stage work per kg. of methane flowing through the compressor is
W ( third stage) = 855 − 718 kJ / kg = 137 kJ / kg

Consequently the total compressor work through all three stages is


W = 196 + 200 + 137 = 533 kJ / kg
b) The liquefaction process is a Joule-Thomson expansion, and therefore occurs at
constant enthalpy. The enthalpy of the methane leaving the cooler at 100 bar and
200 K is 423 kJ/kg. At 1 bar the enthalpy of the saturate vapor is 582 kJ/kg, and
that of the liquid is 71 kJ/kg. Therefore from the energy balance on the throttling
valve and flash drum we have

H in = H out or
H (200 K, 100 bar ) = (1 − x) H (sat'd. vapor, 1 bar) + xH (sat' d. liquid, 1 bar)
kJ kJ kJ
423 = (1 − x) ⋅ 71 + x ⋅ 582
kg kg kg

where x = 0.689 is the fraction of vapor leaving the flash drum, and (1-x) = 0.311
is the fraction of the methane that has been liquefied. Therefore, for each
kilogram of methane that enters the simple liquefaction unit, 689 grams of
methane are lost as vapor, and only 311 grams of LNG are produced. Further,
since 533 kJ of work are required in the compressor to produce 311 grams of
LNG, approximately 1713 kJ of compressor work are required for each kg. of
LNG produced.

c) As in the illustration, we choose the system for writing balance equations to be the subsystem
consisting of the heat exchanger, throttle valve and flash drum (though other choices could be
made). The mass and energy balances for this subsystem (since there are no heat losses to the
outside or any work flows) are

 =M
M  +M or taking M
 = 1 and letting x be the fraction of vapor
3 5' 6 3
1 = (1 − x) + x
M H = M  H + M H
3 3 5' 5' 6 6

1⋅ H (T = 200 K, P = 100 bar) = x ⋅ H (T = 200 K, P = 1 bar) + (1 − x) ⋅ H (sat' d.liquid, P = 1 bar)


kJ kJ kJ
423 = x ⋅ 718 + (1 − x) ⋅ 71
kg kg kg

The solution to this equation is x = 0.544 as the fraction of vapor which is


recycled, and 0.456 as the fraction of liquid.
The mass and energy balances for the mixing of the streams immediately before
the compressor are
 +M
M  =M  ; then basing the calculation of 1 kg of flow into the compressor
5' 1 1'
 
M1' = 1, M5' = 0544
. and M = 0.456
1
However, since both the recycle vapor and the inlet vapor are at 200 K and 1 bar, the gas leaving the
mixing tee must also be at these conditions, so that the inlet conditions to the first compressor are the
kJ
same as in the simple liquefaction process, and H 1' = 718 . Also, all other compressor stages
kg
operate as in the simple liquefaction process.

Therefore, the total compressor work per kg of methane passed through the
compressor is
W = 196 + 200 + 137 = 533 kJ / kg of methane through the compressor.
However, each kg. of
methane through the compressor results in only 0.456 kg. of LNG (the remainder
of the methane is recycled). Consequently the compressor work required per kg.

of LNG produced is (533 kJ/kg)/0.456 kg = 1168 kJ/kg of LNG produced. This is


to be compared to 1713 kJ/kg of LNG produced in the simple liquefaction
process.
5
5.9 (also available as a Mathcad worksheet)
5
5.10
5
5.11 25 bar = 2.5 MPa; 600°C
H = 3686.3 kJ kg ; S = 7.5960 kJ kg K
1 bar 100°C S = 7.3614
150 S = 7.6314
∆ = 0.27 1311
⇒ T = 14344. °C

H = 2776.4 − 131
. = 27633 . kJ kg
W = −3686.3 + 27633. = −923 kJ kg
−16614 kJ mol
aideal gas = −16830 kJ molf
Actual work 784.55
dU
dt
=0= M 1c 1 2 h
 H − H + Q − P dV + W
dt
s

W
H 2 = H 1 +  s = 3686.3 − 784.55 = 290175
.
M1
Final state P = 1 bar ; H = 290175.
H (1 bar, 200° C) = 28753
. UV ∆T
=
50
99.6 ∆H 99.6
H (1 bar, 250° C) = 2974.9 W
. − H (1 bar, 200° C) = 26.45
290175
50
T = 200 + 26.45 = 21328
.
99.6
S(1 bar, 250) = 8.0333
S(1 bar, 200) = 7.8343
S(1 bar, 213.28) = 7.8872
dS
dt
=0= M c 1 h
 S − S + S ;
2 gen

Sgen
= − S2 − S1 = 7.8872 − 7.5960 = 0.2912 kJ kg K
M
.
52416 kJ kg K
(ideal gas = 5468
. )
PR: T = 600° C ; P = 25 bar
H = 216064
. × 104
S = 14.74377
Now P = 1 bar , S = 14.74377 . Guess T = 213° C .

T S H
213 19.67116 727195
.
150 14.74399 5034.86
W
= 5034.86 − 21606.4 = −1657154
. J mol

M
Actual work ⇒ H f = 752059
.

T S H
213 19.67116 7271.95
230 20.90787 7883.64
220 20.18472 7523.41
219.9 20.17742 7519.80
219.92 20.17888 7520.53

S = 2017888
. − 14.74377 = 543511
.
5
5.12 First write balance equations for the contents of the turbine. This is a steady-state,
constant-volume, adiabatic system.
M.B. 0 = M 1 + M 2 ⇒ M 2 = − M 1
a
E.B. 0 = M 1H 1 + M 2 H 2 + Ws ⇒ −Ws = N 1 H1 − H 2 f
 0
S.B. 0= M  S + Q
 S + M + Sgen
1 1 2 2
T
Maximum work ⇒ reversible process ⇒ Sgen = 0 (see Sec. 3.2) ⇒ S1 = S2 .
Thus the inlet and exit turbine conditions are connected by a vertical line on the
Mollier diagram.
(a) H = 2880 J g
1
Vertical line H 2 = 2270 J g , T2 = 99.65° C
⎯ ⎯⎯⎯⎯⎯⎯⎯→
connecting 540 K,
P2 = 1 bar
Saturation T FG IJ
36 bar and 1 bar for P = 1 bar H K
Ws
− = H 1 − H 2 = (2880 − 2270) J g = 610 J g

M
(b) Two stage turbine. Use balance equations above for each stage.

Stage 1
T1 = 540 K = 26685
. °C H i = 2770 J g
P1 = 36 bar T ∗ = 118° C
E D
H 1 = 2880 J g P∗ = 185
. bar
S1 = 6.28 J g K ⎯
⎯→  ∗
S = 6.28 J g K

After reheating in the reactor

Stage 2
P∗ = 185
. bar H 2 = 2440 J g

T = 26685. °C T2 = 99.63° C
E D
H = 2960 J g P2 = 1 bar
S = 6.70 J g ⎯
⎯→ S2 = 6.70

Note: There is no work done in reheating step (no shaft work, and no change of reheater
boundaries). Only energy transfer step is heating; Q M
 = 2960 − 2770 = 190 J g

Ws
Total work out/g = − = (2880 − 2770) + (2960 − 2440) = 630 J g
M
a20 J g additional workf
(c) Similar to part (b) above, except that intermediate pressure is 6.0 bar

Q M
 = 445 J g
− Ws M
 = 685 J g a75 J g additional workf
5
5.13
New ⎛ kJ ⎞ ⎛ kJ ⎞
Path
Point (State) P(MPa) Ŝ ⎜ ⎟ Ĥ ⎜ ⎟
T(°C) ⎝ kg ⎠ ⎝ kg ⎠
Path
1 Sat’d Liq 100 0.10135 1.3069 419.04
Isentropic È
2 ~103 3.0 1.3069 422.07
Isobaric Ç
3 400 3.0 6.9212 3230.9
Isentropic È
4 0.10135 6.9212
Isobaric Ç
1 100 0.10135 1.3069 419.04

Note that P = 0.10135 MPa Ŝ =6.9212 corresponds to a V + L mixture. Let x


represent the quality, or the fraction of steam that is vapor.
Ŝ = x ⋅ 7.3549 + (1 − x ) ⋅1.3069 = 6.9212
6.9212 − 1.3069
x= = 0.9283
7.3549 − 1.3069

Cycle can not function as specifiec because turbine would have a V + L mixture
Liquid droplets in a high speed turbine destroys turbine

Boiler e
J-T valve
d
f

Pump WP Turb. WT

Condenser
c g
b)
(State) Path ⎛ kJ ⎞ ⎛ kJ ⎞
Point P(MPa) Ŝ ⎜ ⎟ Ĥ ⎜ ⎟
Path T(°C) ⎝ kg ⎠ ⎝ kg ⎠
1 Sat’d Liq 100 0.10135 1.3069 419.04
Isentrope È
2 ~103 3.0 1.3069 422.07
Isobaric Ç
3 400 3.0 6.9212 3230.9
Isenthalpic È
4 372°C ~ 0.720 7.5085 3230.9
Isentrope Ç
1 126.2 0.10135 7.5085 2734.7

Use Enthalpy – Entropy diagram Fig. 3.3-1a to get approximate temperature and pressure
(values are approximate as they were read from that figure). Also, vertical line
(isentrope) to 0.1 MPa = 100 kPa remains in vapor region so no liquid in the turbine
system can operate.

c) ŴP =3.03 kJ/kg as before


Q̂ B =3230.9–422.1 = 2808.8 kJ/kg
ŴT =2734.7–3230.9=–496.2 kJ/kg
Q̂C =419.04–2734.7 = –2315.7 kJ/kg as before

work out 496.2 − 3.03


d) Efficiency = η = = = 0.1756
heat in 2808.8
(compared to 0.29 for original cycle)
5
5.14 Methane – 10 MPa, 180 K → 0.1 MPa
From Fig. 3.3-2 methane is a liquid at these conditions.
Ĥ = 340 kJ/kg; Ŝ ≅ 2.38 kJ/kg K
Line of Constant Entropy to 0.1 MPa → Two-phase region at 0.1 MPa
Hˆ = 72 kJ/kg H ˆ = 581 kJ/kg
L V

Sˆ L ∼ 0.8 kJ/kg Sˆ V ≅ 5.25 kJ/kg ⋅ K


Explosion would be a process at constant cntropy …
First determine fraction of liquid after explosion
2.38 = 0.8 L + 5.25 (1 – L) = 5.25 + (0.8 – 5.25) L
5.25 − 2.38
L= = 0.6449 (liquid) and V=0.3551 (vapor)
5.25 − 0.8
Enthalpy of methane after Explosion
H = 0.6449 × 72 + 0.3551 × 581 = 252.7 kJ/kg

Uf – Ui = W so need internal energies, not enthalpies


ˆ = 0.0033 m3 / kg
V ˆ =∼ 0.0023 m3 / kg
V Vf ,V = 0.55 m3 / kg
i f ,L

Only correct U for final vapor


kJ m3 Pa J kJ
Û f ,V = 581 − 0.1 MPa × 0.55 × 106 ×1 × 10−3
kg kg MPa m3 Pa J
kJ kJ
= 581 − 55 = 526
kg kg
kJ
Û f = 0.6449 × 72 + 0.3551× 526 = 232.2
kg
kJ
W = 1000 kg × ( 232.2 − 340 ) = −1.078 × 105 kJ
kg
TNT = 4600 kJ/kg

The possible explosion here would release the energy of approximately 23.4 kg of
TNT, a big blast! (A very likely scenario in an actual explosion is that the methane
release would form a traveling vapor cloud, would mix with air and eventually reach an
ignition source resulting in a secondary chemical explosion, which would release much
more energy).
5
5.15

d Boiler e

WP Pump WT

Condenser
c f

QC

(State) P Ŝ Ĥ V̂
Point T(°C)
Path (MPa) (kJ/kg K) (kJ/kg) (m3/kg)
1 Sat’d Liq 100 0.10135 1.3069 419.04 0.001044
Isentrope È
2 Comp. Liq ~103 5 1.3069 ~423
Isobaric Ç
3 700°C 5 7.5122 3900.1
Irrev.
4 0.10135
Isobar È
1 Sat’d Liq 0.10135 1.3069 419.04

New turbine produces 100 MW of power with a steam flow of 89 kg/S

 +M
M.B. around turbine M  =0 ⇒ M
 = −M

3 4 4 3

E.B. around turbine


 Hˆ  ˆ  6 J kJ
M 3 3 − M 3 H 4 = − W = 100 MW = 100 × 10 = 1× 105
s s
kg ⎛ kJ ˆ ⎞ 5 kJ
89 ⎜ 3900.1 − H 4 ⎟ = 1× 10
s ⎝ kg ⎠ s
105 kJ
Ĥ 4 = 3900.1 − = 3900.1 − 1123.6 = 2776.6
89 kg

Therefore, steam leaving turbine is at P = 0.10135 MPa


and Ĥ =2776.6 kJ/kg

a) From Steam Tables, T = 150°C and Ŝ =7.6134 kJ/kg K


b) Entropy balance
kg kJ kJ
 Sˆ − M
M 3 3
 Sˆ + Sˆ = 0
3 4 gen S gen = 89 ( 7.6134 − 7.5122 ) = 9.01
s kg ⋅ K s⋅K
If the turbine was isentropic, P = 0.10135 MPa and Ŝ =7.5122 kJ/kg·K

Using linear interpolation


T − 100 7.5122 − 7.3614
= = 0.5984
150 − 100 7.6134 − 7.3614
T = 100 + 0.5984¯50 = 100 + 29.9 = 129.9°C
and an interpolation on enthalpy
Ĥ − 2676.2 ˆ = 2676.2 + 60 = 2736.2 kJ/kg
= 0.5984; H
2776.4 − 2676.2
So for an isenthalpic (reversible) turbine
∆H = 3900.1 – 2736.2 = 1163.9 kJ/kg
While for the real turbine
∆H = 1123.6 kJ/kg

1123.6
Turbine Efficiency = = 0.965
1163.9

c) Work to turbine
kJ kg kJ
≅4 × 89 = 356 = 356 kW
kg s s
5
5.16 a) From the isobutane thermodynamic properties figure in the problem, with the
kJ
condenser operating at 320 K, P≈0.675 MPa. Also, Ĥ ( leaving condenser ) ∼ 273
kg
b) Isobutane entering the turbine is at 100 MPa and 480 K. From the figure,
kJ ˆ kJ
Ĥ=775.6 and S=2.259 . Also, isobutane leaving the turbine is at P = 0.675 MPa and
kg kg ⋅ K
kJ kJ
Ŝ=775.6 , which corresponds to about 360 K and Ĥ=665 .
kg ⋅ K kg
c) From the energy balance, the work produced in the turbine is
dU
dt
= 0 = M in H (
ˆ −H
in
ˆ )
out + W so that
-W
M in
= 775.6 − 665 = 110.6
kJ
kg
kJ
3000
Since 3 MW=3000 kW=3000 kJ/s the isobutane flow rate is s = 27.1 kg .
kJ s
110.6
kg
d)
e) Pump (no heat flow):
m3 106 Pa J kJ kJ
W = ( ∆P ) V = 910 − 0.675)MPa ⋅ 0.0019 ⋅ ⋅1 ⋅10−3 = 17.7
kg MPa Pa ⋅ m 3
J kg
kJ kg kJ
W = 17.7 × 27.1 = 479.7
kg s s
Turbine: 3 MW = 3000 kJ/s
kJ kg kg
Condenser: Q = -(665-273) = -392 kJ/kg, Q = −392 × 27.1 = −10623
kg s s
kJ kg kg
Boiler: Q = (776.65-(273+17)) = 486.7 kJ/kg, Q = 486.7 × 27.1 = 13.188
kg s s
net work out 3000 − 479.7
Efficiency = η = = = 0.191 or 19.1%
heat in at boiler 13188
5
5.17
2
0.5 kg/s 100°C, steam
1 1 kg/s

300°C e
1 MPa 0.5 kg/s 100°C, sat’d liq

Streams T P Condition Ĥ kJ/kg Ŝ kJ/kg


1 300°C 1 MPa sup heated 3051.2 7.1229
2 100°C
0.10135
3 100°C sat’d liq 419.04 1.3069
MPa

Energy balance
kg  =0
1 × 3051.2 − 0.5 × H
ˆ − 0.5 × 1344.0 + W
2
s
but W = −1.15 MW = −1500 kJ
s
3051.2 = 0.5 × H − 0.5 × 419.04 − 1500
ˆ
2

kJ
Ĥ 2 = 2683.4
kg

kJ kJ
Now at at 0.1 MPa and T = 100o C Hˆ V = 2676.1 and Sˆ = 7.3549
kg kg ⋅ K

So steam is slightly superheated and Quality is 1


Entropy balance (using Sˆ at saturation)

1 × 7.1229 – 0.5 × 1.3069 – 0.5 × 7.3549 + S gen = 0


S gen = 0.5 (1.3069 + 7.3549 ) − 7.1229
= −2.792 kJ / K − S
So process is not possible
(using better estimate, by interpolation, for exit entropy of steam would have no
effect on conclusion).
6

6.1 Using the Mollier diagram

F ∂T I = F ∆T I = (510 − 490)° C
µ=
H ∂P K H ∆P K c1241
H . × 10 − 7.929 × 10 hPa
H
7 6

= 4.463 × 10−6 ° C Pa = 4.463 ° C MPa


F ∂T I ≈ F ∆T I = (510 − 490)° C
κS =
H ∂P K H ∆P K c1069
S .S× 10 − 9.515 × 10 hPa
7 6

= 1702
. × 10−5 ° C Pa = 17.02 ° C MPa
a∂H ∂S f =
a f a f a f a f
∂ H, T
×
∂ S, P
=
∂ H, T
×
∂ S, P
a∂H ∂S f a f a f a f a f
T

P ∂ S, T ∂ H, P ∂ H, P ∂ S, T
∂aT , H f ∂a P, S f µ
= × = = 0.262 ( unitless)
∂a P, H f ∂aT , S f κS
6

6.2 (a) Start from eqn. 6.4-27

z LMN FH I OP
V
dP
H (T , P) − H IG (T , P) = RT ( Z − 1) +
V =∞
T
dT K V
− P dV
Q
RT a(T ) ∂P F I R da dt
P= − 2
V − b V + 2bV − b 2
;
∂T H K V
= − 2
V − b V + 2bV − b2
so

H (T , P) − H IG (T , P)

z LMN RST UV OP
V
R da dt RT a(T )
= RT (Z − 1) + − − +
V =∞
T
V − b V 2 + 2bV − b2 W
V − b V 2 + 2bV − b2
dV
Q
F da I z
V
dV
H
= RT (Z − 1) + a + T
dT K V =∞ V + 2bV − b
2 2

From integral tables we have

z dx
a ′x 2 + b′x + c′
=
1
ln
2a ′x + b′ − b′2 − 4a ′c′
b′ − 4a ′c′ 2a ′x + b′ + b′ − 4a ′c′
2 2
for 4a ′c′ − b′2 < 0

In our case a′ = 1 , b′ = 2b , c = −b2 ; so c h


4a ′c′ − b′2 = 4 ⋅ 1⋅ −b2 − (2b)2 = −8b2 and

(b′)2 − 4a ′c′ = 8b2 = 2 2b .

H (T , P) − H IG (T , P)

= RT (Z − 1) +
aa − Tda dT f LMln 2V + 2b − 2 2b − ln
2V + 2b − 2 2b OP
2 2b MN 2V + 2b + 2 2b V 2V + 2b + 2 2b V =∞ PQ
= RT (Z − 1) +
aa − Tda dT f ln V + d1 − 2 ib
2 2b V + d1 + 2 ib

or finally

H (T , P) − H IG (T , P) = RT ( Z − 1) +
aTda dT − af lnLM Z + d1 + 2 iB OP
2 2b MN Z + d1 − 2 iB PQ
(b) This part is similar except that we start from eqn. (6.4-28)

z LMNFH I OP
V
dP R
S (T , P) − S IG (T , P) = R ln Z +
V =∞
dT K V

V
dv
Q
z LMN OP
V
R da dT R
= R ln Z + − 2 − dV
V =∞
V − b V + 2bV − b V
2
Q
= R ln Z + R ln
V −b
V
+
d
da dT V + 1 + 2 b i V

ln
V V =∞ 2 2b V + 1 − 2 bd i V =∞

da dT F Z + d1 + 2 i B I
= R ln(Z − B) + lnG J
2 2b H Z + d1 − 2 i B K
6
LM F ∂ P I OP FG ∂ U IJ
MN GH ∂ T JK
dU = CVdT + T − P dV . = CV ;
6.3 Start with eqn. (6.2-21):
V PQ
Thus
H∂TK V

FG ∂ U IJ = C + LMT FG ∂ P IJ − POPFG ∂ V IJ and


H∂TK P MN H ∂ T K PQH ∂ T K
V
V P

FG ∂ U IJ − FG ∂ U IJ = LMT FG ∂ P IJ − POPFG ∂ V IJ .
H ∂ T K H ∂ T K MN H ∂ T K PQH ∂ T K
P V V P
(a) Ideal gas PV = RT
FG ∂ P IJ − P = 0⇒
FG ∂ U IJ = FG ∂ U IJ
T
H∂TK V
H∂TK H∂TK P V
(b) van der Waals gas
∂P FG IJ ∂P LM F I OP
MN GH JK
RT a R a
P= − = ; ⇒ T −P =
V −b V2
;
∂T H K V V −b ∂T V PQ V2
RdT RT 2a
Also: dP = − dV + 3 dV
V − b (V − b)2
V


FG ∂ V IJ = R (V − b) = LM T − 2a(V − b) OP −1

H ∂ T K RT (V − b) − 2a V N (V − b) RV Q
P
2 3 3

⇒G
F ∂ u IJ − FG ∂ u IJ = a
H ∂ T K H ∂ T K V T (V − b) − 2a(V − b) RV
P V
2

aRV (V − b)
=
RTV 3 − 2a(V − b)2
(c) The Virial Equation of State

PV B C B
= 1 + + 2 + " = 1 + ∑ ii
RT V V i =1 V

RT B RT
or P = + ∑ i i +1
V i =1 V

FG ∂ P IJ = R + ∑ B R + ∑ RT F dB I ← derivative,
Note: This is a total

H∂TK V V V H dT K
i i
i +1
since B isi +1 i
V i =1 i =1
a function of only temperature

⇒ TG
F ∂ P IJ − P = ∑ RT dB
H∂TK
i
i +1
V d ln T
V i =1

Also need a∂ V ∂ T f , but this is harder to evaluate alternatively. Since


P
FG ∂ V IJ FG ∂ P IJ FG ∂ T IJ = −1 ⇒
FG ∂ V IJ =−
a∂ P ∂ T f V
H ∂ T K H ∂ V K H ∂ PK
P T V
H∂ TK P a∂ P ∂ V f T

FG ∂ P IJ
H∂TK V
is given above.

FG ∂ P IJ =−
RT
−∑
(i + 1) Bi RT
H ∂V K T V 2 i =1 V i + 2
FG
V RT V + ∑ Bi RT V i +1 + ∑ RT V i +1 dBi d ln T d ia fIJK
F ∂ V IJ
⇒G =
H i =1
H∂ TK FG
T RT V + ∑ (i + 1) Bi RT V i +1
IJ
H K
P
i =1

RT B RT
Using = P − ∑ i i +1 , we get
V i =1 V

FG ∂ V IJ =
d
V P + ∑ RT V i +1 dBi d ln T ia f
H∂ TK P T P + ∑ iBi RT Va f i +1

and

FG ∂ U IJ − FG ∂ U IJ R dBi P + ∑ RT V
=∑ i
i +1
LM d
dBi d ln T ia f OP
H∂TK H∂TK
P V V d ln T MN
P + ∑ iBi RT V i +1 PQ
z LMNFH dPdT IK − VR OPQdv
V
S (T , P) − S IG (T , P) = R ln Z +
V =∞ V

z LMNV R− b − V +da2bVdT− b OP
V
R
= R ln Z + −
V =∞
2 2
V Q
dV

= R ln Z + R ln
V −b
V
+
d
da dT V + 1 + 2 b i V

ln
V V =∞ 2 2b V + 1 − 2 bd i V =∞

da dT F Z + d1 + 2 i B I
= R ln(Z − B) + lnG J
2 2b H Z + d1 − 2 i B K
6
6.4 (a) Start from

µ=−
1 FG IJ OP ⇒ C = − 1 LMV − T FG ∂ V IJ OP
V −T
LM
∂V
CP H KQ
N
∂T µN P
H∂ TK Q P
P

L F ∂ V IJ OP = −T FG ∂ aV T fIJ and C = T FG ∂ aV T fIJ .


but MV − T G
2

N H∂ TK Q H ∂ T K µ H ∂T K
2
P
P P P

FG ∂ aV T fIJ = µ C ; integrate F V I − F V I = z µ C dT . T2 , P
(b)
H ∂T K T P
2
P
HTK HTK TT2 , P T1 , P T1 , P
2
P

z µCP
T2 , P
a f a f TT + T
Thus V T2 , P = V T1, P 2
2
T2
dT .
1 T1 , P
6
RT a
6.5 P= −
V −b V2

FG ∂ P IJ = − RT + 2a
H ∂ V K (V − b) V T
2 3
(1)

FG ∂ P IJ = 2RT − 6a
2

H ∂V K (V − b) V
2
T
3 4
(2)

at the critical point P → PC , T → TC and V → VC

FG ∂ P IJ = 0⇒
RTC
=
2a
H ∂V K T aV C −b f
2
V C3
(1')

FG ∂ P IJ
2
= 0⇒
2 RTC
=
6a
H∂V K 2
T aVC −b f 3
V C4
(2')

Dividing (1') by (2') ⇒


1
2
a 1
VC −b = VC ⇒ b =
3
V
3
ffrom (1')

V 3C RTC (3b)3 RTC 27bRTC 9VC RTC


a= = = =
a
2 VC −b f
2
2(3b − b)2 8 8

PV V a 8a a a
Also = − ; and PC = − =
RT V − b RTV 27b(2b) 9b2 27b2

⇒ ZC =
PCV C
=
VC

a
=
VC

9 8 V C RTC a f
RTC V C − b RTCVC 2 3VC V C RTC a f
3 9 3
= − = = 0.375
2 8 8
6
dS =
CP
dT −
∂V FG IJ
6.6
T ∂T H K P
dP [eqn. (6.2-20)]

CP* R
For the ideal gas d S IG = dT − dP . Thus, at constant temperature
T P

d
d S − S IG = i LMN RP − FGH ∂∂ VT IJK OPQdP and
P

z RST FG IJ UVdP
∂V
P
R
S (T , P) − S IG (T , P) − S (T , P = 0) − S IG (T , P = 0) = −
P=0
P H KW
∂T P

However, S (T , P = 0) − S IG (T , P = 0) = 0 , since all fluids are ideal at P = 0. Also


a f
PV = Z Tr , Pr RT . Thus

FG ∂ V IJ =
1 RS a
RZ Tr , Pr + RTf FG a
∂ Z Tr , Pr fIJ UV
H∂ TK P P T ∂T H KW P

and

R

∂VFG IJ =−
R
ka FG IJ
Z Tr , Pr − 1 −
RT ∂ Z
f p
P ∂T H K P P H K P ∂T P

⇒ S (T , P) − S (T , P) = − R z M
IG L Z − 1 + T FG ∂ Z IJ OPdP
T, P

N P P H∂ TK Q T , P =0 P

= −R z M
L ZaT , P f − 1 + T F ∂ Z I OPdP
Tr , Pr

MN P
Tr , Pr = 0
r
G J
P H ∂ T K PQ
r
r r

r r Pr
r
6
6.7 (a) Ideal gas

PV = NRT

N=
c
(50 bar ) 100 m3 h = 1421
. kmol
−2
(27315
. + 150)K × 8.314 × 10 bar ⋅ m3 kmol K

Energy balance, closed nonflow system

z
∆U = Q − PdV = Q + W .

However, for ideal gas ∆U = 0 since T is constant (isothermal). Thus

z
W = −Q = − PdV = − z NRT
V
V P
dV = − NRT ln 2 = − NRT ln 1
V1 P2
F 50 I
. kmol × 8.314 J mol K × (27315
= −1421 . + 150) × ln
H 300K
= 8959
. × 103 kJ = 8959
. MJ
Q = −8959
. MJ

Also, by Ideal Gas Law at fixed T and N

P1 50
1 1 = PV
PV 2 2 ⇒ V2 = V1 = 100 m3 × = 16.67 m3
P2 300

(b) Corresponding states

H IG − H
Tr Pr Z S IG − S
TC
150 + 27315
. 50
initial state = 1391
. = 0.679 0.94 0.7 0.4
304.2 7376
.
300
final state 1.391 = 4.067 0.765 4.5 2.4
73.76
cal cal
mol K mol K

PV 1421
. PV
Number of moles of gas = N = = = 1512 . =
. kmol ( 1421 from above)
ZRT 0.94 RT
ZNRT
Final volume = V f =
Pf
. × 8.314 × 10−2 × (27315
0.765 × 1512 . + 150)
= = 1356
. m3
300
Energy balance on gas: ∆U = Q + W
Q
Entropy balance on gas processes in gas are reversible: ∆S = + Sgen ⇒ Sgen = 0 . Therefore
T
Q
∆S = or Q = T∆S
T

c h mc
∆S = S f − Si = N S f − S i = N S f − S IG
f + S IG
f − Si
IG
h c
− S i − S iIG h c hr
RS
= N −2.4 × 4184 − 8.314 ln
P
. )V
− (−0.4 × 4184
Uf

T .
P Wi

R
= N S−8.368 − 8.314 ln
300 U
V = −2326
T 50 W
. N J K

Q = T∆S = (27315
. + 150) × 1512
. kmol × (−2326
. ) = −14885
. MJ
W = ∆U − Q = N U f − U i − Q = N H f − H i − N Pf V f − PV
i i −Q

= N TC
LM dH f − H IG
f i + cH IG
− H IG h −TC
d H − H i − Z RT
i
IG
i
OP
+ Zi RTi − Q
MN TC
f
0
Since process
i
TC
f f
PQ
is isothermal.

= 1512
LM304.2(−4.5 − (−0.7)) × 4184
. − 8.314O
PQ + 14885. MJ
. kmol
N×(27315
. + 150) × (0.765 − 0.94)
= 1512
. × 103 −48365
. + 6157
. J + 14885
. MJ = −638.2 + 14885
. MJ
= 850.3 MJ

(c) Peng-Robinson E.O.S.


Using the program PR1 with T = 27315 . , P = 1 bar as the reference state, we obtain
T = 150° C , P = 50 bar
Z = 0.9202 ; V = 0.6475 × 10−3 m3 mol ; H = 4702.48 J mol ; S = −17.57 J mol K .
T = 150° C , P = 300 bar
Z = 0.7842 ; V = 0.9197 × 10−4 m3 mol ; H = −60.09 J mol ; S = −4124
. J mol .
V 100 m3
N= = = 154.44 kmol
V 0.6475 × 10−3 m3 mol
. + 150) × 154.44 × (−4124
Q = TN∆ S = (27315 . − (−17.57)) = −1546.9 MJ
W = ∆U − Q = ( H − PV ) f − ( H − PV )i − Q
= N ( H − PV ) f − ( H − PV )i − Q
LM−60.09 − 300 × 0.9197 × 10 −4
OP
= 154.44M×10 J bar ⋅ m − 4702.48
5 3
PP × 10 + 1546.9 × 10
3 6

MMN+50 × 0.6475 × 10 × 10 −3 5
PQ
= 88525. MJ
{Note that N, Q and W are close to values obtained from corresponding states.}
6
6.8
FG ∂ T IJ =
∂ (T , S ) ∂ (T , S ) ∂ ( P, T )
= ⋅ =−
∂ ( S , T ) ∂ ( P, T )
H ∂ PK S a f
∂ ( P, S ) ∂ ( P, T ) ∂ P , S ∂ (S , P) ∂ (T , P)
−a∂ S ∂ Pf a∂ V ∂ T f = VαT
= =
a∂ S ∂ T f
T P

PC T C P P

and

κS
=
a fa
1 V ∂ V dP f S
=
∂ (V , S ) ∂ ( P, S ) ∂ (V , S ) ∂ ( P, T )
= ⋅
κT a fa
1 V ∂ V dP f T ∂ (V , T ) ∂ ( P, T ) ∂ (V , T ) ∂ ( P, S )

=
∂ (S ,V ) ∂ (T , P)
⋅ =
∂S FG IJ ⋅ FG ∂ T IJ =
CV T

C
= V
∂ (T ,V ) ∂ ( S , P) ∂T H K H∂ SK
V P T CP CP
6
FG ∂ H IJ = ∂ (H, T ) = ∂ ( H, T ) ⋅ ∂ ( P, T ) = FG ∂ H IJ FG ∂ P IJ
6.9 (a)
H ∂ V K ∂ (V , T ) ∂ ( P, T ) ∂ (V , T ) H ∂ P K H ∂ V K
T T T

Since G
F ∂ P IJ ≠ 0 (except at the critical point)
H ∂V KT

FG ∂ H IJ FG ∂ H IJ = 0
= 0 if
H ∂V K T
H ∂ PK T

FG ∂ S IJ = ∂ (S , P) = ∂ (S, P) ⋅ ∂ (T , P) = FG ∂ S IJ ⋅ FG ∂ T IJ
(b)
H ∂ V K ∂ (V , P) ∂ (T , P) ∂ (V , P) H ∂ T K H ∂ V K
P P P

=
C 1 F dT I
P
⋅ ⋅V G
T V H dV K
J = a1 V faC∂ VTV∂ T f = TVC α ⇒ FGH ∂∂ VS IJK
P
P

P
P

P
~ α −1
6
6.10 (a) We start by using the method of Jacobians to reduce the derivatives

FG ∂ T IJ =
∂ (T , H ) ∂ (T , H ) ∂ (T , P) ∂ T ,V
= ⋅ ⋅
a f
H∂V K H ∂ (V , H ) ∂ (T , P) ∂ (T ,V ) ∂ (V , H )

=−
∂ ( H , T ) ∂ ( P, T ) ∂ (V , T )
=−
∂H FG IJ a∂ P ∂ V f
H K a∂ H ∂ T f
T
∂ ( P, T ) ∂ ( H ,V ) ∂ (T ,V ) ∂P T V

=−
a∂ H ∂ V f
ad H ∂ T f
T

Now from Table 6.1 we have that

FG ∂ H IJ =V −T
FG ∂ V IJ FG ∂ H IJ = CP + V − T
LM FG ∂ V IJ OPFG ∂ P IJ
H ∂ PK T
H∂ TK P
and
H∂TK V N H∂ TK P QH ∂ T K V

alternatively, since H = U + PV

FG ∂ H IJ = FG ∂ U IJ + FG ∂ ( PV )IJ F dP I
H∂TK H∂TK H ∂T K
V V V
= CV + V
H dT K V

Thus

FG ∂ T IJ =
a f
− ∂ P ∂V T V − T ∂V ∂ T a f =
− V ∂ P ∂Va f + Ta∂ P ∂ T f
T V
H∂V K a f + V a∂ P ∂ T f
P

H CV + V ∂ P ∂ T V CV V

FG ∂ P IJ FG ∂ V IJ =−
FG ∂ P IJ
Note: I have used
H ∂V K H∂ TK T P
H∂TK V
.

FG ∂ T IJ =
∂ (T , S ) ∂ (T , S ) ∂ (V , T )
= ⋅ =−
∂ (S , T ) ∂ (T ,V )

H∂V K S ∂ (V , S ) ∂ (V , T ) ∂ (V , S ) ∂ (V , T ) ∂ (S ,V )

=
FG ∂ S IJ FG ∂ T IJ =−
T ∂P FG IJ
H∂V K H ∂ SK T V CV ∂ T H K V

(b) For the van der Waals fluid

FG ∂ P IJ =
R ∂P FG IJ =
− RT 2a
+ 3
H∂TK V V −b
,
∂V H K T (V − b) V
2

Thus
FG ∂ T IJ =
n
− − RTV (V − b)2 + 2a V 2 + RT (V − b) s
H∂V K H CV + V R V − b

after simplification we obtain

FG ∂ T IJ =
− 2a(V − b)2 − RTV 2b
H∂V K H CC (V − b)2V 2 + R(V − b)V 3

and

FG ∂ T IJ =−
RT
H∂V K S CV (V − b)
6
6.11 There are a number of ways to solve this problem. The method I use is a little unusual, but the
simplest that I know of. At the critical point all three roots of V are equal, and equal to V C .
Mathematically this can be expressed as V − V Ca f
3
= 0 which, on expansion, becomes

V 3 − 3V CV 2 + 3V C2 V − V C3 = 0 (1)
compare this with

RT a RT a
P= − = − 2
V − b V (V + b) + b(V − b) V − b V + 2bV − b2

which multiplying through by the denominators can be written as

F RT I F 2bRT a I FG
RTb2 ab IJ
H
V 3 +V 2 b −
P K H
+ −3b2 −
P
+ V + b3 +
P P K H

P
=0
K (2)

Comparing the coefficients of V in Eqns. (1) and (2) gives TC , PC

RTC
V 2: b − = −3V C (3)
PC

2bRTC a
V : − 3b2 − + = 3V C2 (4)
PC PC

RTC 2 ab
V 0: b 3 + b − = −V 3C (5)
PC PC

From Eqn. (3)

PCb −3PCVC PCb


−1 = = −3ZC or = 1 − 3ZC (6)
RTC RTC RTC

1− y
For convenience, let y = 1 − 3ZC or ZC = . Then
3

PCb
=y
RTC

From Eqn. (4)


−3
FG P b IJ − 2FG P b IJ + aP
2
= 3ZC2
H RT K H RT K aRT f
C C C
2
C C C

aPC 3(1 − y )2
⇒ −3 y 2 + 2 y + =
a RT f C
2
9

or expanding and rearranging

aPC
aRT f 2
=
1
3
c
10 y 2 + 4 y + 1 h (7)
C

Finally from eqn. (5)

FG P b IJ + FG bP IJ − FG P b IJ FG aP IJ = −Z
3 2

H RT K H RT K H RT K H a RT f K
C C C C 3
2 C
C C C C

y + y − y ⋅ c10 y + 4 y + 1h = − (1 − y)
3 2 1 1 2 3
3 27

or

64 y 3 + 6 y 2 + 12 y − 1 = 0 (8)

This equation has the solution y = 0.077796074

RTC
⇒ b = 0.077796074 (from Eqn. (6))
PC

a = 0.457235529
a RT f C
2
(from Eqn. (7))
PC

1− y
Also ZC = = 0.307401309 .
3
Note that we have equated a and b to TC and PC only at the critical point. Therefore these functions
could have other values away from the critical point. However, as we have equated functions of V ,
we have assumed a and b would only be functions of T. Therefore, to be completely general we
could have

a RT f α FG T IJ
a = 0.457235529
2

HT K
C
P C C

RT F T I
b = 0.077796074 βG J
HT K
C
P C C

FTI
with α G J → 1 as
T FTI
→ 1 and β G J → 1 as
T
→1.
HT K
C T C H T K T C C
In fact, Peng and Robinson (and others) have set β = 1 at all temperatures and adjusted α as a
function of temperature to give the correct vapor pressure (see chapter 5).
6
6.12 (also available as a Mathcad worksheet)

dN
= N 1 + N 2 = 0 ⇒ N 2 = − N 1
M.B.
.N .
N2
dt
Q
1
dU
.Q E.B. = N 1 H 1 + N 2 H 2 + Q = 0 ⇒ = H 2 − H1
dt N1

Also, now using the program PR1 with T = 27315


. , P = 1 bar reference state we obtain

T = 100° C T = 150° C
P = 30 bar P = 20 bar
Z = 0.9032 Z = 0.9583
V = 0.9340 × 10−3 m3 mol V = 01686
. × 10−2 m3 mol
H = 3609.72 J mol H = 6796.06 J mol
S = −1584
. J mol K S = −4.68 J mol
Q
= 6796.06 − 3609.72 = 3186.34 J mol
N
6
6.13 (also available as a Mathcad worksheet)

Since process is adiabatic and reversible ∆S = 0 or Si = S f , i.e.,


S (310 K, 14 bar ) = S (T = ?, 345 bar ) . Using the program PR1 with the T = 27315
. K and P = 1
bar reference state we obtain T = 310 K , P = 14 bar , Z = 0.9733 , V = 01792
. × 10−2 m3 mol ,
H = 10908.3 J mol and S = 1575
. J mol K .
By trial and error (knowing P and S , guessing T) we obtain T = 34191 . K , P = 345 bar ,
Z = 0.9717 , V = 08007
. × 10−4 m3 mol , H = 18860.9 J mol , S = 1575
. J mol K
⇒ Tf = 34191
. K.
System = contents of compressor
dN
M.B.: = 0 = N 1 + N 2 ⇒ N 2 = − N 1
dt
volume of
compressor
constant
adiabatic
dU 0 dV 0
E.B.: = 0 = N 1 H 1 + N 2 H 2 + Q +Ws − P
dt dt
W 
WS = − N 1 H 1 + N 2 H 2 or S = H 2 − H 1 = 18860.9 − 10908.3 = 7952.6 J mol
N
6
FG P + a IJ (V − b) = RT ⇒ PV = V − a
6.14 (a)
H VK 2
RT V − b RTV
PV
= lim G
F V − a IJ = 1
i) lim
P→0
V →∞
RT V →∞ H V − b RTV K
ii) B = lim V F
PV I RV a U
P→0
V →∞
H RT − 1K = lim V STV − b − RTV − 1VW
V →∞

= lim V S
RV − (V − b) − a UV = lim RS bV − a UV = b − a
V →∞ T (V − b) RTV W T(V − b) RT W RT
V →∞

iii) C = lim V G
F PV − 1 − B IJ = lim V RSbV − b(V − b) UV = lim b V = b
2

H RT V K
2 2
P→0
V →∞
V →∞ T V −b W V −b V →∞

⇒ C = b2
F PV − 1I = 0 ⇒ B = 0
(b) At the Boyle temperature: lim V
P→0 H RT K
a a 9V c RTc V
0=b− , TB = but a = , b = c (Eqns. 4.6-3a)
RTB Rb 8 3
9 8V c RTc 27
TB = = Tc = 3375
. Tc
RVc 3 8
6
6.15 (a) From table: TB ~ 320 K , i.e., B TB = B(320 K) = 0a f
The inversion temperature is the temperature at which

FG ∂ T IJ =0=−
1 LM
V −T
∂V FG IJ OP
H ∂ PK H CP N ∂T H KQ P

FG ∂ T IJ =
∂ LM RT + BOP = R + dB
H ∂ PK P ∂T P N P Q P dT
V − TG
F ∂ V IJ =
RT
+B−
RT
−T
dB
= B−T
dB
H∂ TK P P P dT dT

dB
Thus, T inv is the temperature at which B − T =0.
dt
Plot up B vs. T, obtain dB dT either graphically, or numerically from the tabular data. I find
T inv ~ 600 K . Also, dB dT decreases with increasing temperature (i.e.,
3 3
dB dT ~ 4.56 cm mol K at 87.5 K and 0.027 cm mol K at 650 K. Presumably it is negative
at even higher temperature!)
(b) Generally

µ=
FG ∂ T IJ =−
1 RS
V −T
∂V FG IJ UV = − 1 RSB − T dB UV
H ∂ PK H CP T ∂T H K W C T dT W
P P

dB
Using the data in the table it is easy to show that for T < T inv , B − T < 0 ⇒ µ > 0 , while for
dT
dB
T > T inv , B − T > 0⇒ µ < 0.
dT
(c) Since Fig. 2.4-3 for nitrogen is an H-P plot is easiest to proceed as follows

FG ∂ T IJ = −1 −a∂ H ∂ Pf
=
H ∂ P K a∂ H ∂ T f a∂ P ∂ H f a∂ H ∂ T f
T

H P T P

Since a∂ H ∂ T f = C is > 0 and less than ∞ [Except at a phase transition—see Chap. 6 and
P P
Problem 6.1—however, µ has no meaning in the two-phase region], if adT dPf is to be zero, H
then ad H dPf must equal zero. That is, an inversion point occurs when isotherms are parallel
T
to lines of constant H (vertical line). This occurs at low pressures (ideal gas region) and at high
pressures (nonideal gas region). See, for example, T = −200° C isotherm near 30 MPa (which
is off the figure).
To identify the inversion temperatures of nitrogen we can use Fig. 2.4-2b, a temperature-
entropy diagram. From part a of this problem we note that at T inv
V = T inv
FG ∂ V IJ ⇒ P = −T inv
FG ∂ P IJ
H∂ TK P
H∂TK P

Thus at each inversion temperatures T inv we can find a density (or pressure) for which this
equation is satisfied. Unfortunately, it is difficult to read the figure.
6
0.03 m3
6.16 18 kg ; V = = 1667
. × 10−3 m3 kg . Using Fig. 3.3-2 we find P ≈ 91 bar
18 kg
. K and V = 1667
Using the program PR1, with T = 42315 . × 10−3 m3 kg , we find, by trial-and-error
that P = 108.0 bar .
6
6.17 Using the program PR1 we find at 300°C and 35 bar; Z = 0.6853 ; V = 0.9330 × 10−3 m3 mol ;
H = 21,033 J mol = 21033 . kJ mol and S = 7.06 J mol K .
To use the principle of corresponding states we will assume the state of T = 16° C and P = 01 . bar
is an ideal gas state (i.e., don’t need corrections for nonideality at this condition). At 300°C and 35
bar we have

300 + 27315
.
Tr = = 10302
.
2832. + 27315
.
35
Pr = = 0.76754
456
.

d i
We find Z = 0.71 ; H IG − H TC = 8.37 J mol ; S IG − S = 7113
. J mol K .
From Appendix A.II

CP* = 22.243 + 0.05977T − 3499


. × 10−5 T 2 + 7.464 × 10−9 T 3

z
57315
.
∆ H IG = CP*dT = 11910 J mol
289.15

z
57315
.
CP* F 35 bar I = −20.386 J mol K
∆ S IG =
289.15
T
dT − R ln
H 0.1 bar K
Thus

H (T = 300° C, 35 bar )

= H (16° C, 0.1 bar ) + ∆ H IG + TC


FG H − H IJ
IG

H T KC 300°C, 35 bar
= 0 + 11910 − (2832 . )(8.37) = 7253 J mol
. + 27315
S (T = 300° C, 35 bar ) = 0 − 20.386 − 7113
. = 27.499 J mol K

0.71 × 8.314 × 10−5 × 57315


.
Finally PV = ZRT ; V = = 0.9666 × 10−3 m3 mol .
35
6
6.18 Equation of state P(V − b) = RT
FG ∂ P IJ =
R P
= ;
∂V FG IJ =
R V −b
=
∂P FG IJ =−
P
(a)
H∂TK V V −b T ∂T H K P P T
; and
∂V H K T V −b
Thus

CP = CV + T
FG ∂ V IJ FG ∂ P IJ = CV + T
R P
⋅ = CV + R
H∂ TK H∂ TK P V P T

for CP ( P, T ) = CP* (T ) , we must have that

FG ∂ C IJ = −T
FG ∂ V IJ = 0
2

H∂PK H∂T K
P
2
T P

FG ∂ V IJ
2
=
∂ F∂V I
G J =
∂ R
= 0 ⇒ CP (T , P) = CP* (T )
H∂T K 2
P
∂ T H∂ TK
P ∂T P P P

Similarly, for CV (V , T ) = CV* (T ) , we must have that


FG ∂ C IJ =T
FG ∂ P IJ
2
= 0.
H ∂V K H∂T K
V
2
T V

FG ∂ P IJ
2
=
∂ ∂P FG IJ =
∂ R
a f
= 0 ⇒ CV T , V = CV* (T )
H∂T K 2
V
∂TV ∂T H K V ∂ T V V −b

(b) First case is clearly a Joule-Thomson expansion ⇒ H = constant

FG ∂ T IJ 1 LM ∂V FG IJ OP = − 1 L RT + b − RT O = − b
H ∂ PK H
=−
CP
V −T
N ∂T H K Q C NM P P P QP
P C P

Since CP is independent of P, integration can be done easily

z
T2
CP (T )dT = −b P2 − P1 a f
T1

to proceed, we need to know how C p depends on T. If C p is independent of T we have

T2 = T1 −
b
CP
a
P2 − P1 f (1)
Eqn. (1) also holds if CP is a function of T, but then it is the average heat capacity over the
temperature interval which appears in Eqn. (1).
The second expansion is at constant entropy (key words are reversible and adiabatic)

FG ∂ T IJ =−
a∂ S ∂ Pf =+
a∂ V ∂ T f =
T R
z
T
2

⇒ CP
dT
=R z
P
2
dP
H ∂ PK a∂ S ∂ T f
T P

S P CP T CP P T
T P
P
1 1

If CP is independent of T, then

T2 = T1
FG P IJ R CP

HPK
2
; (2)
1

more complicated expression arises if CP = CP (T ) .


6
6.19 (also available as a Mathcad worksheet)
General:
mass balance: N1i = N1f + N 2f (1)
energy balance: N1i U 1i = N1f U 1f + N 2f U 2f (2)
state variable constraints: T1 f = T2f = T f ; P1 f = P2f = P f ; V1 = V2 ⇒ U 1f = U 2f
(a) Ideal gas solution
Eqns. of state: PV = NRT ; U = CVT − CPTR
P1i 2 P f
From M.B. get: = f
T1i Tf
1 i
From E.B. get: T1i = T f ⇒ T1i = T f = 20° C ; P f = P1 = 250 bar .
2
(b) Corresponding states solution

PV = ZNRT or PV = ZRT
d i d
U (T ) = U (T ) + U − U IG (T ) = U IG (T ) + H − H IG
IG
i
T
− PV + ( PV )IG

= U IG (T ) + d H − H i + (1 − Z )( PV )
IG IG

= U IG (T ) + d H − H i + (1 − Z ) RT
IG

From the mass balance

P1i 500 × 107 Pf Pf


= = 1398
. × 105
= 2 × ; or = 6.990 × 104
Z i T1i (20 + 27315
. ) × 122
. ZfTf ZfTf

where we have used

FG
Z i = Z Tr =
20 + 27315
.
= 1538
. , Pr =
5 × 107
= 10.77 = 122
IJ
H 190.7 4.64 × 106
.
K
From energy balance

c h c
N1i U 1i = N1f U 1f + N 2f U 2f = N1f + N 2f U f = N1f + N 2f U i ⇒ U f = U i h
where we have used the fact that U 1f = U 2f . Since T1 f = T2f and P1 f = P2f . But

b g b g c
U 1f − U 1i = 0 = U IG T f − U IG T i + H − H IG h
T f ,P f
c
− H − H IG h
T i , Pi

+(1 − Z ) RT T f , P f − (1 − Z ) RT T i , Pi

and
c h c h c h
U IG T f − U IG T i = Cv T f − T i = 27.25 J mol K T f − 29315
. K c h
dH − H i IG
T i , Pi
= −18.0 × 190.7 = −3432.6 J mol
(1 − Z ) RT T i , P i = −0.22 × 8.314 × 29315
. = −536.2 J mol
. {−27.25 + 0.22 × 8.314} + 18.0 × 190.7 + H − H IG
⇒ 0 = 27.25Tf + 29315 d i
T f ,P f
+ (1 − Z ) RT T f , P f

d
27.25Tf + H − H IG i
T f ,P f
+ (1 − Z ) RT T f , P f = 4,021 (1)

To be solved along with

Pf
= 6.99 × 104 (2)
TfZf

I found the solution by making a guess for T f , using eqn. (2) and Fig. 6.6-3 to find P f , by trial
and error. Then, guessed T f and computed P f were tested in eqn. (1). Solution found:
T f ~ 237 K = −3615
. ° C ; P f ~ 1011
. × 107 Pa = 1011. bar .
(c) The van der Waals gas
1
We know that: U i = U f and V i = V f . To evaluate the final temperature we start from
2

F ∂ T IJ
dT = G
F ∂ T IJ
dV + G dU ⇒ T − T i along = z FGH
V f
∂T IJ
H∂V K H∂U K K
f
dV
U V
path of
const. U Vi
∂V U

but

FG ∂ T IJ =−
a∂ U ∂ V f =−
a
T ∂ P ∂T f
V +P
=−
a
H∂V K a∂ U ∂ T f
T

U V CV CVV 2

Now, by Eqn. (6.2-36)

FG ∂ C IJ = T FG ∂ P IJ for vdW eos P = RT − a


2

H ∂V K H ∂ T K
V

T V −b V
2
V
2

FG ∂ P IJ = R ; FG ∂ P IJ = 0 ⇒ C is independent of volume ⇒ C
2
= CV*
H ∂ T K V −b H ∂ T K
V
2
V
V V

but CV* = CP* − R = 35565


. − 8.314 = 27.25 J mol K

⇒T −T = −f i
V

z
f
a
dV = −
a
z
V f
dV
=+
a 1 1
− i
FG IJ
Vi
CVV 2
CV Vi
V 2
CV V f
V H K
a 1
=− since V i = V f .
2CVV i 2
Thus the first step is to find V i .

F a I
GG P + JJ dV − bi = RT
i

H dV i K
i
i 2

⇒ V i = 6.678 × 10−5 m3 mol ; V f = 1336


. × 10−4 m3 mol .
−0.2283 Pa ⋅ m6 mol2
T f − Ti = = −62.73 K
a f c
2 × 27.25 J mol K × 6.678 × 10−5 m3 mol h
⇒ T f = 29315
. − 62.73 = 230.42 K = −42.73° C
RT f a 8.314 × 230.42 0.2283
Pf = − = −
d
V −b
f
Vf i c h 2
.
1336 −4
× 10 − 4.269 × 10 −5
c.
1336 × 10−4 h
2

Pf = 8.286 × 106 Pa = 82.86 bar

(d) Here we will use the program PR1. Using the 273.15 K and 1 bar reference state we find that at
the initial conditions Z = 11005
. , V i = 05365
. × 10−4 m3 mol , H i = −328185 . J mol and
S i = −5912
. J mol K . Therefore

U i = H i − PiV i = −328185
. − 500 × 05365
. × 10−4 × 105 = −5964.35 J mol

Now since U f = U i = −5964.35 J mol and


V f = 2V i = 1073
. × 10−4 m3 mol .
We must, by trial-and-error, find the temperature and pressure of the state having these
properties. I find the following as the solution Tf = 230.9 K ; P ~ 99 bar (for which
V = 01075
. m3 mol and U = −5968.4 J mol ). To summarize, we have the following answers
for the different parts of the problem:

Pf Tf
Ideal gas 250 bar 293.15 K
Corresponding states 101.1 bar 237 K
van der Waals 82.86 bar 230.42 K
Peng-Robinson 99 bar 230.9 K

Once again, the ideal gas solution is seriously in error.


6
6.20 Mass balance (system = both tanks): N1i = N1f + N 2f
energy balance (system = both tanks): N1i U 1i = N1f U 1f + N 2f U 2f
entropy balance (system = portion of initial contents of tank 1, also in there finally): S 1i = S 1f
V1 V V
Also, P1 f = P2f = P f ; N1i = i
; N1f = 1f and N 2f = 2f
V1 V1 V2
P1i P1 f P2f
(a) Ideal gas solution: obtain = + from mass balance and
T1i T1 f T2f

P1i = P1 f + P2f = 2 P f ⇒ P f = 250 bar = 2.5 × 107 Pa from energy balance


FG P IJf R CP
F 1I 8.314 35.565
T1 f = T1i
HPK
1

1
i
⇒ T1 f = (20 + 27315
. )
H 2K = 249.3 K

= −239
. ° C from entropy balance

and

1 2 1
f
= i − f ⇒ T2f = 3559
. K = 82.7° C
T2 T1 T1

Also

N1f P1 f V1 RT1i 2.5 × 107 29315


.
= ⋅ i = ⋅ = 0588
.
N1i f
RT1 P1 V1 249.3 500. × 107

and

N 2f
= 1−
FG IJ = 0.412
N1f
N1 i
H K
N1i

(b) Corresponding States Solution:


29315
. 5 × 107
Initial conditions Tr = = 1538
. ; Pr = = 10.77 ; ⇒ Z = 122
. ;
190.7 4.64 × 106
H IG − H
= 18.0 J mol K ; S IG − S = 9.6 J mol K .
TC
Mass balance:

P1i
= Pf
1 RS
+
1 UV = 50. × 10 = 1398
7
× 10 5
(1)
Z1i T1i T
Z1f T1 f Z2f T2f W 122
. × 29315
.
.
Entropy balance:

d
S 1f − S 1i = 0 = S 1 − S 1IG i + dS
f IG, f
1 i d
− S 1IG, i − S 1 − S 1IG i
i

or

T1 f Pf
dS − S i
1
IG f
1 + CP ln
.
29315
− R ln
. × 107
50
= −9.6 (2)

Energy balance:

d
N1i U 1i = N1f U 1f + N 2f U 2f but N1i = N1f + N 2f ⇒ N1f U 1f − U 1i + N 2f U 2f − U i2 = 0 i d i
or

P1 f V1 P fV
f
Z1 RT1 f nd i d is nd
H 1f − P1 f V 1f − H 1i − P1iV 1i + f2 2 f H 2f − P2f V 2f − H 1i − P1iV 1i = 0
Z2 RT2
i d is
1
Z1f T1 f
nd i d i d i
H 1f − H 1f , IG + H 1f , IG − H 1i , IG − H 1i − H 1i , IG − Z1f RT1 f + Z1i RT1i s
1
nd i d i d
+ f f H 2f − H 2f , IG + H 2f , IG − H 1i , IG − H 1i − H 1i , IG − Z2f RT2f + Z1i RT1i = 0
Z2 T2
i s
Substituting in the known values gives

. cT − 29315
ndH − H i + 35565 . h − 8.314 Z T + 6,4065
.s
1 f f , IG f f f
1 1 1 1 1
Z1f T1 f

. cT − 29315
ndH − H i + 35565 . h − 8.314Z T + 6,4065. s = 0 (3)
1
+ f f
f
2
f , IG
2 2
f f f
2 2
Z T
2 2

Eqns. (1-3) now must be solved. One possible procedure is


i) Guess P f
ii) Use Eqn. (2) to find T1 f
iii) Use Eqn. (1) to find T2f
iv) Use Eqn. (3), together with T1 f and T2f to see if guessed P f is correct. If not, go back to
step i.
After many iterations, I found the following solution P f = 97.87 bar ; T1 f = 2216 . K;
T2f = 259.4 K ; N1f N1i = 0.645 ; N 2f N1i = 0.355 .
(c) Peng-Robinson equation of state
Here we use the equations

N1i = N1f + N 2f (4)


N1i U 1i = N1f U 1f + N 2f U 2f with U = H − PV (5)
S 1i = S f (6)
P1 =
f
P2f =P f
and N1i = V1 V 1i ; N1f = V1 V 1f ; N 2f = V2 V 2f = V1 V 2f since V1 = V2 (value of V1 cancels out
of problem, so any convenient value may be used). Procedure I used to solve problem was as
c h
follows. From PR1 we know V 1i ⇒ N1i and S 1i given initial conditions. Then
1. Guess value of T1 f , find P1 f = P f that satisfies S 1f = S 1i
2. Use T1 f , P f and V 1f to get N1f ; then N 2f = N1i − N1f so V 2f is known.
3. From P f and V 2f find (trial-and-error with PR1) T2f
4. See if eqn. (5) energy balance is satisfied; if not go back to step 1. After a number of
iterations I find . bar ; T1 f = 222.3 K ; T2f = 2555
Pf = 1036 . K; N1f N1i = 0.619 ;
N 2f N1i = 0.381 .
Summary
ideal gas Corresponding P-R E.O.S.
(part a) states (part b) (part c)
Pf 250 bar 97.87 103.6
T1 f 249.3 K 221.6 K 222.3 K
T2f 355.9 K 259.4 K 255.5 K
N1f N1i 0.588 0.645 0.619
N 2f N1i 0.412 0.355 0.381

Clearly, the ideal gas assumption is seriously in error!


6
6.21 System = contents of compressor. This is a steady-state, open constant volume system.
dN
mass balance: = 0 = N 1 + N 2
dt
dU dV 0
energy balance: = 0 = N 1 H 1 + N 2 H 2 + Q + Ws − P
dt dt
 a
⇒ 0 = N1 H 1 − H 2 + Q + Ws f
 

Q Q
entropy balance:
dS
dt
.
= 0 = N 1 S 1 + N 2 S 2 + + Sgen = N 1 S 1 − S 2 +
T T
a f
0
Thus,

Q = −TN 1 S 1 − S 2a f
Q
N
= Q = T S 2 − S1 a f
1

and

WS + Q
= W + Q = H 2 − H1
N 1
(a) Corresponding states solution

a f nd i d i d
Q = T S 2 − S 1 = T S 2 − S 2IG + S 2IG − S 1IG − S 1 − S 1IG is
R IG PU
= T Sd S − S i − d S − S i − R ln V IG 2
T 2 2 1
PW 1
1

37315
. 1 50
Now Tr = = 0.92 ; Pr , 1 = ~ 0.009 ; Pr , 2 = = 0.443 . Thus
4056. 112.8 112.8

Q = − RT ln
50 RSd
+ T S 2 − S IG i d
− S 1 − S 1IG i UV
1 T 2 Pr ,2 = 0.444
Tr = 0.92
Pr ,1 = 0.009
Tr = 0.92 W
= −8.314 × 37315 . (−523
. × ln 50 + 37315 . − 0) = −14,0881
. J mol

and

W + Q = H 2 − H 1 = ( H 2 − H 2IG ) + ( H IG
2 − H 1IG ) − ( H 1 − H 1IG )
0 since
T = constant

Thus
W + Q = TC
|RSdH 2 − H 2IG i − d H − H i |UV = 4056. × (−6.28 + 0)
1
IG
1

|T TC T C |W
= −2547.2 J mol
W = −Q − 2547.2 = 11540
, .9 J mol

(b) Clausius gas

P(V − b) = RT ; V =
RT
+b ;
∂V FG IJ =
R
P ∂T H K P P

Thus

z FGH ∂∂ SP IJK
z FGH ∂∂ VT IJK dP = −R z P1 dP = −R ln PP
P2 P2 P2
∆S = dP = − 2

P1 T P1 P P1 1

Q = T∆S = − RT lnF I = −12,1365


50
H 1K . J mol

F ∂ H IJ dP = z LMV − T FG ∂ V IJ OPdP = z L RT + b − RT OdP


∆H = z G
P2 P2 P2

H ∂ PK
P1 N H ∂ T K Q NM P
T P1 P P QP P1

= z bdP = ba P − P f = 182.8 J mol


P2

2 1
P1

So W + Q = 182.8 J mol and

W = −Q + 182.8 = 12,1365
. + 182.8 = 12,319.3 J mol

(c) Peng-Robinson equation of state


Using the program PR1 we find (for T = 27315
. and P = 1 bar ideal gas reference state) that

100°C, 1 bar 100°C, 50 bar


V 0.3089 × 10−1 3
m mol 0.4598 × 10−3
H 3619.67 J/mol 1139.65
S 11.32 J/mol K –25.94

Note, from PR1, the vapor pressure of NH 3 at 100°C is 62.58 bar. Therefore, use vapor
solution to P-R equation.
a f
Then Q = T S 2 − S 1 = −13,9036
. J mol

W + Q = H 2 − H 1 = 1139.65 − 3619.67 = −2480.0 J mol

and W = −Q − 2480.0 = 11,4236


. J mol .
6
6.22 (also available as a Mathcad)

Considering the gas that is in the tank finally as the system, this is a closed system undergoing a
reversible, adiabatic expansion. Therefore S i = S f .

(a) d S =
CP
dT −
FG IJ dP but with P(V − b) = RT or V = RT + b . Then FG ∂ V IJ
∂V
=
R
T H K
∂T P P H∂ TK P P
; also

FG ∂ C IJ = −T FG ∂ V IJ = −T ∂ R = 0 ⇒ C is independent of pressure.
2

H ∂ P K H∂T K
P
Therefore
T ∂T P
2
P P
P

CP = CP* . Thus

z z
Tf Pf
CP* 1
0 = ∆S = dT − R dP
Ti
T P
P
i
(1)
This has the solution Pf = 1310
. bar . Now to find the initial and final molar volumes we use
RT
V= +b
P
V i = 0.000709 m3 mol
V f = 0.00197 m3 mol

Nf Vi
So that = = 0.3595 (or 35.95%)
Ni Vf
(b) Corresponding states

d
0 = S f − S i = S f − S IG
f + S f − Si
IG IG
i d
− S i − S iIG i d i
Initial state

400
Tr = = 13149
. S IG − S = 0.49
304.2 ⇒
= 0.67787 Z = 0.906
50
Pr =
7376
.

df − Si
S IG IG
i
= −12.4009 − 8.314 ln
Pf
50

(As given by eqn. (1) above. Why?)


Guess for final state (use P from part a), then iterate. Final solution is Pf = 1152
. bar for which
Pr = 0156
. , S IG − S ≅ 0.29 and Z f = 0.939 .
Nf
=
c P Z RT h = P
f f f f

Zi Ti 1152
=
.
×
400 0.906
× = 0.2964 (or 29.64% )
Ni a P Z RT f Z T
i i i f f Pi 50 300 0.939

(c) Peng-Robinson equation of state


Use program PR1 with given heat capacity constants to find a pressure at 300 K which has the
same entropy as the state T = 400 K , P = 50 bar . By trial-and-error we find that
P = 1337
. bar (somewhat higher than the previous cases). Also, V i = 05982. × 10−3 m3 mol
and

Nf Vi
. × 10−2 m3 mol ⇒
V f = 0175 = = 0.3416 (or 34.16%)
Ni Vf
6
6.23 There are two obvious ways to proceed.
1) retain T and P as the independent variables since we have a program, PR1 that calculates
V (T , P) , H (T , P) and S (T , P) . We can then use
U (T , P) = H (T , P) − PV = H (T , P) − ZRT (where Z = Z (T , P) ) (1)
G(T , P) = H (T , P) − TS (T , P) (2)
and
A(T , P) = G − PV = U − TS = H − PV − TS = H − ZRT − TS (3)
Now we will write
H (T , P) = H IG (T ) + RT ( Z − 1) +
T da dT − a
X (4)
a f
2 2b
and
S (T , P) = S IG (T , P) + R ln(Z − B) +
T da dT
X (5)
a f
2 2b
where for convenience, I have used

X = ln
LM d
Z + 1+ 2 B i OP
MN d
Z + 1− 2 B i PQ
Then we find

U (T , P) = U IG (T ) +
a
T da dT − a f
X where U IG = H IG − RT (6)
2 2b
a
G(T , P) = G IG (T , P) + RT (Z − 1) − ln(Z − B) − X (7)
2 2b

and
a
A(T , P) = AIG (T , P) − RT ln( Z − B) −
X (8)
2 2b
Thus we can either use eqns. (1 to 3) and previously calculated values Z, H and S , or modify
PR1 to use Eqns. (6-8) instead of Eqns. (4 and 5).
2) The second alternative is to take T and V as the independent variables and start from

LM F ∂ P I OP ∂PFG IJ
MN GH ∂ T JK
CV
dU = CVdT + T − P dV and d S = dT +
V PQ T ∂T H K V
dV

to get
LMF ∂ P I OP
z
T, V

MNGH ∂ T JK
R
S − S IG = −
PQ
dV
T , V =∞ V V
and
LMT F ∂ P I OP
z
T, V

MN GH ∂ T JK
U − U IG = − P dV
T , V =∞ V PQ
Then put in the Peng-Robinson equation of state and from build up a procedure to calculate S ,
H , U , P, A and G with T and V as the independent variables. We will not follow this alternative
further.
6
6.24 We will do these calculations using

G = H − T S and A = G − PV = H − PV − TS

As an example, consider the T = 0° C isotherm P = 1 bar , H = −74214


. J mol
S = −2.59 J mol K
G = −74214 . × (−2.59) = −34.68 J mol ,
. − 27315
V = 22.6800 m3 kmol
A = −34.68 − 1 bar × 0.02268 m3 mol × 105 J = −2302.7 J mol

P = 5 bar H = −786.05 J mol


G = 3608.9 J mol
S = −16.09 J mol K ⇒
A = 1352.4 J mol
V = 0.004513 m3 mol
P = 10 bar H = −840.75 J mol
G = 51658
. J mol
S = −2199
. J mol K ⇒
A = 29238
. J mol
V = 0.002242 m3 mol
P = 20 bar H = −949.56 J mol
G = 67150
. J mol
S = −28.06 J mol K ⇒
A = 45010
. J mol
V = 0.001107 m3 mol
P = 40 bar H = −116397
. J mol
G = 8232.4 J mol
S = −34.40 J mol K ⇒
A = 60688
. J mol
V = 0.0005409 m3 mol
P = 60 bar H = −1372.64 J mol
G = 9099.9 J mol
S = −38.34 J mol K ⇒
A = 6978.9 J mol
V = 0.0003532 m3 mol
P = 80 bar H = −157376
. J mol
G = 9704.6 J mol
S = −4129
. J mol K ⇒
A = 76230
. J mol
V = 0.0002602 m3 mol
P = 100 bar H = −176561
. J mol
G = 10,168.3 J mol
S = −4369
. J mol K ⇒
A = 8116.3 J mol
V = 0.0002052 m3 mol
Similarly, G and A at other points could be computed, though this will not be done here.
6
1
6.25 (a) ρ=
V

∂ρ=∂
FG 1 IJ = −V −2
∂V ⇒
FG ∂ P IJ = −V 2
FG ∂ P IJ =+
a
V 2 ∂ S ∂V P f
HV K H∂ ρK S
H ∂V K S a
∂S ∂PV f
by eqn. (6.1-6a)

Now

dS =
CV
dT +
FG IJ dV ⇒ FG ∂ S IJ = C FG ∂ T IJ
∂P
H K H ∂ PK T H ∂ PK
V
T ∂T V V V

dS =
C F ∂ V IJ dP ⇒ FG ∂ S IJ = C FG ∂ T IJ
dT + G
H∂ TK H∂V K T H∂V K
P P
T P P P

⇒G
F ∂ P IJ = V C FG ∂ T IJ ⋅ T FG ∂ P IJ = V γ FG ∂ T IJ FG ∂ P IJ = −γ V 2
FG ∂ P IJ
H ∂ ρK
2
T H∂V K C H ∂ T K
2
H∂V K H ∂ T K H ∂V K
P

S P V V P V T
(by eqn. (4.1- 6a))

Thus vS = −γ V 2
FG ∂ P IJ = γV 2
FG ∂ T IJ FG ∂ P IJ
H ∂V K T
H∂V K H ∂ T K
P V
.

CP C +R R
(b) γ = for the ideal gas CP = CV + R ⇒ γ = V = 1+
CV CV CV
For the Clausius Gas

γ =
CP CV + T ∂ V ∂ T
=
a f a∂ P ∂ T f
P V
= 1+
T ∂VFG IJ FG ∂ P IJ
CV CV H K H∂TK
CV ∂ T P V

with P(V − b) = RT

FG ∂ V IJ =
R V −b
=
∂P FG IJ =
R
H∂ TK P P T
and
∂T H K V V −b

Thus

T V −b R R
γ = 1+ ⋅ ⋅ = 1+
CV T V − b CV

To show that Cv ≠ Cv (V ) we start from eqn. (6.2-35)


FG ∂ C IJ =T
FG ∂ P IJ but FG ∂ P IJ = R ; FG ∂ P IJ = 0 for ideal gas
2 2

H ∂V K H∂T K H∂TK V H∂T K


V
2 2
T V V V

and G
F ∂ P IJ = R ; FG ∂ P IJ = 0 for Clausius gas]
2

H ∂ T K V −b H ∂ T K
V
2
V

F ∂ C IJ = 0 for the ideal and Clausius Gases


⇒G
H ∂V K
V

T P
(c) vS (ideal gas) = γ V 2 = γPV = γRT
V T
T R V
vS (Clausius gas) = γ V 2 = γRT
V −b V −b V −b
V
= vS (ideal gas)
V −b at same T and V
6
6.26 Preliminaries
Pressure = outward force per unit area exerted by gas
Force = tensile force exerted on fiber — at mechanical equilibrium fiber exerts an equal and opposite
inward force
⇒ In all thermodynamic relations replace P by − F A and V by LA, and they will be applicable to
fiber.

In particular, in place of S = S (T ,V ) and dS =


∂S
dT +
∂S FG IJ
dV . We will use S = S T , L
FG IJ a f
∂T V ∂V T H K H K
and dS =
FG ∂ S IJ dT +
FG ∂ S IJ
H∂ TK L
H ∂ LK T
dL . Also

FG ∂ S IJ = C ⇒ FG ∂ S IJ = C
H∂ TK T H∂ TK T
V L

V L

FG ∂ S IJ = FG ∂ P IJ ⇒ FG ∂ S IJ = −FG ∂ F IJ
H ∂ T K H ∂ T K H ∂ LK H ∂ T K
T V T L
1 P 1 F
dS = dU + dV ⇒ dS = dU = dL
T T T T

(a) From the above dS =


FG ∂ S IJ dT +
FG ∂ S IJ dL =
CL
dT +
∂SFG IJ
H∂ TK L
H ∂ LK T T ∂L H K T
dL and the analog of the

Maxwell relation

FG ∂ S IJ = FG ∂ P IJ ⇒
FG IJ
1 ∂S
=−
1 ∂F FG IJ
H∂V K H ∂ T K T V A ∂LH K T A ∂T H K L
we get

dS =
CL
dT −
∂F FG IJ dL =
CL
dT − γ L − L0 dL a f
T ∂T H K L T

(b) dU = TdS − PdV ⇒ dU = TdS + FdL


a f
= CLdT − γT L − L0 dL + γT L − L0 dL = CLdT a f
(Note: This is analog of ideal gas expression U = U (T ) or dU = Cv dT )
C
(c) dS = L dT − γ L − L0 dL
T
a f
a f z RST a f UVW
L ,T
CL
⇒ S ( L, T ) − S L0 , T0 = dT − γ L − L0 dL
L ,T
T
a f a f
0 0

Choosing the path L0 , T0 → L0 , T → ( L, T ) yields


z α + βT
za
T , L0
a f f
T,L
S ( L, T ) − S L0 , T0 = dT − γ L − L0 dL
T0 , L0
T T , L0

γ
= α ln
T
T0
a
+ β T − T0 − L − L0
2
f a f
2

h a f
(d) A reversible (slow), adiabatic expansion ⇒ S L f , Tf − S Li , Ti = 0 c
0 = mS c L , T h − S a L , T fr − kS a L , T f − S a L , T fp
f f 0 0 i i 0 0

FT I f γ
= α lnG J + β cT − T h − c L − L h − a L − L f
2 2
HTK i 2
f i f 0 i 0

Need to solve this transcendental equation to find Tf .

(e) dU = CLdT ⇒
FG ∂ U IJ = 0 ⇒ FU = 0
H ∂ LK T

FS = −T
FG ∂ S IJ k a
= −T −γ L − L0 fp = γTa L − L f = −T FGH ∂∂ FT IJK
H ∂ LK T
0
L
6
6.27 (a)

dU = TdS − PdV + GdN ⇒


FG ∂ U IJ =T
H∂SK V ,N

FG ∂ U IJ = − P ;
;
H ∂V K S,N
and

FG ∂ U IJ = G
H∂ NK S ,V
Now equating mixed second
derivatives

∂ FG ∂ U IJ =
∂ ∂UFG IJ ⇒
FG ∂ T IJ =−
FG ∂ P IJ
∂V S, N
H∂SK V ,N ∂ S V ,N ∂ VH K S,N
H∂V K S, N
H∂SK V, N
(1)

∂ FG ∂ U IJ =
∂ FG ∂ U IJ ⇒G
F ∂ T IJ = −G
F ∂ G IJ
∂N S ,V
H∂SK V ,N ∂ S V ,N H∂ NK S ,V
H∂ NK S ,V
H∂SK V,N
(2)

and

∂ FG ∂ U IJ ∂ FG ∂ U IJ FG ∂ P IJ F ∂ GI
∂N S ,V
H ∂V K S,N
=
∂V S,N
H∂ NK S ,V
⇒−
H∂ NK S ,V
=
H ∂V K S, N
(3)

(b), (c), and (d) are derived in


similar fashion.
Solutions to Chemical and Engineering Thermodynamics, 3e

6.27 (a)

dU = TdS − PdV + GdN ⇒


FG ∂ U IJ = T
H∂SK V ,N

F ∂ U IJ = − P ; and FG ∂ U IJ = G
; G
H ∂V K S,N
H∂ NK S ,V
Now equating mixed second derivatives

∂ FG ∂ U IJ =
∂ ∂U FG IJ ⇒
FG ∂ T IJ =−
FG ∂ P IJ
∂V S, N
H∂SK V ,N ∂ S V ,N ∂ V H K S,N
H∂V K S, N
H∂SK V, N
(1)

∂ FG ∂ U IJ =
∂ ∂U FG IJ ⇒
FG ∂ T IJ =−
FG ∂ G IJ
∂N S ,V
H∂SK V ,N ∂ S V ,N ∂ N H K S ,V
H∂ NK S ,V
H∂SK V,N
(2)

and

∂ FG ∂ U IJ ∂ FG ∂ U IJ FG ∂ P IJ F ∂ GI
∂N S ,V
H ∂V K S,N
=
∂V S,N
H∂ NK S ,V
⇒−
H∂ NK S ,V
=
H ∂V K S, N

(3)

(b), (c), and (d) are derived in similar fashion.

6.28 (also available as a Mathcad worksheet)


(a) The procedure that will be used is to first
identify the temperature at which µ = 0 ,
and then show that
µ < 0 at larger temperatures, and µ > 0 at
lower temperatures. The starting point is,
from Sec. 6.2

µ=
FG ∂ T IJ =−
V
(1 − Tα )
H ∂ PK H CP

where, from Illustration 6.2-4, for the van


der Waals gas,

TV 2a
α −1 = − (V − b) and
V − b RV 2
R| U|
µ=−
V
1− S| a 1
f V|
CP T
V V − b − 2a(V − b) RTV 2 W
Simplifying yields
Solutions to Chemical and Engineering Thermodynamics, 3e

µ=−
a f
V b V − b − 2a(V − b) RTV
2

a f
CP V V − b − 2a(V − b) RTV 2

V RTb (V − b) − 2a(V − b) V
2
=−
CP RTV (V − b) − 2a(V − b) V 2
(1)

Now for µ to be zero, either the


numerator must be zero, or the
denominator infinity. Only the former is
possible. Thus,

2a(V − b) (V − b) 2a(V − b)2


T inv = ⋅ = the desired
RV 2 b RV 2b
expression

to determine the sign of the Joule-


Thomson coefficient in the vicinty of the
inversion temperature, we will replace T
in eqn. (1) by T inv + δ , where δ may be
either positive or negative. The result is

V Rb (V − b) δ
µ=−
CP RV (V − b) δ + 2a(V − b)2 bV 2

It is easily shown that the denominator is


always positive. Thus,
µ is proportional to − δ ⇒ if T > T inv , so
that δ > 0 , µ < 0 . Alternatively, if
T < T inv , δ < 0 and µ > 0 .
9 V
(b) Using a = V C RTC and b = C
8 3

9 a
V −V C 3 27 V f
F I a3V − 1f
2 2 2
T inv = 2 ⋅ V C RTC
8 d
2
RV V C 3
=
i4
TC C
3 H K V r
2

9 F V I a3V − 1f 3a3V − 1f
2 2 2
T inv
⇒ =
H K ⋅3 = =T
C r r inv
2 2 2 r
TC 4 3 V V r 4V
C r
(2)

(c) Expression above gives Trinv = Trinv Vr ; a f


what we want is Trinv as a function of Pr .
Thus look at
Solutions to Chemical and Engineering Thermodynamics, 3e

FG P + 3 IJ a3V − 1f = 8T ⇒ P = 8T − 3
H VK
r
r r r r
r
2
3V − 1 V r r
2

(3)

Choose Vr as independent variable; use


Eqn. (2) to get Trinv , and use Eqn. (3) to
get Pr . Results are tabulated and plotted
below.

Vr Trinv T inv (K) Pr P T inv (° C)

(
b
a
r
)
0 0 9 0 0 –
. . 4 1
5 7 . 7
5 6 8
5 .
5
5
0 1 1 5 1 –
. . 8 . 9 8
6 4 3 6 0 9
2 5 . 2 . .
5 5 6 2 8 6
2
0 2 2 7 2 –
. . 6 . 7 1
7 0 2 9 0 0
5 4 . 7 . .
8 5 7 7 7
1 3 3 9 3 1
. . 7 . 0 0
0 0 8 0 5 5
. . .
6 5 4
1 3 4 8 2 1
. . 5 . 9 8
2 6 8 6 3 4
5 3 . 4 . .
1 2 9
1 4 5 8 2 2
. . 1 . 7 4
5 0 5 0 1 2
0 8 . . .
3 3 5 1
1 4 5 7 2 2
. . 5 . 4 8
7 4 8 3 9 4
5 2 . 4 . .
2 1 4 3 9
Solutions to Chemical and Engineering Thermodynamics, 3e

2 4 5 6 2 3
. . 9 . 2 1
0 6 1 7 9 8
8 . 5 . .
8 6 1 1 4

6.29 (also available as a Mathcad worksheet)


6
6.30 For an isothermal process
involving a fluid described by the
Redlich-Kwong equation of state
develop expressions for the
changes in
(a) internal energy,
(b) enthalpy, and
(c) entropy
in terms of the initial temperature
and the initial and final
volumes.
For your information, the
Redlich-Kwong equation of
state is

RT a
P= −
V −b T ⋅V ⋅ (V + b)

and

z dx 1 x F I
= ln
x ( x + c) c x+c H K
LM F ∂ P I − POPdV
MN GH ∂ T JK PQ
dU = T
V

L R + a1 2fat − RT −
= MT ⋅
a OP
N V − b T V (V + b) V − b T
32
V (V + b)
12
dV
Q
−adV
=
2 TV (V + b)

a f a f
U T ,V 2 − U T ,V 1 = −
a
z
V2
dV
=−
a
ln
LM
V 2 V1 +b OP
2 T V1
V (V + b ) 2 Tb V N
2 + b V1 Q

a f a f a f a f
H T ,V 2 − H T ,V 1 = U T ,V 2 − U T ,V 1 + P2V 2 − PV
1 1

=−
a LV aV + bf OP + RT LM V − V OP
lnM ⋅
2 Tb N V aV + bf Q
2 1 2 1

1 2 NV − b V − b Q
2 1

a L 1 1 O
− M −
T NV + b V + b Q
2
P1
dS =
FG ∂ P IJ dV = LM R + a OPdV
H∂TK V NV − b 2T V (V + b) Q 32

S aT ,V f − S aT ,V f = R ln
V −b
+
a LV aV + bf OP
lnM ⋅
V − b 2T b N V aV + bf Q
2 2 1
2 1 32
1 1 2

a f a f a f a
a f a f f
G T ,V 2 − G T ,V 1 = H T ,V 2 − TS T ,V 2 − H T ,V 1 − T S T ,V 1

=−
a LV aV + bf OP + RT LM V − V OP
lnM ⋅
2 Tb N V aV + bf Q
2 1 2 1

1 NV − b V − b Q
2 2 1

a L 1 1 O
− M −
T NV + b V + b Q
2
P 1

− RT ln
V −b

a LV aV + bf OP
ln M ⋅
V − b 2 Tb N V aV + bf Q
2 2 1

1 1 2

=−
a LV aV + bf OP + RT LM V − V OP
ln M ⋅
Tb N V aV + bf Q
2 1 2 1

1 NV − b V − b Q
2 2 1

a L 1
− M − 1 OP − RT lnFGH VV ++ bb IJK
T NV + b V + b Q
2 1
2

1
6
6.31
X
Joule-Thomson Expansion
P1 = 25 bar , T1 = 300° C ,
P2 = 1 bar , T2 = ?
(a) Ideal gas-enthalpy is
independent of pressure
⇒ T2 = 300° C
(b) van der Waals gas

2c
H 2 = H 1 ⇒ H2 − H IG + H IG h c
2 − H1
IG
− H 1 − H 1IG = 0 h c h
z z
fL
MMNT FGH ∂∂ TP IJK − POPPQdV + C dT − RT aZ − 1f
a
V T2 , P1 T2

⇒ 0 = RT a Z − 1f +
2 2
*
p 1 1
V =∞ V T1

z
a fL
MMNT FGH ∂∂ TP IJK − POPPQdV = 0
V T1, P1


V =∞ V

a F ∂ PI
P=
RT

V −b V H ∂ T K
; G 2 J = V R− b ; T FGH ∂∂ TP JIK − P = VRT− b − VRT− b + Va
V V
2
=
a
V2

z z z
V2 T V
a f
dV 2 V
a f
1
dV
⇒ 0 = RT2 Z2 − 1 + a 2
+ C p dT − RT1 Z1 − 1 − a 2
V =∞ V T1 V =∞ V

z z
V2 T
a
0 = RT2 Z2 − 1 + a f dV 2 V
2
+ C p dT − RT1 Z1 − 1 a f
V =∞ V T1

0 = P2V 2
F 1 − 1 IJ + z ca + bT + cT
− RT − aG
T2
2
+ dT 3 dTh
HV V K
2
2 1 T1

− PV
1 1 + RT1

FG 1 − 1 IJ + aaT − T f + b cT
0 = P2V 2 − RT2 − a 2
− T12 h
HV V K 2 2 1
2 1 2

+ cT − T h + cT − T h − PV + RT
c d
3 3 4 4
2 1 2 1 1 1 1
3 4

Solved together with vdW


EOS
T = 57507
. K, ≈ 3019
. °C
(T increases?)
(c) Peng Robinson EOS
Thermodynamic properties
relative to an ideal gas at
273.15 K and 1 bar.

H (300° C, 25 bar ) = 9.4363 × 103 J mol

After some trial and error

H (2741
. ° C, 1 bar ) = 9.4362 × 103 J / mol
Close enough
So the solution is
T=274.1oC

(d) Steam tables

H (300° C, 25 bar ) = 30088


. kJ kg

H (T = ?, 1 bar ) = 30088. kJ kg
H (T = 250° C, 1 bar ) = 2974.3
H (T = 300° C, 1 bar ) = 3074.3
⇒ T ≅ 267° C = 540 K
6

6.32 The solution is available only as


a Mathcad worksheet.
6

6.33

A( N ,V , T ) = − kT ln Q( N ,V , T ) = − kT ln
LM f (T ) V Za N V , T f OP
N N N

N N! Q
F N , TI + kT ln N !
= − NRT ln f (T ) − NkT lnV − NkT ln Z
HV K
Stirling’s approximation
ln N ! = N ln N − N

F N , TI + NkT ln N − NkT
HV K
A( N ,V , T ) = − NkT ln f (T ) − NkT lnV − kT ln Z

FG ∂ A IJ = − P = − NkT − NkT ∂ ln Za N V , T f ∂ a N V f
H ∂V K T,N
V ∂aN V f ∂V T T

NkT N kT ∂ ln Z a N V , T f
2
P= −
V V ∂aN V f
2
T

FG ∂A IJ = −S = − Nk ln f (T ) − NkT d ln f (T ) − Nk lnV − k ln Z F N , TI
H∂ TKV ,N
dT HV K
∂ ln Z a N V , T f
− NkT + Nk ln N − kT
∂T N ,V

d ln f (T ) N N F I F I
S = Nk ln f (T ) + NkT
dT
− Nk ln
V
+ k ln Z
V H K
,T
H K
+ NkT
∂ ln Z N V , T a − kT
f
∂T N ,V

G molecule =
FG ∂ A IJ = − kT ln f (T ) − kT ln V − kT
a f ∂aN V f
∂ ln Z N V , T
H∂ NK T ,V a f ∂N
∂ NV V V

a f
− NkT ln Z N V , T + kT ln N + kT − kT

G = N ⋅ G molecule = − NkT ln f (T ) − NkT ln V −


a f
NkT ∂ ln Z N V , T
V ∂ NV a f
F N , TI
+ NkT ln N − NkT ln Z
HV K
As a check
F N , TI + NkT ln N
G = A + PV = − NkT ln f (T ) − NkT ln V − NkT ln Z
HV K
− NkT + M
L NkT − NkT ∂ ln Za N V , T f OPV
N V V ∂aN V f Q 2
T

= − NkT ln f (T ) − NkT lnV − NkT ln Z F , T I + NkT ln N


N
HV K

− NkT + NkT −
a f
NkT ∂ ln z N V , T
V ∂ NV a f T

F N , TI + NkT ln N
= − NkT ln f (T ) − NkT ln V − NkT ln Z
HV K

a f
NkT ∂ ln Z N V , T
V ∂ NV a f
which checks!

∂ F AI = − U
∂ T V, HTK T
N
2

∂ LM− Nk ln f (T ) − Nk lnV − Nk ln Z F N , TI + Nk ln N − Nk OP
=
∂T N
V, N
HV K Q
L d ln f (T ) − Nk F ∂ ln Za N V , T fI OP
= M− Nk
MN GH ∂ T JK P
dT Q N, V

U = NkT 2
d ln f (T )
+ NkT 2
FG a
∂ ln Z N V , T fIJ
dT H ∂T K N, V

F dU I d ln f (T ) d ln f (T )
2
CV =
H dT K N,V
= 2 NkT
dT
+ NkT 2
dT 2

+ 2 NkT
FG ∂ ln Za N V , T fIJ + NkT 2
FG ∂ 2
a
ln Z N V , T fIJ
H ∂T K N, V H ∂T K N, V

H = U + PV

= NkT 2
d ln f (T )
+ NkT 2
FG a
∂ ln Z N V , T fIJ
dT H ∂T K N ,V

+
NkT NkT ∂ ln Z N V , T

a f
V V ∂ NV a f T

etc.
6

6.34

d H res = d H − d H IG = CPdT + V − T
LM FG ∂ V IJ OPdP − C dT
H∂ TK Q
*

N P
P

c LM FG ∂ V IJ OPdP
h
= CP − CP* dT + V − T
N H∂ TK Q P

dU res
L F ∂ P IJ − POPdV
= cC − C hdT + MT G
N H∂TK Q
*
V V
V

∂P FG IJ CV − CV* LMF ∂ P I OP
MNGH ∂ T JK
CV P
dS = dT + dV ⇒ d S res = dT + −
T ∂T H K V T V T
dV
PQ
or d S res =
CV − CV*

LMFG ∂ V IJ −
V OP
NH ∂ T K
dP
T P T Q
c h
dG res = d H res − d TS res = d H res − Td S res − S resdT

c h
= CP − Cp* dT + V − T
LM FG ∂ V IJ OPdP − cC h
− CP* dT − T
LM FG ∂ V IJ OP
− V dP
N H∂ TK Q P
P
N H∂ TK P Q
− S dT
res

dG res = S resdT

z LMMN FGH ∂P IJ OP
V
H (T , P) − H IG (T , P) = RT ( Z − 1) + − P dV
V =∞
T
∂T K V PQ
RT a
P= − 2
V −b V
FG ∂ P IJ = R
H ∂ T K V −b
V

TG
F ∂ P IJ − P = RT − RT + a =
a
H∂TK V
V −b V −b V 2
V2
H (T , P) − H IG (T , P) = RT ( Z − 1) + z
V
a 1 1
dV = RT (Z − 1) − a −
LM OP
V =∞ V 2
V ∞ N Q
a RT a P
= RT ( Z − 1) − = RT (Z − 1) − ⋅
V P V RT
aP
= RT ( Z − 1) −
ZRT

aP
H (T , P) − H IG (T ) = RT ( Z − 1) − = H res
ZRT

a f
U res = U (T , P) − U IG (T ) = H T , P − PV − H IG − PV IGd i
PV PV IG
= H res (T , P) − RT
RT
+
RT
a f
RT = H res T , P + RT (1 − Z )

aP
= H (T , P) − RT (Z − 1) =
res
ZRT

z LMMNFGH ∂P IJ OP
V
R
S res = S (T , P) − S IG (T , P) = − −
V =∞
∂T K V V PQ
dV

z LMN OP
V
R R (V − b) V
=− − dV = − R ln + R ln
V =∞
V −b V Q (V → ∞) − b (V → ∞)
V Z Pb
S res (T , P) = R ln = R ln ; B=
V −b Z−B RT
6
6.35 a) The Soave-Redlich-
Kwong equation of state is
RT a(T )
P= −
V −b V V −b a f
Rewrite this in the power
series of V

V3 −
RT 2
V + −b2 −FH
RT
b−
a(T )
V−
a(T )b
=0 IK
P P P P

Notice that the three roots of


volume at the critical point
are identical so we can write

bV − V g C
3
=0 or
V − 3V C V
3 2
+ 3V 2C V − V 3C = 0

At critical point, the second


and fourth equations must be
satisfied simultaneously.
Consequently, the
coefficients of each power of
V must be the same. Thus,

RTC
3V C =
PC

3V C2 = -b 2 -
RTC
b−
a TC a f and
PC PC

V 3C =
a TCa f
b
PC
Solving the above three
RTC
VC =
3PC

b= c 3
h
2 − 1 V C = 0.08664
RTC
PC
and

a f
a TC =
PC V 3C
= 0.42748
a f
RTC
2

b PC
Also

a f
a(T ) = a TC α (T ) = 0.42748
a RT fC
2
α (T )
PC
equations together for a(T), b and
Vc, we get
b)
PCV C PCV C RTC 1
ZC = = =
RTC RTC 3PC 3
6
6.36 (also available as a
Mathcad worksheet)
RT a(T )
P= −
V − b V (V + b)

z LMMN FGH ∂P IJ OP
V
H (T , P) − H IG (T , P) = RT (Z − 1) + − P dV
V =∞
T
∂T K V PQ
FG ∂ P IJ = R − 1 da(T )
H ∂ T K V − b V (V + b) dT
V

F ∂ P IJ − P = RT − T da(T ) − RT + a
TG
H∂TK V V − b V (V + b) dT V − b V (V + b)

=
1 LMa − T da(T ) OP
V (V + b) N dT Q

So the integral to be done is

z
V
1
dV = z
V
dV
=
1 1 ⋅V + b LM OP
V

V =∞
V (V + b) V =∞
V (V + b) (−b)
ln
V N Q
V =∞

1 V +b
= − ln
LM
1 V +b
+ ln
OP LM OP LM
1 V +b
= − ln
OP
b V bN V Q N Q
V =∞ b VN Q
So

z
LMT FG ∂ P IJ − POPdV
V

H (T , P) − H IG (T , P) = RT (Z − 1) +
MN H ∂ T K PQ
V =∞ V

a − T ada dT f F V + b I
lnG
= RT (Z − 1) −
b H V JK
T ada dT f − a L Z + a Pb RT f O
ln M
= RT (Z − 1) +
b N Z PQ
T ada dT f − a L Z + B O
ln M
= RT (Z − 1) +
b N Z PQ
LMF ∂ P I OP
z
V = ZRT P

MNGH ∂ T JK
R
S (T , P) − S IG (T , P) = R ln Z + −
PQ
dV
V =∞ V V
FG ∂ P IJ −
R
=
R

1 da R

H∂TK V V V − b V (V + b) dT V

z LM R − 1 da − R OPdV
V = ZRT P

V =∞
NV − b V (V + b) dT V Q
V −b V da 1 V + b
= R ln − R ln + ln
V − b V →∞ V V →∞ dT b V

= R ln
V − b da 1
+
Z + Pb RTLM a f OP
V dT b
ln
Z N Q
F
Z−B da 1 IZ+B LM OP
= R ln
H Z
+
dT b
ln
K Z N Q
F Z − B I + da 1 lnLM Z + B OP
S (T , P) − S IG (T , P) = R ln Z + R ln
H Z K dT b N Z Q
da 1 L Z + B O
dT b NM Z QP
= R ln(Z − B) + ln

aP Z
G res = H res − TS res = RT (Z − 1) − − RT ln
zRT Z−B

Redlich-Kwong

RT a
P= −
V −b TV (V + b)
FG ∂ P IJ = R + a1 2fa
H ∂ T K V − b T V (V + b)
V
32

F ∂ P IJ − P = RT + a1 2fa − RT +
TG
a
H∂TK VV −b TV (V + b) V − b TV (V + b)

=
a3 2fa = 3a
TV (V + b) 2 TV (V + b)
z
V
3a dV
H (T , P) − H IG (T , P) = RT ( Z − 1) +
V =∞ 2 T
V (V + b)
3a V
= RT ( Z − 1) + ln
2b T V +b
3a Z
= RT ( Z − 1) + ln
2b T Z + bP RT a f
3a Z
= RT ( Z − 1) + ln
2b T Z + B

3a Z
U (T , P) − U IG (T , P) = ln
2b T Z + B

z
LMFG ∂ P IJ − R OPdV
V

S (T , P) − S IG (T , P) = −
MNH ∂ T K V PQ
V =∞ V

zL R + a1 2fa − R OPdV
V

=− M
V =∞
NV − b T V (V + b) V Q
32

= R ln
V

a1 2fa ln V
V − b T 3 2b V + b
Z a Z
= R ln − 3 2 ln
Z − B 2T b Z + B
6
6.37
6
6.38
6
6.39
Critical properties for carbon dioxide:
6
Tc := 304.2 K Pc := 7.376⋅ 10 Pa ω := 0.225
Soave-Redlich-Kwong Equation of State and Constants:
3
joule Pa⋅ m ⎛ R2⋅ Tc2 ⎞
R := 8.31451 or R⋅ Tc
K⋅ mol ( K⋅ mol) a1 := 0.42748⋅ ⎜ b := 0.08664⋅
⎝ Pc ⎠ Pc

( )⎛
2

α ( T) := ⎢ 1 + 0.480 + 1.574⋅ ω − 0.176⋅ ω ⋅ ⎜ 1 −
2 T⎞⎤
⎥ a( T) := a1⋅ α ( T)
⎣ ⎝ Tc ⎠ ⎦

R⋅ T a ( T)
P( V, T) := −
V− b V⋅ ( V + b )

Data given in the problem:


5 5 3
T := ( 150 + 273.15) K P1 := 50⋅ 10 Pa P2 := 300⋅ 10 Pa V1total := 100 m
T
V := R⋅ (Initial guess needed for solver)
P1

Solving for the initial molar volume and the number of moles of carbon dioxide:

Given P1 P( V, T)
V1total 5
V1molar:= Find( V) N := N = 1.518 × 10 −4
V1molar V1molar = 6.587 × 10

Solving for the final molar volume and the final total volume:
( R ⋅ T)
V :=
P2
Given P2 P( V, T)
−5
V2molar:= Find( V) V2molar = 9.805 × 10
V2total := V2molar⋅ N (i) V2total = 14.885

Calculating the amount of work done to compress the gas:


V2molar
⌠ 8
Work := N⋅ ⎮ −P( V, T) dV ( ii) Work = 8.823 × 10 joule

V1molar
Since the temperature is constant, the change in enthalpy, H(T, P2) - H(T,P1), is just equal
to Hdep(T,P2) - Hdep(T,P1) :

d
a ( T) − T⋅ a ( T)
Hdep ( T , P) := R⋅ T⋅ ⎛⎜
P⋅ V
− 1⎞ + ⋅ ln⎛⎜ ⎞
dT V
⎝ R⋅ T ⎠ b ⎝ V+ b⎠
3 3
H( T , P) := Hdep ( T , P) H( T , P2) = −4.382 × 10 H( T , P1) = −7.314 × 10

Q := N⋅ [ ( H( T , P2) − H( T , P1) ) − ( P2⋅ V2molar − P1⋅ V1molar) ] − Work


8
Q = −3.837 × 10 joule
6
6.40
6
6.41
3
mol := 1 bar := 101300Pa
⋅ joule bar⋅ m
RE := 8.314⋅ RG := 0.00008314⋅
mol⋅ K mol⋅ K

Property Data Tc := 304.2⋅ K Pc := 73.76⋅ bar om := 0.225


(T in K, P in bar):
kap := 0.480 + 1.574⋅ om − 0.176⋅ om⋅ om
Initial Conditions: Ti := 400⋅ K Pi := 50⋅ bar
2 2
RG⋅ Tc RG ⋅ Tc
SRK Constants : b := 0.08664⋅ ac := 0.42748⋅
Pc Pc
Initial temperature T := Ti
Note that these are being defined as a 2
⎡ ⎛ T ⎞⎤
function of temperature since we will need to alf( T) := 1⋅ ⎢ 1 + kap⋅ ⎜ 1 − ⎥
interate on temperature later to obtain the final ⎣ ⎝ Tc ⎠ ⎦ a( T) := ac ⋅ alf( T)
state of the system d
Da( T) := a( T)
joule − 2 joule
Heat capacity Cp1 := 22.243⋅ Cp2 := 5.977⋅ 10 ⋅ dT
constants mol⋅ K mol ⋅ K
− 5 joule − 9 joule
Cp3 := −3.499⋅ 10 ⋅ Cp4 := 7.464⋅ 10 ⋅
mol⋅ K mol⋅ K
Find initial molar volume and number of moles RG⋅ Ti −4 3
Start with initial guess for volume, m^3/mol V := V = 6.6512× 10 m
Pi

RG⋅ T a ( T)
Solve SRK EOS for initial volume Given Pi − Vi := Find( V)
V− b V⋅ ( V + b )
−4 3
Vi = 6.09602× 10 m

DELSi:= ⎡⎢ln⎡⎢( Vi − b ) ⋅ ⎤ + Da( T) ⋅ ln⎛ Vi + b ⎞⎤ ⋅ RE


Pi
Entropy departure at the ⎥ ⎜ ⎥
initial conditions ⎣ ⎣ RG⋅ T⎦ b ⋅ RG ⎝ Vi ⎠⎦

-1 Pi⋅ Vi
DELSi = −2.37467K joule Zi := Zi = 0.91653
RG⋅ T

⎛ T⋅ d a( T) − a( T) ⎞ ⎛ Zi + b⋅ Pi ⎞

DelHi := RE⋅ T⋅ ( Zi − 1) +
⎝ dT ⎠ ⋅ ln⎜⎜ RG⋅ T
b ⎝ Zi ⎠
3
DelHi = −1.24253× 10 joule

Final temperature is 300 K, and final pressure is unknown; will be found by equating the initial and final
entropies. Guess final temperature is 10 bar
RG⋅ T
Pf := 10⋅ bar T := 300⋅ K V := −3 3
Pf V = 2.4942× 10 m
Temperature part of ideal gas entropy change

ti := 400 tf := 300

DSidealT := Cp1⋅ ln⎛⎜


tf ⎞
+ Cp2⋅ ( tf − ti) +
Cp3 (
⋅ tf
2 2
− ti ) + Cp4 ⋅(tf 3 − ti3)
⎝ ti ⎠ 2 3

-1
DSidealT = −11.24332K joule

Note: To use the given and find commands for variable with different dimensions such as P and V, will have
to convert to dimensionless variables so as not to have a units conflict. Define x=V/b and y=P/Pc
V
initial guess x := x = 83.95847 y := 0.5
b
Given
RG⋅ 300⋅ K a( 300⋅ K)
y ⋅ Pc −
x⋅ b − b x⋅ b ⋅ ( x⋅ b + b )

DSidealT − RE⋅ ln⎛⎜


y ⋅ Pc ⎞
+ ⎡⎢ln⎡⎢( x⋅ b − b ) ⋅ ⎤ + Da( 300⋅ K) ⋅ ln⎛ x + 1 ⎞⎤ ⋅ RE − DELSi
y ⋅ Pc
0 ⎥ ⎜ x ⎥
⎝ Pi ⎠ ⎣ ⎣ RG⋅ 300⋅ K⎦ b ⋅ RG ⎝ ⎠⎦

Y := Find( x, y ) ⎛ 53.6541⎞
Y= ⎜
⎝ 0.19654⎠

−3 3
Vf := Y ⋅ b Vf = 1.59393× 10 m
0
6
Pf := Y ⋅ Pc Pf = 1.46849× 10 Pa
1

Pf Vi
= 14.49643 Fraction mass remaining in tank = = 0.38245
bar Vf
th
Solutions to Chemical and Engineering Thermodynamics, 4 ed
6.42 (also available as a Mathcad worksheet. In fact, that file contain graphs and other information.)
FG
∂ CV
=T
IJ
∂2P FG IJ
∂ CP
= −T
∂ 2V FG IJ FG IJ
Easier to work with
H
∂V T K
∂T V
2
H
than with
K
∂P T ∂ T2 P
.
H K H K
CV (V , T ) − CV (V = ∞, T ) = CV (V , T ) − CV* (T ) = z
V
FG ∂ P IJ dV 2

V =∞
H∂ T K
T 2

= 0.75
a V +b
V
= 0.75
a F V + b IJ
lnG
T3 2
ln
V V =∞ T HV K 32

So CV (V , T ) = CV* (T ) + 0.75
a FG
V +b IJ
T 3 2b
ln
H
V K
. Clearly as

V → ∞ (ideal gas) we recover CV = CV* .


Procedure: Choose collection of V ’s
Calculate CV − CV* for given V and T
Calculate P from RK EOS get CV − CV* vs. P
Next use

CP = CV − T
FG ∂ V IJ FG ∂ P IJ 2
= CV − T
a∂ P ∂ T f 2
V
H ∂ PK H ∂ T K
T V a∂ P ∂ V f T

to convert from CV to CP . Have done both parts using MATHCAD. See the MATHCAD F file.
th
Solutions to Chemical and Engineering Thermodynamics, 4 ed
6.43 (also available as a Mathcad worksheet)

RT a PV B C
a) P = − ; = 1 + + 2 +"= Z
V −b TV (V + b) RT V V

PV V V a V a
Z= = − = −
RT V − b RT TV (V + b) V − b RT (V + b)
32

1 aV 1 a /V
= − = −
1 − b / V RT 3 2 (1 + b / V ) 1 − b / V RT 3 2 (1 + b / V )
Now expanding in a power series in 1V
a /V F a 1 I
Z = 1+ b /V −
RT 3 2 H
= 1+ b −
RT 3 2 V K
a a
B =b− 32
; B = 0; b =
RT RTB3 2
a F I
a 23
TB3 2 =
bR
⇒ TB =
H K
bR
= 8765
. K

b) Using the Redlich-Kwong parameters

FaI 23
TB =
H bR K = 8765
. K
th
Solutions to Chemical and Engineering Thermodynamics, 4 ed
6.44 (also available as a Mathcad worksheet)

V a
Z= −
V − b RT 3 2 (V + b)

FG ∂ Z IJ =
1 ∂V FG IJ −
V ∂V FG IJ +
a ∂V FG IJ
H ∂ PK T V −b ∂ P H K T (V − b)2 ∂ P H K T RT 3 2 (V + b)2 ∂ P H K T
RT a
P= −
V −b T V 2 + bV c h
FG ∂ P IJ =−
RT
+
a
(2V + b)
H ∂V K T (V − b)2
T V 2 + bV d i
2

FG ∂ P IJ = 0 ; lim
FG ∂ P IJ =∞
lim
P→0
V →∞
H ∂V K T
P →∞
V →∞
H ∂V K T

FG ∂ Z IJ = 0 = LM 1 − V + a OPFG ∂ V IJ
lim
P→0
V →∞
H ∂ PK T NV − b (V − b) RT (V + b) QH ∂ P K 2 32 2
T

FG ∂ Z IJ = 1 (V − b) − mV (V − b) r + ma RT (V + b) r 2 32 2

H ∂ P K m− RT (V − b) r + {a T cV + bV h }(2V + b)
T
2 2 2

lim G
F ∂ Z IJ = V − b − V (V − b) = b 2

P →∞
V →b
H ∂ P K − RT (V − b)
T RT 2

F ∂ Z IJ = 1 V − 1 V + a c RT V h
lim G
32

P→0
V →∞
H ∂ P K − RT (V ) + a d TV i ⋅ 2V
T
2 4

a d RT V i aV
32 2
a 2
= =− =−
− RT V 2
RT V 2 32
RT 3 2
th
Solutions to Chemical and Engineering Thermodynamics, 4 ed
6.45 a) The Redlich-Kwong equation of state is

RT a
P= −
V −b T V (V + b )

which we rewrite as follows

PV V a
= − so that
RT V − b RT (V + b)
1.5

PV V a b a
−1 = − 1− = − and
RT V −b RT 1.5(V + b) V − b RT 1.5(V + b)
F
PV I V a V
V
H
RT K
−1 = b −
V − b RT 1.5 V + b

VF
PV I F PV − 1I = Lim LMb V − a V OP a
Lim P → 0
H RT − 1K = Lim V →∞ V
H RT K N V − b RT
V →∞ 1.5
V +b
=b−
Q RT 1.5
= B (T )

To proceed further, we now need to have expressions for a and b in terms of the critical properties. To obtain these we
proceed as in Problem 4.35 and rewrite this in the power series of V

RT 2 RTF a ab I
V3 −
P
V + −b2 −
PHb−
TP
V−
TP
=0
K
Notice that the three roots of volume at the critical point are identical so we can write

bV − V g C
3
=0 or
V 3 − 3V C V 2 + 3V C2 V − V 3C = 0

At critical point, the second and fourth equations must be satisfied simultaneously. Consequently, the coefficients of each
power of V must be the same. Thus,

RTC
3V C =
PC
RTC a
3V 2C = -b2 - b− and
PC TC PC
a
V 3C = b
TC PC

Solving the above three equations together for a(T), b and Vc, we get

RTC
VC =
3 PC

b= d 3
i
2 − 1 V C = 0.08664
RTC
PC
and

R 2TC2.5
a = 0.42748
PC
th
Solutions to Chemical and Engineering Thermodynamics, 4 ed
a RT RT 2.5
So B(T ) = b − 1.5
= 0.08664 C − 0.42748 C1.5
RT PC PC T
For n-pentane, TC=469.6 K and PC = 33.74 bar

The resulting virial coefficient as a function of temperature is shown below.

0
.0

B( T ) 0.001

0.002
200 400 600 800 1000 1200 1400 1600
1.5 .10
200 T 3
th
Solutions to Chemical and Engineering Thermodynamics, 4 ed

6.46 Inversion temperature µ = 0 =


FG ∂ T IJ = − V 1 − Tα ⇒ αT = 1
H ∂ PK C H P

1 F∂V I
α= G
V H∂ TK
J but FGH ∂∂ VT IJK FGH ∂∂ VP IJK FGH ∂∂ TP IJK = −1 by triple product rule.
P P T V

FG ∂ V IJ = −1
=−
a∂ P ∂ T f V
H ∂ T K a∂ P ∂ V f a∂ T ∂ Pf a∂ P ∂ V f
P T V T

vdW EOS; P =
RT a
− 2;
∂P FG IJ =
R
V −b V ∂T H K V V −b
FG ∂ P IJ = − RT + 2a
H ∂ V K (V − b) V
T
2 3

FG ∂ V IJ = − R (V − b)
H ∂ T K n(− RT ) (V − b) s + 2a V
P
2 3

1 F∂V I k− R V (V − b) p
α= G J =
V H∂ TK P n− RT (V − b) s + 2a V 2 3

− RT V (V − b)
Tα = 1 ; =1
n− RT (V − b)2 + 2a V 3 s
RT RT 2a
= − 3
V (V − b) (V − b) V
2

TR
LM 1 − 1 OP = − 2a
NV (V − b) (V − b) Q V 2 3

TR 2a
V − b −V = − 3
V (V − b)2 V
bRT 2a
= 3
V (V − b)2
V

T=
2a (V − b)2 2a
= 1−
b FG IJ 2

bR V 2
bR V H K (1)

RT a
also P = − (2)
V −b V2
Choose V
Calculate Tinv from Eqn. (1)
Calculate P from Eqn. (2)
Solution done with MATHCAD (see MATHCAD worksheet).
th
Solutions to Chemical and Engineering Thermodynamics, 4 ed
(b) RK EOS
RT a
P= −
V −b TV (V + b)
FG ∂ P IJ =−
RT
+
a
+
a
H ∂V K T (V − b)2
TV (V + b)
2
TV (V + b)2
FG ∂ P IJ =
R
+
12 a a f
H∂TK V V − b T 3 2V (V + b)

α=
1 ∂V FG IJ =−
a
1 ∂ P ∂T f
H K
V

V ∂T P a
V ∂ P ∂V f T

=
1 l
− R (V − b) + 1 2 a T 3 2V (V + b) a f q
m r n
V − RT (V − b)2 + a TV 2 (V + b) + a TV (V + b)2 s n s
Tα = 1
− RT

12a a f
=−
RT
+
a
+
a
V (V − b) T 1 2V 2 (V + b) (V − b)2 TV 2 (V + b) TV (V + b)2


LM
RT 1

1
=
OP a LM 1 + 1 + 1 2 OP
N
V −b V V −b Q TV (V + b) NV V + b V Q

RT LV − b − V O
M P =
RTb
=
a LM a3 2f(V + b) + V OP
V − b N V (V − b) Q V (V − b) TV (V + b) N V (V + b) Q
2

RTb a 5V + 3b
=
V (V − b)2
2 TV (V + b) V (V + b)
RTb a(5V + 3b)
=
(V − b)2
2 TV (V + b)2
a(5V + 3b) (V − b)2
T3 2 =
2V (V + b)2 Rb
RS a L5 + 3 b O (V − b) UV 2 23

T 2 Rb MN V PQ (V + b) W
T= 2

6.47 Using a Mathcad program for the Soave-Redlich-Kwong EOS we find


H (300° C, 25 bar ) = 9.45596 × 103 J mol
(relative to ideal gas at 273.15 K and 1 bar).
Now by trial and error until enthalpies match
H (274.5° C, 1 bar ) = 9.45127 × 103
H (274.6° C, 1 bar ) = 9.45486 × 103 ⇒ T = 274.6o C
Close enough
Note that this solution is only very slightly different from that obtained with the Peng-Robinson equation
(274.1oC compared to 274.6oC obtained here).

6.48 Only available as a Mathcad worksheet.

6.49 Available as a Mathcad worksheet.


th
Solutions to Chemical and Engineering Thermodynamics, 4 ed
(b) RK EOS
RT a
P= −
V −b TV (V + b)
FG ∂ P IJ =−
RT
+
a
+
a
H ∂V K T (V − b)2
TV (V + b)
2
TV (V + b)2
FG ∂ P IJ =
R
+
12 a a f
H∂TK V V − b T 3 2V (V + b)

α=
1 ∂V FG IJ =−
a
1 ∂ P ∂T f
H K
V

V ∂T P a
V ∂ P ∂V f T

=
1 l
− R (V − b) + 1 2 a T 3 2V (V + b) a f q
m r n
V − RT (V − b)2 + a TV 2 (V + b) + a TV (V + b)2 s n s
Tα = 1
− RT

12a a f
=−
RT
+
a
+
a
V (V − b) T 1 2V 2 (V + b) (V − b)2 TV 2 (V + b) TV (V + b)2


LM
RT 1

1
=
OP a LM 1 + 1 + 1 2 OP
N
V −b V V −b Q TV (V + b) NV V + b V Q

RT LV − b − V O
M P =
RTb
=
a LM a3 2f(V + b) + V OP
V − b N V (V − b) Q V (V − b) TV (V + b) N V (V + b) Q
2

RTb a 5V + 3b
=
V (V − b)2
2 TV (V + b) V (V + b)
RTb a(5V + 3b)
=
(V − b)2
2 TV (V + b)2
a(5V + 3b) (V − b)2
T3 2 =
2V (V + b)2 Rb
RS a L5 + 3 b O (V − b) UV 2 23

T 2 Rb MN V PQ (V + b) W
T= 2

6.47 Using a Mathcad program for the Soave-Redlich-Kwong EOS we find


H (300° C, 25 bar ) = 9.45596 × 103 J mol
(relative to ideal gas at 273.15 K and 1 bar).
Now by trial and error until enthalpies match
H (274.5° C, 1 bar ) = 9.45127 × 103
H (274.6° C, 1 bar ) = 9.45486 × 103 ⇒ T = 274.6o C
Close enough
Note that this solution is only very slightly different from that obtained with the Peng-Robinson equation
(274.1oC compared to 274.6oC obtained here).

6.48 Only available as a Mathcad worksheet.

6.49 Available as a Mathcad worksheet.


th
Solutions to Chemical and Engineering Thermodynamics, 4 ed
3
mol := 1 bar := 101300Pa
⋅ joule bar ⋅ m
RE := 8.314⋅ RG := 0.00008314⋅
mol⋅ K mol⋅ K

Property Data Tc := 5.19⋅ K Pc := 2.27⋅ bar om := −0.387 Cp := 2.5⋅ RE


(T in K, P in bar):
kap := 0.37464 + 1.54226om
⋅ − 0.26992om
⋅ ⋅ om
3
Initial Conditions and total volume Vt: Ti := 298⋅ K Pi := 400⋅ bar Vt := 0.045⋅ m
2 2
RG⋅ Tc RG ⋅ Tc
Peng-Robinson Constants : b := 0.07780⋅ ac := 0.45724⋅
Pc Pc
Initial temperature T := Ti
Note that these are being defined as a 2
⎡ ⎛ T ⎞⎤
function of temperature since we will need to alf( T) := 1⋅ ⎢ 1 + kap⋅ ⎜ 1 − ⎥
interate on temperature later to obtain the final ⎣ ⎝ Tc ⎠ ⎦ a( T) := ac ⋅ alf( T)
state of the system d
Da( T) := a ( T)
dT
Find initial molar volume and number of moles RG⋅ Ti −5 3
Start with initial guess for volume, m^3/mol V := V = 6.19393× 10 m
Pi

RG⋅ T a ( T)
Solve P-R EOS for initial volume Given Pi − Vi := Find( V)
V− b V⋅ ( V + b ) + b ⋅ ( V − b )

Initial molar volume and −5 3 Vt


number of moles Vi = 7.10667× 10 m N := N = 633.20762
Vi


DELSi:= ⎢ ln⎡⎢( Vi − b ) ⋅
Pi ⎤ + Da( T) ⋅ ln⎡ Vi + ( 1 + 2) ⋅ b ⎤ ⎤ ⋅ RE
Entropy departure at the ⎥ ⎢ ⎥⎥
initial conditions ⎣ ⎣ RG⋅ T⎦ 2⋅ 2⋅ b ⋅ RG ⎣ Vi + ( 1 − 2) ⋅ b ⎦ ⎦

-1 Pi⋅ Vi
DELSi = 0.0151K joule Zi := Zi = 1.14736
RG⋅ T

T⋅
d
a ( T) − a ( T) ⎡ Zi + ( 1 + 2) ⋅ b ⋅ Pi ⎤
dT ⎢ RG⋅ T ⎥
DelHi := RE⋅ T⋅ ( Zi − 1) + ⋅ ln⎢ ⎥ DelHi = 274.27781joule
2⋅ 2⋅ b ⎢ Zi + ( 1 − 2) ⋅ b⋅ Pi ⎥
⎣ RG⋅ T ⎦

Final pressure is 1.013 bar, and final temperature is unknown; will be found by equating the initial and final
entropies. Guess final temperature is 30 K
RG⋅ T
Pf := 1.013⋅ bar T := 25.7⋅ K V := −3 3
Pf V = 2.10928× 10 m

Note: To use the given and find command for variables with different dimensions such as T and V, will have
to convert to dimensionless variables so as not to get a units conflict. Define x=V/b, y=T/Tc
initial guess x := 10 y := 5
th
Solutions to Chemical and Engineering Thermodynamics, 4 ed
initial guess x := 10 y := 5

Given
RG⋅ y ⋅ Tc a( y ⋅ Tc)
Pf −
x⋅ b − b x⋅ b ⋅ ( x⋅ b + b ) + b ⋅ ( x⋅ b − b )

Cp⋅ ln⎛⎜
y ⋅ Tc ⎞
− RE⋅ ln⎛⎜
Pf ⎞ ⎡
+ ⎢ RE⋅ ln⎡⎢( x⋅ b − b ) ⋅
Pf ⎤ + Da( y⋅ Tc) ⋅ RE⋅ ln⎡ x⋅ b + ( 1 + 2) ⋅ b ⎤ ⎤ − DELS
0 ⎥ ⎢ ⎥⎥
⎝ Ti ⎠ ⎝ Pi ⎠ ⎣ ⎣ RG⋅ y ⋅ Tc⎦ 2⋅ 2⋅ b ⋅ RG ⎣ x⋅ b + ( 1 − 2) ⋅ b ⎦ ⎦

Y := Find( x, y ) ⎛ 150.89161⎞
Y= ⎜
⎝ 5.27265 ⎠

−3 3
V := Y ⋅ b V = 2.2315× 10 m Tf := Y ⋅ Tc Tf = 27.36506K Final temperature
0 1

Pf ⋅ V
Zf := Zf = 1.05794
RG⋅ T

Tf ⋅
d
a( Tf ) − a( Tf ) ⎡ Zf + ( 1 + 2) ⋅ b ⋅ Pf ⎤
dTf ⎢ RG⋅ Tf ⎥
DelHf := RE⋅ Tf ⋅ ( Zf − 1) + ⋅ ln⎢ ⎥
2⋅ 2⋅ b ⎢ Zf + ( 1 − 2) ⋅ b⋅ Pf ⎥
⎣ RG⋅ Tf ⎦

DelHf = 11.61232joule

Uf - Ui = (Hf - Zf*R*Tf) - (Hi - Zi*R*Ti) = Hf - Hi - Zf*R*Tf + Zi*R*Ti


= (Hf-HfIG) + HfIG -(Hi - HiIG) -HiIG - Zf*R*Tf + Zi*R*Ti
= DelHf - DelHi + Cp*(Tf-Ti) - Zf*R*Tf + Zi*R*Ti

DelU := [ DelHf − DelHi + Cp⋅ ( Tf − Ti) − Zf ⋅ RE⋅ Tf + Zi⋅ RE⋅ Ti] ⋅ N


6
DelU = −2.08062× 10 joule
DelU
TNTeq := − TNTeq = 0.45231kg
joule
4600000⋅
kg
6
6.50 We start from
F ∂P I
d S = CVdT +
H ∂T K V
dV

Since the entropy at 0 K is not a function of temperature, it follows that CV = 0.


Also, since the entropy is not a function of specific volume, it follows that

F ∂P I
H ∂T K V
=0

However, by the triple product rule


F ∂P I F ∂V I FG ∂T IJ = −1 or
H ∂T K H ∂P K H ∂V K
V T P

F ∂P I = −FG ∂P IJ F ∂V I = 0
H ∂T K H ∂V K H ∂T K
V T P
but from the thermodynamic stability condition
FG ∂P IJ <0
H ∂V K T
which implies that
F ∂V I 1 ∂VF I
H ∂T K P
= 0 and α =
V ∂TH K P
=0
6
6.51 Rewrite the Clausius equation as
RT ⎛ ∂V ⎞ R
V= +b Then ⎜ ⎟ = ;
P ⎝ ∂T ⎠ P T
⎛ ∂2V ⎞ ⎛ ∂V ⎞
⎟⎟ = 0 (which means CP is independent of pressure and equal to CP ) and V − T ⎜
*
⎜⎜ 2 ⎟ =b
⎝ ∂T ⎠P ⎝ ∂T ⎠ P

(a) Therefore

z z
T2 P2
∆H = H (T2 , P2 ) − H (T1, P1) = CP* (T )dT + bdP = 0 or
T1 P1

β γ
α (T2 − T1) + (T22 − T12 ) +
(T23 − T13 ) + b( P2 − P1) = 0
2 3
is the line of constant enthalpy.
b)

z z FH I
T2 P
CP* (T ) 2
∂V
∆S = S (T2 , P2 ) − S (T1, P1) =
T1
T
dT −
1P
∂T K P
dP = 0

z z
T2 P
CP* (T ) 2
R
= dT − dP = 0
T1
T P
P
1

or
T2 γ P
α ln + β (T2 − T1 ) + (T22 − T12 ) − R ln 2 = 0
T1 2 P1
is the line of constant entropy.

c) For the fluid to have a Joule-Thomson inversion temperature


F ∂T I
µ=
H ∂P K H
must undergo a sign change. However

LMV − T F ∂V I OP RT + b − T R
µ=F I =−N
∂T H ∂T K Q = − P P =− b
H ∂P K C
H C P C
P
P P

This is always negative, so the Clausius does not have a Joule-Thomson inversion
temperature.
6
6.52
RT RT ⎛ ∂V ⎞ R ⎛ ∂V ⎞ RT RT
P= and C*P = α + β T + γT 2 ; so V= + b; ⎜ ⎟ = ; and V-T ⎜ ⎟ = + b− =b
V−b P ⎝ ∂T ⎠ P P ⎝ ∂T ⎠P P P

P = 0,T1 P = 0,T2 P2 ,T2


⎡ ⎛ ∂V ⎞ ⎤ ⎡ ⎛ ∂V ⎞ ⎤
∆H = ∫
P1 ,T1
⎢V − T ⎜

⎟ ⎥ dP +
⎝ ∂T ⎠P ⎦ P = 0,T1

C*P dT + ∫
⎢V − T ⎜
P = 0,T2 ⎣
⎟ ⎥ dP = 0
⎝ ∂T ⎠ P ⎦
( for constant enthalpy )

β 2 γ
=b ( 0-P1 ) + α ( T2 − T1 ) +
2
( ) (
T2 − T12 + T23 − T13 + b(P2 − 0) = 0
3
)
β γ
⇒ bP + αT + T 2 + T 3 for line of constant enthalpy
2 3
P = 0,T1 P = 0,T2 P2 ,T2
⎛ ∂V ⎞ C*P ⎛ ∂V ⎞
∆S = ∫
P1 ,T1
⎜ ⎟ dP +
⎝ ∂T ⎠ P ∫
P = 0,T
T
dT +
P = 0,T
⎜ ∫⎟ dP = 0
⎝ ∂T ⎠ P
( for constant entropy )
1 2

⎛P ⎞ ⎛T ⎞ γ
=Rln ⎜ 2 ⎟ + αln ⎜ 2 ⎟ + β ( T2 − T1 ) + T22 − T12 = 0
⎝ P1 ⎠ ⎝ T1 ⎠ 2
( )
γ
⇒ b ln P + α ln T + β T + T 2 for line of constant entropy
2
6
6.53 Using the P-R EOS program for the initial conditions of 100 bar and 35.5oC
we have for the liquid that
VL=1.02858¯10-4 m3/mol, HL = -105055.59 J/mol and SL = -386.03 J/(mol K). At
this temperature (since the process is isothermal) the VLE conditions are
Pvap=0.0378 bar and
VL=1.04023¯10-4 m3/mol, HL = -106742.98 J/mol and SL = -385.00 J/(mol K) and
VV=5.21479¯10-1 m3/mol, HV = -77713.36 J/mol and SV = -262.84 J/(mol K).
a) Therefore the volume change is
∆V=(5000¯1.04023¯10-4 + 5000¯5.21479¯10-1 - 10000¯1.02858¯10-4) =
26069¯10-4 = 2. 6069 m3
b) The entropy balance is
dS Q 
= + S gen ; now since T is constant, and the fluid is undergoing a uniform expansion, S gen =0,
dt T
kJ
so that ∆S=-1000 × ( 5 × 385.00 + 5 × 262.84 − 10 × 386.03) = 621.1 and
K
Q=T∆S=237.65 × 621.1=147,604 kJ
6
6.54 From Eq. 6.5-9
V
⎛ ∂2P ⎞
CV ( V, T ) = CV ( V = ∞, T ) + T ∫ ⎜ 2 ⎟ dV
V =∞ ,T ⎝ ∂T ⎠ V

RT a(T)
P= −
V − b V ( V + b) + b ( V − b)
⎛ ∂P ⎞ R da(T) / dT ⎛ ∂2P ⎞ d 2 a(T) / dT 2
⎜ ∂T ⎟ = V − b − V V + b + b V − b ; ⎜ 2⎟ −
⎝ ⎠V ( ) ( ) ⎝ ∂T ⎠ V V ( V + b ) + b ( V − b )
V
d 2 a(T) dV
CV ( V, T ) = C*V − T
dT 2 V∫=∞ V(V + b) + b(V − b)
but
dV 1 ⎡ V + b(1 − 2 ) ⎤
∫ V(V + b) + b(V − b) = 2 ln ⎢ ⎥
2b ⎣⎢ V − b(1 − 2 ) ⎥⎦
so that
d 2 a(T) 1 ⎡ V + b(1 − 2 ) ⎤
CV ( V, T ) = C*V − T ln ⎢ ⎥
dT 2 2b ⎢⎣ V − b(1 − 2 ) ⎥⎦
2

Finally
2
R 2 TC2 ⎡ ⎛ T ⎞⎤
a(T) = 0.42788 α(T) where α(T) = ⎢1 + κ ⎜ 1 − ⎟⎥
⎜ TC ⎟⎠ ⎥⎦
PC ⎢⎣ ⎝
dα(T) ⎡ ⎛ T ⎞⎤ ⎡ 1 1 ⎤
so that = 2 ⋅ ⎢1 + κ ⎜ 1 − ⎟⎟ ⎥ ⋅ ⎢ −κ ⋅ ⋅ ⎥ and
⎜ TC ⎠ ⎥⎦ ⎢⎣ 2 T ⋅ TC
dT ⎢⎣ ⎝ ⎥⎦
⎡ ⎤
d 2 α(T) ⎡ ⎛ T ⎞⎤ ⎢ κ 1 ⎥ ⎛ κ2 ⎞
= 2 ⋅ ⎢1 + κ ⎜ 1 − ⎟⎥ ⋅ ⎢ ⋅ ⎥ + ⎜⎜ ⎟⎟
⎜ TC ⎟⎠ ⎥⎦ ⎢ 4 3
dT 2 ⎢⎣ ⎝ 2 ⎥ ⎝ 2 ⋅ T ⋅ TC ⎠
⎣ T TC ⎦
⎧ ⎡ ⎤ ⎫
d 2 a(T) R 2 TC2 ⎪⎪ ⎡ ⎛ T ⎞⎤ ⎢ κ 1 ⎥ ⎛ κ2 ⎞ ⎪⎪
= 0.42788 ⋅ ⎨2 ⋅ ⎢1 + κ ⎜1 − ⎟⎟ ⎥ ⋅ ⎢ ⋅ 3 ⎥ + ⎜⎜ ⎟⎟ ⎬

⎥ ⎝ 2 ⋅ T ⋅ TC
2 PC ⎪ ⎢⎣ TC ⎠ ⎥⎦ ⎢ 4
dT ⎝ 2 ⎠⎪
⎩⎪ ⎣ T TC ⎦ ⎪⎭
6
6.55
V ( T, P ) = a + bT − CP and U ( T,P ) = dT + eP
⎛ ∂V ⎞ ⎛ ∂S ⎞ ⎛ ∂S ⎞ ⎛ ∂U ⎞ 1
⎜ ⎟ = b = −⎜ ⎟ = −⎜ ⎟ ⎜ ⎟ = − e; ⇒ e= − bT
⎝ ∂T ⎠P ∂
⎝ ⎠T
P ⎝ ∂U ⎠T ⎝ ∂P ⎠T T
But it was claimed that a, b, c, d and e are constants, which is not the case given the
interrelationship between b and e given above.
6
6.56
a) Rewriting the equation of state as
RT aP ⎛ ∂V ⎞ R aP ⎛ ∂2V ⎞ 2aP
V = b+ + ; ⎜ ⎟ = − ; and ⎜⎜ 2 ⎟⎟ = 3
P T ⎝ ∂T ⎠ P P T 2 ⎝ ∂T ⎠P T
P P
⎛ ∂2V ⎞
( )
2 2

∫ ∫
2aP a
CP ( P2 , T ) = CP ( P1 , T ) − T ⎜⎜ 2 ⎟⎟ dP = CP ( P1 , T ) − T dP = CP ( P1 , T ) − 2 P22 − P12
P1 ⎝
∂T ⎠ P P1
T 3
T

and
m3 K (144 − 1) bar J
2
14300 J
CP (12 bar,T ) = CP (1 bar,T ) − 10−3 × 2 2
× 105 3
= 33.5 − 2
bar mol T K bar m T mol K

J
CP (12 bar,T=273.15 K ) = 33.5 − 0.193 = 33.307
mol K
J
Therefore, CP (12 bar,T=573.15 K ) = 33.5 − 0.044 = 33.446
mol K
J
So that the mean value over the temperature range is 33.377
mol K
0 bar, 300 K 400 K 12 bar, 400 K
⎡ ⎛ ∂V ⎞ ⎤ ⎡ ⎛ ∂V ⎞ ⎤
b) ∆H = ∫
4 bar, 300 K
⎢ V-T ⎜
⎣ ⎝
⎟ ⎥dP +
∂T ⎠ P ⎦
300 K

C*P dT +
0 bar, 400 K
⎢ V-T ⎜
⎣ ⎝ ∫ ⎟ ⎥dP
∂T ⎠ P ⎦

and
RT aP ⎛ ∂V ⎞ R aP ⎛ ∂V ⎞ RT aP RT aP 2aP
V = b+ + ; ⎜ ⎟ = − ; and V − T ⎜ ⎟ = b+ + − + = b+
P T ⎝ ∂T ⎠ P P T 2 ⎝ ∂T ⎠P P T P T T

0 bar, 300 K 400 K 12 bar, 400 K


⎡ 2aP ⎤ ⎡ 2aP ⎤
∆H = ∫
4 bar, 300 K
⎢ b + T ⎥dP +
⎣ ⎦ 300 K

33.5dT +
0 bar, 400 K
⎣ ∫
⎢ b + T ⎥dP

0 bar, 300 K 12 bar, 400 K
⎛ aP 2 ⎞ ⎛ aP 2 ⎞ J
= ⎜⎜ bP + ⎟⎟ + 33.5 × ( 400 − 300 ) + ⎜⎜ bP + ⎟⎟ = 3444.7
⎝ T ⎠ 4 bar, 300 K ⎝ T ⎠ 0 bar, 400 K mol
0 bar, 300 K 400 K 12 bar, 400 K
⎛ ∂V ⎞ ⎛ ∂V ⎞
*
CP
∆S = − ∫
4 bar, 300 K
⎜ ⎟ dP +
⎝ ∂T ⎠ P 300 K
T ∫
dT −
0 bar, 400 K
⎜ ∫
⎟ dP
⎝ ∂T ⎠ P


= − −R ln − ×
2 2 5⎤
0 a 0 − 4 × 10 ⎥ ( ⎡
12 ⎢ )
+ 33.5ln − − R ln − ×
2 2 5⎤
12 a 12 − 0 × 10 ⎥
= 18.771
J ( )
⎢ 4 2 T 2 ⎥ 4 ⎢ 0 2 T 2 ⎥ mol K
⎣ ⎦ ⎣ ⎦
th
Solutions to Chemical and Engineering Thermodynamics, 4 ed
6.57
( a)
1 To calculate points at geometrically increasing values of the molar volume
10 at ten points per decade
zz := 10

i (Pa⋅ m3)
i := 0 , 1 .. 30 V := 0.0001zz
⋅ units m^3/m R := 8.314 units
i mol⋅ K
van der Waals parameters
(Pa⋅m6)
3
m
a := 0.1386 −5 units
units b := 3.864⋅ 10 mol
2
mol

( R⋅ 100) a ( R⋅ 150) a
P100 := − ( R⋅ 125) a P150 := −
P125 := −
V −b
(Vi)2 V −b
(Vi)2
i i
V −b
(Vi)2
i i i
i

( R⋅ 200) a
P200 := −
V −b
(Vi)2
i
i

1.4 .10
7

1.2 .10
7

1 .10
7

P100i 8 .10
6

P125i
6 .10
6
P150i

P200i
4 .10
6

2 .10
6

2 .10
6
4 3.5 3 2.5 2 1.5 1
log( Vi)
th
Solutions to Chemical and Engineering Thermodynamics, 4 ed

(b)
T := 300
Ideal gas guess
( R ⋅ T) a ( R ⋅ T)
P( V) := − VinitIG:= for initial volume V := VinitIG
V− b 2 100000
V
Given
P( V) 100000
Vinit := Find( V) Vinit = 0.025 VinitIG = 0.025

Ideal gas guess


( R ⋅ T)
VfinIG:= for initial volume V := VfinIG
7
10
Given
7
P( V) 10
−4 −4
Vfin := Find( V) Vfin = 2.395 × 10 VfinIG = 2.494 × 10

Vfin
⌠ 4
W := −⎮ P( V) dV W = 1.145 × 10 J

Vinit

( c) For the ideal gas

VfinIG
⌠ ( R ⋅ T)
WIG := −⎮
4
dV WIG = 1.149 × 10 J
⎮ V

VinitIG
Almost identical to the vdW result

Note that if one wanted to do the integrals analytically in Mathcad instead of numerically,
the following would be obtained

⌠ vf
( RT) ⌠ ( RT)
⎮ dV → RT⋅ ln( V) ⎮ dV → ln( vf ) ⋅ RT − ln( vi) ⋅ RT
⎮ V ⎮ V
⌡ ⌡
vi


⎮ ⎛ RT − aa ⎞ dV → ln( V − bb ) ⋅ RT + aa
⎮ ⎜ V − bb 2 V
⎮ ⎝ V ⎠

vf

⎮ ⎛ RT − aa ⎞ dV → ( ln( vf − bb ) ⋅ RT⋅ vf + aa ) − ( ln( vi − bb ) ⋅ RT⋅ vi + aa)
⎮ ⎜ V − bb 2 vf vi

⌡ ⎝ V ⎠
vi
6
6.58 For the gas in the pump, the process is adiabatic and reversible (uniform
compression). Therefore, the process is isentropic (whether or not it is an ideal
gas).
a) For the ideal gas case
Tf P Tf 5.15
∆ S = C*P ln − R ln f = 0 = 29.3ln − 8.314 ln ⇒ T = 472.96 K
Ti Pi 298.15 1.013
Pi Vi Pf Vf V P T 1.013 × 472.96
Now since from the ideal gas law = ⇒ f = i× f = = 0.312
Ti Tf Vi Ti Pf 5.15 × 298.15
Therefore, the pump handle will be pushed down 50 × (1 − 0.312 ) = 34.40 cm
b) Using the P-R EOS program
At T = 298.15 K and P = 1.013 bar, V = 2.4459¯10-2 m3/mol and S = -0.13 J/(mol
K).
Now at P = 5.15 bar, guess T until one obtains S = -0.13 J/(mol K) (can use ideal
gas temperature as initial guess). Find that T = 473.2 K, for which V =
7.65012¯10-3 m3/mol.
⎛ 7.65013 × 10−3 ⎞
Therefore, the pump handle will be pushed down 50 × ⎜⎜ 1 − ⎟ = 34.36 cm
⎝ 2.4459 × 10−2 ⎟⎠
6
6.59 a) By the triple product rule
⎛ ∂T ⎞ ⎛ ∂U ⎞ ⎛ ∂V ⎞
⎜ ⎟ ⎜ ⎟ ⎜ ⎟ = −1
⎝ ∂V ⎠U ⎝ ∂T ⎠V ⎝ ∂U ⎠T
⎛ ∂T ⎞ ⎛ ∂T ⎞ ⎛ ∂U ⎞
⎜ ⎟ = −⎜ ⎟ ⎜ ⎟
⎝ ∂V ⎠U ⎝ ∂U ⎠V ⎝ ∂V ⎠T
and dU = Td SHJ − PdV
⎛ ∂U ⎞ ⎛ ∂S ⎞ ⎛ ∂S ⎞ ⎛ ∂P ⎞
Therefore ⎜ ⎟ =T⎜ ⎟ − P and using ⎜ ⎟ =⎜ ⎟ gives
⎝ ∂V ⎠T ⎝ ∂V ⎠T ⎝ ∂V ⎠T ⎝ ∂T ⎠V
⎛ ∂U ⎞ ⎛ ∂P ⎞ ⎛ ∂T ⎞ 1
⎜ ⎟ =T⎜ ⎟ − P and ⎜ ⎟ =
⎝ ∂V ⎠T ⎝ ∂T ⎠V ⎝ ∂U ⎠V C V

⎡ ⎛ ∂P ⎞ ⎤
⎢T ⎜ ⎟ − P⎥
⎢ ⎝ ∂T ⎠V ⎦⎥
so that ξ = − ⎣
CV

∂P
b) Ideal gas PV = RT so that ⎛⎜ ⎞⎟ = = , so ξ =0
R P
⎝ ∂T ⎠V V T
c)
RT B (T ) RT ⎛ ∂P ⎞ R B (T ) R RT dB (T )
P= + ⋅ ; ⎜ ⎟ = + ⋅ + ⋅
V V V ⎝ ∂T ⎠V V V V V2 dT
⎛ ∂P ⎞ RT 2 dB (T ) RT 2 dB (T )
T⎜ ⎟ − P = ⋅ ; and ξ = - ⋅
⎝ ∂T ⎠V V2 dT V 2 ⋅ CV dT

d)
RT a(T )
P= −
V − b V (V + b ) + b (V − b )
⎛ ∂P ⎞ R 1 da (T )
⎜ ⎟ = − ⋅
⎝ ∂T ⎠V V − b V (V + b ) (
+ b V − b ) dT
da (T )
T − a (T )
⎛ ∂P ⎞ dT
T⎜ ⎟ − P = ; and
⎝ ∂T ⎠V V (V + b ) + b (V − b )
da (T )
a(T ) − T
ξ= dT ⋅
(V (V + b ) + b (V − b ) ) ⋅ CV

where CV is a function of temperature and specific volume.


th
Solutions to Chemical and Engineering Thermodynamics, 4 ed
RT a(T )
P= −
V − b V (V + b ) + b (V − b )
⎛ ∂P ⎞ R 1 da (T )
⎜ ⎟ = − ⋅
⎝ ∂T ⎠V V − b V (V + b ) + b (V − b ) dT
da (T )
d) T − a (T )
⎛ ∂P ⎞ dT
T⎜ ⎟ − P = ; and
⎝ ∂T ⎠V V (V + b ) + b (V − b )
da (T )
a(T ) − T
ξ= dT ⋅
(V (V + b ) + b (V − b ) ) ⋅ CV
where CV is a function of temperature and specific volume.

6.60 Reactor is I, initially evacuated tank is II. The mass balance, as in Problem 4.45, is Ni = N I + N II , except that here,
unlike there, here we use the virial gas equation of state.
The energy balance of the reactor and tank is NiU i = N I U I + N II U II .The entropy balance on just the contents of the
reactor that undergo a uniform expansion leads to
S I = constant . The equation of state is
PV B(T) 0.056 m3 J
= 1+ with B(T) = 5.86 × 10−5 − ; C*P = 73.2
RT V T mol mol K
V =∞ ,Ti Tf Vf ,Tf
⎛ ∂P ⎞ C*V ⎛ ∂P ⎞
∆S = 0 = ∫
Vi ,Ti
⎜ ⎟
⎝ ∂T ⎠ V
dV +
T
T ∫dT +
V =∞ ,T
∫ ⎜ ⎟ dV
⎝ ∂T ⎠ V
i f

V ⎡ 1 1⎤ T
= Rln f − 5.86 × 10−5 R ⎢ − ⎥ + ( 73.2 − 8.314 ) ln f
Vi ⎣ Vf Vi ⎦ Ti
RT RTB(T) RT RT × 5.86 × 10−5 R × 0.056
since P = + = + −
V V2 V V2 V2
⎛ ∂P ⎞ R R × 5.86 × 10−5
⎜ ⎟ = +
⎝ ∂T ⎠ V V V2
Also, for later reference
V =∞ ,Tref
⎡ ⎛ ∂P ⎞ ⎤ ⎡ ⎛ ∂P ⎞ ⎤
T V,T
∆U= ∫
Vref ,Tref
⎢T ⎜ ⎟
⎣⎢ ⎝ ∂T ⎠ V
− P ⎥
⎥⎦
dV +
Tref

C*V dT + ⎢T ⎜ ∫
⎟ − P ⎥ dV
⎢ ⎝ ∂T ⎠ V
V =∞ ,T ⎣ ⎥⎦

⎛ ∂P ⎞ ⎛ ∂P ⎞ RT RT × 5.86 × 10−5 RT RT × 5.86 × 10−5 R × 0.056 R × 0.056


and T ⎜ ⎟ −P =⎜ ⎟ = + − − + =
⎝ ∂T ⎠ V ⎝ ∂T ⎠ V V V2 V V2 V2 V2
so that
V =∞ ,Tref T V,T
R × 0.056 R × 0.056
U= ∫
Vref ,Tref
V2
dV + ∫
Tref
C*V dT + ∫
V =∞ ,T
V2
dV

Rest of the problem solution is in MATHCAD.


th
Solutions to Chemical and Engineering Thermodynamics, 4 ed

−5
R := 8.314 RG := 8.314⋅ 10 Tref := 273.15 Pref := 1

−5 0.056
B( T) := 5.86⋅ 10 − Cv := 73.2 − R
T

Calculate reference volume (for later)

( RG⋅ Tref )
Vref := Vref = 0.023
Pref

Given

( Pref⋅ Vref) B( Tref )


1+
RG⋅ Tref Vref

Vref := Find( Vref) Vref = 0.023

Pinit := 200 Tinit := 400 + 273.15

( RG⋅ Tinit) −4
Vig := Vig = 2.798 × 10
Pinit

Vinit := Vig

Given

( Pinit⋅ Vinit) B( Tinit)


1+
RG⋅ Tinit Vinit

−4
Vinit := Find( Vinit) Vinit = 2.526 × 10 Calculated initial molar volume

1
Ninit := 3
Vinit Ninit = 3.959 × 10

Calculate initial molar internal energy


−1
Uinit := R⋅ 0.056⋅ ⎛⎜ ⎞ + Cv⋅ ( Tinit − Tref )
1 4
+ Uinit = 2.413 × 10
⎝ Vref Vinit ⎠
th
Solutions to Chemical and Engineering Thermodynamics, 4 ed
Guess final pressure and temperature in tank 1; use ideal gas result as first guess. The
result below is the result of several iterations.
T1f
Pf := 41.86 T1f := 560.7 V1f := RG⋅ −3
Pf V1f = 1.114 × 10
Find final temperature and molar volume in reactor for this final pressure guess

Given

( Pf ⋅ V1f) B( T1f)
1+
RG⋅ T1f V1f

0 R⋅ ln⎛⎜
V1f ⎞
− 5.86⋅ 10 ⋅ R⋅ ⎛⎜
1 ⎞
+ ( 73.2 − R) ⋅ ln⎛⎜
−5 1 T1f ⎞

⎝ Vinit ⎠ ⎝ V1f Vinit ⎠ ⎝ Tinit ⎠
x := find( T1f , V1f)
⎛ 548.698 ⎞
x= ⎜
−3
⎝ 1.044 × 10 ⎠
T1f := x V1f := x
0 1
1
N1f := N1f = 957.446
V1f

Calculate final molar internal energy of gas in reactor


−1
U1f := R⋅ 0.056⋅ ⎛⎜ ⎞ + Cv⋅ ( T1f − Tref )
1
+
⎝ Vref V1f ⎠ U1f = 1.745 × 10
4

Calculate final number of moles in tank and molar volume based on guess Pf

3 4
N2f := Ninit − N1f N2f = 3.002 × 10 V2f := −3
N2f V2f = 1.333 × 10
Now use this to calculate temperature in tank

T2f := Tinit

Given
( Pf ⋅ V2f) B( T2f)
1+
RG⋅ T2f V2f
T2f := find( T2f)
T2f = 682.947
Calculate final molar internal energy of gas in tank

U2f := R⋅ 0.056⋅ ⎛⎜ ⎞ + Cv⋅ ( T2f − Tref )


1 −1 4
+ U2f = 2.626 × 10
⎝ Vref V2f ⎠

Is energy balance satisfied?

Del := Ninit⋅ Uinit − N1f⋅ U1f − N2f⋅ U2f

3
Del = 1.321 × 10

Results

Pf is 40.00 del is 6,977,000


Pf is 45.00 del is -11,800,00
Pf is 42.00 del is -524,100
Pf is 41.90 del is -148,800 whuch us close enough
Pf is 41.86 del is 1321

T1f = 548.698 N1f = 957.446


3
N2f = 3.002 × 10
T2f = 682.947

Compare with ideal gas result


T1f = 560.7 and T2f = 708.7

So the results are not drastically different than the ideal gas result.

Note that in the next problem we solve this using the P-R EOS. In that case the
final pressure is 37.9 bar, T1f = 544.3 and T2f = 686.4, which are close to the results
obtained here (except for the pressure).
6
6.61 Reactor is I, initially evacuated tank is II. The mass balance, as in Problem 4.45, is
Ni = N I + N II , except that here, unlike there, we can not use the ideal gas equation of state.
The energy balance of the reactor and tank is NiU i = N IU I + N IIU II
The entropy balance on just the contents of the reactor that undergo a uniform expansion leads to
S I = constant . Using the Peng-Robinson program with heat capacity that is a function of
temperature, we obtain that
m3
V i = 2.8366 ⋅10−4 , so that Ni = 3525 mol
mol
J −4 J
H i = 21418.62 , so that U i = H i − PV
i i = 21418.62 − 200 ⋅ 2.8366 ⋅ 10 ⋅ 10 = 15745.4
5
mol mol
J
S i = 1.84
mol ⋅ K
The calculational procedure is as follows. Guess a final pressure, find the temperature in the reactor
for which the entropy is equal to Si. Using that T and P, calculate (using the P-R EOS) the molar
volume and molar enthalpy, from which the number of moles in the reactor and its internal energy
can be computed. The use the mass balance to compute the number of moles in tank II, the molar
volume in tank II and the known pressure to calculate the temperature in tank II. Then see if the
energy balance is satisfied. If not, repeat the calculation with another guessed value for the final
pressure.

Start with the ideal gas solution of problem 4.45 of P = 40 bar. Find
J
for S i = 1.84 the temperature is T I = 548.2 K
mol ⋅ K
m3
V I = 1.11024 ⋅10−3 , so that N I = 900.7 mol, N II = 3525 − 900.7 = 2624.3 mol,
mol
4 m3
and V II = = 1.5242 ⋅10−3
2624.3 mol
J J
H = 13393.4
I
, so that U = H − P I V I = 13393.4 − 40 ⋅1.11024 ⋅10−3 ⋅105 = 8952.4
I I
mol mol
By trial-and-error, we find that the conditions in the tank II are
m3
for V II = 1.5242 ⋅10−3 , the temperature is T II = 733.3 K
mol
J J
H II = 27465.8 , so that U II = H II − P II V II = 27465.8 − 40 ⋅ 1.5242 ⋅10−3 ⋅105 = 21396
mol mol
and the energy balance is
?
3525 ⋅15745.4 = 5.550 ⋅108 = 900.7 ⋅ 8952.4 + 2624.3 ⋅ 21396 = 6.421 ⋅108
So that the energy balance is NOT satisfied.

Try another guess… P I= 37.9 bar


J
for S i = 1.84 the temperature is T I = 544.3 K
mol ⋅ K
m3
V I = 1.16366 ⋅10−3 , so that N I = 859.4 mol, N II = 3525 − 859.4 = 2665.6 mol,
mol
4 m3
and V II = = 1.5006 ⋅ 10−3
2665.6 mol
J J
H I = 13154.6 , so that U I = H I − P I V I = 13154.6 − 37.9 ⋅1.16366 ⋅10−3 ⋅105 = 8744.3
mol mol
By trial-and-error, we find that the conditions in the tank II are
m3
for V II = 1.5006 ⋅10−3 , the temperature is T II = 686.4 K
mol
J J
H II = 23712.8 , so that U II = H II − P II V II = 23712.8 − 37.9 ⋅1.5006 ⋅10−3 ⋅105 = 18025.5
mol mol
and the energy balance is
?
3525 ⋅15745.4 = 5.550 ⋅108 = 859.4 ⋅ 8744.3 + 2665.6 ⋅18025.5 = 5.556 ⋅108
Which is close enough, so that we can consider the energy balance to be satisfied. The final
conditions then are a pressure of 37.9 bar, TI of 544.3 K and TII of 686.4 K.

This is to be compared with the ideal gas, constant heat capacity solution of a pressure of 40 bar,
TI of 560.7 K and TII of 708.7 K.

To compare better with the


previous ideal gas solution, we
could repeat the calculation with
the constant ideal gas heat
capacity of 73.2 J/mol. Doing
this, we find that the final
pressure is again about 37.9 bar,
TI=546.4 K, and TII=685.0 K.
6
⎛ ∂V ⎞ ⎛ ∂S ⎞
6.62 ⎜ ∂T ⎟ = − ⎜ ∂P ⎟ (eqn. 6.2-13)
⎝ ⎠S ⎝ ⎠V

and eqn. 6.2.19


CV ⎛ ∂P ⎞ 1 ⎛ ∂V ⎞
dS = dT + ⎜ ⎟ dV κT = −
T ⎝ ∂T ⎠ V V ⎝⎜ ∂P ⎠⎟T
⎛ ∂S ⎞ C V ⎛ ∂T ⎞
⎜ ∂P ⎟ = T ⎜ ∂P ⎟
⎝ ⎠V ⎝ ⎠V

( ∂ V / ∂T )S CV ⎛ ∂T ⎞ ⎛ ∂T ⎞
=− ⎜ ⎟
( ∂ V / ∂T )P T ⎜⎝ ∂P ⎟⎠ V ⎝ ∂ V ⎠ P
⎛ ∂T ⎞ ⎛ ∂ V ⎞ ⎛ ∂P ⎞
Triple product Rule ⎜ ⎟ ⎜ ⎟ ⎜ ⎟ = −1
⎝ ∂ V ⎠ P ⎝ ∂P ⎠T ⎝ ∂T ⎠ V
⎛ ∂T ⎞ ⎛ ∂P ⎞ ⎛ ∂T ⎞
So that ⎜ ⎟ = −⎜ ⎟ ⎜ ⎟
⎝ ∂ V ⎠P ⎝ ∂ V ⎠T ⎝ ∂P ⎠ V
( ∂ V / ∂T )S CV ⎛ ∂T ⎞ ⎛ ∂P ⎞
2
CV ⎛ ∂T ⎞
2

a) =+ ⎜ ⎟ =−
( ∂ V / ∂T )P T ⎜⎝ ∂P ⎟⎠ V ⎝ ∂ V ⎠T TVκ T ⎜⎝ ∂P ⎟⎠ V
⎛ ∂V ⎞
b) CV > 0 Since ⎜ ⎟ <0 so κ T > 0
⎝ ∂P ⎠T

⇒ ( ∂ V ∂T )S and (∂V ∂T )P must have opposite signs


6
6.63 Consider the process to be continuous (not a batch piston-and-cylinder
operation). In this case the energy and entropy balances for the adiabatic process
are
dN  +N    
=0=N in out ⇒ N out = − N in = N
dt
dU  + NH
  dS   
=0=W in − NH out ; = 0 = NSin − NSout + Sgen
dt dt
For minimum work, S = 0 so that S = S
gen in out

Using the P-R EOS program, at the initial conditions of 298 K and 0.1 bar
VL = 1.69343¯10-4 m3/mol, HL = -40618.11 J/mol and SL = -103.60 J/(mol K)
Now at 25 bar, by trial and error T = 298.75 K and
VL = 1.68831¯10-4 m3/mol, HL = -40199.36 J/mol and SL = -103.60 J/(mol K).
Therefore W = Hout – Hin = -40199.35-(-40618.11) = 418.8 J/mol
6
6.64
( a)
From Peng-Robinson EOS
−3
−5 VV:= 2.4432310

VL := 8.433⋅ 10

3 4
Ni := Ni = 3.557 × 10 HL := −1457.86
VL
HV := −16758.39
4
Nf := 0.4⋅ Ni Nf = 1.423 × 10

L := Nf V := 0
given
L+ V Nf

⋅ + VL⋅ L
VVV 3

Z := find( L, V) ⎛ 1.347 × 104 ⎞


Z=⎜
L := Z
0
V := Z
1 ⎝ 763.068 ⎠
V
Fraction vapor, f f := f = 0.054
V+ L

so 5.4% of the propane remaining in the tank is vapor


(b)
Mass balance
4
Nout := Ni − Nf Nout = 2.134 × 10

Energy balance 0=L*HL+V*HV+Nout*HV-Ni*HL+Q

Q := Ni⋅ HL − L⋅ HL − V⋅ HV − Nout ⋅ HV

8
Q = 3.383 × 10 J heat must be added to vaporize the propane
6
6.65 In this problem, methane is to undergo an isentropic compression from 20oC and
some unknown initial pressure to 120oC and 4 MPa = 40 bar. The problem is to find
the initial pressure. Using one of the P-R EOS programs and the ideal gas heat
capacity for methane in the PROPERTY program, we have at 40 bar and 120oC =
393.15 K that
S = -21.00 J/(mol K) and H = 3194.23 J/mol. By trial and error, we find that at 10.975
bar and 20oC = 293.15 K
S = -21.00 J/(mol K) and H = -381.95 J/mol. Therefore, the lowest pressure allowed
is 10.95 bar. A lower pressure would result in the temperature from the outlet to the
compressor being above the safety limit.
The work done in the compressor, from the energy balance is
dU 
W J

= 0 = NH  
in − NH out + W; or 
= H out − Hin = 3194.23 − (−381.95) = 3576.18
dt N mol
6
6.66 By the mass and energy balances for a Joule-Thomson expansion we have
H in = H out or H (10 bar, 420 K ) = H ( 0.1 bar, T =? ) . Since the final pressure is so low, we
will assume the exiting gas is ideal. Therefore
(H − H ) IG
10 bar, 420 K
+ H 10
IG
bar, 420 K = H 0.1 bar, T
IG

T T

( H − H IG )
10 bar, 420 K
= H 0.1
IG
bar, T − H 10 bar, 420 K =
IG
∫ CP* dT = ∫ ( 29.93 + 0.093T ) dT
420 K 420 K

=27.93 ⋅ (T − 420 ) +
0.093 2
2
T − 4202 ( )
Now
V (T , P )
⎡ ⎛ ∂P ⎞ ⎤
H (T , P ) − H IG (T , P ) = RT ( Z − 1) +
V =∞
∫ ⎢T ⎜ ⎟
⎣⎢ ⎝ ∂T ⎠V
− P ⎥dV
⎥⎦
and
PV B (T ) B (T ) RT RTB (T )
= Z = 1+ so that Z − 1 = ; P= +
RT V V V V2
⎛ ∂P ⎞ R RB (T ) RT dB (T )
⎜ ∂T ⎟ = V + + 2 ; and
⎝ ⎠V V2 V dT
⎛ ∂P ⎞ RT RTB (T ) RT 2 dB (T ) RT RTB (T ) RT 2 dB (T )
T⎜ ⎟ −P= + + 2 − − = 2
⎝ ∂T ⎠V V V2 V dT V V2 V dT

V (T , P )
RTB (T ) RT 2 dB RTB (T ) dB ⎛ 1 1 ⎞
H (T , P ) − H IG (T , P ) =
V
+ ∫
V =∞
2
V dT
dV =
V
− RT 2 ⎜ − ⎟=
dT ⎝ V ∞ ⎠

RT ⎡ dB ⎤
= ⎢ B (T ) − T dT ⎥
V ⎣ ⎦

Also
256,100 dB (T ) 256,100
B (T ) = 297.6 − ; = ;
T dT T2
dB (T ) 256,100 256,100 2 ⋅ 256,100
B (T ) − T = 297.6 − − = 297.6 −
dT T T T
⎡ dB (T ) ⎤ 2 ⋅ 256,100 cm3 m3
⎢ B (T ) − T dT ⎥ = 297.6 − = −921.9 = −921.9 × 10−6
⎣ ⎦T =420 420 mol mol

from
PV B (T )
= 1+
RT V
One obtains at 420 K and 10 bar
m3 ⎛ −6 m ⎞
3
V = 3.1453 × 10−6 ⎜⎜ Note, for the ideal gas, i.e., B = 0, V = 3.4912 × 10 ⎟
mol ⎝ mol ⎟⎠
RT ⎡ dB ⎤
H (T , P ) − H IG (T , P ) = ⎢ B (T ) − T dT ⎥ = −1023.5
V ⎣ ⎦

So that the equation to be solved is −1023.5 = 27.93 ⋅ (T − 420 ) +


0.093 2
2
(
T − 4202 )
The solution to this equation is T = 404.56 K so that the temperature change is -15.44 K
th
Solutions to Chemical and Engineering Thermodynamics, 4 ed

So that the equation to be solved is −1023.5 = 27.93 ⋅ (T − 420 ) +


2
(
0.093 2
T − 4202 )
The solution to this equation is T = 404.56 K so that the temperature change is -15.44 K

6.67
T B( T)
P( T) := R⋅ + R ⋅ T⋅ Cv( T) := 27.93 + 0.093⋅ T − 8.314
V 2
V
d
B( T)
−5 d R B( T) dT
R := 8.314⋅ 10 P( T) → + R⋅ + R ⋅ T⋅
dT V 2 2
V V
d
⌠ B( T)
⎮ d B( T ) dT
P( T) dV → R⋅ ln( V) − R⋅ − R ⋅ T⋅
⎮ dT V V

B( T) := ⎛⎜ 297.6 −
256100⎞ −6
⋅ 10 d 2561
⎝ T ⎠ B( T) →
dT 2
10000⋅ T

d
DB( T) := B( T) + T⋅ B( T) −4 This combination is a constant
dT DB( 420) = 2.976 × 10

−4
B( 420) = −3.122 × 10
Solving for volumes
420
RTP1 := R⋅
10
V1 :=
⎡ 2 (
⎣ RTP1 + RTP1 − 4⋅ RTP1⋅ B( 420) )0.5⎤⎦ V1 = 3.78 × 10
−3
2

404.56
RTP2 := R⋅
0.1
V2 :=
⎡ 2 (
⎣ RTP2 + RTP2 − 4⋅ RTP2⋅ B( 404.56) )0.5⎤⎦ V2 = 0.337
2

Using Eq. (6.4-17a)

Term1 := 8.314⋅ ln⎛⎜


V2 ⎞
Term1 = 37.325
⎝ V1 ⎠
DB( 420) ⋅ 8.314
Term2 := Term2 = 0.655
V1
DB( 404.56) ⋅ 8.314 −3
Term3 := Term3 = 7.349 × 10
V2
404.56
⌠ Cv( T)
Term4 := ⎮ dT Term4 = −2.171
⎮ T

420

DelS := Term1 + Term2 − Term3 + Term4

J
DelS = 35.801 mol⋅ K
6
6.68 Energy and entropy balances are
dU dV  dS 
= −P = W and = Sgen since system is closed and adiabatic;
dt dt dt
since process is done slowly, we can assume it is done reversibly (Sgen = 0). Therefore the balances become

∆U = W and S=constant
Initial condition is T = 160 K at which the pressure which equals the vapor pressure
is P = 16.14 bar. Using the P-R EOS program, given that only liquid is present, we
have that VL = 4.49387¯10-5 m3/mol, HL = -11187.98 J/mol and
SL = -83.27 J/(mol K). The procedure now is to use the P-R EOS program, guess a
temperature, compute the vapor-liquid equilibrium properties at that temperature
and see if the entropy at 25% vapor and 75% liquid equals the entropy of the initial
state. We find that at T=118.5 K, Pvap = 1.7453 bar
VL = 3.46573¯10-5 m3/mol, HL = -13592.88 J/mol and SL = -100.11 J/(mol K) and
VV = 5.36446¯10-3 m3/mol, HV = -5605.12 J/mol and SV = -32.71 J/(mol K)
and SLV = 0.25SV + 0.75SL= -83.26 J/(mol K).

Now to compute the work, we need the internal energies, which are obtained from U = H
L L V J
– PV from which we obtain Uinit = -12605, U final = -13598, and U final = -6541.3 , and
mol
1000 g J 1000 g
∆U = W = (0.25U fin
V
+ 0.75U fin
L
− U init
L
)× = −573.37 × = −35, 740 J
g mol 16.049 g
16.049
mol mol
7

7.1 (also available as a Mathcad worksheet)


(a) G = H − TS at P = 2.5 MPa and T = 22399. ° C = 49714
. K
G V = H V − TS V = 28031
. − 49714
. × 6.2575 = −307.8 J g
equal with
|UV
G L = H L − TS L = 96211
. − 49714
. × 2.5547 = −307.9 J g
the accuracy
of tables
|W
(b) T (° C) T (K) HV − TS V GL
225 49815
. 2806.3 − 49815
. × 6.2639 = −3141
. J g
250 52315
. . − 52315
28801 . × 6.4085 = −472.5
300 57315
. . − 57315
30088 . × 6.6438 = −7991
.
350 62315
. 3126.3 − . × 68403
62315 . = −1136.2
400 67315 . 3239.3 − 67315 . × 7.0148 = −1482.7
(Note: All Gibbs free energies are relative to the internal energy and entropy of the liquid phase
being zero at the triple point. Since H L ~ U L , and G L = H L − TS L , we have that G L = 0 at
the triple point.)
(c) T (° C) T (K) HL − TS L GV
160 43315 . .
67555 − 43315. × 19427
. = −1659 . J g
170 44315 . 719.21 − 44315 . × 2.0419 = −1857 .
180 45315 . .
76322 − 45315. × 21396
. = −206.3
190 46315 . 807.62 − 46315 . × 2.2359 = −227.9
200 47315 . 852.45 − 47315 . × 2.3309 = −250.4
210 48315 . 897.76 − 48315 . × 2.4248 = −2738 .
RESULTS
(d) T (° C) 150 160 180 200 220 224
c 3
h
V m kg 0.001091 0.001102 0.001127 0.001157 0.001190 0.001197
to
0.07998

T (° C) 225 250 300 350 400


c
V m kg3
h 0.08027 0.08700 0.09890 010976
. 012010
.
FG ∆H IJ =
H (T + ∆T ) − H (T )
(e) Will compute CP from CP ~
H ∆T K P
∆T
T (° C) 150 170 180 190 200 210 224
4.6225
a
CP kJ kg K f 4.328 4.392 4.430 4.472 4.518 4.572 to
3.200
T (° C) 250 300 350 400
a f
CP kJ kg K 2.952 2.574 2.350 2.260
These results are plotted below.
7
dU dV
7.2 Closed system energy balance: = Q − P
dt dt
dS Q 
Closed system entropy balance: = + Sgen
dt T
(a) System at constant volume and constant entropy

dV dS
= 0 and =0
dt dt
dU Q
⇒ = Q and 0 = + Sgen ⇒ Q = −TSgen
dt T
dU
and = −TSgen ; T > 0 ; Sgen ≥ 0
dt
dU
⇒ ≤ 0 or U = minimum at equilibrium at constant V and S.
dt

(b) System at constant entropy and pressure again Q = −TSgen .


dP dV d
Now = 0⇒ P = ( PV ) . Thus
dt dt dt

dU dV d
= Q − P = −TSgen − ( PV )
dt dt dt

and

dU d d dH
+ ( PV ) = (U + PV ) = = −TSgen ≤ 0
dt dt dt dt

Therefore, enthalpy is a minimum at equilibrium at constant S and P.


7
7.3 (a) The condition for equilibrium at constant T and V is that the Helmholtz free energy A shall be a
minimum.
i) Equilibrium analysis (following analysis in text)

dAI =
FG ∂ A IJ I
dT I +
FG ∂ A IJI
dV I +
FG ∂ A IJ
I
dM I
H∂T K I
V,M
H ∂V K I
T, M
H∂ M K I
T ,V

but dT I = 0 , since temperature is fixed, and

FG ∂ A IJI
= − P I and
FG ∂ A IJ
I
= G I
H ∂V K I
T, M
H∂ M K
I
T ,V

Thus, following the analysis in the text, we obtain

c h c h
dA = − P I − P II dV I + G I − G II dM I ⇒ P I = P II and G I − G II

ii) Stability analysis:


Here again we follow analysis in Sec. 7.2—and find

1 2
d A = AVV (dV )2 + 2 AVM (dV )(dM ) + AMM (dM )2 ≥ 0
2

This can be rewritten as

1 2
2
a f
d A = θ 1 dx1 2 + θ 2 dx2 a f 2
≥0

where

AMM AVV − AVM


2
A2
θ 1 = AVV ; θ 2 = = AMM − VM
AVV AVV

and
AVM
dx1 = dV + dM , dx2 = dM
AVV

Thus, θ 1 ≥ 0 and θ 2 ≥ 0

θ1 =
FG ∂ A IJ
2
=
∂ FG ∂ A IJ =

(− P ) ⇒ −
FG ∂ P IJ ≥0
H ∂V K 2
T, M
∂V T, M
H ∂V K T, M ∂V T, M
H ∂V K T, M
FG ∂ P IJ ≤ 0 as previously found
or
H ∂V K T, M
2
AVM
θ 2 = AMM − ≥0
AVV

AVV = −
FG ∂ P IJ ; AVM =
∂ ∂AFG IJ = −FG ∂ P IJ
H ∂V K T, M ∂ M ∂VH K H∂ MK T, M T ,V
;

AMM =
∂ FG ∂ A IJ = FG ∂ G IJ
∂M T ,V
H∂ MK H∂ MK T ,V T ,V

Now using (on a mass basis)

LMFG ∂ G IJ
FG ∂ G IJ =
1
− G
OP
MNH ∂ M K
H∂ MK T ,V
M T ,V PQ

Also, dG = VdP − SdT + GdM
F ∂ G IJ
⇒G =V
FG ∂ P IJ + G and
H∂ MK T, V
H∂ MK T, V

FG ∂ G IJ =
1 ∂P FG IJ = V
∂PFG IJ = AMM
H∂ MK T, V
M
V
∂M H K T, V ∂M H K T, V

so

AMM −
2
AVM
= V
∂P FG IJ −
a
−∂P ∂M f 2

H K −a∂ P ∂ V f
T ,V

AVV ∂M T ,V T, M

=
FG ∂ P IJ LMV + FG ∂ V IJ FG ∂ P IJ OP
H∂ MK T ,V N H ∂ PK H ∂ M K Q T, M T ,V

=G
F ∂ P IJ
H∂ MK T ,V
MNLV − FGH ∂∂ VM IJK OPQ byrule;theEqn.
T,P
triple product
(4.1- 6)

Since

FG ∂ V IJ A2
= V ⇒ AMM − VM = θ 2 = 0!
H∂ MK T,P AVV

(b) The Gibbs free energy must be a minimum for a system constrained at constant T and P
i) Equilibrium analysis

dG I =
FG ∂ G IJ I
dT I +
FG ∂ G IJ I
dP I +
FG ∂ G IJ I
dM I
H∂T K I
P, M
H∂ P K I
T, M
H∂ M K I
T,P

Since T and P are fixed,

dG I =
FG ∂ G IJ I
dM I = G IdM I
H∂ M K I
T,P
Thus
c h
dG = G I − G II dM I = 0 and G I = G II
ii) Stability analysis

1 2
d G = G MM (dM )2 =
∂ 2G FG IJ (dM )2 > 0
2 ∂ M2 H K T,P

Now

FG ∂ G IJ
2
=
FG ∂ G IJ Eqn.

1 LMFG ∂ G IJ OP
− G =
1  
G−G
H∂ M K 2
T,P
H∂ MK T,P
4.9-10 M MNH ∂ M K T,P PQ
M

Thus G MM ≡ 0 , and stability analysis gives no useful information.


7
7.4 (a) At constant M, T and V, A should be a minimum. For a vapor-liquid mixture at constant M, T
and V we have:

A = AL + AV

and at equilibrium dA = 0 = dAL + dA V . Thus

m r m
dA = 0 = − P LdV L − S LdT L + G LdM L + − P VdV V − S VdT V + G VdM V r
but

M = constant ⇒ dM L + dM V = 0 or dM L = −dM V
V = constant ⇒ dV L + dV V = 0 or dV L = −dV V
T = constant ⇒ dT L + dT V = 0
c h c h
⇒ dA = − P L − P V dV L + G L − G V dM L = 0

Since dV L and dM L are independent variations, we have that

P L = P V ; and G L = G V

also T L = T V by constraint that T is constant and uniform.


(b) At constant M, T, and P, G = minimum or dG = 0 or equilibrium.

m r m
dG = V LdP L − S LdT L + G LdM L + V VdP V − S VdT V + G VdM V r
and M = constant ⇒ dM = −dM L V

P = constant ⇒ dP L = dP V = 0
T = constant ⇒ dT L = dT V = 0
c h
⇒ dG = G dM L + G VdM V = G V − G L dM L = 0
L

or G L = G V for vapor-liquid equilibrium at constant T and P.

(Also, T and P are uniform—this is implied by constraints.)


7
7.5 From Sec. 7.2 we have

CP = CV − T
FG ∂ V IJ FG ∂ P IJ 2
= CV − T
FG ∂ V IJ FG ∂ P IJ
2

H ∂ PK H ∂ T K
T V
H ∂ T K H ∂V K
P T

It is the last form of the equation which is useful here now T > 0 and ∂ V ∂ T a f
2
P ≥ 0 . However

FG ∂ P IJ RS< 0
H ∂ V K T= 0 at critical point or limit of stability
T

Thus CP > CV in general; except that CP = CV


i) at the critical point or limit of stability of a single phase.
ii) For the substances with zero valuer (or very small value) of the coefficient of thermal expansion
a fa f
α = 1 V ∂ V ∂ T P such as liquids and solids away from the critical point.
7
7.6 Stability conditions for a fluid are

CV > 0 and
FG ∂ P IJ <0
H ∂V K T

for a fiber these translate to

CL > 0 and
FG ∂ F IJ >0
H ∂ LK T

Now CL = α + βT ; if CL > 0 for all T, then CL > 0 at T ⇒ 0 implies α > 0 ; CL > 0 as T → ∞


a f
implies β > 0 . Also, ∂ F ∂ L T = γT > 0 since T > 0 , this implies γ > 0 .
7
1 P
7.7 dU = Td S − PdV ⇒ d S = dU + dV
T T
FG ∂ S IJ =
1 ∂S P FG IJ
= . These relations, together with the equation
Thus
H∂UK V T
and
∂V U T H K
U V
S = S o + α ln + β ln o (1)
Uo V

will be used to derive the required equation.


[Note that Eqn. (1), which is of the form S = S (U ,V ) is a fundamental equation of state , in the
sense of Sec. 7.2.]
FG ∂ S IJ =α
FG U IJ −1
1 α 1
= = ⇒ U = αT
(a)
H∂UK V
HU K
o
U o
U T
(2)

FG ∂ S IJ =
P

Vo 1

β
= . Thus PV = βT .
(b)
H∂V K U
T V Vo V
(3)

[Clearly, the fluid with an equation of state given by (1) is an ideal gas with constant heat
capacity]
(c) Stability criteria:

FG ∂ S IJ < FG ∂ S IJ FG ∂ S IJ
2 2 2 2
FG ∂ S IJ
2
<0;
FG ∂ S IJ
2
<0
H ∂ U∂ V K H ∂ V K H ∂ U K 2
U
2
V
and
H∂V K 2
U
H∂U K 2
V
Now

FG ∂ S IJ
2
=
∂ α α
= − 2 < 0 ⇒ α > 0 for fluid
U|
H∂U K 2
V
∂U V U U |V to be
FG ∂ S IJ
2
=
∂ β
=−
β
< 0 ⇒ β > 0 || stable
H∂V K 2
U
∂V U V V2 W
[Note: α , β > 0 by problem statement.]
and
∂ 2S
=
∂ ∂S FG IJ =
∂ β
≡0
∂ U ∂ V ∂ U V ∂V H K U ∂U V V
Thus, the stability criteria yield

α=
FG ∂ U IJ = CV > 0 since α and β are positive constants
H∂TK V
from
Eqn. (2)
and

β=−
FG IJ
V2 ∂ P
>0⇒
FG ∂ P IJ <0
T ∂VH K T
H ∂V K T

Thus, fluid is always stable and does not have a first order phase transition.
7

7.8 a
At limit of stability ∂ P ∂ V fT = 0 for the van der Waals equation:
FG P + a IJ (V − b) = RT
H VK 2

So that at limit of stability

FG ∂ P IJ =0=
− RT 2a
+ = 0 ; or
H ∂V K T V −b V3
2a
=
RT
=
1 RT
=
1 FG a
P+ 2
IJ FG IJ
V 3
(V − b)2
V −b V −b V −b H V K H K
Thus P = a(V − 2b) V 3 ; or using

VC 3Vr − 2
a = 3PCVC2 and b = ⇒ Pr =
3 Vr3

To obtain the envelope, we compute Pr for various values of Vr

Vr 10 2 1 0.8 0.7
Pr 0.028 0.5 1.0 0.781 0.0343

a f
Notice, that the critical point Vr = 1, Pr = 1 is the upper limit of metastability (i.e., Pr ≤ 1 ), as well
as the limit of single phase stability.
7
7.9 T and P will be taken as the independent variables at a second order phase transition

G I = G II ; S I = S II , since S = −
FG ∂ G IJ
Then
H∂TK P
and

V I = V II
F ∂ G IJ
, where V = G
H ∂ PK T

and, of course, T I = T II and P I = P II .


From S I = S II we have that along the 2nd order phase transition curve that d S I = d S II or

FG ∂ S IJ
I
FG ∂ S IJ dP = FG ∂ S IJ dT + FG ∂ S IJ
dT +
I II II

H ∂T K H∂ PK H ∂T K H ∂ P K
P T P T
dP


C I
F ∂ V IJ dP = C dT − FG ∂ V IJ dP
dT − G
P
I II
P
II

T H ∂T K T H ∂T K
P P

Thus

FG ∂ P IJ =
CPI − CPII
H∂TK along
transition od
T ∂VI ∂ T i − d∂ V
P
II
∂T it P
(1)
curve

Similarly, equating dV I = dV II yields

FG ∂ V IJ
I
dT +
FG ∂ V IJ I
dP =
FG ∂ V IJII
dT +
FG ∂ V IJ
II

H ∂T K P
H∂ PK T
H ∂T K P
H ∂PK T
dP

Thus

F dP I od
− ∂VI ∂ T i − d∂ V II
it
∂T
H dT K along
transition
=
d∂ V I
T
P
∂ Pi − d∂ V II
∂ Pi
T
P
(2a)
curve

However, since V I = V II , we can divide numerator and denominator by V and obtain

FG ∂ P IJ =
α I − α II
H∂TK along
transition κ TI − κ TII
(2b)
curve
Note: The Clausius-Clapeyron equation is

FG ∂ P IJ =
H I − H II
T
H∂TK along
transition V I − V II
(3)
curve

However, this form is indeterminate for a 2nd order phase transition. Applying L’Hopital’s rule to
eqn. (3) taking derivatives of numerator and denominator with respect to T at constant P

FG ∂ P IJ =
CPI − CPII
T
H∂TK along
transition d i d
∂ V I ∂ T − ∂ V II ∂ T
which is eqn. (1)!
i
curve

Similarly, applying L’Hopital’s rule, but now taking derivatives with respect to P at constant T.

FG ∂ P IJ =
c∂ H I
h c h
∂ P T − ∂ H II ∂ P
H∂TK c∂ V ∂ Ph − c∂ V ∂ Ph
T
T along I II
transition T T
curve

VI − T c∂ V ∂ T h − V + T c∂ V
I II II
∂T h
= P P

c∂ V ∂ Ph − c∂ V ∂ Ph
I
T
II
T

but V I = V II so that

F ∂ P IJ
⇒ TG = −T
d∂ V I
∂T i − d∂ V II
∂T i
H∂TK
P P
along
transition
curve
d∂ V I
∂ Pi − d∂ V
T
II
∂ Pi
T

which is eqn. (2a)!


7

F dP I ∆H dP H − H S . × 105 J kg
335
7.10 (a)
H dT K along
transition
=
T ∆V

d ln T
= L
VL − VS
=
−0.000093 m3 kg
curve

= −361
. × 103 J m3

dP T
= −361
. × 109 J m3 = −361
. × 109 Pa ⇒ P2 − P1 = −361
. × 109 ln 2
d ln T T1

or T2 = T1 exp
RS −2.985 × 10 a P − P fUV
−10

T Pa W 2 1

dP ∆H RT
(b) = but ∆V ≈ V V ~
dT transition T∆V
along
P
curve

dP ∆ HP d ln P ∆ H
= ; =
dT RT 2 dT RT 2

If we assume that ∆H is constant, then

P2
=−
∆H 1 1

FG
⇒ T2 =
1

R P IJ LM OP −1
ln
P1 R T2 T1 H ln 2
T1 ∆ H P1 K N Q
(c) Denver: P2 = 8.46 × 105 Pa

m
. exp −2.985 × 10−10 8.46 × 104 − 1013
T2F = 27315 . c
× 105 hr
. K = 0° C (freezing point essentially unchanged )
= 27315

T2B =
LM 1 − 8.314 × 10 ln 8.46 × 10 OP 3 4 −1
= 368.08 K = 94.9° C
N 37315
. 2.255 × 10 × 18 1013
. × 10 Q
6 5
7
7.11 This problem involves the application of the Clausius-Clapeyron equation. We will assume that the
heats of fusion, sublimation and vaporization are all constant. Thus we will use
P
ln 2 = −
FG
∆H 1 1

IJ
in all cases. Now ∆ vap H = ∆ sub H − ∆ fus H . To calculate ∆ sub H we
P1 H
R T2 T1 K
will use the following sublimation data:

State 1: Triple point; T = 112.9° C = 386.05 K ; P = 1157


. × 104 Pa
State 2: T = 1054. ° C = 37855. K ; P = 8.00 × 10 Pa
3

⎛ 1.157 × 10 ⎞
4
∆ H⎛ 1 1 ⎞
⇒ ln ⎜⎜ 3 ⎟⎟ = 0.369 = − sub ⎜ − ⎟
⎝ 8.00 × 10 ⎠ R ⎝ 386.05 378.55 ⎠
⇒ ∆ sub H = 5.980 × 104 J mol
∆ fus H = 1.527 × 104 J mol ⇒ ∆ vap H = ( 5.980 − 1.527 ) × 104
= 4.453 × 104 J mol

and ∆ vap H R = 5356 K . To find the normal boiling temperature we again use Clausius-Clapeyron
equation.

FG 1013
. × 10 I −4.453 × 10 F 1
5
J = 8.314 GH T − 3861.05IJK
4 State 1 = T.P.
ln
H 1157
. × 10 K
4
2 State 2 = N.B.P.
⇒ T2 = 457.7 K = 184.5° C

Experimental value = 183° C ; difference due to assumption that ∆H vap is a constant.


7

7.12 (a) At equilibrium P sat (ice) = P sat (water )


Equating the ln P sat ’s gives

614.01 5432.8
288962
. − = 26.3026 − ⇒ T = 2731
. °C
T T

and

61401 .
ln P sat (ice) = 288962
. − = 6.4096 ⇒ P sat = 607.7 Pa
2731
.

B d ln P ∆ H
(b) ln P = A − and = also
T dT RT 2

d ln P B
= + 2 ⇒ ∆H = B ⋅ R
dT T

Thus

∆H
= 61401
. and ∆ sub H = 5.105 × 104 J mol
R ice→ vapor

∆H
= 5432.8 and ∆ vap H = 4.517 × 104 J mol
R water → vapor

∆ fus H = ∆ H ice → water


= ∆sub H − ∆ vap H = 5.880 × 103 J mol
7
7.13 (Also available as a Mathcad worksheet. The Mathcad solution includes graphs.)

(a) Use the Clausius-Clapeyron equation

∆ vap H ln ( P2 P1 )
=
R 1 T1 − 1 T2

1
Plotting ln P vs. and graphically taking slope, I find ∆ vap H ~ 42700 J mol .
T
(b) The vapor pressure is low enough that the ideal gas approximation should be valid—thus

d ln P vap ∆ ln P vap ∆ vap H


= =
dT ∆T RT 2

either graphically or analytically, we find

∆ vap H ~ 313, 600 J mol


7
7.14 (a) Start with Eqn. (7.4-6)

RS 1 FV − RT I dPUV ⇒ ln f = F PV − 1I dP
T RT z H P K W P z H RT K
P P
f = P exp
0 0

1 d ( PV ) dV dZ dV
but dP = − = − so
P PV V Z V

z z F I z FGH IJ
Z V V
f dZ PV dV Z P 1
ln =
P Z =1
(Z − 1) −
Z V =∞ RT
−1
V H
= (Z − 1) − ln −
K −
1 V =∞ RT V
dV
K

or

z FGH IJ
V
f 1 RT
= (Z − 1) − ln Z + − P dV
ln
P RT V =∞ V K (Eqn. 7.4-8)

(b) Z = 1 +
B(T )
and P =
RT
1+
B FG IJ
V V V H K
f B
= − ln 1 +
BFG
+
1 IJ z LMNFGH
V
RT RT RT B
− − ⋅
IJ OP
ln
P V V H RT V =∞ K V V V V
dV
KQ
z
V
B 1 ZB
= − ln Z + B 2
dV = − ln Z
V V =∞ V
V
(c) vdW e.o.s.

RT a PV V a
P= − ; Z vdW = = −
V −b V2 RT V − b V 2 RT

z FGH IJ z FGH IJ
V V V
RT RT RT a V a
− P dV = − + 2 dV = RT ln −
V =∞
V K V =∞
V V −b V K
V − b V V =∞
V a
= RT ln −
V −b V

so

f vdW V a
ln = ln − + ( Z − 1) − ln Z
P V − b RTV
F Pb I a RT P
H
= ln Z − ln Z −
RT

K ⋅
RT PV RT
⋅ + (Z − 1) − ln Z

A
= ( Z − 1) − − ln(Z − B)
Z

aP Pb
where A = and B = .
( RT )2
RT
(d) Peng-Robinson equation of state. Start with

z FG RT − PIJ dV = z LM RT − RT +
V
a OPdV
V =∞
HV K N V V − b V (V + b) + b(V − b) Q
V =∞

= RT ln
V

a
lnM
LV + d1 + 2 ib OP
V − b 2 2b MNV + d1 − 2 ib PQ

= RT ln
Z

a LV + d1 + 2 ib OP
ln M
Z − B 2 2b MNV + d1 − 2 ib PQ

[See solution to Problem 6.2 for integral]. Therefore

f PR
= ( Z − 1) − ln Z + ln
Z

a V + 1+ 2 b LM d i OP
ln
P
ln
Z − B 2 2bRT V + 1 − 2 b MN d i PQ
= ( Z − 1) − ln( Z − B) −
a V + 1+ 2 b d i
ln
2 2bRT V + 1 − 2 b d i
7

7.15 (a) f Hliq2S = f Hvap


2S
a f
; f vap = P f P , where the fugacity coefficient, f P will be gotten from
corresponding states.

20
PC , H 2S = 89.42 bar ⇒ Pr = = 0.2237
89.42
. + 27315
255 .
TC , H 2S = 3732
. K ⇒ Tr = = 08002
.
3732
.
ZC , H 2S = 0.284, which is reasonably close to 0.27

f
From Fig. 7.4-1, = 0.765 , f H 2 S = 20 × 0.765 = 153
. bar .
P
(b) For a liquid, from Eqn. (7.4-18)

FfI LM OP
MN z
P
V
f = P vap
H PK sat
exp
P vap
RT
dP
PQ
Since P vap = 6.455 × 103 Pa at the temperature of interest, we will assume that a f Pf
sat ~ 1.
Also, we will consider the liquid to be incompressible. Thus

c h
z z V P − P vap
P P
V V
dP = dP =
P vap
RT RT P vap
RT

and

LMV c P − P h OP = 6455 expL 0.018( P − 6455) OPa


vap
f H 2 S = P vap exp
MN RT PQ MN8.314 × 10 × 310.6 PQ 3

so that

Pressure, Pa fH2S, Pa Reported


P = 10
. × 10 7 6,921 6,925
. × 107
50 9,146 9,175
. × 108
10 12,960 12,967
7
7.16 (also available as a Mathcad worksheet)
(a) There are (at least) two ways to solve this problem. One way is to start from
RS I UV
z FH
P
1 RT
f = P exp
T
RT 0
V−
P K W
dP

or

z FH I
P
f RT
RT ln
P
=
0
V−
PKdP

RT 8.314 × 10−6 MPa ⋅ m3 mol K × (27315


. + 400)K 0.310748 3
= −3
= m kg
P P(MPa ) × 18.01 g mol × 10 kg g P
From Steam Tables T = 400° C

P MPa V m3 kg V − RT P
0.01 31.063 –0.0118
0.05 6.029 –0.00596
0.10 3.103 –0.00448
0.2 1.5493 –0.00444
0.3 1.0315 –0.00433
0.4 0.7726 –0.00427
0.5 0.6173 –0.00420
0.6 0.5137 –0.00421
0.8 0.3843 –0.00413
1.0 0.3066 –0.00415
1.2 0.2548 –0.00416
1.4 0.2178 –0.00416
1.6 0.19005 –0.00417
1.8 0.16847 –0.00417
2.0 0.15120 –0.00417

By numerical integration of this data we find that

f
RT ln ~ −0.0084 MPa ⋅ m3 kg
P
f −0.0084 MPa ⋅ m3 kg
⇒ ln = = −0.027032
P 0.310748 MPa ⋅ m3 kg

so f P = 0.97333 and f = 1947. MPa . A second way to use the steam tables is to assume that
steam at 400°C and 0.01 MPa is an ideal gas. From the steam tables, at these conditions, we have
H = 3279.6 kJ kg ; S = 9.6077 kJ kg K
⇒ G = H − TS = 3279.6 − 67315
. × 9.6077 = −3187.8 kJ kg
= −57412.7 kJ kmol = G(400° C, 0.01 MPa ) = G IG (400° C, 0.01 MPa )
Also

z
2 MPa
G IG (T = 400° C, 2 MPa) − G IG (T = 400° C, 0.01 MPa ) = V IG dP
P = 0.01 MPa

z
2 MPa
RT
G IG (T = 400° C, 2 MPa) = −57412.7 kJ kmol + dP
0.01
P
= −57412.7 + 8.314 ln 200 = −27760.3 kJ kmol

Also, from steam tables

G(T = 400° C, 2 MPa) = (3247.6 − 67315


. × 71271
. ) × 18.01
= −2791563
. kJ kmol
f FG
G − G IG IJ F
−2791563
. + 27760.3 I
P
= exp
HRT
= exp
K H
8.314 × 67315
. K
= 0.9726
f = 0.9726 × 2 MPa = 1945
. MPa
a
(b) Corresponding states TC = 647.3 K, PC = 22.048 MPa, w = 0.344 f
400 + 27315
. 2
Tr = = 104
. ; Pr = = 0.0907
647.3 22.048

From corresponding states chart (actually from Table in Hougen, Watson and Rogatz, Vol. II, p.
601) we have

f
= 0.983 ⇒ f = 1966
. MPa
P

(c) Using the program PR1 we find

f = 19.40 bar = 1940


. MPa

Comment: The steam table results are probably the most accurate, and the corresponding states
results the least accurate. Note that with the availability of the computer program PR1, the P-R
e.o.s. is the easiest to use. The results would be even more accurate if the PRSV equation was
used.
7
PV B C RT BRT CRT
7.17 (a) = 1+ + 2 +" ; ⇒ V = + + +"
RT V V P V P V 2P
Thus

PV B
= 1+
RT RT P + BRT V P + CRT V 2 P + "
C
+ +"
RT P + BRT V P + CRT V 2 P + "

= 1+
a
B P RT f
C P RT 2
+
a f
k p k
1 + B RT P + " −1 + C RT P + " −2 + " 1 + 2 B V + " p
Now keeping terms of order 1, B, B 2 and C only yields

PV P F I F1 − B P + "I + CF P I 2

RT
= 1+ B
RT H K H RT K H RT K
+ cC − B hF
BP PI 2
= 1+
H RT K + "
2
RT

(b) V =
RT
P
c
+ B + C − B2 hFH RTP IK + " and V IG
=
RT
P
. Therefore

hFH RTP IK + " and


V − V IG = B + C − B 2 c
R 1 LB + cC − B h P + "OdPUV
T RT z MN
P
= expS
f
P RT
0
PQ W 2

or

R|
BP C − B
2
c hF P I 2 U|
f
= exp + S| H RT K +" V|
P RT 2 T W
We will consider a number of alternatives for using the virial coefficient data. The first is to
start with Eqn. (7.4-6a)

RS
z FHV − RTP IK dPUVW = expRST RT1 z dV − V idPUVW
P P
f 1
= exp IG
(1)
P RT T 0 0
zd
P
thus we need to evaluate the integral 1 RT i
V − V IG dP . Since the truncated virial equation
0

P RT = 1 V + B V 2 + C V 3 can not easily be solved for V as a function of T and P, the


following procedure will be used:
i) Choose values of V and compute

P=
RT FG
B C
1+ + 2
IJ and P IG
=
RT
V H
V V K V

ii) Plot P and P IG as a function of V


iii) Use these two plots to obtain V IG and V (real gas) at the same value of P, also compute
V (T , P) − V IG (T , P)
iv) Finally, use a numerical or graphical integration scheme to get f P as a function of P

Same representative values of V − V IG are given below

c h
P 106 Pa 1 2 3 4 5 6 7 8
−dV − V i
IG
0.187 0.180 0.187 0.223 0.206 0.211 0.201 0.180
m3 kmol

c h
P 106 Pa 9 10 11 12 13 14 15
−dV − V i
IG
0.1573 0.1384 0.1215 0.1069 0.0944 0.0834 0.0739
m3 kmol

Using the data and performing the integration we obtain

c
P 106 Pa h 1 3 5 7 9 10 11 13 15
f P 0.939 0.822 0.703 0.602 0.527 0.499 0.475 0.439 0.412

FG ∂ P IJ dV and, with
An alternative is to note that dP =
H ∂V K T

RT F B CI F ∂ P IJ = FG − 1 − 2 B − 3C IJ RT
P= G 1 + + J , that G
V H V V K 2
H ∂V K H V V V K T
2 3 4

and
z z
F ∂ P IJ dV − 1 RT dP
z
P V ( P) P
1
RT
cV − V h IG
VG
dP =
1
H ∂ V K RT P
RT
0 V ( 0) T 0

L F 1 2 B 3C I dP O
z z
V ( P) P

= lim M− G + + J dV − P
MN a f H V V V K
P0 → 0
V P0
2
PP
Q
3
P0

L F 1 − 1 IJ + 3C FG 1 − 1 IJ OP
= lim M− ln PV + ln PV a P f + 2 BG
N
P0 → 0 H V ( P ) V a P f K 2 H V ( P) V a P f K Q
0 0
0
2 2
0

2 B 3C
= − ln PV + ln RT + + 2
V 2V
[Note: lim V P0 = ∞ ]
P0 → 0
a f
Thus

f
= exp ln +
RS
RT 2 B 3C
+ 2 =
RT
exp
2 B 3C
+ 2
UV RS UV
P PV V T 2V PV V 2V W T W
but PV RT = 1 + B / V + C / V 2 and

f
=
m
exp 2 B V + 3C 2V 2
=
r
exp −0.3326 V + 0.01938 V 2 m r (2)
P 1+ B V + C V 2 1 − 01663
. V + 0.012921 V 2
for V in m3 kmol .
The use of this equation leads to results that are somewhat more accurate than the graphical
integration scheme. Still another possibility is to use the results of part (a) which yields

f
P
m
= exp −0.00619 P − 10207
. × 10−5 P 2 for P in bar r (3)

The results of using this equation are listed below.


a f
Finally we can also compute f P using corresponding states (Figure 7.4-1). For methyl
fluoride TC = 317.7 K and PC = 5875 . MPa (note ZC is unknown is V C has not been
measured). Thus, Tr = (27315
. + 50) 317.7 = 1017
. , and for each pressure Pr can be computed,
and f P found from Fig. 7.4-1.
The results for each of the calculations are given below:

P (bar) 10 30 50 70 100 130 150


eqn. (1) f P 0.939 0.822 0.703 0.602 0.499 0.439 0.412
eqn. (2) f P 0.939 0.822 0.710 0.607 0.503 0.442 0.416
eqn. (3) f P 0.939 0.823 0.715 0.617 0.486 0.376 0.314
Corresponding 0.96 0.85 0.72 0.60 0.47 0.39 0.345
states f P

Note that at low pressure, all the results for f P are similar. At high pressures, however, the
results differ. Equation (3) is approximate, and probably the least accurate. Equation (2) should
be the most accurate, except that there is a question as to how accurate it is to use an equation of
state with only the second and third virial coefficients for pressures as high as 150 bar.
From Steam Tables T = 400° C

P MPa V m3 kg V − RT P
0.01 31.063 –0.0118
0.05 6.029 –0.00596
0.10 3.103 –0.00448
0.2 1.5493 –0.00444
0.3 1.0315 –0.00433
0.4 0.7726 –0.00427
0.5 0.6173 –0.00420
0.6 0.5137 –0.00421
0.8 0.3843 –0.00413
1.0 0.3066 –0.00415
1.2 0.2548 –0.00416
1.4 0.2178 –0.00416
1.6 0.19005 –0.00417
1.8 0.16847 –0.00417
2.0 0.15120 –0.00417

By numerical integration of this data we find that

f
RT ln ~ −0.0084 MPa ⋅ m3 kg
P
f −0.0084 MPa ⋅ m3 kg
⇒ ln = = −0.027032
P 0.310748 MPa ⋅ m3 kg

so f P = 0.97333 and f = 1947. MPa . A second way to use the steam tables is to assume that
steam at 400°C and 0.01 MPa is an ideal gas. From the steam tables, at these conditions, we have
H = 3279.6 kJ kg ; S = 9.6077 kJ kg K
⇒ G = H − TS = 3279.6 − 67315
. × 9.6077 = −3187.8 kJ kg
= −57412.7 kJ kmol = G(400° C, 0.01 MPa ) = G IG (400° C, 0.01 MPa )
Also

z
2 MPa
G IG (T = 400° C, 2 MPa) − G IG (T = 400° C, 0.01 MPa ) = V IG dP
P = 0.01 MPa

z
2 MPa
RT
G IG (T = 400° C, 2 MPa) = −57412.7 kJ kmol + dP
0.01
P
= −57412.7 + 8.314 ln 200 = −27760.3 kJ kmol

Also, from steam tables

G(T = 400° C, 2 MPa) = (3247.6 − 67315


. × 71271
. ) × 18.01
= −2791563
. kJ kmol
f FG
G − G IG IJ F
−2791563
. + 27760.3 I
P
= exp
HRT
= exp
K H
8.314 × 67315
. K
= 0.9726
f = 0.9726 × 2 MPa = 1945
. MPa
a
(b) Corresponding states TC = 647.3 K, PC = 22.048 MPa, w = 0.344 f
400 + 27315
. 2
Tr = = 104
. ; Pr = = 0.0907
647.3 22.048

From corresponding states chart (actually from Table in Hougen, Watson and Rogatz, Vol. II, p.
601) we have

f
= 0.983 ⇒ f = 1966
. MPa
P

(c) Using the program PR1 we find

f = 19.40 bar = 1940


. MPa

Comment: The steam table results are probably the most accurate, and the corresponding states
results the least accurate. Note that with the availability of the computer program PR1, the P-R
e.o.s. is the easiest to use. The results would be even more accurate if the PRSV equation was
used.
7

7.18 (a) Assume the vapor phase is ideal, and that ∆ vap H is approximately constant (or an average
∆ vap H can be used).

P2
=−
FG
∆ H vap 1 1

IJ
ln
P1 R H
T2 T1 K (1)

⎛ 2.026 ⎞ −∆ vap H ⎛ 1 1 ⎞
⇒ ln ⎜ ⎟= ⎜ − ⎟
⎝ 1.013 ⎠ R ⎝ 222.0 + 273.15 178.0 + 273.15 ⎠
∆ vap H
⇒ = 3.52 × 103 K ∆ vap H = 2.93 × 104 J mol
R

(b) ∆ vap H (T ) = H ( sat. vap,T ) − H ( sat. liq,T )

= ⎡ H ( sat. vap, T ) − H IG (T ) ⎤ − ⎡ H ( sat. liq., T ) − H IG (T ) ⎤


⎣ ⎦ ⎣ ⎦
⎡⎛ H − H IG ⎞ ⎛ H − H IG ⎞ ⎤
⇒ ∆ vap H (T ) = TC ⎢⎜ ⎟ −⎜ ⎟⎟ ⎥
⎢⎜⎝ TC ⎟ ⎜
⎠sat. vap, T ⎝ T ⎠sat. liq., T ⎥⎦

T 200 + 27315.
(c) Tr = = = 0851
.
TC 2831 . + 27315
.

FG H − H IJ
IG
= 506
FG H − H IJ
IG
= 44.69 J mol K
H T K C sat. vap
Tr =0.851
. J mol K and
H T K C sat. liq.
Tr =0.851

∆ vap H (T ) = 556.45 K [ 44.69 − 5.06] = 2.205 × 104 J mol

(d) The reason for the discrepancy is probably not the inaccuracy of corresponding states (since
ZC = 0.272 which is close to 0.27) but rather the assumption of an ideal vapor phase in the
Clausius-Clapeyron equation. We correct for gas-phase nonideality below.

at T = 178° C , Tr = 0811
. , Z = 08
. 2
T = 222° C , Tr = 0890
. , Z = 0.71

The average value of the compressibility is

1
Z= (082
. + 0.71) = 0.765
2
We now replace eqn. 1 with

P2 −∆ vap H ⎛ 1 1 ⎞
ln
P1
=
ZR ⎝ T2 T1 ⎠
4
(
⎜ − ⎟ ⇒ ∆ vap H = 0.765 × 2.93 × 10 J mol = 2.24 × 10 J mol
4
)
which is in much better agreement with the result of part (c). A better way to proceed would be
to compute the compressibility as a function of temperature, i.e., find Z = Z (T , P) and then
integrate
dP ∆ vap H P
=
dT Z (T , P ) RT 2

rather than use an average value of Z, i.e. Z .


7

7.19 Basis: vessel volume 1 m3 (cancels out of problem)


x = fraction of vessel filled with liquid water
x (1 − x)
N = total number of moles of water = L + (per 1 m3 of vessel)
V VV
1) Total number of moles same at all conditions
2) x is the same at initial loading and at critial point
State 1—low pressure V V >> V L

x (1 − x) x
⇒ N1 = + ≈ L
VL VV V

1− x x 1
State 2—Critical point V V = V L = V C ⇒ N 2 = + = but
VC VC VC

x 1 VL
N1 = N 2 ⇒ L
= or x =
V VC VC

(a) Using steam tables

V L (25° C) = 0.001003 m3 kg ; V C = 0.003155 m3 kg


0.001003
x= = 0.3179
0.003155

⇒ Initial fill should contain 31.79% of volume with liquid (which was reported in the Chemical
and Engineering News article).
(b) Peng-Robinson equation of state

V L (25° C) = 0.2125 × 10−4 m3 mol

and the P-R equation of state predicts ZC = 0.3074 (solution to Problem 4.11b) so

ZC RTC 0.3074 × 8.314 × 10−6 MPa m3 mol K × 647.3 K


VC = =
PC 22.048 MPa
= 0.75033 × 10−4 m3 mol ⇒ x = 0.2832

or an initial fill of 28.32% of volume with liquid.


7
7.20 (a) One theory for why ice skating is possible is because ice melts due to the pressure put upon it
under the ice skates, and then refreezes when skate leaves and the pressure is released. Skate
actually moves over a film of water on the sheet of ice. To find the lowest temperature we use
the Clapeyron equation to calculate the change in freezing point as a result of the applied
pressure. Properties of ice:

ρ = 0.90 g cc ⇒ V S = 111
. cc g
∆ fus Hˆ = ∆sub Hˆ − ∆ vap Hˆ = 2834.8 − 2501.3 = 333.5 J g (at 0.01°C)
(Appendix III)
FG ∂ P IJ =
∆H

∂P
=
3335. Jg
H∂TK sat T ∆V  ∂ T .
27315 K × ( . ) cc g
1 − 111
= −1110
. J cc K = −1114
. bar K
FG ∂ T IJ ~ −0.0090 K bar ⇒ ∆T = −0.009 K bar (∆P)
or
H ∂ PK
Assume 70 kg person on 0.6 cm2 skate area (well sharpened)

70
∆P = = 116.7 kg cm2 × 0.9807 bar kg cm2 = 114.4 bar
0.6
⇒ ∆T = −103
. °C

assuming skate makes complete contact with ice. If the surface is irregular (as it is) maybe
contact only over 10% of area. In this case ∆T = −9.3° C . My observation in Minnesota was
that it was possible to skate down to ~ −20° C (5% contact area??). Of course, the
thermodynamic model for this process may be incorrect. Other possibilities include the melting
of ice as a result of friction, or by heat transfer from the skater’s foot to the ice. I believe the
thermodynamic theory to be the a reasonable explanation of the phenomena.
(b) Since ∆ fus Hˆ > 0 and ∆ fusVˆ > 0 for CO2 and most other materials, freezing point will be
elevated not depressed. Liquid film can not form and ice skating is impossible.
(c) More difficult to quantify. Similar to (a) for freezing point depression, which on release of
pressure causes refreezing and formation of snowball; but in this case there is also considerable
heat transfer from the hands to the surface of the snowball that causes melting.
th
Solutions to Chemical and Engineering Thermodynamics, 4 ed.

7.21 (also available as a Mathcad worksheet)


5
From Eq. 7.7-4 ∆ T=∆ P*∆ V*T/∆ H bar := 10 ⋅ Pa

⎛ −6 m
3⎞
Water 1000⋅ bar ⋅ ⎜ −0.0906⋅ 10 ⋅ ⋅ 273.1⋅ K
∆T :=
⎝ g ⎠ ∆T = −7.412K
joule
333.8⋅
g
⎛ −6 m ⎞
3
Acetic acid ⎜
1000⋅ bar ⋅ 0.0159510
⋅ ⋅ ⋅ 289.8⋅ K
∆T :=
⎝ g ⎠
∆T = 2.472K
joule
187⋅
g
⎛ −6 m ⎞
3
Tin ⎜
1000⋅ bar ⋅ 0.0038910
⋅ ⋅ ⋅ 505.0⋅ K
∆T :=
⎝ g ⎠
∆T = 3.352K
joule
58.6⋅
g
⎛ −6 m ⎞
3
Bismuth 1000⋅ bar ⋅ ⎜ −0.00342⋅ 10 ⋅ ⋅ 544⋅ K
∆T :=
⎝ g ⎠ ∆T = −3.53K
joule
52.7⋅
g
th
Solutions to Chemical and Engineering Thermodynamics, 4 ed.

7.22 (also available as a Mathcad worksheet)

Ag(l) Tmax 2485 Tmin 1234 a 14260 b 0.458 c 0


Tmax Tmin .
Yi ( i) Tmin
10 T, K ∆ Hsub, J/mol
Yi ∆ H Yi , a , b , c

1.234 . 10 1.139 . 10
3 5

1.359 . 10 1.134 . 10
3 5

1.484 . 10 1.129 . 10
3 5

1.609 . 10 1.124 . 10
3 5

1.734 . 10 1.12 . 10
3 5

1.86 . 10 1.115 . 10
3 5

1.985 . 10 1.11 . 10
3 5

2.11 . 10 1.105 . 10
3 5

2.235 . 10 1.1 . 10
3 5

2.36 . 10 1.096 . 10
3 5

2.485 . 10 1.091 . 10
3 5

BeO(s) Tmax 2800 Tmin 298 a 34230 b 0.869 c 0


Tmax Tmin .
Yi ( i) Tmin
10 T, K ∆ Hm, J/mol
Yi ∆ H Yi , a , b , c
298 2.824 . 10
5
548.2
2.806 . 10
5
798.4
2.788 . 10
5
1.049 . 10
3

2.77 . 10
5
1.299 . 10
3

2.752 . 10
5
1.549 . 10
3

2.734 . 10
5
1.799 . 10
3

2.716 . 10
5
2.049 . 10
3

2.698 . 10
5
2.3 . 10
3

2.68 . 10
5
2.55 . 10
3

2.662 . 10
5
2.8 . 10
3

2.644 . 10
5
th
Solutions to Chemical and Engineering Thermodynamics, 4 ed.

Ge(s) Tmax 1210 Tmin 298 a 20150 b 0.395 c 0


Tmax Tmin .
Yi ( i) Tmin
10 T, K ∆ Hm, J/mol
Yi ∆ H Yi , a , b , c
298 1.665 . 10
5
389.2
1.662 . 10
5
480.4
1.659 . 10
5
571.6
1.656 . 10
662.8 5

1.654 . 10
754 5
845.2
1.651 . 10
5
936.4
1.648 . 10
5
1.028 . 10
3

1.645 . 10
5
1.119 . 10
3

1.642 . 10
5
1.21 . 10
3

1.639 . 10
5

1.636 . 10
5

Mg(s) Tmax 924 Tmin 298 a 7780 b 0.371 c 0


Tmax Tmin .
Yi ( i) Tmin
10 T, K DHm, J/mol
Yi ∆ H Yi , a , b , c
298 6.376 . 10
4
360.6
6.357 . 10
4
423.2
6.338 . 10
4
485.8
6.318 . 10
548.4 4

6.299 . 10
611 4
673.6
6.28 . 10
4
736.2
6.261 . 10
4
798.8
6.241 . 10
861.4 4

924
6.222 . 10
4

6.203 . 10
4

6.183 . 10
4
th
Solutions to Chemical and Engineering Thermodynamics, 4 ed.

Mg(l) Tmax 1380 Tmin 924 a 7750 b 0.612 c 0


Tmax Tmin .
Yi ( i) Tmin
10 T, K DHm, J/mol
Yi ∆ H Yi , a , b , c
924 5.973 . 10
4

969.6
5.95 . 10
4
1.015 . 10
3
5.927 . 10
4
1.061 . 10
3
5.904 . 10
4
1.106 . 10
3
5.88 . 10
4
1.152 . 10
3
5.857 . 10
4
1.198 . 10
3
5.834 . 10
4
1.243 . 10
3
5.811 . 10
4
1.289 . 10
3
5.788 . 10
4
1.334 . 10
3
5.764 . 10
4
1.38 . 10
3
5.741 . 10
4

NaCl(s)
0.46 . 10
3
Tmax 1074 Tmin 298 a 12440 b 0.391 c
Tmax Tmin .
Yi ( i) Tmin
10 T, K DHm, J/mol
Yi ∆ H Yi , a , b , c
298 1.021 . 10
5
375.6
1.017 . 10
5
453.2
1.012 . 10
5
530.8
1.006 . 10
5
608.4
1 . 10
686 5
763.6
9.94 . 10
4
841.2
9.871 . 10
4
918.8
9.799 . 10
996.4 4

1.074 . 10
3
9.721 . 10
4

9.639 . 10
4

9.552 . 10
4
th
Solutions to Chemical and Engineering Thermodynamics, 4 ed.

Si(s)
Tmax 1683 Tmin 1200 a 18000 b 0.444 c 0
Tmax Tmin .
Yi ( i) Tmin
10 T, K DHm, J/mol
Yi ∆ H Yi , a , b , c

1.2 . 10 1.452 . 10
3 5

1.248 . 10 1.45 . 10
3 5

1.297 . 10 1.449 . 10
3 5

1.345 . 10 1.447 . 10
3 5

1.393 . 10 1.445 . 10
3 5

1.442 . 10 1.443 . 10
3 5

1.49 . 10 1.442 . 10
3 5

1.538 . 10 1.44 . 10
3 5

1.586 . 10 1.438 . 10
3 5

1.635 . 10 1.436 . 10
3 5

1.683 . 10 1.434 . 10
3 5

7.23
The metal tin undergoes a transition from a gray phase to a white phase at 286 K at ambient
pressure. Given that the enthalpy change of this transition is 2090 kJ/mole and that
the volume change of this transition is -4.35 cm3/mole, compute the temperature at which
this transition occurs at 100 bar.

5
From Eq. 7.7-4 ∆ T=∆ P*∆ V*T/∆ H bar := 10 ⋅ Pa

⎛ −6 m
3 ⎞
99⋅ bar ⋅ ⎜ −4.35⋅ 10 ⋅ ⋅ 286⋅ K
∆T :=
⎝ mole ⎠
∆T = −5.893 × 10
−3
K
joule
2090000⋅
mole

7.24 For the solid-liquid transition

FG ∂ P IJ =
∆Hf

∂PFG IJ =
∆Hf
=
127 J g
= 964.3 J cc
H∂TK eq T∆V H
∂ ln T K eq ∆V .
01317 cc g
FG ∂ P IJ = 964.3 J cc = 964.3 × 106 J m3 = 9643 bar = 9.643 × 108 Pa
H ∂ ln T K eq
T2
⇒ P2 = P1 + 9643 × ln
T1

⇒ T TP = T1 exp
RS P − P UV = 278.7 expRS P − 10 Pa UV
TP
1
TP 5
(1)
T 9.643 × 10 Pa W 8
T 9.643 × 10 Pa W 8
7
7.23

The metal tin undergoes a transition from a gray phase to a white phase at 286 K at ambient
pressure. Given that the enthalpy change of this transition is 2090 kJ/mole and that
the volume change of this transition is -4.35 cm3/mole, compute the temperature at which
this transition occurs at 100 bar.

5
From Eq. 7.7-4 ∆ T=∆ P*∆ V*T/∆ H bar := 10 ⋅ Pa

⎛ −6 m
3 ⎞
99⋅ bar ⋅ ⎜ −4.35⋅ 10 ⋅ ⋅ 286⋅ K
∆T :=
⎝ mole ⎠ −3
∆T = −5.893 × 10 K
joule
2090000⋅
mole
7
7.24 For the solid-liquid transition

FG ∂ P IJ =
∆Hf

∂P FG IJ =
∆Hf
=
127 J g
= 964.3 J cc
H∂TK eq T∆V ∂ ln TH K eq ∆V .
01317 cc g
FG ∂ P IJ = 964.3 J cc = 964.3 × 106 J m3 = 9643 bar = 9.643 × 108 Pa
H ∂ ln T K eq
T2
⇒ P2 = P1 + 9643 × ln
T1

⇒ T TP = T1 exp
RS P − P UV = 278.7 expRS P − 10 Pa UV
TP
1
TP 5
(1)
T 9.643 × 10 Pa W 8
T 9.643 × 10 Pa W 8

For the solid-vapor transition, assuming an ideal vapor phase

P2
=−
∆ H sub 1 1

FG IJ
ln
P1 R T2 T1 H K
∆ H sub lna P P f ln 26.67 1333
. a f
⇒− = 2
= 1
= −5513 K
R 1 T2 − 1 T1 .
3696 × 10−3 − 3822
. × 10−3

⇒ ln
P2
= −5513
1 1

FG IJ
P1 T2 T1 H K
and

T TP =
LM 1 − 01814 × 10 −3 P TP OP −1
=
1000
NT 1
. ln
P1 Q .
3696 − 01814
. c
ln P TP 26.67 h (2)

Solving Eqns. (1) and (2) simultaneously gives

P TP = 0.483 bar = 4.83 kPa and T TP = 278.7 K

[The melting temperature of benzene ~ triple point temperature = 553


. ° C = 278.7 K , which agrees
exactly with our prediction].
7
7.25 First, at 298.15 K, lets relate the Gibbs free energy at any pressure to the value
given at 1 bar.
F ( P − 1)bar × 3510 kg × 1000 g × 1 mol I
. , P) = G (298, P = 1 bar ) + G
G (29815
G m kg 12 g J
3
JJ
dia dia
GH 8.314 × 10 mol bar m
−5
K
× 29815
. K
3

K
F ( P − 1)bar × 2220 kg × 1000 g × 1 mol I
G
G (29815, P) = G (298, P = 1 bar ) + G
m kg 12 g J
3
JJ
g g
GH 8.314 × 10 mol K × 29815
bar
−5 m 3
. K K
Note that
1 m3 1 kg g m3
V dia = × × 12 = 34188
. × 10−6
3510 kg 1000 g mol mol
1 m3 1 kg g m3
Vg = × × 12 = 54054
. × 10−6
2220 kg 1000 g mol mol
Therefore
G dia (29815
. , P) − G g (29815, P) = G dia (298, P = 1 bar ) − G g (298, P = 1 bar )
F ( P − 1)bar × (34188 m I 3

G
+G
. − 54054
. ) × 10
mol J
−6

JJ
GH 8.314 × 10 mol bar m
−5
K
3
× 29815
. K K
at equilibrium at 298.15 K we have
0 = 2900 - 0 - ( P -1) × 8.0143 × 10-5
2900
P = 1+ = 36185
. × 106 bar = 36.185 Mbar
8.0143 × 10−5

To find the transition pressure at other temperatures we use the Clapeyron


equation
J
(2.377 − 5740
. )
F ∂P I ∆S Pa m3
H ∂T K =
sat ∆V
= mol K
(3.4188 - 5.4054) × 10−6
m3
× 1
J
mol
6 Pa bar
= 16928
. × 10 = 16.928
K K
which indicates that for every degree K increase about 298.15 K we need to
increase the pressure
by 16.928 bar. However, this is a small percentage increase compared to the
36.185 MPa pressure
required at 298.15 K. So the transition is essentially (within engineering accuracy) only very
weakly dependent on pressure.
7
7.26 Mass balance: M f = Mi
Energy balance: U f = Ui
Equilibrium criterion: G S = G L = G V ; also S = maximum
We will assume the vapor phase is an ideal gas.
Properties of the triple point (a convenient reference state):

U (liquid, T = 0° C, triple point ) = 0 (reference state)


 
U (solid, T = 0° C) = − ∆H (solid → vapor ) = −335 J g
since ∆V ≅ 0

U ( vapor, T = 0° C) = ∆H vap − RT


1 mol
= 2530 J g − 8.314 × 27315
. J mol ×
18 g
= 24038
. J g

Energy content of original system:

U (liquid, T = −10° C) = 0 + 4.22 J g° C × (−10° C) = −42.2 J g


U (solid, T = −10° C) = −335 J g + 21. J g° C × (−10° C) = −356 J g
U ( vapor, T = −10° C) = 24038. J g + CV (−10° C) where CV ~ CP − R
F 8.314I
= 24038
H
. + 2.03 −
18 K(−10° C) = 23881
. J g

Also, initial specific volume of vapor phase

RT 8.314 Pa ⋅ m3 mol K × 26315


. K
VV = = = 7609 m3 mol
P 287.6 Pa

Ratio of mass initially present in vapor to mass in liquid

. × 10−5 m3 mol
M V V L 18
= = = = 2.37 × 10−6
ML VV 7.609 m3 mol

⇒ Negligible mass of system initially present in gas phase!


Since the initial system is a subcooled liquid + vapor, the following possibilities exist for
the equilibrium state.
(1) All the liquid freezes and some vapor remains (i.e., a solid-vapor system at equilibrium).
The energy released (heat of fusion) would then go to heat the system—Since 335 J/g are
released, and CP ~ 4.22 J g° C , too much energy is released for only the solid and vapor to be
present.
(2) Some of the liquid vaporizes and some freezes, so that a solid-liquid-vapor mixture is
present. Thus, the system is at the triple point at equilibrium. [The energy released in
solidification of the water goes to heat the system up to 0°C, the triple point temperature.]
We will consider this second possibility here; as a first guess, the small amount of vapor will not
be included in the calculations.

Let x = fraction of liquid that solidified


M L = initial mass of liquid in the system

Energy balance

M LU L (−10° C) = (1 − x ) M LU L (0° C) + xM LU S (0° C)


− 42.2 J = (1 − x )(0) + x(−335)
⇒ x = 0126
. fraction that is solid
1 − x = 0874
. fraction that is liquid
Now lets go back and determine the amount of vapor in the system

triple point pressure = 0.611 kPa = 611 Pa


A Steam tables
As a first approximation, assume that the vapor still occupies about 1/2 of the total volume—
this is reasonable, since we expect little volume change of the condensed phase due either to
freezing or thermal expansion.

MV
=
18 c
. × 10−5 m3 mol × 611 Pah = 4.84 × 10−6
M 8.314 Pa ⋅ m3 mol K × 27315
. K

If we now include this amount of vapor in the energy balance, it makes only an insignificant
change in the computed solid and liquid fractions. ⇒ We will neglect the presence of the vapor.
To compute the entropy change, we notice that if we started with 1 gram of liquid, the net
change in the system would be

1 g liquid (−10° C) .
0874 g liquid (0° C)
+ → 0126
. g solid (0° C)
−6
2.37 × 10 g vapor (−10° C, 287.6 Pa) 4.84 × 10−6 vapor (0° C, 611 Pa)

Entropy changes
1 g liquid (–10°C) → 1 g liquid (0°C)

Tf .
27315
∆S = CP ln = 4.22 J gK ln = 01574
. J gK
Ti .
26315

0.126 g liquid (0°C) → 0.126 g solid (0°C)

∆G = 0 = ∆H − T∆S


∆H ∆H
∆S = and ∆S = M
T T
−335 J g
∆S = × 0126
. g = −01545
. J K
27315
. K

(4.84 − 2.37) × 10−6 = 2.47 × 10−6 g liquid (0° C) → 2.47 × 10−6 g vapor (0° C)
∆H vap 2530
∆S = 2.47 × 10−6 = 2.47 × 10−6 × = 0.229 × 10−4 J K
T 27316
.

⇒ vapor makes a negligible contribution in computing ∆S

∆S = 01574
. − 01545
. = 0.0029 J K for each gram of water present.
7
7.27 Gibbs Phase Rule (Eqn. 7.6-1) F = 3 − P
(a) Only solids can have many as 3 phases present. These can only exist at a single point of T and
P as there are no degrees of freedom.
If only 2 solid phases, F = 1 ⇒ a single degree of freedom. Can fix either T or P.
If only one solid phase present, then F = 2 , both T and P can vary independently.
(b) If a liquid is also present
Liquid alone F = 3−1 = 2 T and P independently
variable
Liquid + 1 solid F = 3− 2 = 1 Can vary T or P, the
other follows
Liquid + 2 solids F=0 Only a single point
(c) Same as (b), with vapor replacing liquid in discussion above.
(d) If liquid and vapor are already present, then P = 2
⇒ 1 degree of freedom if no solid is present (i.e., either T or P can be fixed, not both)
If liquid and vapor and solid are present, P = 3
⇒ F = 0 Triple point is a unique point on phase diagram.
7
7.28 Criterion for equilibrium at constant T and V as that A = minimum. However, from Problem 7.4
this implies G V = G L (not AV = A L !)
a f
Now G = A + PV and P = − ∂ A ∂ V T . Thus, at equilibrium

G = A −G
V F ∂ A IJ V = G = A − FG ∂ A IJ V
V
V
V L L
L
L
H ∂V K T
H ∂V K L
T
(1)

Also at equilibrium P = P ⇒ G
V F ∂ A IJ = FG ∂ A IJ
L
V L

H ∂V K H ∂V K
V
T
L
T
⇒ Pressures are equal when derivatives of A with respect to V are equal. The derivative of A with
respect to V is the tangent to the curve in the A − V plane. Thus both curves must have a common
tangent if equilibrium is to occur, and the slope of this tangent line is the negative of the equilibrium
pressure. Now must prove that the points of intersection of the tangent line and A curve are points
at which G V = G L . From the figure we have AV = AL + ∂ A ∂ V a f dV
T
V
i
− V L but this is exactly
what is required by Eqn. (1) for G = G ! So we have indeed identified the equilibrium state.
V L

VL VV

⎛ ∂A⎞ V
⎜⎜

(
⎟⎟ V − V L )
⎝ V ⎠
7
7.29 (a) From the equilibrium criteria we have T V = T L , P V = P L and

G =G ⇒V L
z
V L, T
VdP = 0 or
VV

z FG ∂ P IJ dV = 0
V ,TV
V L
V
H ∂V K T

(b) For the van der Waals equation we have

8Tr 3
Pr = − 3 (1)
3Vr − 1 Vr

and

FG ∂ P IJ = 6 − 24T =6
RS 1 − 4T UV
H ∂ V K V a3V − 1f
r r r

r r
3
r
2
TV a3V − 1f W
r
3
r
2

Thus

z V FGH ∂∂ VP IJK dV = 0 = z FGH V1 − a3V4T−V1f IJK dV


VrV VrV
r r r
r r 2 2 r
VrL r VrL r r

VrV
0=−
1
Vr

4Tr
9
a VrV
ln 3Vr − 1 − 3Vr − 1 −1 L
Vr
f a f
VrL

and

c3V − 1h + c3V
r
L
V
−1 h − c3V − 1h
−1 L −1
+
9 VL
1 − rV = 0
FG IJ
ln
c3V − 1h
r
V r r
4TrVrL
Vr H K (2)

Another equation arises from the fact that P V = P L ⇒ PrV = PrL so that, using Eqn. (1)

8Tr 3 8Tr 3
− = − ,
3Vr − 1 VrV
V
c h 2
3Vr − 1 VrL
L
c h 2

or, solving for the reduced temperature

Tr =
3 {c
1 VrV − 1 VrL h c h}
2 2

mc
8 1 3VrV − 1 − 1 3VrL − 1 h c hr (3)

This is an independent equation between Tr , VrV and VrL . Using Eqn. (3) in Eqn. (2) gives'
R| c3V − 1h U| + c3V − 1h − c3V − 1h
lnS
L

|Tc3V − 1h V|W
r V −1 L
V r r
r
(4)
6V V R 1
V L
1 U
+
r
r r

r
L S r
V

V + V T 3V − 1 3V − 1W
V V=0r
L

(c) Procedure used in solution


i) Guess (or choose) a value of VrV , compute VrL which satisfies Eqn. (4).
ii) Use VrV and VrL so obtained to compute Tr from Eqn. (3).
iii) Use Vr and Tr and Eqn. (1) to get Pr = Prvap . [Note: This calculation was done on a
digital computer.] Results are at end of problem solution.
(d) The Clausius-Clapeyron equation can be written as

dP ∆H P dP vap ∆ H vap
= = C r =
dT coexistance
curve
T∆V TC dTr T∆V

or

dPrvap ∆ H vap 1 ∆ H vap 3V C 9b ∆ H vap


2
= = =
dTr Tr ∆V PC Tr ∆V a a Tr ∆V r

Since a = 3PCVC2 ; VC = 3b . Now 9b a has units of (energy)−1 , so define a reduced heat of


vaporization, ∆Hrvap , to be 9b a ∆ H vap . Thus

dPrvap
∆ H rvap = Tr ∆Vr
dT
Thus ∆Hrvap can be computed by taking derivatives of results of part (c). (This was done
graphically). The results are as shown below.
V rV V rL Prvap Tr Tr ∆Hrvap
3 × 103 0.3690 2.617 × 10−4 0.2948
0.3124 9.1
1 × 103 0.3745 8.772 × 10−4 0.3300
0.3341 8.69
−3
8 × 10 2
0.3758 . × 10
1123 0.3382
0.3477 8.69
−3
5 × 102
0.3789 1894
. × 10 0.3571
0.3686 8.51
3 × 102 0.3829 33503
. × 10−3 0.3802
0.3986 8.77
−3
. × 10
15 2
0.3896 7.299 × 10 0.4171
0.4297 8.87
100 0.3947 0.01153 0.4423
0.4522 8.70
75 0.3985 0.01597 0.4621
0.4704 8.52
60 0.4020 0.02056 0.4787
0.4859 8.54
50 0.4052 0.02527 0.4931
0.5025 8.62
40 0.4095 0.03254 0.5119
0.5251 8.53
30 0.4158 0.04505 0.5383
0.5591 8.85
20 0.4268 0.07112 0.5799
0.5964 8.01
15 0.4365 0.09810 0.6129
0.6389 7.81
10 0.4538 0.1534 0.6649
0.6807 7.51
8 0.4658 0.1954 0.6965
0.7184 7.33
6 0.4849 0.2650 0.7402
0.7550 6.97
5 0.4998 0.3198 0.7697
0.7885 6.64
4 0.5226 0.3996 0.8072
0.8323 5.86
3 0.5596 0.5240 0.8573
0.8922 4.89
2 0.6410 0.7332 0.9270
0.9354 3.94
1.8 0.6710 0.7899 0.9437
0.9567 3.22
1.5 0.7364 0.8830 0.9697
0.9849 1.43
1.0 1.0 1.0 1.0

Results are plotted below.

The reduced vapor pressure and heat of vaporization for the van der Waals fluid.
7
7.30 (a) Restricted form of Gibbs Phase Rule: F = 3 − P
⇒ P must be 3 or less
⇒ quaternary point can not exist in a 1-component fluid.
(b) 2 phases ⇒ F = 3 − 2 = 1 degree of freedom.
Thus, if any property of one of the phases is specified, this is sufficient to fix all of the
thermodynamic properties of both phases! However, if only the total molar volume of the two-
phase system (or some other two-phase property), i.e., V = x IV I (T , P) + x IIV II (T , P) , this is not
sufficient to solve for x I , V I and V II . That is, many choices of T and P can yield the same
value of the total molar volume by varying the mass distribution between the two phases.
Consider now the situation in which the total molar volume and total molar enthalpy is
specified. In this case we have

1 = x I + x II
V = x IV I (T , P) + x IIV II (T , P)
and H = x I H I (T , P) + x II H II (T , P)

The unknowns here are x I , x II and either T or P (note that since the system has only one degree
of freedom, either T or P, but not both are independent variables). Thus, given equation of state
information (relating V and H to T and P, and the two-phase coexistance curve), the equations above
provide 3 equations to be solved for the 3 unknowns.
7
7.31 (a) d S =
FG ∂ S IJ dT +
FG ∂ S IJ
H∂ TK P
H ∂ PK T
dP . Thus

FG ∂ S IJ =
FG ∂ S IJ + FG ∂ S IJ FG ∂ P IJ
H∂ TK sat
curve
H ∂ T K H ∂ PK H ∂ T K
P T
sat
curve

but
FG ∂ S IJ =
CP ∂P FG IJ =
∆ H vap
H∂ TK P T
;
∂T H K sat
curve
T∆V vap
and, by the Maxwell relations
FG ∂ S IJ =−
FG ∂ V IJ = −V α
H ∂ PK T
H∂ TK P

Thus

Csat = T
FG ∂ S IJ = CP − αV
∆ H vap
= CP −
∂V FG IJ ∆ H vap
H∂ TK sat
curve
∆V vap
∂T H K P ∆V vap
and

= CPi − α iV i
∆ H vap
= CPi −
∂V FG IJ i
∆ H vap
H K
i
Csat ;
∆V vap
∂T P ∆V vap
where i denotes the phase.
(b) For the liquid

∆V vap >> V L , and α i ~ 0 ⇒ Csat


L
~ CPL

For the vapor we will use a f a f aT − T f


CP ~ H T2 − H T1 2 1 and
a f
∂ V ∂ T P ~ V T2 − V T1 a f a f aT − T f and 2 1

∆ H vap
V
Csat = CPV − α VV V
∆V vap

[ CP and ∂ V ∂ Ta f P will be evaluated using finite differences above and steam tables. In each
case T1 will be taken as the saturation temperature, and T2 to be the next higher temperature in
the steam tables.]
Thus, at 100°C (0.1 MPa)

2776.4 − 26761
.
CPV = = 2.006 kJ kg K ;
50
F dV I 19364
. − 16958
.
H dT K P
=
50
= 0.00481 m3 kg K

at 370°C (21.0 MPa) CPV = 14.6 kJ kg K and


F I
dV
H K
dT P
= 0.000141 m3 kg K . Thus, at 100°C

2257.0 kJ kg
V
Csat = 2.006 kJ kg K − 0.00481 m3 kg K ×
16719
. m3 kg
= −4.488 kJ kg K

at 370°C

441.6 kJ kg
V
Csat = 14.6 kJ kg K − 0.000141 m3 kg K ×
0.0027 m3 kg
= −8.359 kJ kg K
7
7.32 (a) Multiply out the terms, and this is easily proved.
∂ 2S ∂ 1 1 ∂T F I ∂ 2S FG IJ FG IJ
(b) SUN = =
∂ N∂ U ∂ N U ,V T
=− 2
H K
T ∂ N U ,V
=
∂ U∂ N H K H K
=
∂ FG ∂ S IJ
∂U N ,V
H∂ NK U ,V

but dS =
1 P G
dU + dV − dN ⇒
∂S FG IJ =−
G
T T T ∂N H K U ,V T
so that

∂ 2S ∂ G F I 1 ∂G FG IJ FG IJ
G ∂T
=
∂ U∂ N ∂ U

T H K V,N
=−
T ∂U H K N ,V

H K
T2 ∂ U V ,N

1 F ∂ GI FG ∂ T IJ 1 F ∂ GI F ∂ T I
=− G J
TN H ∂ U K V
+
G
NT 2 H∂U K V
=− G JG J
TN H ∂ T K H ∂ U K V V
+
G
NT 2Cv

Now dG = VdP − SdT ; thus

FG ∂ G IJ =V
FG ∂ P IJ −S
H∂TK V
H∂TK V

and

SUN = −
1 ∂P |RS FG IJ −S +
|UV G
=
−V ∂ P FG IJ +
G + TS
CV NT
V
∂T |T H K V |W NT CV NCv T ∂ T
2
H K V NT 2CV

but G + TS = H . Thus

SUN = −
1 ∂TFG IJ =−
V ∂P FG IJ +
H
H K
T2 ∂ N U ,V NCVT ∂ T H K V NT 2CV
∂ ∂S ∂ P F I = 1 FG ∂ P IJ − P FG ∂ T IJ
(c) SVN = =
∂ N ∂V ∂ N T H K T H∂ NK T H∂ NK U ,V U ,V
2
U ,V

=
∂ ∂S
= −G
F ∂ aG T fIJ = − 1 FG ∂ G IJ + G FG ∂ T IJ
∂V ∂ N H ∂V K T H ∂V KU ,N T H∂V K U ,N
2
U ,N

1 F ∂ GI
=− G J + NTG FGH ∂∂ VT IJK = − NT1 FGH ∂∂ GT IJK FGH ∂∂ VT IJK + NTG FGH ∂∂ VT IJK
NT H ∂ V K U
2
U U U
2
U

=G
F ∂ T IJ 1 LMG − T FG ∂ G IJ OP
H ∂ V K NT MN H ∂ T K PQ
U
2
U
but
LM F ∂ P I OP FG ∂ T IJ ∂P LM F I OP
MN GH ∂ T JK MN GH JK
1
dU = CVdT + T − P dV ⇒ =− −P
V PQ H∂V K U CV
T
∂T V PQ
Also

FG ∂ G IJ =
a
f a f a f
∂ G, U
=
∂ G, U ∂ T , V

H∂TK U a
f a f a f
∂ T, U ∂ T, V ∂ T, U

=
a∂ G ∂ T f a∂ U ∂ V f − a∂ G ∂ V f a∂ U ∂ T f
V T T V
a∂ U ∂ V f T

=G
F ∂ G IJ − C a∂ G ∂ V f ;
H ∂ T K a∂ U ∂ V f
V T

V T

dG = VdP − SdT ; G
F ∂ G IJ = V FG ∂ P IJ − S and FG ∂ G IJ = V FG ∂ P IJ
H∂TK H∂TK V
H ∂V K H ∂V K V T T

Thus,

FG ∂ G IJ = V FG ∂ P IJ − S − C V a∂ P ∂ V f
H∂TK H∂TK
V
T a∂ P ∂ T f − P
T

U V V

G − TG
F ∂ G IJ = H − TS − VT FG ∂ P IJ + T S + C TV a∂ P ∂ V f
H∂TK H∂TK
V
T a∂ P ∂ T f − P
T

U V V

|RST FG ∂ P IJ − P|UV|RSG − T FG ∂ G IJ |UV


|T H ∂ T K |W|T H ∂ T K |W
V U

R| F ∂ P IJ U|VR|ST FG ∂ P IJ − PU|V + C TV FG ∂ P IJ
= S H − VT G
T| H ∂ T K W|T| H ∂ T K W| V
H ∂V K
V
V
T

so that, finally,

SVN =
FG IJ
1 ∂P

FG IJ
P ∂T
H K
T ∂N U ,V
H K
T2 ∂ N U ,V

=−
1 |UVR|ST FG ∂ P IJ
|RSH − VT FG ∂ P IJ −P −
|UV V ∂P FG IJ
NCVT 2 |W|T H ∂ T K
|T H ∂ T K V V |W NT ∂ V H K T

F ∂ S IJ = ∂ RS− G UV = − 1 FG ∂ G IJ
=G
2
+
G ∂T FG IJ
(d) S NN
H∂ N K ∂ N T T W T H∂ NK
2
U ,V U ,V U ,V T2 ∂ N H K U ,V

1 |RF ∂ G I |U G FG ∂ T IJ
S G J − GV +
eqn.
= −
A4-10 TN |TH ∂ N K |W NT H ∂ N K
U ,V
2
U ,V
Now

dG = VdP − SdT + GdN ⇒


FG ∂ G IJ =V
FG ∂ P IJ −S
FG ∂ T IJ +G
H∂ NK U ,V
H∂ NK U ,V
H∂ NK U ,V
⇒ S NN = −
FG IJ
V ∂P FG IJ + G FG ∂ T IJ
+
S ∂T
H K
TN ∂ N H K T N H∂ NK
U ,V TN ∂ N U ,V
2
U ,V

V F ∂ PI H F∂TI V F ∂ PI HF∂TI
=− G J
TN H ∂ N K
+ G J
T N H∂ NK
U ,V
=− G
2
T H∂ NK
J + G J
T H∂ NKU ,V U ,V U ,V

but from above

1 ∂T FG IJ =
V ∂P FG IJ −
H
T2 ∂ N H K U ,V NCVT ∂ T H K V NT 2CV

and, from equating the two expressions for SVN

FG IJ
1 ∂P
=
a
−H ∂ P ∂ T f
V
+
V ∂P FG IJ 2

V ∂P FG IJ
H K
T ∂N U ,V NCVT NCV ∂ T H K V NT ∂ VH K T

Putting these expressions together yields

=
2 HV ∂ P FG IJ −
H2

V2 ∂ P ∂ T a f 2
V
+
FG IJ
V2 ∂ P
S NN
NCVT ∂ T H K V NCVT 2 NCV H K
NT ∂ V T

(e) It is now simple algebra to combine the expressions above, and those in Sec. 5.2, and show that

θ3 =
SUU S NN − SUN
2
− UU VN
a
S S − SUV SUN f 2

SUU b
SUU SUU SVV − SVN
2
g
is exactly zero!!
7
7.33 (Mathcad worksheets in the Mathcad Utilities Directory are also available to do these calculations)
Students in my thermodynamics courses have produced thermodynamics diagrams for many fluids
using the program PR1 and following the methods in illustrations of Chapters 6 and 7. The
following figures are examples of some of these diagrams. It should be noted that all of these
diagrams are in qualitative, but not quantitative agreement with thermodynamic diagrams generated
using more accurate equations of state. In particular, liquid densities are not predicted very
accurately from the Peng-Robinson e.o.s. so that the location of the two-phase dome is somewhat
shifted as are the other thermodynamic properties. Diagrams for other substances will be found in
the file named “Other figs”. Also, the VISUAL BASIC computer program will generate many of
these diagrams quickly and easily.

Thermodynamic properties of nitrogen by Tom Petti


Pressure-volume diagram for nitrogen Peng-Robinson eos.
7

7.34 Thermodynamic properties of water (steam) by Allen Donn.


Pressure-volume diagram of steam computed with the Peng-Robinson equation of state
Temperature-entropy diagram of steam computed with the Peng-Robinson equation of state.

T= 300°C T= 425°C

T= 200°C

T= 150°C
Pressure-enthalpy diagram of steam computed with the Peng-Robinson equation of state.
7
7.35(a) This would be a difficult problem if it were not for the availability of the program PR1. Using this
program, the critical properties and the heat capacity data in the text, and the T = 27315
. K,
P = 1 bar reference state (which cancels out of the problem) we find for ethylene

85 bar and 25° C = 29815


. K
H = −6388 J mol
S = −52.79 J mol K

By trial and error, using guessed values of T until we obtain P vap = 10 bar , we obtain

. K ; S V = −29.44 J mol K ; V V = 01536


T = 22135 . × 10−2 m3 mol ;
S L = −78.71 J mol K ; V L = 05454
. × 10−4 m3 mol .

Now considering the fluid initially in the tank that will be in the tank finally as the system we
have

Ni = N f and S i = S f

(b) Now there can not be only vapor in the tank (entropy too high) or only liquid (entropy too low),
so there must be two phase mixture. Let x L = mass (or mole) fraction of liquid. Thus:

c h
x L S L + 1 − x L S V = S i = −52.79 J mol K

c h
x L (−78.71) + 1 − x L (−29.44) = −52.79
−52.79 + 29.44
xL = = 0.474; x V = 0526
.
−78.71 + 29.44

Thus, 47.4 wt % of fluid in tank is liquid, and 52.6% is vapor. Based on 1 mole in tank we have

V = 0.474 × 05454
. × 10−4 + 0526
. × 01536
. × 10−2 = 8.338 × 10−4 m3 mol
0.474 × 05454
. × 10−4
volume % liquid = × 100 = 31%
.
8.338 × 10−4
volume % vapor = 96.9%
7
7.36 (also available as a Mathcad worksheet)
FG ∂ P IJ =
∆H
H∂TK sat T∆V
ZRT
Assume V V >> V L ⇒ ∆V ~ V V = '
P

FG ∂ P IJ =
∆H

FG
∂ ln P IJ =
∆H
H∂TK sat
2
ZRT P H
∂T K sat ZRT 2

but

FG ∂ ln P IJ ∂ F 5622.7 I
H ∂T K sat
=
∂T
.
H
43552 −
T
− 4.70504 ln T
K
5622.7 4.70504 1
=+ 2
− = 2 (5622.7 − 4.70504T )
T T T

Thus

∆H 1
2
= 2 (5622.7 − 4.70504T )
ZRT T

or

∆H = ZR(5622.7 − 4.70504T ) = 31,602 J mol at 75°C


31,602
Z= = 0.9539
8.314 × (5622.7 − 4.70504 × (27315
. + 75))

but

PV B
= 1 + = Z = 0.9539
RT V

so

B
= 0.9539 − 1 = −0.04607 ; B = −0.04607V
V

Then V = 0.9539 RT P . To find P use


5622.7
ln P vap = 43552
. − − 4.70504 ln(27315
. + 75)
(27315
. + 75)
P vap = 08736
. bar
0.9539 × 8.314 × 10−5 × (27315
. + 75)
V= = 31606
. × 10−2 m3 mol
08736
.

and

B = −1456
. × 10−3 m3 mol
7
7.37
We start with Eqn. (7.7-4), the Clapeyron equation

FG ∂ P IJ
sat
=
∆H
H ∂T K G I = G II
T∆V

From the problem statement ∆H = 48.702 kJ mol , but no data on ∆V is given. Also
P sat
= 1013
. bar at T = 185
. ° C = 29165
. K . Based on other hydrocarbons, we can guess that

∆ fus V ~ 1 to 2 × 10−4 m3 kg

We will use this as an estimate and determine the effect on Tm . Also, the molecular weight of
hexadecane is 226.45. Thus

dP 48.702 kJ mol × 1000 J kJ


=
d ln T δ × 10−4 m3 kg × 22654
. g mol × 1 kg 1000 g
214.98
= × 107 J m3
δ
[where δ is 1 or 2]
214.98 × 10 7
214.98 × 102
= J m3 × 10−2 bar ⋅ m3 kJ × 10−3 kJ J = bar
δ δ
21498 T2 21498
dP = d ln T ⇒ ( P − 1013
. bar ) = ln
δ δ . K
29165

T2 = 29165 LM. )×δ


(200 − 1013
=
294.36 if δ = 1 OP RS
. exp
N
21498 29710
. if δ = 2 Q T
So the freezing point is raised between 2.7 and 5.5 K, depending on the (unknown) value of
∆ fus V .
7
7.38 (also available as a Mathcad worksheet)

This is a one-component adiabatic flash process. I will assume that only vapor + liquid are present,
and then show that this is indeed the case.
There are two ways to solve this problem. One is to calculate all the thermodynamic properties, and
the second is to use the steam tables. Both methods will be considered here
(1) Calculating all thermodynamic properties, and assuming the vapor phase is ideal.
energy balance: 10. U L (T = 95° C) = (10 − x )U L (T ) + xU V (T )
equilibrium requirements: T L = T V ; P L = P V ; and G L = G V ⇒ P = P vap
Also, using data supplied earlier,

F 5432.8 I
H
P vap = exp 14.790 −
T K
and by the ideal gas law

N V RT x 18 mol × 8.314 × 10−5 (bar ⋅ m3 mol K)T


P= =
VV 1 × 10−3 − (10 − x ) 106
volume taken up
by liquid

Equating P and P vap we have

x 18 × 8.314 × 10−5 T F 5432.8 Ix 8314


. T
−3
1 × 10 − (10 − x) 10 6 H
= exp 14.790 −
T K
= ⋅
18 1000 − (10 − x )
Also we have for the internal energies

U L (T = 0° C) = 0 reference state
U L (T = 95° C) = 95° C × 4184
. J g° C = 397.48 J g
U (T ) = (T − 27315
L
. ) × 4184
. assuming C = constant
v
8.314 × 27315
.
U V (T = 0° C) = ∆H vap − RT = 2260 − = 213383
. J g
18
U V (T ) = 213383
. + (T − 27315. ) × 2.09

so that

10 × 397.48 = (10 − x) × 4184


. × (T − 27315
. ) + x 213383
. + (T − 27315
. ) × 2.09

I find that the solution to these equations is

T = 352.68 K and P = 05411


. bar
x = 0.3289 g
This is so far above the melting point of water, that the presence of an ice phase is impossible.
(2) Using the steam tables
energy balance:

10 ⋅ U L (T = 95° C) = 10 × 397.88 = (10 − x)U L (T ) − xU V (T )




both at saturation

also P = P sat (T ) and

V = 0.001 m3 = (10 − x)V L (T ) + xV V (T )

Procedure
i) Guess T, get P sat (T ) , V L (T ) , V V (T ) , U L (T ) and U V (T ) from steam tables
ii) See if Eqns. (1) and (2) are satisfied by using Eqn. (2) to get x, and then seeing if Eqn. (1) is
satisfied.
For example, guess T = 80° C :
V L = 1029
. × 10−6 m3 g U L = 334.86 J g
V V = 3407
. × 10−3 m3 g U V = 2482.2 J g
P = 47.39 kPa

⇒ x(eqn. (2)) = 0.29058 ; x(eqn. (1)) = 0.29348

by iteration and interpolation

T = 79.4° C , P = 0.4739 bar and x = 0.29 grams .

Difference between this solution and the previous one is due to the inaccuracies of the
approximate vapor pressure equation in Part 1, and the assumption of constant heat capacities.
7
7.39 All the P-V data for this problem was obtained with a simple basic language program written for this
problem. Calculations were done for n-butane as a representative fluid. The van der Waals loop
region is shown on the diagram. What is interesting is that, in addition to the van der Waals loop,
there is much structure in the P- V plot. Much of it occurs in the region of b > V and V < 0 , so that
it has no relevance to our calculations. In the region V > b there is only the van der Waals loop
behavior at low reduced temperatures, and the hyperbolic behavior ( PV = RT ) at very high
temperatures. The main point is that the cubic equations we use exhibit quite complicated P- V
behavior, but only relatively simple behavior in the region of interest to us, which is V > b .
P- V diagram for n-butane calculated with the Peng-Robinson equation of state for realizable ( V > b ) and
physically unrealizable ( V < b ) regions.
7
7.40 Let TE = the equilibrium transition temperature when both solid phases are stable.

dG = VdP − SdT

Also dH = TdS + VdP so at constant pressure

FG ∂ S IJ =
FG IJ
1 ∂H
=
Cp
H∂ TK P
H K
T ∂T P T

⇒ phase with higher heat capacity will have a higher entropy since the entropy of both phases are
zero at 0 K.
Then, again at constant pressure

FG ∂ G IJ = −S
H∂TK P

Since both phases have the same Gibbs free energy at the temperature, T, this implies that the
substance with the larger entropy (which arises from larger heat capacity) will have the lower Gibbs
free energy, and therefore be the stable phase.
7
PV B(T ) RT B(T ) RT
7.41 = 1+ ; P= +
RT V V V2
FG ∂ P IJ =−
RT −2 B(T ) RT
<0
H ∂V K T V2
;
V3
2 B(T ) 1 V3 V V
3
> − 2 ; B(T ) > − 2 = − ; B(T ) > −
V V V 2 2 2
Back to virial eq.
PV 2
− V − B(T ) = 0 ; V =
+1 ± 1 + 4 PB RT a f
RT 2 P RT
RT RT 4 PB
V= ± 1+
2P 2P RT

B>−
1 RTLM OPLM
1± 1+
4 PB OP
2 2P N QNRT Q
B RT
>−
1 ± 1 + 4 PB RTa f 4P
fluid will be stable

V
In fact, B(T ) > − is sufficient since B(T ) << V in all conditions where second virial coefficient is
2
used.
V RT
Approximation B(T ) > − ⇒ B(T ) ~> − for stability.
2 2P
7
7.42 Easy way
dU dV dS Q
= Q− P ; = + Sgen
dt dt dt T
System of constant entropy Q = −TSgen
Also constant pressure
dU dV d
= −TSgen − P = −TSgen − ( PV )
dt dt dt
dU d d dH
+ ( PV ) = (U + PV ) = = −TSgen ≤ 0
dt dt dt dt
⇒ H = maximum at equilibrium
dH = 0
d 2H ≥ 0 ⇒
dH = TdS + VdP

d 2H =
FG ∂ T IJ (dS ) + FG ∂ T IJ dSdP + FG ∂ V IJ dPdS +
FG ∂ V IJ (dP)2
H∂ SK
2

P
H ∂ PKS
H∂ SK P
H ∂ PK S

d H =G
F ∂ T IJ a∂ S f ≥ 0 ⇒ FG ∂ T IJ ≥ 0
2
H∂ SK H∂ SK
2

P P

FG ∂ T IJ = T ≥ 0 ⇒ C > 0
H∂ SK C
P P
P

More theoretically correct way


Equilibrium criterion for a closed system at constant entropy and pressure.
dU dV dS Q
= Q− P ; = + Sgen
dt dt dt T
dS
Q=T − TSgen
dt
dU dS dV
=T −P − TSgen
dt dt dt
dS dV d
Constant entropy = 0 ; constant pressure P = ( PV )
dt dt dt
dU d d
⇒ = − ( PV ) − TSgen ; (U + PV ) = −TSgen
dt dt dt
dH dH
= −TSgen ≤ 0 ⇒ ≤ 0 ⇒ H = minimum stability
dt dt
d 2 H > 0 but dH = TdS + VdP + Gi dN
c
d 2 H = HSS
I
+ HSS
II
hc h + c H + H hcdN h
dS I
2 I
NN
II
NN
I 2

+ 2c H + H hdS dN > 0
I
SN
II
SN
I I

N +N
N H cdS h + 2 N H dS dN c h
I II
I 2 2
= I II
I I
SS
I I
SN
I I
+ N I HNN
I
dN I >0
N N
Making a transformation of variables
H
dx1 = dS I + SN dN ; dx2 = dN I
HNN

θ 1 = NHSS ; θ 2 =
c NH SS NHNN − N 2 HSN
2
h
NHSS
As a check
θ 1dx12 + θ 2dx22
FG
= NHSS dS I +
HSN IJ 2
+
NHSS NHNN − N 2 HSN
2
2

H HSS
dN I
K NHSS
dN I

2 H SN I I H2 I2
= NH SSdSI + 2NH SS ⋅ dS dN + NH SS ⋅ SN
2 dN
H SS H SS
2
2 NH SN 2
+ NH NN dN I − dN I
H SS
which is correct so
θ 1dx12 + θ 2dx22 ≥ 0 ⇒ θ 1 > 0; θ 2 > 0

θ 1 = NHSS = N
FG ∂ H IJ = N ∂ FG ∂ H IJ = N FG ∂ T IJ > 0
2

H ∂ S K ∂ S H ∂ S K H∂ SK
2
P P P

but d S =
C
dT − G
F ∂ V IJ ; FG ∂ S IJ = C ⇒ FG ∂ S IJ = NC
H∂ TK H∂ TK T H∂ TK T
P P P
T P P P
T
⇒N > 0 ; N > 0, T > 0 ⇒ CP > 0
NCP
Second criterion
NHSS NHNN − N 2 HSN 2
; HNN =
IJ
∂ 2H
=

(G ) =
∂G FG FG IJ
NHSS K
∂ N S, P ∂ N
2
S, P ∂N H H K S, P

H =
∂ FG ∂ H IJ = ∂ (T ) = FG ∂ T IJ
SN
∂N H∂SK S,P ∂N H∂ NK
P, N
S,P
S,P

NH − N
H
= NG
F ∂ G IJ − N a∂ T ∂ N f
2 2
S,P
=?
H∂ NK a∂ T ∂ S f
SN
NN
H SS S, P P, N
7
RT
7.44 Clausius EOS: P =
V − b(T )
FG ∂ P IJ <0
Condition for stability is
H ∂V K T
For the Clausius equation
FG IJ
∂P
=−
RT
Since R > 0 , T > 0 and (V − b)2 > 0 .
H K
∂V T (V − b)2

Then G
F ∂ P IJ must be negative or
H ∂V K
T

FG ∂ P IJ < 0 ⇒ Single phase is stable at all conditions.


H ∂V KT
7
7.45 See solution to Problem 7.41. If fluid is unstable, then a vapor-liquid phase transition can occur.
7
7.46 Redlich-Kwong equation of state
f
=
1
z
V = ZRT P
FG RT − PIJ dV − ln Z + (Z − 1)
ln
P RT V =∞
HV K
z FGH RTV − PIJK dV = z FGH RTV − VRT− b + V (aV(T+)b)IJK dV
V

V =∞

z
V
V V −b dV
= RT ln − RT ln + a (T )
V →∞ (V − b)V →∞ V =∞
V (V + b)
VF I FG IJ
1 V +b Z a Z+B F I
= RT ln
H K H K
V −b
+ a − ln
b V
= RT ln − ln
Z−B b Z H K
lnF
f Z a Z + BI
Z − B bRT H Z K
ln = ln − − ln Z + ( Z − 1)
P
lnF
a Z + BI
bRT H Z K
= − ln( Z − B) − + (Z − 1)

lnF
a Z + Pb RT I
bRT H K
= (Z − 1) − ln(Z − B) −
Z
aP bP
A= 2
; B=
RT RT
f A F
Z+B I
ln
P
= (Z − 1) − ln(Z − B) − ln
B H
Z K
Using the same analysis for the Soave-Redlich-Kwong equation of state leads to the following
f F Pb I
a(T ) LM a
Z + Pb RT fO
ln = (Z − 1) − ln Z −
P H RT

KRTb
ln
NZ QP
= ( Z − 1) − ln( Z − B) −
a (T ) Z+BLM OP
RTb
ln
Z N Q
th
Solutions to Chemical and Engineering Thermodynamics, 4 ed. Chapter 7

7.46 Redlich-Kwong equation of state


f
=
1
z
V = ZRT P
FG RT − PIJ dV − ln Z + (Z − 1)
ln
P RT V =∞
HV K
z FGH IJ z FGH RTV − VRT− b + V (aV(T+)b)IJK dV
V
RT
− P dV =
V K V =∞

z
V
V V −b dV
= RT ln − RT ln + a (T )
V →∞ (V − b)V →∞ V =∞
V (V + b)

F I FG IJ
V 1 V +b Z a Z+B F I
= RT ln
H K H K
V −b
+ a − ln
b V
= RT ln − ln
Z−B b Z H K
lnF
f Z a Z + BI
ln = ln
P

Z − B bRT H Z K
− ln Z + ( Z − 1)

lnF
a Z + BI
= − ln( Z − B) −
bRT H Z K
+ (Z − 1)

lnF
a Z + Pb RT I
= (Z − 1) − ln(Z − B) −
bRT H Z K
aP bP
A= 2
; B=
RT RT
f A Z+B F I
ln
P
= (Z − 1) − ln(Z − B) − ln
B Z H K
Using the same analysis for the Soave-Redlich-Kwong equation of state leads to
the following
f F Pb I
a(T ) LM a
Z + Pb RT fO
PQ
ln = (Z − 1) − ln Z −
P HRT

K
RTb
ln
Z N
= ( Z − 1) − ln( Z − B) −
a (T ) Z+B LM OP
RTb
ln
Z N Q
7.47 Available as a Mathcad worksheet. See Mathcad worksheet for the graphs.

fi Pi . exp fop i 150

100

P
i
50
10 .Pa
5

50
0 0.5 1 1.5 2 2.5
V
i
log
b
th
Solutions to Chemical and Engineering Thermodynamics, 4 ed. Chapter 7

R.T a Pi . Vi
Pi 3 3 1
0.5 . Zi V100 = 5.655 10 m mole
Vi b T Vi . Vi b R. T

Vi a Vi
fop i ln ln Zi Zi 1 . ln
1.5 .
Vi b T R.b Vi b

fi Pi . exp fop i

500

400

300
P
i

10 .Pa
5

200

100

0
0 0.5 1 1.5 2 2.5
V
i
log
b

7
2.5 10

7
2 10

7
1.5 10

f
i
7
1 10

6
5 10

0
0 0.5 1 1.5 2 2.5
V
i
log
b
7
7.48
a) If ethanol is an ideal gas, the
f = P, so that the fugacity of ethanol is 505 kPa

b) Starting from eqn. 7.4-12 we have that

ln (f/P)=(B/V) - lnZ +(Z-1) mol := 1

P := 505000Pa
⋅ T := ( 273.15 + 126) ⋅ K R := 8.314⋅
(Pa⋅m3) B := −523⋅ 10
−6 m

3

mol⋅ K mol
( R ⋅ T) −3 3
V := V = 6.571 × 10 m
P

Given ( P⋅ V) B
1+
R⋅ T V −3 3
V := Find( V) V = 5.998 × 10 m
( P⋅ V)
Z :=
R⋅ T Z = 0.913

fsat := P⋅ exp⎡⎢ ⋅ −ln( Z) + ( Z − 1)⎤⎥


B
5
⎣V ⎦ fsat = 4.592 × 10 Pa
7
7.49
The density of ethanol is 0.789 g/cc at 20 C which we will also use at 126 C, and its mole
weight is 46.07. Therefore its liquid molar volume is
3 6
⎛ 46.07⋅ 10− 3⋅ kg ⎞ kPa := 10 ⋅ Pa MPa := 10 ⋅ Pa

Vl :=
⎝ mol ⎠
−5 3
3 kg Vl = 5.839 × 10 m
0.789⋅ 10 ⋅
3
m
a)
f := fsat ⋅ exp⎡⎢
[ ( 25⋅ MPa − 505⋅ kPa) ⋅ Vl] ⎤
⎥ 5
f = 7.066 × 10 Pa
⎣ R⋅ T ⎦

b)

VV( p ) := Vl⋅ ⎡⎣ 1 + 1.09⋅ 10 ⋅ ( 101.3⋅ kPa − p ) ⎤⎦


−6 −1
⋅ kPa −5 3
VV( 25⋅ MPa) = 5.681 × 10 m
⎛ 1 ⌠ 25⋅ MPa ⎞
f := fsat ⋅ exp
⎜ ⋅⎮ VV( p ) dp 5
⎜ R⋅ T ⌡ f = 7.024 × 10 Pa
⎝ 505⋅ kPa ⎠
th
Solutions to Chemical and Engineering Thermodynamics, 4 ed. Chapter 7

7.50 FUGACITY CALCULATION USING SRK EQUATION

Read in properties for Pentane Tc := 469.6 Pc := 33.74 om := 0.251

kappa calculation kap := 0.480 + 1.574⋅ om − 0.176⋅ om⋅ om

2 2
R ⋅ Tc R ⋅ Tc
S-R-K Constants: R := 0.00008314 b := 0.08664⋅ ac := 0.42748⋅
Pc Pc
Note that these are being defined as a
2
function of temperature. ⎡ ⎛ ⎞⎤
T
alf( T) := 1⋅ ⎢ 1 + kap⋅ ⎜ 1 − ⎥ a( T) := ac ⋅ alf( T)
⎣ ⎝ Tc ⎠ ⎦
a ( T) ⋅ P P⋅ b
CA( T , P) := CB( T , P) := d
2 R⋅ T Da( T) := a ( T)
( R ⋅ T) dT
Z( T , P) := A ← CA( T , P)
B ← CB( T , P)
⎛ −A ⋅ B ⎞
⎜ 2 Vector of coefficients in the SRK equation
V← ⎜
A − B − B⎟ in the form
⎜ −1 ⎟ 0=-A*B+(A-B^2-B)*Z-*Z^2+Z^3

⎝ 1 ⎠
ZZ ← polyroots ( V) Solution to the cubic
for i ∈ 0 .. 2

(ZZi ← 0) if (Im(ZZi) ≠ 0) Set any imaginary roots to zero


ZZ ← sort ( ZZ) Sort the roots

ZZ ← ZZ ⎛ ZZ < 10− 5 ⎞
if
0 ⎝ 0
2 ⎠ Set the value of any imaginary roots
to value of the real root
ZZ ← ZZ if ⎛ ZZ < 10
− 5⎞
2 0 ⎝ 2 ⎠
ZZ
Enter temperature T, and pressure P.

T := 100
C T := 273.15 + T
K P := 50

Fugacity expressions [actually ln(f/P)] for the liquid fl and vapor fv

CA( T , P) ⎛ Z( T , P) 0 + CB( T , P) ⎞
( ) (
fl( T , P) := Z( T , P) − 1 − ln Z( T , P) − CB( T , P) − ) ⋅ ln⎜
0 0 CB( T , P) ⎜ Z( T , P)
⎝ 0 ⎠

CA( T , P) ⎛ Z( T , P) 2 + CB( T , P) ⎞
( ) (
fv( T , P) := Z( T , P) − 1 − ln Z( T , P) − CB( T , P) − ) ⋅ ln⎜
2 2 CB( T , P) ⎜ Z( T , P)
⎝ 2 ⎠
th
Solutions to Chemical and Engineering Thermodynamics, 4 ed. Chapter 7

Fugacity Fugacity coefficient

fugl
fugl := P⋅ exp( fl( T , P) ) fugl = 6.49272 fl( T , P) = −2.04134 phil := phil = 0.12985
P

fugv := P⋅ exp( fv( T , P) ) fugv = 6.49272 fv( T , P) = −2.04134 fugv


phiv := phiv = 0.12985
P

SUMMARY OF RESULTS

T = 373.15
K Vapor pressure, bar P = 50
LIQUID VAPOR
Compressibility Z( T , P) = 0.23249 Z( T , P) = 0.23249
0 2
Fugacity coefficient phil = 0.12985 phiv = 0.12985

Fugacity, bar fugl = 6.49272 fugv = 6.49272


th
Solutions to Chemical and Engineering Thermodynamics, 4 ed. Chapter 7

Read in properties for Benzene Tc := 562.1 Pc := 48.94 om := 0.212

kappa calculation kap := 0.480 + 1.574⋅ om − 0.176⋅ om⋅ om

2 2
R⋅ Tc R ⋅ Tc
S-R-K Constants: R := 0.00008314 b := 0.08664⋅ ac := 0.42748⋅
Pc Pc
Note that these are being defined as a
2
function of temperature. ⎡
alf( T) := 1⋅ ⎢ 1 + kap⋅ ⎜ 1 −
⎛ T ⎞⎤
⎥ a( T) := ac ⋅ alf( T)
⎣ ⎝ Tc ⎠ ⎦
a ( T) ⋅ P P⋅ b
CA( T , P) := CB( T , P) := d
2 R⋅ T Da( T) := a ( T)
( R ⋅ T) dT
Z( T , P) := A ← CA( T , P)
B ← CB( T , P)
⎛ −A ⋅ B ⎞
⎜ 2 Vector of coefficients in the SRK equation
V← ⎜
A − B − B⎟ in the form
⎜ −1 ⎟ 0=-A*B+(A-B^2-B)*Z-*Z^2+Z^3

⎝ 1 ⎠
ZZ ← polyroots ( V) Solution to the cubic
for i ∈ 0 .. 2

(ZZi ← 0) if (Im(ZZi) ≠ 0) Set any imaginary roots to zero


ZZ ← sort ( ZZ) Sort the roots

ZZ ← ZZ ⎛ ZZ < 10− 5 ⎞
if
0 ⎝ 0
2 ⎠ Set the value of any imaginary roots
to value of the real root
ZZ ← ZZ if ⎛ ZZ < 10
− 5⎞
2 0 ⎝ 2 ⎠
ZZ
Enter temperature T, and pressure P.
T := 100
C T := 273.15 + T
K P := 50

Fugacity expressions [actually ln(f/P)] for the liquid fl and vapor fv

CA( T , P) ⎛ Z( T , P) 0 + CB( T , P) ⎞
( ) (
fl( T , P) := Z( T , P) − 1 − ln Z( T , P) − CB( T , P) −
0 0 ) CB( T , P)
⋅ ln⎜
⎜ Z( T , P)
⎝ 0 ⎠

CA( T , P) ⎛ Z( T , P) 2 + CB( T , P) ⎞
( ) (
fv( T , P) := Z( T , P) − 1 − ln Z( T , P) − CB( T , P) −
2 2 ) CB( T , P)
⋅ ln⎜
⎜ Z( T , P)
⎝ 2 ⎠
th
Solutions to Chemical and Engineering Thermodynamics, 4 ed. Chapter 7

Fugacity Fugacity coefficient

fugl
fugl := P⋅ exp( fl( T , P) ) fugl = 2.01968 fl( T , P) = −3.20908 phil := phil = 0.04039
P

fugv := P⋅ exp( fv( T , P) ) fugv = 2.01968 fv( T , P) = −3.20908 fugv


phiv := phiv = 0.04039
P

SUMMARY OF RESULTS

T = 373.15
K Vapor pressure, bar P = 50
LIQUID VAPOR
Compressibility Z( T , P) = 0.17187 Z( T , P) = 0.17187
0 2
Fugacity coefficient phil = 0.04039 phiv = 0.04039

Fugacity, bar fugl = 2.01968 fugv = 2.01968


th
Solutions to Chemical and Engineering Thermodynamics, 4 ed. Chapter 7

Z( T , P ) A CA ( T , P )
B CB( T , P )
A .B
2
Vector of coefficients in the SRK equation
A B B in the form
V
1 0=-A*B+(A-B^2-B)*Z-*Z^2+Z^3

1
ZZ polyroots ( V) Solution to the cubic
for i ∈ 0 .. 2
ZZi 0 if Im ZZi 0
Set any imaginary roots to zero
ZZ sort( ZZ ) Sort the roots
5
ZZ0 ZZ2 if ZZ0 < 10 Set the value of any imaginary roots
5 to value of the real root
ZZ2 ZZ0 if ZZ2 < 10

ZZ
Enter temperature T, and pressure P.
T 100 C T 273.15 T K P 50

Fugacity expressions [actually ln(f/P)] for the liquid fl and vapor fv

CA ( T , P ) . Z( T , P ) 0 CB( T , P )
fl( T , P ) Z( T , P ) 0 1 ln Z( T , P ) 0 CB( T , P ) ln
CB( T , P ) Z( T , P ) 0

CA ( T , P ) . Z( T , P ) 2 CB( T , P )
fv ( T , P ) Z( T , P ) 2 1 ln Z( T , P ) 2 CB( T , P ) ln
CB( T , P ) Z( T , P ) 2
th
Solutions to Chemical and Engineering Thermodynamics, 4 ed. Chapter 7

7.51 (Solution using Mathcad worksheet)

T = -200oC Pvap = 0.10272 bar


Z V = 0.99512 Z L = 4.414 × 10−4
H V = −55795
. × 103 H L = −12994
. × 104
S V = 17.372 S L = −118.74

T = -180oC Pvap = 1.348 bar


Z V = 0.96359 Z L = 4.955 × 10−3
H V = −51095
. × 103 H L = −11948
. × 104
S V = −32.734 S L = −10615
.

T = -160oC Pvap = 6.750 bar


Z V = 08810
. Z L = 0.02307
H V = −4.7953 × 103 H L = −10805
. × 104
S V = −42.099 S L = −95210
.

T = -140oC Pvap = 20.676 bar


Z V = 0.73096 Z L = 0.07305
H V = −4.7988 × 103 H L = −9.4328 × 103
S V = −49.6785 S L = −84.481

T = -130oC Pvap = 32.310 bar


Z V = 0.61800 Z L = 012528
.
H V = −50406
. × 103 H L = −85449
. × 103
S V = −53938
. S L = −78.418

T = -125oC Pvap = 39.554 bar


Z V = 054226
. Z L = 016843
.
H V = −52985
. × 103 H L = −7.9739 × 103
S V = −56.685 S L = −74.744

T = -120oC Pvap = 47.848 bar


Z V = 0.42788 Z L = 0.24887
H V = −58378
. × 103 H L = −71281
. × 103
S V = −61034
. S L = −69.459
th
Solutions to Chemical and Engineering Thermodynamics, 4 ed. Chapter 7

The Mathcad worksheet for this file is shown below.

7.51 VAPOR PRESSURE CALCULATION USING SRK EQUATION

Read in properties for oxygen Tc := 154.6 Pc := 50.46 om := 0.021

Heat capacity constants


−2 −5 −9
Cp := 25.460 Cp := 1.519⋅ 10 Cp := −0.715⋅ 10 Cp := 1.311⋅ 10
0 1 2 3
Reference state and kappa calculation

Trs := 373.15 Prs := 1.013 kap := 0.480 + 1.574⋅ om − 0.176⋅ om⋅ om


2 2
R ⋅ Tc R ⋅ Tc
S-R-K Constants: R := 0.00008314 b := 0.08664⋅ ac := 0.42748⋅
Pc Pc
Note that these are being defined as a
2
function of temperature since we will ⎡ ⎛
alf( T) := 1⋅ ⎢ 1 + kap⋅ ⎜ 1 −
⎞⎤
T
need to interate on temperature. ⎥ a( T) := ac ⋅ alf( T)
⎣ ⎝ Tc ⎠ ⎦
a ( T) ⋅ P P⋅ b
CA( T , P) := CB( T , P) := d
2 R⋅ T Da( T) := a ( T)
( R ⋅ T) dT
Z( T , P) := A ← CA( T , P)
B ← CB( T , P)
⎛ −A ⋅ B ⎞
⎜ 2 Vector of coefficients in the SRK equation
V← ⎜
A − B − B⎟ in the form
⎜ −1 ⎟ 0=-A*B+(A-B^2-B)*Z-*Z^2+Z^3

⎝ 1 ⎠
ZZ ← polyroots ( V) Solution to the cubic
for i ∈ 0 .. 2

(ZZi ← 0) if (Im(ZZi) ≠ 0) Set any imaginary roots to zero


ZZ ← sort ( ZZ) Sort the roots

ZZ ← ZZ ⎛ ZZ < 10− 5 ⎞
if
0 ⎝ 0
2 ⎠ Set the value of any imaginary roots
to value of the real root
ZZ ← ZZ if ⎛ ZZ < 10
− 5⎞
2 0 ⎝ 2 ⎠
ZZ
Enter temperature T, and pressure P.
Depending on what is specified in the Given and Find statements
below, either T or P is specified and the other is an initial guess T := −173.15 C
which may have to be adjusted as the other variable is changed,
especially as the critical point is approached.This worksheet will
probably not converge to a solution when T or P are within 3 to 5% T := 273.15 + T K
of the critical values unless an extremely good initial guess is
provided. One way to obtain a good initial is to start well below the
critical region and step towards it using the result of previous calculations.
P := 0.15
th
Solutions to Chemical and Engineering Thermodynamics, 4 ed. Chapter 7

Fugacity expressions [actually ln(f/P)] for the liquid fl and vapor fv

CA( T , P) ⎛ Z( T , P) 0 + CB( T , P) ⎞
( ) (
fl( T , P) := Z( T , P) − 1 − ln Z( T , P) − CB( T , P) −
0 0 ) CB( T , P)
⋅ ln⎜
⎜ Z( T , P)
⎝ 0 ⎠

CA( T , P) ⎛ Z( T , P) 2 + CB( T , P) ⎞
( ) (
fv( T , P) := Z( T , P) − 1 − ln Z( T , P) − CB( T , P) −
2 2 ) CB( T , P)
⋅ ln⎜
⎜ Z( T , P)
⎝ 2 ⎠

Given fl( T , P) − fv( T , P) 0 Equate log of fugacity coefficients


Solve equality of fugacities
P := Find( P) P = 2.52595

Fugacity Fugacity coefficient

fugl
fugl := P⋅ exp( fl( T , P) ) fugl = 2.38691 fl( T , P) = −0.05662 phil := phil = 0.94495
P

fugv
fugv := P⋅ exp( fv( T , P) ) fugv = 2.38691 fv( T , P) = −0.05662 phiv := phiv = 0.94495
P
Residual entropy for liquid (DELSL) and vapor (DELSV) phases

⎛ ⎛ Z( T , P) 0 + CB( T , P) ⎞ ⎞
(
DELSL:= ⎜ R⋅ ln Z( T , P) − CB( T , P) + ) ⋅ ln⎜
Da( T) 5
⋅ 10
⎜ 0 b ⎜ Z( T , P)
⎝ ⎝ 0 ⎠⎠

⎛ ⎛ Z( T , P) 2 + CB( T , P) ⎞ ⎞
(
DELSV:= ⎜ R⋅ ln Z( T , P) − CB( T , P) + ) ⋅ ln⎜
Da( T) 5
⋅ 10
⎜ 2 b ⎜ Z( T , P)
⎝ ⎝ 2 ⎠⎠

Residual enthalpy for liquid (DELHL) and vapor (DELHV) phases

⎡ T⋅ Da( T) − a( T) ⎛ Z( T , P) 0 + CB( T , P) ⎞⎥⎤ 5


⎢ (
DELHL:= ⎢R⋅ T⋅ Z( T , P) − 1 +
0 ) b
⋅ ln⎜
⎜ Z( T , P) ⎥
⋅ 10
⎣ ⎝ 0 ⎠⎦

⎡ T⋅ Da( T) − a( T) ⎛ Z( T , P) 2 + CB( T , P) ⎞⎥⎤ 5


⎢ (
DELHV:= ⎢R⋅ T⋅ Z( T , P) − 1 +
2 ) b
⋅ ln⎜
⎜ Z( T , P) ⎥
⋅ 10
⎣ ⎝ 2 ⎠⎦

Ideal gas properties changes relative to the reference state

(
Cp ⋅ T − Trs
1
2 )
2
Cp ⋅ T − Trs
2
( 3 3 ) Cp ⋅ T − Trs
3
( 4 4 )
DELHIG:= Cp ⋅ ( T − Trs ) + + +
0 2 3 4

(
Cp ⋅ T − Trs
2 2 ) Cp ⋅ T − Trs ( 3 3 )
DELSIG:= Cp ⋅ ln⎛⎜
T ⎞
− R⋅ 10 ⋅ ln⎛⎜
2 3 5 P ⎞
+ Cp ⋅ ( T − Trs ) + +
0 ⎝ Trs ⎠ 1 2 3 ⎝ Prs ⎠

Total entropy and enthalpy relative to ideal gas reference state


SL := DELSIG+ DELSL SV := DELSIG+ DELSV HL := DELHIG+ DELHL HV := DELHIG+ DELHV
SUMMARY OF RESULTS
th
Solutions to Chemical and Engineering Thermodynamics, 4 ed. Chapter 7

T = 100 K Vapor pressure, bar P = 2.52595


LIQUID VAPOR
−3
Compressibility Z( T , P) = 8.97042× 10 Z( T , P) = 0.94188
0 2
4 3
Enthalpy, J/mol HL = −1.45382× 10 HV = −7.94408× 10

Entropy, J/mol K SL = −111.53504 SV = −45.59335

Fugacity coefficient phil = 0.94495 phiv = 0.94495

Fugacity, bar fugl = 2.38691 fugv = 2.38691


th
Solutions to Chemical and Engineering Thermodynamics, 4 ed. Chapter 7

7.52

Pure component properties calculation using the SRK equation of state

Read in properties for oxygen Tc := 154.6 Pc := 50.46 om := 0.021

Heat capacity constants


−2 −5 −9
Cp := 25.460 Cp := 1.519⋅ 10 Cp := −0.715⋅ 10 Cp := 1.311⋅ 10
0 1 2 3
Reference state and kappa calculation

Trs := 298.15 Prs := 1.0 kap := 0.480 + 1.574⋅ om − 0.176⋅ om⋅ om


2 2
R ⋅ Tc R ⋅ Tc
S-R-K Constants: R := 0.00008314 b := 0.08664⋅ ac := 0.42748⋅
Pc Pc
Note that these are being defined as a
2
function of temperature for convenience. ⎡ ⎛
alf( T) := 1⋅ ⎢ 1 + kap⋅ ⎜ 1 −
⎞⎤
T
⎥ a( T) := ac ⋅ alf( T)
⎣ ⎝ Tc ⎠ ⎦
a ( T) ⋅ P P⋅ b
CA( T , P) := CB( T , P) := d
2 R⋅ T Da( T) := a ( T)
( R ⋅ T) dT
Z( T , P) := A ← CA( T , P)
B ← CB( T , P)
⎛ −A ⋅ B ⎞
⎜ 2 Vector of coefficients in the SRK equation
V← ⎜
A − B − B⎟ in the form
⎜ −1 ⎟ 0=-A*B+(A-B^2-B)*Z-*Z^2+Z^3

⎝ 1 ⎠
ZZ ← polyroots ( V) Solution to the cubic
for i ∈ 0 .. 2

(ZZi ← 0) if (Im(ZZi) ≠ 0) Set any imaginary roots to zero


ZZ ← sort ( ZZ) Sort the roots

ZZ ← ZZ ⎛ ZZ < 10− 5 ⎞
if
0 ⎝ 0
2 ⎠ Set the value of any imaginary roots
to value of the real root
ZZ ← ZZ if ⎛ ZZ < 10 ⎞
− 5
2 0 ⎝ 2 ⎠
ZZ
Enter temperature T, and pressure P.

T := −125
C T := 273.15 + T
K P := 100
th
Solutions to Chemical and Engineering Thermodynamics, 4 ed. Chapter 7

Fugacity expressions [actually ln(f/P)] for the liquid fl and vapor fv


CA( T , P) ⎛⎜
Z( T , P) + CB( T , P) ⎞
( ) ( )
0
fl( T , P) := Z( T , P) − 1 − ln Z( T , P) − CB( T , P) − ⋅ ln
0 0 CB( T , P) ⎜ Z( T , P)
⎝ 0 ⎠

CA( T , P) ⎛ Z( T , P) 2 + CB( T , P) ⎞
( ) (
fv( T , P) := Z( T , P) − 1 − ln Z( T , P) − CB( T , P) −
2 2 ) CB( T , P)
⋅ ln⎜
⎜ Z( T , P)
⎝ 2 ⎠
Fugacity Fugacity coefficient
fugl
fugl := P⋅ exp( fl( T , P) ) fugl = 34.64672 fl( T , P) = −1.05997 phil := phil = 0.34647
P
fugv
fugv := P⋅ exp( fv( T , P) ) fugv = 34.64672 fv( T , P) = −1.05997 phiv := phiv = 0.34647
P
Residual entropy for liquid (DELSL) and vapor (DELSV) phases

⎛ ⎛ Z( T , P) 0 + CB( T , P) ⎞ ⎞
(
DELSL:= ⎜ R⋅ ln Z( T , P) − CB( T , P) + ) ⋅ ln⎜
Da( T) 5
⋅ 10
⎜ 0 b ⎜ Z( T , P)
⎝ ⎝ 0 ⎠⎠
⎛ ⎛ Z( T , P) 2 + CB( T , P) ⎞ ⎞
(
DELSV:= ⎜ R⋅ ln Z( T , P) − CB( T , P) + ) ⋅ ln⎜
Da( T) 5
⋅ 10
⎜ 2 b ⎜ Z( T , P)
⎝ ⎝ 2 ⎠⎠

Residual enthalpy for liquid (DELHL) and vapor (DELHV) phases

⎡ T⋅ Da( T) − a( T) ⎛ Z( T , P) 0 + CB( T , P) ⎞⎤⎥


(
DELHL:= ⎢R⋅ T⋅ Z( T , P) − 1 + ) ⋅ ln⎜
5
⋅ 10
⎢ 0 b ⎜ Z( T , P) ⎥
⎣ ⎝ 0 ⎠⎦
⎡ T⋅ Da( T) − a( T) ⎛ Z( T , P) 2 + CB( T , P) ⎞⎤ 5
⎢ (
DELHV:= ⎢R⋅ T⋅ Z( T , P) − 1 +
2 ) b
⋅ ln⎜
⎜ Z( T , P)
⎥ ⋅ 10

⎣ ⎝ 2 ⎠⎦

Ideal gas properties changes relative to the reference state

(
Cp ⋅ T − Trs
1
2 2 ) Cp ⋅ T − Trs
2
( 3 3 ) (
Cp ⋅ T − Trs
3
4 4 )
DELHIG:= Cp ⋅ ( T − Trs ) + + +
0 2 3 4

Cp ⋅ T − Trs( 2 2 ) Cp ⋅ T − Trs( 3 3 )
DELSIG:= Cp ⋅ ln⎛⎜
T ⎞
− R⋅ 10 ⋅ ln⎛⎜ ⎞
2 3 5 P
+ Cp ⋅ ( T − Trs ) + +
0
⎝ Trs ⎠ 1 2 3 ⎝ Prs ⎠

Total entropy and enthalpy relative to ideal gas reference state


SL := DELSIG+ DELSL SV := DELSIG+ DELSV HL := DELHIG+ DELHL HV := DELHIG+ DELHV
th
Solutions to Chemical and Engineering Thermodynamics, 4 ed. Chapter 7

−2 T
V := Z( T , P) ⋅ 8.314⋅ 10 ⋅ V = 0.04081 −2 T
0 0 P 0 V := Z( T , P) ⋅ 8.314⋅ 10 ⋅
2 2 P V = 0.04081
2

SUMMARY OF RESULTS

T = 148.15
K Vapor pressure, bar P = 100
LIQUID VAPOR
Compressibility Z( T , P) = 0.3313 Z( T , P) = 0.3313
0 2
Volume, m^3/kmol V = 0.04081 V = 0.04081
0 2

3 3
Enthalpy, J/mol HL = −9.29109× 10 HV = −9.29109× 10

Entropy, J/mol K SL = −83.19194 SV = −83.19194

Fugacity coefficient phil = 0.34647 phiv = 0.34647

Fugacity, bar fugl = 34.64672 fugv = 34.64672

Some representative results are shown below.

T (C) -125 -150 -175 -200


P=1 bar
Z 0.9923 0.9872 0.9766 0.9505
V 12.2227 10.1072 7.9693 5.7804
H -4301.41 -4994.48 -5684.02 -6375.9
S -19.97 -25.1 -31.35 -35.49

P=10 bar
Z 0.9193 0.8565 0.03572 0.04292
V 1.1323 0.877 0.02914 0.0261
H -4561.07 -5357.77 -12395.6 -13706.2
S -40.28 -46.18 -106.11 -121.5

P=50 bar
Z 0.1946 0.1647 0.17634 0.21349
V 0.04795 0.03373 0.02878 0.02597
H -8938.78 -10919.2 -12338.7 -13628.9
S -79.34 -93.79 -106.71 -121.86

P=100 bar
Z 0.3313 0.318 0.34788 0.42446
V 0.04081 0.03256 0.02839 0.02581
H -9291.09 -10896.8 -12261.8 -13530.9
S -83.19 -95.02 -107.39 -122.29
.
th
Solutions to Chemical and Engineering Thermodynamics, 4 ed. Chapter 7

7.53 Pure component properties calculation using the SRK equation of state

Read in properties for Water Tc := 647.3 Pc := 220.48 om := 0.344

Heat capacity constants


−2 −5 −9
Cp := 32.218 Cp := 0.192⋅ 10 Cp := 1.055⋅ 10 Cp := −3.593⋅ 10
0 1 2 3
Reference state and kappa calculation

Trs := 373.15 Prs := 1.013 kap := 0.480 + 1.574⋅ om − 0.176⋅ om⋅ om


2 2
R ⋅ Tc R ⋅ Tc
S-R-K Constants: R := 0.00008314 b := 0.08664⋅ ac := 0.42748⋅
Pc Pc
Note that these are being defined as a
2
function of temperature for convenience. ⎡ ⎛ ⎞⎤
T
alf( T) := 1⋅ ⎢ 1 + kap⋅ ⎜ 1 − ⎥ a( T) := ac ⋅ alf( T)
⎣ ⎝ Tc ⎠ ⎦
a ( T) ⋅ P P⋅ b
CA( T , P) := CB( T , P) := d
2 R⋅ T Da( T) := a ( T)
( R ⋅ T) dT
Z( T , P) := A ← CA( T , P)
B ← CB( T , P)
⎛ −A ⋅ B ⎞
⎜ 2 Vector of coefficients in the SRK equation
V← ⎜
A − B − B⎟ in the form
⎜ −1 ⎟ 0=-A*B+(A-B^2-B)*Z-*Z^2+Z^3

⎝ 1 ⎠
ZZ ← polyroots ( V) Solution to the cubic
for i ∈ 0 .. 2

(ZZi ← 0) if (Im(ZZi) ≠ 0) Set any imaginary roots to zero


ZZ ← sort ( ZZ) Sort the roots

ZZ ← ZZ ⎛ ZZ < 10− 5 ⎞
if
0 ⎝ 0 2 ⎠ Set the value of any imaginary roots
to value of the real root
ZZ ← ZZ if ⎛ ZZ < 10 ⎞
− 5
2 0 ⎝ 2 ⎠
ZZ
Enter temperature T, and pressure P.

T := 50
C T := 273.15 + T
K P := 0.15

Fugacity expressions [actually ln(f/P)] for the liquid fl and vapor fv

CA( T , P) ⎛ Z( T , P) 0 + CB( T , P) ⎞
( ) (
fl( T , P) := Z( T , P) − 1 − ln Z( T , P) − CB( T , P) − ) ⋅ ln⎜
0 0 CB( T , P) ⎜ Z( T , P)
⎝ 0 ⎠

CA( T , P) ⎛ Z( T , P) 2 + CB( T , P) ⎞
( ) (
fv( T , P) := Z( T , P) − 1 − ln Z( T , P) − CB( T , P) − ) ⋅ ln⎜
2 2 CB( T , P) ⎜ Z( T , P)
⎝ 2 ⎠
th
Solutions to Chemical and Engineering Thermodynamics, 4 ed. Chapter 7

Fugacity Fugacity coefficient

fugl
fugl := P⋅ exp( fl( T , P) ) fugl = 0.09983 fl( T , P) = −0.40721 phil := phil = 0.6655
P

−3 fugv
fugv := P⋅ exp( fv( T , P) ) fugv = 0.14972 fv( T , P) = −1.83629× 10
phiv := phiv = 0.99817
P
Residual entropy for liquid (DELSL) and vapor (DELSV) phases

⎛ ⎛ Z( T , P) 0 + CB( T , P) ⎞ ⎞
(
DELSL:= ⎜ R⋅ ln Z( T , P) − CB( T , P) + ) ⋅ ln⎜
Da( T) 5
⋅ 10
⎜ 0 b ⎜ Z( T , P)
⎝ ⎝ 0 ⎠⎠

⎛ ⎛ Z( T , P) 2 + CB( T , P) ⎞ ⎞
(
DELSV:= ⎜ R⋅ ln Z( T , P) − CB( T , P) + ) ⋅ ln⎜
Da( T) 5
⋅ 10
⎜ 2 b ⎜ Z( T , P)
⎝ ⎝ 2 ⎠⎠

Residual enthalpy for liquid (DELHL) and vapor (DELHV) phases

⎡ T⋅ Da( T) − a( T) ⎛ Z( T , P) 0 + CB( T , P) ⎞⎤⎥


⎢ (
DELHL:= ⎢R⋅ T⋅ Z( T , P) − 1 +
0 ) b
⋅ ln⎜
⎜ Z( T , P) ⎥
⋅ 10
5

⎣ ⎝ 0 ⎠⎦

⎡ T⋅ Da( T) − a( T) ⎛ Z( T , P) 2 + CB( T , P) ⎞⎤⎥


⎢ (
DELHV:= ⎢R⋅ T⋅ Z( T , P) − 1 +
2 ) b
⋅ ln⎜
⎜ Z( T , P) ⎥
⋅ 10
5

⎣ ⎝ 2 ⎠⎦

Ideal gas properties changes relative to the reference state

(
Cp ⋅ T − Trs
1
2 )
2
Cp ⋅ T − Trs
2
( 3 3 ) Cp ⋅ T − Trs
3
( 4 4 )
DELHIG:= Cp ⋅ ( T − Trs ) + + +
0 2 3 4
(
Cp ⋅ T − Trs
2 2 ) Cp ⋅ T − Trs( 3 3 )
DELSIG:= Cp ⋅ ln⎛⎜
T ⎞
− R⋅ 10 ⋅ ln⎛⎜
2 3 5 P ⎞
+ Cp ⋅ ( T − Trs ) + +
0 ⎝ Trs ⎠ 1 2 3 ⎝ Prs ⎠

Total entropy and enthalpy relative to ideal gas reference state


SL := DELSIG+ DELSL SV := DELSIG+ DELSV HL := DELHIG+ DELHL HV := DELHIG+ DELHV

SUMMARY OF RESULTS

T = 323.15
K Vapor pressure, bar P = 0.15
LIQUID VAPOR
−4
Compressibility Z( T , P) = 1.35706× 10 Z( T , P) = 0.99816
0 2
4 3
Enthalpy, J/mol HL = −4.74037× 10 HV = −1.71382× 10

Entropy, J/mol K SL = −127.05678 SV = 10.96203

Fugacity coefficient phil = 0.6655 phiv = 0.99817

Fugacity, bar fugl = 0.09983 fugv = 0.14972


th
Solutions to Chemical and Engineering Thermodynamics, 4 ed. Chapter 7

7.54 ISENTHALPIC PENG-ROBINSON EQUATION OF STATE CALCULATION

Tc := 154.6 Pc := 50.46 om := 0.021 kap := 0.37464 + 1.54226om


⋅ − 0.26992om
⋅ ⋅ om
−2 −5 −9
Cp := 25.46 Cp := 1.591⋅ 10 Cp := −0.7151⋅ 10 Cp := 1.311⋅ 10
0 1 2 3
2 2
R ⋅ Tc R ⋅ Tc
Peng-Robinson Constants: R := 0.00008314 b := 0.07780⋅ ac := 0.45724⋅
Pc Pc

Input initial temperature and pressure of calculation Ti := 120. K, Pi := 30 bar


Input final pressure Pf := 3.0 bar
Initial state calculations T := Ti P := Pi
2
⎡ ⎛ T ⎞⎤ a ( T) ⋅ P P⋅ b
alf( T) := 1⋅ ⎢ 1 + kap⋅ ⎜ 1 − ⎥ a( T) := ac ⋅ alf( T) CA( T , P) := CB( T , P) :=
⎣ ⎝ Tc ⎠ ⎦
( R ⋅ T)
2 R⋅ T
d
Da( T) := a( T)
dT
Z( T , P) := A ← CA( T , P)
B ← CB( T , P)

(
⎡⎢ − A ⋅ B − B2 − B3 ) ⎥⎤ Vector of coefficients in the PR equation
⎢ 2 ⎥ in the form
V ← ⎢ A − 3⋅ B − 2⋅ B ⎥ 0=-(A*B-B^2-B^3)+(A-3*B^2-2*B)*Z-(1-B)*Z^2+Z^3
⎢ −( 1 − B) ⎥
⎢ ⎥
⎣ 1 ⎦
ZZ ← polyroots ( V) Solution to the cubic

for i ∈ 0 .. 2

(ZZi ← 0) if (Im(ZZi) ≠ 0) Set any imaginary roots to zero


Sort the roots
ZZ ← sort ( ZZ)

ZZ ← ZZ ⎛ ZZ < 10− 5 ⎞
if Set the value of any imaginary roots
0 ⎝ 0 2 ⎠ to value of the real root

ZZ ← ZZ if ⎛ ZZ < 10 ⎞
− 5
2 0 ⎝ 2 ⎠
ZZ
Calculate inital properties Zf ( T , P) := Z( T , P)

⎛ 0.0888⎞
Calculate initial molar volume Z( T , P) ⋅ R ⋅ T
0 3 Z( T , P) = ⎜ 0
and enthalpy and entropy VL := ⋅ 10 ⎜
departure
P ⎝ 0.0888⎠

⎡ T⋅ Da( T) − a( T) ⎡ Z( T , P) 0 + ( 1 + 2) ⋅ CB( T , P) ⎤ ⎤ 5
⎢ (
DELHin:= ⎢ R ⋅ T⋅ Z( T , P) − 1 +
0 ) 2⋅ 2⋅ b
⋅ ln⎢ ⎥ ⎥ ⋅ 10
⎢ Z( T , P) + ( 1 − 2) ⋅ CB( T , P) ⎥ ⎥
⎣ ⎣ 0 ⎦⎦

⎡ ⎡ Z( T , P) 0 + ( 1 + 2) ⋅ CB( T , P) ⎤ ⎤ 5
(
DELSin := ⎢ R⋅ ln Z( T , P) − CB( T , P) + ) ⋅ ln⎢ ⎥ ⎥ ⋅ 10
Da( T)
⎢ 0 2⋅ 2⋅ b ⎢ Z( T , P) + ( 1 − 2) ⋅ CB( T , P) ⎥ ⎥
⎣ ⎣ 0 ⎦⎦
th
Solutions to Chemical and Engineering Thermodynamics, 4 ed. Chapter 7

3
DELHin = −5.9875× 10 DELSin = −40.1647

Guess for final state T := 0.8⋅ Ti P := Pf

Fugacity expressions [actually ln(f/P)] for the liquid fl and vapor fv

CA( T , P) ⎡ Z( T , P) 0 + ( 1 + 2) ⋅ CB( T , P) ⎤
( ) (
fl( T , P) := Z( T , P) − 1 − ln Z( T , P) − CB( T , P) −
0 0 ) 2⋅ 2⋅ CB( T , P)
⋅ ln⎢
⎢ Z( T , P) + ( 1 − 2) ⋅ CB( T , P) ⎥

⎣ 0 ⎦

CA( T , P) ⎡ Z( T , P) 2 + ( 1 + 2) ⋅ CB( T , P) ⎤
( ) (
fv( T , P) := Z( T , P) − 1 − ln Z( T , P) − CB( T , P) −
2 2 ) 2⋅ 2⋅ CB( T , P)
⋅ ln⎢
⎢ Z( T , P) + ( 1 − 2) ⋅ CB( T , P) ⎥

⎣ 2 ⎦

Given fl( T , P) − fv( T , P) 0 T := Find( T) T = 101.906

Residual entropy for liquid (DELSL) and vapor (DELSV) phases

⎡ ⎡ Z( T , P) 0 + ( 1 + 2) ⋅ CB( T , P) ⎤ ⎤ 5
(
DELSL( T , P) := ⎢ R⋅ ln Z( T , P) − CB( T , P) + ) ⋅ ln⎢ ⎥ ⎥ ⋅ 10
Da( T)
⎢ 0 2⋅ 2⋅ b ⎢ Z( T , P) + ( 1 − 2) ⋅ CB( T , P) ⎥ ⎥
⎣ ⎣ 0 ⎦⎦

⎡ ⎡ Z( T , P) 2 + ( 1 + 2) ⋅ CB( T , P) ⎤ ⎤ 5
(
( , P) := ⎢ R⋅ ln Z( T , P) − CB( T , P) + ) ⋅ ln⎢ ⎥ ⎥ ⋅ 10
Da( T)
DELSVT
⎢ 2 2⋅ 2⋅ b ⎢ Z( T , P) + ( 1 − 2) ⋅ CB( T , P) ⎥ ⎥
⎣ ⎣ 2 ⎦⎦

Residual enthalpy for liquid (DELHL) and vapor (DELHV) phases

⎡ T⋅ Da( T) − a( T) ⎡ Z( T , P) 0 + ( 1 + 2) ⋅ CB( T , P) ⎤ ⎤ 5
⎢ (
DELHL( T , P) := ⎢ R⋅ T⋅ Z( T , P) − 1 +
0 ) 2⋅ 2⋅ b
⋅ ln⎢ ⎥ ⎥ ⋅ 10
⎢ Z( T , P) + ( 1 − 2) ⋅ CB( T , P) ⎥ ⎥
⎣ ⎣ 0 ⎦⎦

⎡ T⋅ Da( T) − a( T) ⎡ Z( T , P) 2 + ( 1 + 2) ⋅ CB( T , P) ⎤ ⎤ 5
( , P) := ⎢ R⋅ T⋅ Z( T , P) − 1 +
DELHVT
⎢ ( 2 ) 2⋅ 2⋅ b
⋅ ln⎢ ⎥ ⎥ ⋅ 10
⎢ Z( T , P) + ( 1 − 2) ⋅ CB( T , P) ⎥ ⎥
⎣ ⎣ 2 ⎦⎦
Ideal gas properties changes relative to the initial state

(
Cp ⋅ T − Ti
1
2 2 ) (
Cp ⋅ T − Ti
2
3 3 ) Cp ⋅ T − Ti
3
( 4 4 )
( , P) := Cp ⋅ ( T − Ti) +
DELHIGT + +
0 2 3 4

Cp ⋅ T − Ti ( 2 2 ) Cp ⋅ T − Ti ( 3 3 )
( , P) := Cp ⋅ ln⎛⎜ ⎞ + Cp ⋅ ( T − Ti) + − R⋅ 10 ⋅ ln⎛⎜
T 2 3 5 P⎞
DELSIGT +
0
⎝ Ti ⎠ 1 2 3 ⎝ Pi ⎠
Find vapor-liquid split x := 0.5

Given

x⋅ DELHVT
( , P) + ( 1 − x) ⋅ DELHL( T , P) + DELHIGT
( , P) DELHin

x := Find( x) x = 0.1618 Fraction vapor


th
Solutions to Chemical and Engineering Thermodynamics, 4 ed. Chapter 7

HV := DELHVT
( , P) + DELHIGT
( , P) SV := DELSVT
( , P) + DELSIGT
( , P)

HL := DELHL( T , P) + DELHIGT
( , P) SL := DELSL( T , P) + DELSIGT
( , P)

δH := x⋅ HV + ( 1 − x) ⋅ HL − DELHin δS := x⋅ SV + ( 1 − x) ⋅ SL − DELSin

SUMMARY OF RESULTS

FEED LIQUID VAPOR


Temperature, K Ti = 120 T = 101.906 T = 101.906
Pressure, bar Pi = 30 P=3 P=3

Vapor-liquid split x = 0.1618

−3
Compressibility Z( Ti , Pi) = 0.0888 Z( T , P) = 9.3464× 10 Z( T , P) = 0.9309
0 0 2
Enthalpy, J/mol 3
(relative to feed) 0 HL = −7.0203× 10 HV = −635.2466

Entropy, J/mol K 0 SL = −48.8038 SV = 13.8527


(relative to feed)

Enthalpy change
J/mol δH = 0

Entropy change
J/mol K δS = 1.4957
th
Solutions to Chemical and Engineering Thermodynamics, 4 ed. Chapter 7

7.55
ISENTROPIC PENG-ROBINSON EQUATION OF STATE CALCULATION

Tc := 154.6 Pc := 50.46 om := 0.021 kap := 0.37464 + 1.54226om


⋅ − 0.26992om
⋅ ⋅ om
−2 −5 −9
Cp := 25.46 Cp := 1.591⋅ 10 Cp := −0.7151⋅ 10 Cp := 1.311⋅ 10
0 1 2 3
2 2
R⋅ Tc R ⋅ Tc
Peng-Robinson Constants: R := 0.00008314 b := 0.07780⋅ ac := 0.45724⋅
Pc Pc

Input initial temperature and pressure of calculation Ti := 120. K, Pi := 30 bar


Input final pressure Pf := 3.0 bar
Initial state calculations T := Ti P := Pi
2
⎡ ⎛ T ⎞⎤ a ( T) ⋅ P P⋅ b
alf( T) := 1⋅ ⎢ 1 + kap⋅ ⎜ 1 − ⎥ a( T) := ac ⋅ alf( T) CA( T , P) := CB( T , P) :=
⎣ ⎝ Tc ⎠ ⎦
( R ⋅ T)
2 R⋅ T
d
Da( T) := a( T)
dT
Z( T , P) := A ← CA( T , P)
B ← CB( T , P)

(
⎡⎢ − A ⋅ B − B2 − B3 ) ⎥⎤
Vector of coefficients in the PR equation
⎢ 2 ⎥ in the form
V ← ⎢ A − 3⋅ B − 2⋅ B ⎥ 0=-(A*B-B^2-B^3)+(A-3*B^2-2*B)*Z-(1-B)*Z^2+Z^3
⎢ −( 1 − B) ⎥
⎢ ⎥
⎣ 1 ⎦
ZZ ← polyroots ( V) Solution to the cubic

for i ∈ 0 .. 2

(ZZi ← 0) if (Im(ZZi) ≠ 0) Set any imaginary roots to zero


Sort the roots
ZZ ← sort ( ZZ)

ZZ ← ZZ ⎛ ZZ < 10− 5 ⎞
if Set the value of any imaginary roots
0 ⎝ 0 2 ⎠ to value of the real root
ZZ ← ZZ if ⎛ ZZ < 10
− 5⎞
2 0 ⎝ 2 ⎠
ZZ
Calculate inital properties Zf ( T , P) := Z( T , P)

⎛ 0.0888⎞
Z( T , P) = ⎜
Calculate initial molar volume Z( T , P) ⋅ R⋅ T
0 3 0
and enthalpy and entropy VL := ⋅ 10 ⎜
departure
P ⎝ 0.0888⎠

⎡ T⋅ Da( T) − a( T) ⎡ Z( T , P) 0 + ( 1 + 2) ⋅ CB( T , P) ⎤ ⎤ 5
⎢ (
DELHin:= ⎢ R⋅ T⋅ Z( T , P) − 1 +
0 ) 2⋅ 2⋅ b
⋅ ln⎢ ⎥ ⎥ ⋅ 10
⎢ Z( T , P) + ( 1 − 2) ⋅ CB( T , P) ⎥ ⎥
⎣ ⎣ 0 ⎦⎦

⎡ ⎡ Z( T , P) 0 + ( 1 + 2) ⋅ CB( T , P) ⎤ ⎤ 5
(
DELSin:= ⎢ R⋅ ln Z( T , P) − CB( T , P) + ) ⋅ ln⎢ ⎥ ⎥ ⋅ 10
Da( T)
⎢ 0 2⋅ 2⋅ b ⎢ Z( T , P) + ( 1 − 2) ⋅ CB( T , P) ⎥ ⎥
⎣ ⎣ 0 ⎦⎦
3
DELHin = −5.9875× 10 DELSin = −40.1647
th
Solutions to Chemical and Engineering Thermodynamics, 4 ed. Chapter 7

Guess for final state T := 0.8⋅ Ti P := Pf

Fugacity expressions [actually ln(f/P)] for the liquid fl and vapor fv

CA( T , P) ⎡ Z( T , P) 0 + ( 1 + 2) ⋅ CB( T , P) ⎤
( ) (
fl( T , P) := Z( T , P) − 1 − ln Z( T , P) − CB( T , P) −
0 0 ) 2⋅ 2⋅ CB( T , P)
⋅ ln⎢
⎢ Z( T , P) + ( 1 − 2) ⋅ CB( T , P) ⎥

⎣ 0 ⎦

CA( T , P) ⎡ Z( T , P) 2 + ( 1 + 2) ⋅ CB( T , P) ⎤
( ) (
fv( T , P) := Z( T , P) − 1 − ln Z( T , P) − CB( T , P) −
2 2 ) 2⋅ 2⋅ CB( T , P)
⋅ ln⎢
⎢ Z( T , P) + ( 1 − 2) ⋅ CB( T , P) ⎥

⎣ 2 ⎦

Given fl( T , P) − fv( T , P) 0 T := Find( T) T = 101.906

Residual entropy for liquid (DELSL) and vapor (DELSV) phases

⎡ ⎡ Z( T , P) 0 + ( 1 + 2) ⋅ CB( T , P) ⎤ ⎤ 5
(
DELSL( T , P) := ⎢ R⋅ ln Z( T , P) − CB( T , P) + ) ⋅ ln⎢ ⎥ ⎥ ⋅ 10
Da( T)
⎢ 0 2⋅ 2⋅ b ⎢ Z( T , P) + ( 1 − 2) ⋅ CB( T , P) ⎥ ⎥
⎣ ⎣ 0 ⎦⎦

⎡ ⎡ Z( T , P) 2 + ( 1 + 2) ⋅ CB( T , P) ⎤ ⎤ 5
(
( , P) := ⎢ R⋅ ln Z( T , P) − CB( T , P) + ) ⋅ ln⎢ ⎥ ⎥ ⋅ 10
Da( T)
DELSVT
⎢ 2 2⋅ 2⋅ b ⎢ Z( T , P) + ( 1 − 2) ⋅ CB( T , P) ⎥ ⎥
⎣ ⎣ 2 ⎦⎦

Residual enthalpy for liquid (DELHL) and vapor (DELHV) phases

⎡ T⋅ Da( T) − a( T) ⎡ Z( T , P) 0 + ( 1 + 2) ⋅ CB( T , P) ⎤ ⎤ 5
⎢ (
DELHL( T , P) := ⎢ R⋅ T⋅ Z( T , P) − 1 +
0 ) 2⋅ 2⋅ b
⋅ ln⎢ ⎥ ⎥ ⋅ 10
⎢ Z( T , P) + ( 1 − 2) ⋅ CB( T , P) ⎥ ⎥
⎣ ⎣ 0 ⎦⎦

⎡ T⋅ Da( T) − a( T) ⎡ Z( T , P) 2 + (1 + 2) ⋅ CB( T , P) ⎤ ⎤ 5
( , P) := ⎢ R⋅ T⋅ Z( T , P) − 1 +
DELHVT
⎢ ( 2 ) 2⋅ 2⋅ b
⋅ ln⎢ ⎥ ⎥ ⋅ 10
⎢ Z( T , P) + (1 − 2) ⋅ CB( T , P) ⎥ ⎥
⎣ ⎣ 2 ⎦⎦
Ideal gas properties changes relative to the initial state

(
Cp ⋅ T − Ti
1
2 2) Cp ⋅ T − Ti
2
( 3 3 ) Cp ⋅ T − Ti
3
( 4 4 )
( , P) := Cp ⋅ ( T − Ti) +
DELHIGT + +
0 2 3 4

(
Cp ⋅ T − Ti
2 )
2
Cp ⋅ T − Ti ( 3 )
3
( , P) := Cp ⋅ ln⎛⎜ ⎞ + Cp ⋅ ( T − Ti) + − R⋅ 10 ⋅ ln⎛⎜
T 2 3 5 P⎞
DELSIGT +
0 ⎝ Ti ⎠ 1 2 3 ⎝ Pi ⎠
Find vapor-liquid split x := 0.5

Given

x⋅ DELSVT
( , P) + ( 1 − x) ⋅ DELSL( T , P) + DELSIGT
( , P) DELSin

x := Find( x) x = 0.1379 Fraction vapor


HV := DELHVT
( , P) + DELHIGT
( , P) SV := DELSVT
( , P) + DELSIGT
( , P)

HL := DELHL( T , P) + DELHIGT
( , P) SL := DELSL( T , P) + DELSIGT
( , P)

δH := x⋅ HV + ( 1 − x) ⋅ HL − DELHin δS := x⋅ SV + ( 1 − x) ⋅ SL − DELSin
th
Solutions to Chemical and Engineering Thermodynamics, 4 ed. Chapter 7

SUMMARY OF RESULTS

FEED LIQUID VAPOR


Temperature, K Ti = 120 T = 101.906 T = 101.906
Pressure, bar Pi = 30 P=3 P=3

Vapor-liquid split x = 0.1379

−3
Compressibility Z( Ti , Pi) = 0.0888 Z( T , P) = 9.3464× 10 Z( T , P) = 0.9309
0 0 2
Enthalpy, J/mol 3
(relative to feed) 0 HL = −7.0203× 10 HV = −635.2466

Entropy, J/mol K 0 SL = −48.8038 SV = 13.8527


(relative to feed)

Enthalpy change
J/mol δH = −152.4165

Entropy change
J/mol K δS = 0
th
Solutions to Chemical and Engineering Thermodynamics, 4 ed. Chapter 7

7.56
7.56 ISENTHALPIC S-R-K EQUATION OF STATE CALCULATION

Tc := 154.6 Pc := 50.46 om := 0.021 kap := 0.480 + 1.574⋅ om − 0.176⋅ om⋅ om


−2 −5 −9
Cp := 25.46 Cp := 1.591⋅ 10 Cp := −0.7151⋅ 10 Cp := 1.311⋅ 10
0 1 2 3
2 2
R⋅ Tc R ⋅ Tc
S-R-K Constants: R := 0.00008314 b := 0.08664⋅ ac := 0.42748⋅
Pc Pc

Input initial temperature and pressure of calculation Ti := 120. K, Pi := 30 bar


Input final pressure Pf := 3.0 bar
Initial state calculations T := Ti P := Pi
2
⎡ ⎛ T ⎞⎤ a ( T) ⋅ P P⋅ b
alf( T) := 1⋅ ⎢ 1 + kap⋅ ⎜ 1 − ⎥ a( T) := ac ⋅ alf( T) CA( T , P) := CB( T , P) :=
⎣ ⎝ Tc ⎠ ⎦
( R ⋅ T)
2 R⋅ T
d
Da( T) := a( T)
dT
Z( T , P) := A ← CA( T , P)
B ← CB( T , P)
⎛ −A ⋅ B ⎞
⎜ 2 Vector of coefficients in the S-R-K equation
V← ⎜
A − B − B⎟ in the form
⎜ −1 ⎟ 0=-A*B+(A-B^2-B)*Z-Z^2+Z^3

⎝ 1 ⎠
ZZ ← polyroots ( V) Solution to the cub ic
for i ∈ 0 .. 2

(ZZi ← 0) if (Im(ZZi) ≠ 0) Set any imaginary roots to zero


ZZ ← sort ( ZZ) Sort the roots

ZZ ← ZZ ⎛ ZZ < 10− 5 ⎞
if
0 ⎝ 0 2 ⎠ Set the value of any imaginary roots
to value of the real root
ZZ ← ZZ if ⎛ ZZ < 10
− 5⎞
2 0 ⎝ 2 ⎠
ZZ
Calculate inital properties Zf ( T , P) := Z( T , P)

⎛ 0.1004⎞
Z( T , P) = ⎜
Calculate initial molar volume Z( T , P) ⋅ R⋅ T
0 3 0
and enthalpy and entropy VL := ⋅ 10 ⎜
departure
P ⎝ 0.1004⎠

⎡ T⋅ Da( T) − a( T) ⎛ Z( T , P) 0 + CB( T , P) ⎞⎥⎤ 5


⎢ (
DELHin:= ⎢R⋅ T⋅ Z( T , P) − 1 +
0 ) b
⋅ ln⎜
⎜ Z( T , P) ⎥
⋅ 10
⎣ ⎝ 0 ⎠⎦

⎛ ⎛ Z( T , P) 0 + CB( T , P) ⎞ ⎞
(
DELSin:= ⎜ R⋅ ln Z( T , P) − CB( T , P) + ) ⋅ ln⎜
Da( T) 5
⋅ 10
⎜ 0 b ⎜ Z( T , P)
⎝ ⎝ 0 ⎠⎠
3
DELHin = −6.0618× 10 DELSin = −40.9502
th
Solutions to Chemical and Engineering Thermodynamics, 4 ed. Chapter 7

Guess for final state T := 0.8⋅ Ti P := Pf

Fugacity expressions [actually ln(f/P)] for the liquid fl and vapor fv

CA( T , P) ⎛ Z( T , P) 0 + CB( T , P) ⎞
( ) (
fl( T , P) := Z( T , P) − 1 − ln Z( T , P) − CB( T , P) −
0 0 ) CB( T , P)
⋅ ln⎜
⎜ Z( T , P)
⎝ 0 ⎠

CA( T , P) ⎛ Z( T , P) 2 + CB( T , P) ⎞
( ) (
fv( T , P) := Z( T , P) − 1 − ln Z( T , P) − CB( T , P) −
2 2 ) CB( T , P)
⋅ ln⎜
⎜ Z( T , P)
⎝ 2 ⎠

Given fl( T , P) − fv( T , P) 0 T := Find( T) T = 102.0671

Residual entropy for liquid (DELSL) and vapor (DELSV) phases

⎛ ⎛ Z( T , P) 0 + CB( T , P) ⎞ ⎞
(
DELSL( T , P) := ⎜ R⋅ ln Z( T , P) − CB( T , P) + ) ⋅ ln⎜
Da( T) 5
⋅ 10
⎜ 0 b ⎜ Z( T , P)
⎝ ⎝ 0 ⎠⎠

⎛ ⎛ Z( T , P) 2 + CB( T , P) ⎞ ⎞ 5
(
( , P) := ⎜ R⋅ ln Z( T , P) − CB( T , P) + ) ⋅ ln⎜
Da( T)
DELSVT ⋅ 10
⎜ 2 b ⎜ Z( T , P)
⎝ ⎝ 2 ⎠⎠

Residual enthalpy for liquid (DELHL) and vapor (DELHV) phases

⎡ T⋅ Da( T) − a( T) ⎛ Z( T , P) 0 + CB( T , P) ⎞⎥⎤


⎢ (
DELHL( T , P) := ⎢R⋅ T⋅ Z( T , P) − 1 +
0 ) b
⋅ ln⎜
⎜ Z( T , P) ⎥
⋅ 10
5

⎣ ⎝ 0 ⎠⎦

⎡ T⋅ Da( T) − a( T) ⎛ Z( T , P) 2 + CB( T , P) ⎞⎤ 5
DELHVT
⎢ (
( , P) := ⎢R⋅ T⋅ Z( T , P) − 1 +
2 ) b
⋅ ln⎜
⎜ Z( T , P)
⎥ ⋅ 10

⎣ ⎝ 2 ⎠⎦
Ideal gas properties changes relative to the initial state

Cp ⋅ T − Ti
1
( 2 2 ) (
Cp ⋅ T − Ti
2
3 3 ) Cp ⋅ T − Ti
3
( 4 4 )
( , P) := Cp ⋅ ( T − Ti) +
DELHIGT + +
0 2 3 4

Cp ⋅ T − Ti( 2 2 ) Cp ⋅ T − Ti ( 3 3 )
( , P) := Cp ⋅ ln⎛⎜ ⎞ + Cp ⋅ ( T − Ti) + − R⋅ 10 ⋅ ln⎛⎜ ⎞
T 2 3 5 P
DELSIGT +
0
⎝ ⎠
Ti 1 2 3 ⎝ Pi ⎠
Find vapor-liquid split x := 0.5

Given

x⋅ DELHVT
( , P) + ( 1 − x) ⋅ DELHL( T , P) + DELHIGT
( , P) DELHin

x := Find( x) x = 0.1661 Fraction vapor


th
Solutions to Chemical and Engineering Thermodynamics, 4 ed. Chapter 7

HV := DELHVT
( , P) + DELHIGT
( , P) SV := DELSVT
( , P) + DELSIGT
( , P)

HL := DELHL( T , P) + DELHIGT
( , P) SL := DELSL( T , P) + DELSIGT
( , P)

δH := x⋅ HV + ( 1 − x) ⋅ HL − DELHin δS := x⋅ SV + ( 1 − x) ⋅ SL − DELSin

SUMMARY OF RESULTS

FEED LIQUID VAPOR


Temperature, K Ti = 120 T = 102.0671 T = 102.0671
Pressure, bar Pi = 30 P=3 P=3

Vapor-liquid split x = 0.1661

Compressibility Z( Ti , Pi) = 0.1004 Z( T , P) = 0.0106 Z( T , P) = 0.934


0 0 2
Enthalpy, J/mol 3
(relative to feed) 0 HL = −7.1435× 10 HV = −630.1699

Entropy, J/mol K 0 SL = −49.936 SV = 13.8781


(relative to feed)

Enthalpy change
J/mol δH = 0

Entropy change
J/mol K δS = 1.6121
th
Solutions to Chemical and Engineering Thermodynamics, 4 ed. Chapter 7

7.57 ISENTROPIC S-R-K EQUATION OF STATE CALCULATION

Tc := 154.6 Pc := 50.46 om := 0.021 kap := 0.480 + 1.574⋅ om − 0.176⋅ om⋅ om


−2 −5 −9
Cp := 25.46 Cp := 1.591⋅ 10 Cp := −0.7151⋅ 10 Cp := 1.311⋅ 10
0 1 2 3
2 2
R⋅ Tc R ⋅ Tc
S-R-K Constants: R := 0.00008314 b := 0.08664⋅ ac := 0.42748⋅
Pc Pc

Input initial temperature and pressure of calculation Ti := 120. K, Pi := 30 bar


Input final pressure Pf := 3.0 bar
Initial state calculations T := Ti P := Pi
2
⎡ ⎛ T ⎞⎤ a ( T) ⋅ P P⋅ b
alf( T) := 1⋅ ⎢ 1 + kap⋅ ⎜ 1 − ⎥ a( T) := ac ⋅ alf( T) CA( T , P) := CB( T , P) :=
⎣ ⎝ Tc ⎠ ⎦
( R ⋅ T)
2 R⋅ T
d
Da( T) := a( T)
dT
Z( T , P) := A ← CA( T , P)
B ← CB( T , P)
⎛ −A ⋅ B ⎞
⎜ 2 Vector of coefficients in the S-R-K equation
V← ⎜
A − B − B⎟ in the form
⎜ −1 ⎟ 0=-A*B+(A-B^2-B)*Z-Z^2+Z^3

⎝ 1 ⎠
ZZ ← polyroots ( V) Solution to the cub ic
for i ∈ 0 .. 2

(ZZi ← 0) if (Im(ZZi) ≠ 0) Set any imaginary roots to zero


ZZ ← sort ( ZZ) Sort the roots

ZZ ← ZZ ⎛ ZZ < 10− 5 ⎞
if
0 ⎝ 0 2 ⎠ Set the value of any imaginary roots
to value of the real root
ZZ ← ZZ if ⎛ ZZ < 10
− 5⎞
2 0 ⎝ 2 ⎠
ZZ
Calculate inital properties Zf ( T , P) := Z( T , P)

⎛ 0.1004⎞
Z( T , P) = ⎜
Calculate initial molar volume Z( T , P) ⋅ R⋅ T
0 3 0
and enthalpy and entropy VL := ⋅ 10 ⎜
departure
P ⎝ 0.1004⎠

⎡ T⋅ Da( T) − a( T) ⎛ Z( T , P) 0 + CB( T , P) ⎞⎤ 5
⎢ (
DELHin:= ⎢R⋅ T⋅ Z( T , P) − 1 +
0 ) b
⋅ ln⎜
⎜ Z( T , P)
⎥ ⋅ 10

⎣ ⎝ 0 ⎠⎦

⎛ ⎛ Z( T , P) 0 + CB( T , P) ⎞ ⎞ 5
(
DELSin:= ⎜ R⋅ ln Z( T , P) − CB( T , P) + ) ⋅ ln⎜
Da( T)
⋅ 10
⎜ 0 b ⎜ Z( T , P)
⎝ ⎝ 0 ⎠⎠
3
DELHin = −6.0618× 10 DELSin = −40.9502

Guess for final state T := 0.8⋅ Ti P := Pf


th
Solutions to Chemical and Engineering Thermodynamics, 4 ed. Chapter 7

Fugacity expressions [actually ln(f/P)] for the liquid fl and vapor fv

CA( T , P) ⎛ Z( T , P) 0 + CB( T , P) ⎞
( ) (
fl( T , P) := Z( T , P) − 1 − ln Z( T , P) − CB( T , P) −
0 0 ) CB( T , P)
⋅ ln⎜
⎜ Z( T , P)
⎝ 0 ⎠

CA( T , P) ⎛ Z( T , P) 2 + CB( T , P) ⎞
( ) (
fv( T , P) := Z( T , P) − 1 − ln Z( T , P) − CB( T , P) −
2 2 ) CB( T , P)
⋅ ln⎜
⎜ Z( T , P)
⎝ 2 ⎠

Given fl( T , P) − fv( T , P) 0 T := Find( T) T = 102.0671

Residual entropy for liquid (DELSL) and vapor (DELSV) phases

⎛ ⎛ Z( T , P) 0 + CB( T , P) ⎞ ⎞
(
DELSL( T , P) := ⎜ R⋅ ln Z( T , P) − CB( T , P) + ) ⋅ ln⎜
Da( T) 5
⋅ 10
⎜ 0 b ⎜ Z( T , P)
⎝ ⎝ 0 ⎠⎠

⎛ ⎛ Z( T , P) 2 + CB( T , P) ⎞ ⎞
(
( , P) := ⎜ R⋅ ln Z( T , P) − CB( T , P) + ) ⋅ ln⎜
Da( T) 5
DELSVT ⋅ 10
⎜ 2 b ⎜ Z( T , P)
⎝ ⎝ 2 ⎠⎠

Residual enthalpy for liquid (DELHL) and vapor (DELHV) phases

⎡ T⋅ Da( T) − a( T) ⎛ Z( T , P) 0 + CB( T , P) ⎞⎤ 5
⎢ (
DELHL( T , P) := ⎢R⋅ T⋅ Z( T , P) − 1 +
0 ) b
⋅ ln⎜
⎜ Z( T , P)
⎥ ⋅ 10

⎣ ⎝ 0 ⎠⎦

⎡ T⋅ Da( T) − a( T) ⎛ Z( T , P) 2 + CB( T , P) ⎞⎤ 5
DELHVT
⎢ (
( , P) := ⎢R⋅ T⋅ Z( T , P) − 1 +
2 ) b
⋅ ln⎜
⎜ Z( T , P)
⎥ ⋅ 10

⎣ ⎝ 2 ⎠⎦
Ideal gas properties changes relative to the initial state

Cp ⋅ T − Ti
1
( 2 2 ) (
Cp ⋅ T − Ti
2
3 3 ) (
Cp ⋅ T − Ti
3
4 4 )
( , P) := Cp ⋅ ( T − Ti) +
DELHIGT + +
0 2 3 4

Cp ⋅ T − Ti( 2 2 ) Cp ⋅ T − Ti ( 3 3 )
( , P) := Cp ⋅ ln⎛⎜ ⎞ + Cp ⋅ ( T − Ti) + − R⋅ 10 ⋅ ln⎛⎜
T 2 3 5 P⎞
DELSIGT +
0
⎝ Ti ⎠ 1 2 3 ⎝ Pi ⎠
Find vapor-liquid split x := 0.5

Given

x⋅ DELSVT
( , P) + ( 1 − x) ⋅ DELSL( T , P) + DELSIGT
( , P) DELSin

x := Find( x) x = 0.1408 Fraction vapor


th
Solutions to Chemical and Engineering Thermodynamics, 4 ed. Chapter 7

HV := DELHVT
( , P) + DELHIGT
( , P) SV := DELSVT
( , P) + DELSIGT
( , P)

HL := DELHL( T , P) + DELHIGT
( , P) SL := DELSL( T , P) + DELSIGT
( , P)

δH := x⋅ HV + ( 1 − x) ⋅ HL − DELHin δS := x⋅ SV + ( 1 − x) ⋅ SL − DELSin

SUMMARY OF RESULTS

FEED LIQUID VAPOR


Temperature, K Ti = 120 T = 102.0671 T = 102.0671
Pressure, bar Pi = 30 P=3 P=3

Vapor-liquid split x = 0.1408

Compressibility Z( Ti , Pi) = 0.1004 Z( T , P) = 0.0106 Z( T , P) = 0.934


0 0 2
Enthalpy, J/mol 3
(relative to feed) 0 HL = −7.1435× 10 HV = −630.1699

Entropy, J/mol K 0 SL = −49.936 SV = 13.8781


(relative to feed)

Enthalpy change
J/mol δH = −164.5454

Entropy change − 15
J/mol K δS = 7.1054× 10
th
Solutions to Chemical and Engineering Thermodynamics, 4 ed. Chapter 7

7.58 This problem was solved using the attached Mathcad worksheet. The results are

T(oC) Pvap with α(T) Pvap with α =1 (P in kPa)

273.15 0.3137 166.57


283.15 0.5529 221.329
293.15 1.697 288.55
303.15 3.208 369.83
323.25 9.994 580.97
343.15 26.681 867.65
373.15 92.355 1467.0
393.15 186.67 1997.1
423.25 463.23 3016.5
448.15 886.08 4094.2
474.15 1599.4 5456.5
523.15 4065.2 8759.0
623.15 16744 18865
643.15 21060

As can be seen, the S-R-K equation is of comparable accuracy to the P-R


equation. In both cases if the α parameter is set to one, the results are not very
good, indeed quite bad at low temperatures.

The Mathcad worksheet used in solving this problem is given below.


th
Solutions to Chemical and Engineering Thermodynamics, 4 ed. Chapter 7

VAPOR PRESSURE CALCULATION USING SRK EQUATION

Read in properties for Water Tc := 647.3 Pc := 220.48 om := 0.344

kappa calculation kap := 0.480 + 1.574⋅ om − 0.176⋅ om⋅ om


2 2
R⋅ Tc R ⋅ Tc
S-R-K Constants: R := 0.00008314 b := 0.08664⋅ ac := 0.42748⋅
Pc Pc
Note that these are being defined as a
2
function of temperature since we will ⎡ ⎛ T⎞⎤
need to interate on temperature. alf( T) := 1⋅ ⎢ 1 + kap⋅ ⎜ 1 − ⎥ a( T) := ac ⋅ alf( T)
⎣ ⎝ Tc ⎠ ⎦
a ( T) ⋅ P P⋅ b
CA( T , P) := CB( T , P) :=
2 R⋅ T
( R ⋅ T)
Z( T , P) := A ← CA( T , P)
B ← CB( T , P)
⎛ −A ⋅ B ⎞
⎜ 2 Vector of coefficients in the SRK equation
V← ⎜
A − B − B⎟ in the form
⎜ −1 ⎟ 0=-A*B+(A-B^2-B)*Z-*Z^2+Z^3

⎝ 1 ⎠
ZZ ← polyroots ( V) Solution to the cubic
for i ∈ 0 .. 2

(ZZi ← 0) if (Im(ZZi) ≠ 0) Set any imaginary roots to zero


ZZ ← sort ( ZZ) Sort the roots

ZZ ← ZZ ⎛ ZZ < 10− 5 ⎞
if
0 ⎝2 0 ⎠ Set the value of any imaginary roots
to value of the real root
ZZ ← ZZ if ⎛ ZZ < 10
− 5⎞
2 0 ⎝ 2 ⎠
ZZ
Enter temperature T, and pressure P.
Depending on what is specified in the Given and Find statements
below, either T or P is specified and the other is an initial guess
which may have to be adjusted as the other variable is changed,
especially as the critical point is approached.This worksheet will
probably not converge to a solution when T or P are within 3 to 5% T := 643.15
of the critical values unless an extremely good initial guess is
provided. One way to obtain a good initial is to start well below the P := 210
critical region and step towards it using the result of previous calculations.
th
Solutions to Chemical and Engineering Thermodynamics, 4 ed. Chapter 7

Fugacity expressions [actually ln(f/P)] for the liquid fl and vapor fv

CA( T , P) ⎛ Z( T , P) 0 + CB( T , P) ⎞
( ) (
fl( T , P) := Z( T , P) − 1 − ln Z( T , P) − CB( T , P) −
0 0 ) CB( T , P)
⋅ ln⎜
⎜ Z( T , P)
⎝ 0 ⎠

CA( T , P) ⎛ Z( T , P) 2 + CB( T , P) ⎞
( ) (
fv( T , P) := Z( T , P) − 1 − ln Z( T , P) − CB( T , P) −
2 2 ) CB( T , P)
⋅ ln⎜
⎜ Z( T , P)
⎝ 2 ⎠

Given fl( T , P) − fv( T , P) 0 Equate log of fugacity coefficients


Solve equality of fugacities
P := Find( P) P = 210.59561

Fugacity Fugacity coefficient

fugl
fugl := P⋅ exp( fl( T , P) ) fugl = 141.78565 fl( T , P) = −0.39562 phil :=
P

fugv := P⋅ exp( fv( T , P) ) fugv = 141.78565 fv( T , P) = −0.39562

SUMMARY OF RESULTS

T = 643.15
K Vapor pressure, bar P = 210.59561
LIQUID VAPOR
Compressibility Z( T , P) = 0.25283 Z( T , P) = 0.42294
0 2
Fugacity, bar
fugl = 141.78565 fugv = 141.78565
th
Solutions to Chemical and Engineering Thermodynamics, 4 ed. Chapter 7

7.59 (also available as a Mathcad worksheet)


The solution is that the final temperature is 131.34 K, and the final pressure is
37.036 bar.
7.59 Using SRK EOS with the approximate two-constant heat capacity expression

Property Data Tc := 126.2 Pc := 33.94 om := 0.04 Cp1 := 27.2 Cp2 := 0.0042


(T in K, P in bar):
R := 0.00008314 kap := 0.480 + 1.574⋅ om − 0.176⋅ om⋅ om
Initial Conditions (Vt=total volume, m^3): Ti := 170 Pi := 100 Vt := 0.15
2 2
R⋅ Tc R ⋅ Tc
Peng-Robinson Constants: b := 0.08664⋅ ac := 0.42748⋅
Pc Pc
Initial temperature T := Ti
Note that these are being defined as a 2
⎡ ⎛ T ⎞⎤
function of temperature since we will need to alf( T) := 1⋅ ⎢ 1 + kap⋅ ⎜ 1 − ⎥
interate on temperature later to obtain the final ⎣ ⎝ Tc ⎠ ⎦ a( T) := ac ⋅ alf( T)
state of the system d
Da( T) := a ( T)
dT
Find initial molar volume and number of moles R⋅ Ti
Start with initial guess for volume, m^3/mol V :=
Pi

R⋅ T a ( T)
Solve P-R EOS for initial volume Given Pi − Vi := Find( V)
V− b [ V⋅ ( V + b ) ]

−4 Vt 3
Initial molar volume and Vi = 1.02 × 10 Ni := Ni = 1.471 × 10
number of moles Vi

DELSi:= ⎡⎢R⋅ ln⎡⎢( Vi − b ) ⋅ ⎤ + Da( T) ⋅ ln⎛ Vi + b ⎞⎤ ⋅ 105


Pi
Entropy departure at the ⎥ ⎜ ⎥
initial conditions ⎣ ⎣ R⋅ T⎦ b ⎝ Vi ⎠⎦

0.15
Now consider final state Nf := Ni − 10⋅ 50 Vf := V := Vf
Nf
Type out final number of −4
moles and specific volume Nf = 971.269 Vf = 1.544 × 10

Final pressure, will change in course R⋅ T a ( T)


of solving for the final temperature Pf ( T) := −
V− b V⋅ ( V + b )

V + b ⎞⎤
DELS( T) := ⎡⎢R⋅ ln⎡⎢( V − b ) ⋅
Pf ( T) ⎤
⋅ ln⎛⎜
Entropy departure Da( T) 5
at final conditions ⎥+ ⎥ ⋅ 10
⎣ ⎣ R⋅ T ⎦ b ⎝ V ⎠⎦

27.2⋅ ln⎛⎜ ⎞ + 0.0042⋅ ( T − Ti) − R⋅ 105⋅ ln⎛ Pf( T) ⎞ + DELS( T) − DELS


Solve for final Given T
temperature using 0 ⎜ Pi
⎝ Ti ⎠ ⎝ ⎠
S(final)-S(initial)=0
T := Find( T)
Type out solution T = 131.34 Pf ( T) = 37.076
th
Solutions to Chemical and Engineering Thermodynamics, 4 ed. Chapter 7

7.60 a) At a given temperature, the stability limit of a fluid is determined by the


following criterion (Note that this leads to the spinodal curve)
FG IJ
∂P
=0
H K
∂V T

For the given EOS, the stability limit of a fluid undergoing a pressure change at
constant temperature is
FG IJ
∂P RT BRT
= − 2 −2 3 −3 4 = 0
CRT
H K
∂V T V V V
or V 2 + 2 BV + 3C = 0

In order to have a phase transition, there must be two distinct stability limits,
i.e., the above quadratic equation must have two different roots of V. Therefore,
(2 B) 2 − 4 × 1 × (3C) > 0
or B 2 > 3C

b) According to Illustration 5.2-1


∂P LM F I OP
NH K
dU = CVdT + T − P dV
∂T V Q
LM F ∂P I OP
N H ∂T K
But for the given EOS T −P =0
V Q
Therefore,
z z
∆U = dU = CV (V , T )dT

FG ∂C IJ =T
FG ∂ P IJ
2
= 0 (Because B and C are not functions of T)
H ∂V K H ∂T K
V
Since 2
T V

Therefore Cv = Cv (T ) = CV* = a + bT
and

z
T2
b 2
∆U = (a + bT )dT = a(T2 − T1) + (T2 − T12 )
T1
2
The internal energy change is the same for an ideal gas.

c) According to Eqn 6.2-19


C F I
∂P
d S = V dT +
T H K
∂T V
dV so that

TF I
∂P FG RT + BRT + CRT IJ
H K
FG ∂T IJ = − ∂T = P = H V V V K V
2 3

H ∂V K S C a + bT V a + bT

For an ideal gas,


RT FG IJ
FG ∂T IJ =
P
=
V
=
H K
RT
H ∂V K S a + bT a + bT V (a + bT )
7
∆ F→ D V > 0 and ∆ F→ D H > 0, and that for an equibrium transition (i.e., G F = G D )
7.62 a) Since ∆ F→ D H ⎛ ∂G ⎞ the
it follows that ∆ F→D S = > 0, so that SD > SF . Further, since ⎜ ⎟ =S
T ⎝ ∂T ⎠P
slope of the Gibbs energy versus temperature must be greater for the denatured of
unfolded protein than for the folded protein. This is shown below:

G F

T*
If T < T* then GF > GD, so that the folded state or natural state is the stable
equilibrium state. If T > T* then GF < GD, so that the unfolded state is then the
stable equilibrium state. So T* is the “melting temperature” of the protein.

⎛ dP ⎞ ∆ F→D H
b) Since, by the Clapeyron equation ⎜ ⎟= > 0 , it follows that as the pressure on the
⎝ dT ⎠ T∆ F→ D V
protein is increases, T* will increase. That is, the protein will unfolded or denature at a
higher temperature as the pressure is increased.

The density of ethanol is 0.789 g/cc at 20 C which we will also use at 126 C, and its mole
weight is 46.07. Therefore its liquid molar volume is
3 6
⎛ 46.07⋅ 10− 3⋅ kg ⎞ kPa := 10 ⋅ Pa MPa := 10 ⋅ Pa

Vl :=
⎝ mol ⎠
−5 3
3 kg Vl = 5.839 × 10 m
0.789⋅ 10 ⋅
3
m
a)
f := fsat ⋅ exp⎡⎢
[ ( 25⋅ MPa − 505⋅ kPa) ⋅ Vl] ⎤
⎥ f = 7.066 × 10 Pa
5
⎣ R⋅ T ⎦

b)

VV( p ) := Vl⋅ ⎡⎣ 1 + 1.09⋅ 10 ⋅ ( 101.3⋅ kPa − p ) ⎤⎦


−6 −1
⋅ kPa −5 3
VV( 25⋅ MPa) = 5.681 × 10 m
⎛ 1 ⌠ 25⋅ MPa ⎞
f := fsat ⋅ exp
⎜ ⋅⎮ VV( p ) dp 5
⎜ R⋅ T ⌡ f = 7.024 × 10 Pa
⎝ 505⋅ kPa ⎠
th
Solutions to Chemical and Engineering Thermodynamics, 4 ed. Chapter 7

7.62 a) Since
∆ F→ D V > 0 and ∆ F→ D H > 0, and that for an equibrium transition (i.e., G F = G D )
∆ F→ D H ⎛ ∂G ⎞
it follows that ∆ F→D S = > 0, so that SD > SF . Further, since ⎜ ⎟ =S
T ⎝ ∂T ⎠P
the slope of the Gibbs energy versus temperature must be greater for the
denatured of unfolded protein than for the folded protein. This is shown below:

G F

T*

If T < T* then GF > GD, so that the folded state or natural state is the stable
equilibrium state. If T > T* then GF < GD, so that the unfolded state is then the
stable equilibrium state. So T* is the “melting temperature” of the protein.

⎛ dP ⎞ ∆ F→D H
b) Since, by the Clapeyron equation ⎜ ⎟= > 0 , it follows that as the
⎝ dT ⎠ T∆ F→ D V
pressure on the protein is increases, T* will increase. That is, the protein will
unfolded or denature at a higher temperature as the pressure is increased.

7.63 The VISUAL Basic program Peng-Robinson Equation of State is used for the
Peng-Robinson predictions, the correlation in the Visual Basic Property program will be
used for the ‘literature data”, and the MATHCAD worksheet PRVAPP.MCD is used for
the a(T) = 1 calculation, since it is the easiest to modify. The results are as follows.

T( K) P (bar) literature, P(bar), P-R a(T) P(bar), P-R a(T)=1


115 1.297 1.343 3.452
120 1.884 1.943 4.467
130 3.636 3.723 7.085
140 6.377 6.496 10.59
150 10.39 10.54 15.11
160 15.94 16.14 20.76
170 23.36 23.61 27.65
180 33.01 33.27 35.89
190 45.42 45.48 45.59

and, of course at Tc = 190.4, both P-R calculations give P = 46.0 bar = Pc, since the E.O.S.
parameters have been fitted to give the correct critical point conditions (since α(TC ) = 1. )

Name : methane
Formula: CH4
Molecular weight (g/mol) = 16.043
Normal boiling point (K) = 111.6
th
Solutions to Chemical and Engineering Thermodynamics, 4 ed. Chapter 7

Critical temperature (K) = 190.4


Critical pressure (bar) = 46.0
Critical volume (cm3/mol) = 99.2
Critical compressibility factor = 0.288
Pitzer's acentric factor = 0.011
Isobaric heat capacity of the ideal gas (J/mol.K)
Cp = A + B*T + C*T^2 + D*T^3
A = 1.925e+01
B = 5.213e-02
C = 1.197e-05
D = -1.132e-08
------------------------------------------------------------------
** Vapor-Liquid Equilibrium **

Temperature (K) = 111.6


Vapor pressure (bar) = 1.0249

Vapor Liquid
Compressibility factor 0.96673 0.00373
Specific volume (m^3/mol) 8.75155e-03 3.37236e-
05
Density (g/cm^3) 1.83316e-03 4.75720e-
01
Fugacity (bar) 0.99183 0.99186
Fugacity coeff 0.96771 0.96774
Enthalpy departure (J/mol) -73.98 -8266.44
Enthalpy ideal (J/mol) -5661.75 -5661.75
Enthalpy (J/mol) -5735.73 -13928.18
Entropy departure (J/mol K) -0.39 -73.80
Entropy ideal (J/mol K) -29.21 -29.21
Entropy (J/mol K) -29.60 -103.01
** Vapor-Liquid Equilibrium **

Molar volume (m^3/mol) Density (g/cm^3)


T (K) P (bar) Vapor Liquid Vapor Liquid
115.0 1.343 6.82605e-03 3.41685e-05 2.35026e-03 4.69525e-01
120.0 1.943 4.85847e-03 3.48771e-05 3.30207e-03 4.59986e-01
125.0 2.726 3.54925e-03 3.56597e-05 4.52011e-03 4.49891e-01
130.0 3.723 2.65044e-03 3.65291e-05 6.05295e-03 4.39184e-01
135.0 4.969 2.01609e-03 3.75015e-05 7.95749e-03 4.27797e-01
140.0 6.496 1.55770e-03 3.85977e-05 1.02992e-02 4.15647e-01
145.0 8.342 1.21889e-03 3.98450e-05 1.31620e-02 4.02636e-01
150.0 10.539 9.63678e-04 4.12800e-05 1.66477e-02 3.88639e-01
155.0 13.128 7.67798e-04 4.29535e-05 2.08948e-02 3.73497e-01
160.0 16.143 6.15040e-04 4.49387e-05 2.60845e-02 3.56998e-01
165.0 19.625 4.93846e-04 4.73457e-05 3.24858e-02 3.38848e-01
170.0 23.612 3.96190e-04 5.03529e-05 4.04932e-02 3.18611e-01
175.0 28.146 3.15943e-04 5.42758e-05 5.07781e-02 2.95583e-01
180.0 33.270 2.48265e-04 5.97618e-05 6.46204e-02 2.68449e-01
185.0 39.030 1.88258e-04 6.85514e-05 8.52179e-02 2.34029e-01
190.0 45.478 1.18996e-04 9.18577e-05 1.34820e-01 1.74651e-01
th
Solutions to Chemical and Engineering Thermodynamics, 4 ed. Chapter 7

Volume-temperature coexistence curve of methane


Vapor Pressure of methane
190
50

180
45

170 40

160 35

150 30

25
140
20
130
15
120
10

110
5
0.00001 0.00010 0.00100 0.01000

V (m^3/mol) 0
110 120 130 140 150 160 170 180 190

Vapor Liquid Temperature (K)

Modified MATHCAD worksheet for Peng-Robinson vapor pressure calculation with a(T) = 1.
th
Solutions to Chemical and Engineering Thermodynamics, 4 ed. Chapter 7

Tc := 190.4 Pc := 46.0 om := 0.011 kap := 0.37464 + 1.54226om


⋅ − 0.26992om
⋅ ⋅ om
2 2
R ⋅ Tc R ⋅ Tc
Peng-Robinson Constants: R := 0.00008314 b := 0.07780⋅ ac := 0.45724⋅
Pc Pc
a( T) := ac a ( T) ⋅ P P⋅ b d
CA( T , P) := CB( T , P) := Da( T) := a( T)
2 R⋅ T dT
( R ⋅ T)
Z( T , P) := A ← CA( T , P)
B ← CB( T , P)

(
⎡⎢ − A ⋅ B − B2 − B3 ) ⎥⎤ Vector of coefficients in the PR equation
⎢ 2 ⎥ in the form
V ← ⎢ A − 3⋅ B − 2⋅ B ⎥ 0=-(A*B-B^2-B^3)+(A-3*B^2-2*B)*Z-(1-B)*Z^2+Z^3
⎢ −( 1 − B) ⎥
⎢ ⎥
⎣ 1 ⎦
ZZ ← polyroots ( V) Solution to the cubic

for i ∈ 0 .. 2
(ZZi ← 0) if (Im(ZZi) ≠ 0) Set any imaginary roots to zero
Sort the roots
ZZ ← sort ( ZZ)

ZZ ← ZZ ⎛ ZZ < 10− 5 ⎞
if Set the value of any imaginary roots
0 ⎝ 0 2 ⎠ to value of the real root
ZZ ← ZZ if ⎛ ZZ < 10
− 5⎞
2 0 ⎝ 2 ⎠
ZZ

T := 170 K P := 20 bar

Fugacity expressions [actually ln(f/P)] for the liquid fl and vapor fv

CA( T , P) ⎡ Z( T , P) 0 + ( 1 + 2) ⋅ CB( T , P) ⎤
( ) (
fl( T , P) := Z( T , P) − 1 − ln Z( T , P) − CB( T , P) −
0 0 ) 2⋅ 2⋅ CB( T , P)
⋅ ln⎢
⎢ Z( T , P) + ( 1 − 2) ⋅ CB( T , P) ⎥

⎣ 0 ⎦

CA( T , P) ⎡ Z( T , P) 2 + ( 1 + 2) ⋅ CB( T , P) ⎤
( ) (
fv( T , P) := Z( T , P) − 1 − ln Z( T , P) − CB( T , P) −
2 2 ) 2⋅ 2⋅ CB( T , P)
⋅ ln⎢
⎢ Z( T , P) + ( 1 − 2) ⋅ CB( T , P) ⎥

⎣ 2 ⎦

Given fl( T , P) − fv( T , P) 0 Equate log of fugacity coefficients


Solve equality of fugacities
P := Find( P) P = 27.65147

Fugacity Fugacity coefficient


fugl
fugl := P⋅ exp( fl( T , P) ) fugl = 19.90655 fl( T , P) = −0.32863 phil := phil = 0.71991
P
fugv
fugv := P⋅ exp( fv( T , P) ) fugv = 19.90655 fv( T , P) = −0.32863 phiv := phiv = 0.71991
P
SUMMARY OF RESULTS
T = 170 K Vapor pressure, bar P = 27.65147
LIQUID VAPOR
Compressibility Z( T , P) = 0.10692 Z( T , P) = 0.60688
0 2
Fugacity coefficient phil = 0.71991 phiv = 0.71991

Fugacity, bar fugl = 19.90655 fugv = 19.90655


th
Solutions to Chemical and Engineering Thermodynamics, 4 ed

7.42 Easy way


dU dV dS Q
= Q− P ; = + Sgen
dt dt dt T
System of constant entropy Q = −TSgen
Also constant pressure
dU dV d
= −TSgen − P = −TSgen − ( PV )
dt dt dt
dU d d dH
+ ( PV ) = (U + PV ) = = −TSgen ≤ 0
dt dt dt dt
⇒ H = maximum at equilibrium
dH = 0
d 2H ≥ 0 ⇒
dH = TdS + VdP

d 2H =
FG ∂ T IJ (dS ) + FG ∂ T IJ dSdP + FG ∂ V IJ dPdS +
FG ∂ V IJ (dP)2
H∂ SK
2

P
H ∂ PK H∂ SK S P
H ∂ PK S

d H =G
F ∂ T IJ a∂ S f ≥ 0 ⇒ FG ∂ T IJ ≥ 0
2
H∂ SK H∂ SK
2

P P

FG ∂ T IJ = T ≥ 0 ⇒ C > 0
H∂ SK C
P P
P

More theoretically correct way


Equilibrium criterion for a closed system at constant entropy and pressure.
dU dV dS Q
= Q− P ; = + Sgen
dt dt dt T
dS
Q=T − TSgen
dt
dU dS dV
=T −P − TSgen
dt dt dt
dS dV d
Constant entropy = 0 ; constant pressure P = ( PV )
dt dt dt
dU d d
⇒ = − ( PV ) − TSgen ; (U + PV ) = −TSgen
dt dt dt
dH dH
= −TSgen ≤ 0 ⇒ ≤ 0 ⇒ H = minimum stability
dt dt
d 2 H > 0 but dH = TdS + VdP + Gi dN

c
d 2 H = HSS
I
+ HSS
II
hc h + c H + H hcdN h
dS I
2 I
NN
II
NN
I 2

+ 2c H + H hdS dN > 0
I
SN
II
SN
I I

N +N
N H cdS h + 2 N H dS dN c h
I II
I 2 2
= I II
I I
SS
I I
SN
I I
+ N I HNN
I
dN I >0
N N
Making a transformation of variables
H
dx1 = dS I + SN dN ; dx2 = dN I
HNN
th
Solutions to Chemical and Engineering Thermodynamics, 4 ed. Chapter 7

θ 1 = NHSS ; θ 2 =
c NH SS NHNN − N 2 HSN
2
h
NHSS
As a check
θ 1dx12 + θ 2dx22

= NHSS
FG dS + H I
dN J
2
+
NHSS NHNN − N 2 HSN
2
2

H H K
I I
SN
dN I
SS NHSS

2 H SN I I H2 I2
= NH SSdSI + 2NH SS ⋅ dS dN + NH SS ⋅ SN
2 dN
H SS H SS
2
2 NH SN 2
+ NH NN dN I − dN I
H SS
which is correct so
θ 1dx12 + θ 2dx22 ≥ 0 ⇒ θ 1 > 0; θ 2 > 0

θ 1 = NHSS = N
FG ∂ H IJ = N ∂ FG ∂ H IJ = N FG ∂ T IJ > 0
2

H ∂ S K ∂ S H ∂ S K H∂ SK
2
P P P

but d S =
C
dT − G
F ∂ V IJ ; FG ∂ S IJ = C ⇒ FG ∂ S IJ = NC
H∂ TK H∂ TK T H∂ TK T
P P P
T P P P
T
⇒N > 0 ; N > 0, T > 0 ⇒ CP > 0
NCP
Second criterion
NHSS NHNN − N 2 HSN 2
; HNN =
∂ 2H
=

(G )
FG =
∂G IJ FG IJ
NHSS ∂ N S, P ∂ N
2
H
S, P ∂N K H K S, P

HSN =
∂ FG ∂ H IJ = ∂ (T ) = FG ∂ T IJ
∂N H∂SK ∂N
S,P P, N
H∂ NK S,P
S,P

−N
H 2
= NG
F ∂ G IJ − N a∂ T ∂ N f 2
S,P
=?
H∂ NK a∂ T ∂ S f
SN
NHNN
H SS S, P P, N
7.43 Available as a Mathcad worksheet
RT
7.44 Clausius EOS: P =
V − b(T )
FG ∂ P IJ <0
Condition for stability is
H ∂V K T
For the Clausius equation
FG IJ
∂P
=−
RT
Since R > 0 , T > 0 and (V − b)2 > 0 .
H K
∂V T (V − b)2

Then G
F ∂ P IJ must be negative or
H ∂V KT

FG ∂ P IJ < 0 ⇒ Single phase is stable at all conditions.


H ∂V K T
7.45 See solution to Problem 7.41. If fluid is unstable, then a vapor-liquid phase
transition can occur.
th
Solutions to Chemical and Engineering Thermodynamics, 4 ed. Chapter 7

7.46 Redlich-Kwong equation of state


f
=
1
z
V = ZRT P
FG RT − PIJ dV − ln Z + (Z − 1)
ln
P RT V =∞
HV K
z FGH IJ z FGH RTV − VRT− b + V (aV(T+)b)IJK dV
V
RT
− P dV =
V K V =∞

z
V
V V −b dV
= RT ln − RT ln + a (T )
V →∞ (V − b)V →∞ V =∞
V (V + b)

F I FG IJ
V 1 V +b Z a Z+B F I
= RT ln
H K H K
V −b
+ a − ln
b V
= RT ln − ln
Z−B b Z H K
lnF
f Z a Z + BI
ln = ln
P

Z − B bRT H Z K
− ln Z + ( Z − 1)

lnF
a Z + BI
= − ln( Z − B) −
bRT H Z K
+ (Z − 1)

lnF
a Z + Pb RT I
= (Z − 1) − ln(Z − B) −
bRT H Z K
aP bP
A= 2
; B=
RT RT
f A Z+B F I
ln
P
= (Z − 1) − ln(Z − B) − ln
B Z H K
Using the same analysis for the Soave-Redlich-Kwong equation of state leads to
the following
f F Pb I
a(T ) LM a
Z + Pb RT fO
PQ
ln = (Z − 1) − ln Z −
P HRT

K
RTb
ln
Z N
= ( Z − 1) − ln( Z − B) −
a (T ) Z+B LM OP
RTb
ln
Z N Q
7.47 Available as a Mathcad worksheet. See Mathcad worksheet for the graphs.

fi Pi . exp fop i 150

100

P
i
50
10 .Pa
5

50
0 0.5 1 1.5 2 2.5
V
i
log
b
th
Solutions to Chemical and Engineering Thermodynamics, 4 ed. Chapter 7

R.T a Pi . Vi
Pi 3 3 1
0.5 . Zi V100 = 5.655 10 m mole
Vi b T Vi . Vi b R. T

Vi a Vi
fop i ln ln Zi Zi 1 . ln
1.5 .
Vi b T R.b Vi b

fi Pi . exp fop i

500

400

300
P
i

10 .Pa
5

200

100

0
0 0.5 1 1.5 2 2.5
V
i
log
b

7
2.5 10

7
2 10

7
1.5 10

f
i
7
1 10

6
5 10

0
0 0.5 1 1.5 2 2.5
V
i
log
b
th
Solutions to Chemical and Engineering Thermodynamics, 4 ed. Chapter 7

7.48
a) If ethanol is an ideal gas, the
f = P, so that the fugacity of ethanol is 505 kPa
b) Starting from eqn. 7.4-12 we have that

ln (f/P)=(B/V) - lnZ +(Z-1) mol := 1

P := 505000Pa
⋅ T := ( 273.15 + 126) ⋅ K R := 8.314⋅
(Pa⋅m3) B := −523⋅ 10
−6 m

3

mol⋅ K mol
( R ⋅ T) −3 3
V := V = 6.571 × 10 m
P

Given ( P⋅ V) B
1+
R⋅ T V −3 3
V := Find( V) V = 5.998 × 10 m
( P⋅ V)
Z :=
R⋅ T Z = 0.913

fsat := P⋅ exp⎡⎢ ⋅ −ln( Z) + ( Z − 1)⎤⎥


B
5
⎣V ⎦ fsat = 4.592 × 10 Pa

7.49 Available as a Mathcad worksheet

The density of ethanol is 0.789 g/cc at 20 C which we will also use at 126 C, and its mole
weight is 46.07. Therefore its liquid molar volume is
3 6
⎛ 46.07⋅ 10− 3⋅ kg ⎞ kPa := 10 ⋅ Pa MPa := 10 ⋅ Pa

Vl :=
⎝ mol ⎠
−5 3
3 kg Vl = 5.839 × 10 m
0.789⋅ 10 ⋅
3
m
a)
f := fsat ⋅ exp⎡⎢
[ ( 25⋅ MPa − 505⋅ kPa) ⋅ Vl] ⎤
⎥ f = 7.066 × 10 Pa
5
⎣ R⋅ T ⎦

b)

VV( p ) := Vl⋅ ⎡⎣ 1 + 1.09⋅ 10 ⋅ ( 101.3⋅ kPa − p ) ⎤⎦


−6 −1
⋅ kPa −5 3
VV( 25⋅ MPa) = 5.681 × 10 m
⎛ 1 ⌠ 25⋅ MPa ⎞
f := fsat ⋅ exp
⎜ ⋅⎮ VV( p ) dp 5
⎜ R⋅ T ⌡ f = 7.024 × 10 Pa
⎝ 505⋅ kPa ⎠
th
Solutions to Chemical and Engineering Thermodynamics, 4 ed. Chapter 7

7.50 FUGACITY CALCULATION USING SRK EQUATION

Read in properties for Pentane Tc := 469.6 Pc := 33.74 om := 0.251

kappa calculation kap := 0.480 + 1.574⋅ om − 0.176⋅ om⋅ om

2 2
R ⋅ Tc R ⋅ Tc
S-R-K Constants: R := 0.00008314 b := 0.08664⋅ ac := 0.42748⋅
Pc Pc
Note that these are being defined as a
2
function of temperature. ⎡ ⎛ ⎞⎤
T
alf( T) := 1⋅ ⎢ 1 + kap⋅ ⎜ 1 − ⎥ a( T) := ac ⋅ alf( T)
⎣ ⎝ Tc ⎠ ⎦
a ( T) ⋅ P P⋅ b
CA( T , P) := CB( T , P) := d
2 R⋅ T Da( T) := a ( T)
( R ⋅ T) dT
Z( T , P) := A ← CA( T , P)
B ← CB( T , P)
⎛ −A ⋅ B ⎞
⎜ 2 Vector of coefficients in the SRK equation
V← ⎜
A − B − B⎟ in the form
⎜ −1 ⎟ 0=-A*B+(A-B^2-B)*Z-*Z^2+Z^3

⎝ 1 ⎠
ZZ ← polyroots ( V) Solution to the cubic
for i ∈ 0 .. 2

(ZZi ← 0) if (Im(ZZi) ≠ 0) Set any imaginary roots to zero


ZZ ← sort ( ZZ) Sort the roots

ZZ ← ZZ ⎛ ZZ < 10− 5 ⎞
if
0 ⎝ 0
2 ⎠ Set the value of any imaginary roots
to value of the real root
ZZ ← ZZ if ⎛ ZZ < 10
− 5⎞
2 0 ⎝ 2 ⎠
ZZ
Enter temperature T, and pressure P.

T := 100
C T := 273.15 + T
K P := 50

Fugacity expressions [actually ln(f/P)] for the liquid fl and vapor fv

CA( T , P) ⎛ Z( T , P) 0 + CB( T , P) ⎞
( ) (
fl( T , P) := Z( T , P) − 1 − ln Z( T , P) − CB( T , P) − ) ⋅ ln⎜
0 0 CB( T , P) ⎜ Z( T , P)
⎝ 0 ⎠

CA( T , P) ⎛ Z( T , P) 2 + CB( T , P) ⎞
( ) (
fv( T , P) := Z( T , P) − 1 − ln Z( T , P) − CB( T , P) − ) ⋅ ln⎜
2 2 CB( T , P) ⎜ Z( T , P)
⎝ 2 ⎠
th
Solutions to Chemical and Engineering Thermodynamics, 4 ed. Chapter 7

Fugacity Fugacity coefficient

fugl
fugl := P⋅ exp( fl( T , P) ) fugl = 6.49272 fl( T , P) = −2.04134 phil := phil = 0.12985
P

fugv := P⋅ exp( fv( T , P) ) fugv = 6.49272 fv( T , P) = −2.04134 fugv


phiv := phiv = 0.12985
P

SUMMARY OF RESULTS

T = 373.15
K Vapor pressure, bar P = 50
LIQUID VAPOR
Compressibility Z( T , P) = 0.23249 Z( T , P) = 0.23249
0 2
Fugacity coefficient phil = 0.12985 phiv = 0.12985

Fugacity, bar fugl = 6.49272 fugv = 6.49272


th
Solutions to Chemical and Engineering Thermodynamics, 4 ed. Chapter 7

Read in properties for Benzene Tc := 562.1 Pc := 48.94 om := 0.212

kappa calculation kap := 0.480 + 1.574⋅ om − 0.176⋅ om⋅ om

2 2
R⋅ Tc R ⋅ Tc
S-R-K Constants: R := 0.00008314 b := 0.08664⋅ ac := 0.42748⋅
Pc Pc
Note that these are being defined as a
2
function of temperature. ⎡
alf( T) := 1⋅ ⎢ 1 + kap⋅ ⎜ 1 −
⎛ T ⎞⎤
⎥ a( T) := ac ⋅ alf( T)
⎣ ⎝ Tc ⎠ ⎦
a ( T) ⋅ P P⋅ b
CA( T , P) := CB( T , P) := d
2 R⋅ T Da( T) := a ( T)
( R ⋅ T) dT
Z( T , P) := A ← CA( T , P)
B ← CB( T , P)
⎛ −A ⋅ B ⎞
⎜ 2 Vector of coefficients in the SRK equation
V← ⎜
A − B − B⎟ in the form
⎜ −1 ⎟ 0=-A*B+(A-B^2-B)*Z-*Z^2+Z^3

⎝ 1 ⎠
ZZ ← polyroots ( V) Solution to the cubic
for i ∈ 0 .. 2

(ZZi ← 0) if (Im(ZZi) ≠ 0) Set any imaginary roots to zero


ZZ ← sort ( ZZ) Sort the roots

ZZ ← ZZ ⎛ ZZ < 10− 5 ⎞
if
0 ⎝ 0
2 ⎠ Set the value of any imaginary roots
to value of the real root
ZZ ← ZZ if ⎛ ZZ < 10
− 5⎞
2 0 ⎝ 2 ⎠
ZZ
Enter temperature T, and pressure P.
T := 100
C T := 273.15 + T
K P := 50

Fugacity expressions [actually ln(f/P)] for the liquid fl and vapor fv

CA( T , P) ⎛ Z( T , P) 0 + CB( T , P) ⎞
( ) (
fl( T , P) := Z( T , P) − 1 − ln Z( T , P) − CB( T , P) −
0 0 ) CB( T , P)
⋅ ln⎜
⎜ Z( T , P)
⎝ 0 ⎠

CA( T , P) ⎛ Z( T , P) 2 + CB( T , P) ⎞
( ) (
fv( T , P) := Z( T , P) − 1 − ln Z( T , P) − CB( T , P) −
2 2 ) CB( T , P)
⋅ ln⎜
⎜ Z( T , P)
⎝ 2 ⎠
th
Solutions to Chemical and Engineering Thermodynamics, 4 ed. Chapter 7

Fugacity Fugacity coefficient

fugl
fugl := P⋅ exp( fl( T , P) ) fugl = 2.01968 fl( T , P) = −3.20908 phil := phil = 0.04039
P

fugv := P⋅ exp( fv( T , P) ) fugv = 2.01968 fv( T , P) = −3.20908 fugv


phiv := phiv = 0.04039
P

SUMMARY OF RESULTS

T = 373.15
K Vapor pressure, bar P = 50
LIQUID VAPOR
Compressibility Z( T , P) = 0.17187 Z( T , P) = 0.17187
0 2
Fugacity coefficient phil = 0.04039 phiv = 0.04039

Fugacity, bar fugl = 2.01968 fugv = 2.01968


th
Solutions to Chemical and Engineering Thermodynamics, 4 ed. Chapter 7

Z( T , P ) A CA ( T , P )
B CB( T , P )
A .B
2
Vector of coefficients in the SRK equation
A B B in the form
V
1 0=-A*B+(A-B^2-B)*Z-*Z^2+Z^3

1
ZZ polyroots ( V) Solution to the cubic
for i ∈ 0 .. 2
ZZi 0 if Im ZZi 0
Set any imaginary roots to zero
ZZ sort( ZZ ) Sort the roots
5
ZZ0 ZZ2 if ZZ0 < 10 Set the value of any imaginary roots
5 to value of the real root
ZZ2 ZZ0 if ZZ2 < 10

ZZ
Enter temperature T, and pressure P.
T 100 C T 273.15 T K P 50

Fugacity expressions [actually ln(f/P)] for the liquid fl and vapor fv

CA ( T , P ) . Z( T , P ) 0 CB( T , P )
fl( T , P ) Z( T , P ) 0 1 ln Z( T , P ) 0 CB( T , P ) ln
CB( T , P ) Z( T , P ) 0

CA ( T , P ) . Z( T , P ) 2 CB( T , P )
fv ( T , P ) Z( T , P ) 2 1 ln Z( T , P ) 2 CB( T , P ) ln
CB( T , P ) Z( T , P ) 2
th
Solutions to Chemical and Engineering Thermodynamics, 4 ed. Chapter 7

7.51 (Solution using Mathcad worksheet)

T = -200oC Pvap = 0.10272 bar


Z V = 0.99512 Z L = 4.414 × 10−4
H V = −55795
. × 103 H L = −12994
. × 104
S V = 17.372 S L = −118.74

T = -180oC Pvap = 1.348 bar


Z V = 0.96359 Z L = 4.955 × 10−3
H V = −51095
. × 103 H L = −11948
. × 104
S V = −32.734 S L = −10615
.

T = -160oC Pvap = 6.750 bar


Z V = 08810
. Z L = 0.02307
H V = −4.7953 × 103 H L = −10805
. × 104
S V = −42.099 S L = −95210
.

T = -140oC Pvap = 20.676 bar


Z V = 0.73096 Z L = 0.07305
H V = −4.7988 × 103 H L = −9.4328 × 103
S V = −49.6785 S L = −84.481

T = -130oC Pvap = 32.310 bar


Z V = 0.61800 Z L = 012528
.
H V = −50406
. × 103 H L = −85449
. × 103
S V = −53938
. S L = −78.418

T = -125oC Pvap = 39.554 bar


Z V = 054226
. Z L = 016843
.
H V = −52985
. × 103 H L = −7.9739 × 103
S V = −56.685 S L = −74.744

T = -120oC Pvap = 47.848 bar


Z V = 0.42788 Z L = 0.24887
H V = −58378
. × 103 H L = −71281
. × 103
S V = −61034
. S L = −69.459
th
Solutions to Chemical and Engineering Thermodynamics, 4 ed. Chapter 7

The Mathcad worksheet for this file is shown below.

7.51 VAPOR PRESSURE CALCULATION USING SRK EQUATION

Read in properties for oxygen Tc := 154.6 Pc := 50.46 om := 0.021

Heat capacity constants


−2 −5 −9
Cp := 25.460 Cp := 1.519⋅ 10 Cp := −0.715⋅ 10 Cp := 1.311⋅ 10
0 1 2 3
Reference state and kappa calculation

Trs := 373.15 Prs := 1.013 kap := 0.480 + 1.574⋅ om − 0.176⋅ om⋅ om


2 2
R ⋅ Tc R ⋅ Tc
S-R-K Constants: R := 0.00008314 b := 0.08664⋅ ac := 0.42748⋅
Pc Pc
Note that these are being defined as a
2
function of temperature since we will ⎡ ⎛
alf( T) := 1⋅ ⎢ 1 + kap⋅ ⎜ 1 −
⎞⎤
T
need to interate on temperature. ⎥ a( T) := ac ⋅ alf( T)
⎣ ⎝ Tc ⎠ ⎦
a ( T) ⋅ P P⋅ b
CA( T , P) := CB( T , P) := d
2 R⋅ T Da( T) := a ( T)
( R ⋅ T) dT
Z( T , P) := A ← CA( T , P)
B ← CB( T , P)
⎛ −A ⋅ B ⎞
⎜ 2 Vector of coefficients in the SRK equation
V← ⎜
A − B − B⎟ in the form
⎜ −1 ⎟ 0=-A*B+(A-B^2-B)*Z-*Z^2+Z^3

⎝ 1 ⎠
ZZ ← polyroots ( V) Solution to the cubic
for i ∈ 0 .. 2

(ZZi ← 0) if (Im(ZZi) ≠ 0) Set any imaginary roots to zero


ZZ ← sort ( ZZ) Sort the roots

ZZ ← ZZ ⎛ ZZ < 10− 5 ⎞
if
0 ⎝ 0
2 ⎠ Set the value of any imaginary roots
to value of the real root
ZZ ← ZZ if ⎛ ZZ < 10
− 5⎞
2 0 ⎝ 2 ⎠
ZZ
Enter temperature T, and pressure P.
Depending on what is specified in the Given and Find statements
below, either T or P is specified and the other is an initial guess T := −173.15 C
which may have to be adjusted as the other variable is changed,
especially as the critical point is approached.This worksheet will
probably not converge to a solution when T or P are within 3 to 5% T := 273.15 + T K
of the critical values unless an extremely good initial guess is
provided. One way to obtain a good initial is to start well below the
critical region and step towards it using the result of previous calculations.
P := 0.15
th
Solutions to Chemical and Engineering Thermodynamics, 4 ed. Chapter 7

Fugacity expressions [actually ln(f/P)] for the liquid fl and vapor fv

CA( T , P) ⎛ Z( T , P) 0 + CB( T , P) ⎞
( ) (
fl( T , P) := Z( T , P) − 1 − ln Z( T , P) − CB( T , P) −
0 0 ) CB( T , P)
⋅ ln⎜
⎜ Z( T , P)
⎝ 0 ⎠

CA( T , P) ⎛ Z( T , P) 2 + CB( T , P) ⎞
( ) (
fv( T , P) := Z( T , P) − 1 − ln Z( T , P) − CB( T , P) −
2 2 ) CB( T , P)
⋅ ln⎜
⎜ Z( T , P)
⎝ 2 ⎠

Given fl( T , P) − fv( T , P) 0 Equate log of fugacity coefficients


Solve equality of fugacities
P := Find( P) P = 2.52595

Fugacity Fugacity coefficient

fugl
fugl := P⋅ exp( fl( T , P) ) fugl = 2.38691 fl( T , P) = −0.05662 phil := phil = 0.94495
P

fugv
fugv := P⋅ exp( fv( T , P) ) fugv = 2.38691 fv( T , P) = −0.05662 phiv := phiv = 0.94495
P
Residual entropy for liquid (DELSL) and vapor (DELSV) phases

⎛ ⎛ Z( T , P) 0 + CB( T , P) ⎞ ⎞
(
DELSL:= ⎜ R⋅ ln Z( T , P) − CB( T , P) + ) ⋅ ln⎜
Da( T) 5
⋅ 10
⎜ 0 b ⎜ Z( T , P)
⎝ ⎝ 0 ⎠⎠

⎛ ⎛ Z( T , P) 2 + CB( T , P) ⎞ ⎞
(
DELSV:= ⎜ R⋅ ln Z( T , P) − CB( T , P) + ) ⋅ ln⎜
Da( T) 5
⋅ 10
⎜ 2 b ⎜ Z( T , P)
⎝ ⎝ 2 ⎠⎠

Residual enthalpy for liquid (DELHL) and vapor (DELHV) phases

⎡ T⋅ Da( T) − a( T) ⎛ Z( T , P) 0 + CB( T , P) ⎞⎥⎤ 5


⎢ (
DELHL:= ⎢R⋅ T⋅ Z( T , P) − 1 +
0 ) b
⋅ ln⎜
⎜ Z( T , P) ⎥
⋅ 10
⎣ ⎝ 0 ⎠⎦

⎡ T⋅ Da( T) − a( T) ⎛ Z( T , P) 2 + CB( T , P) ⎞⎥⎤ 5


⎢ (
DELHV:= ⎢R⋅ T⋅ Z( T , P) − 1 +
2 ) b
⋅ ln⎜
⎜ Z( T , P) ⎥
⋅ 10
⎣ ⎝ 2 ⎠⎦

Ideal gas properties changes relative to the reference state

(
Cp ⋅ T − Trs
1
2 )
2
Cp ⋅ T − Trs
2
( 3 3 ) Cp ⋅ T − Trs
3
( 4 4 )
DELHIG:= Cp ⋅ ( T − Trs ) + + +
0 2 3 4

(
Cp ⋅ T − Trs
2 2 ) Cp ⋅ T − Trs ( 3 3 )
DELSIG:= Cp ⋅ ln⎛⎜
T ⎞
− R⋅ 10 ⋅ ln⎛⎜
2 3 5 P ⎞
+ Cp ⋅ ( T − Trs ) + +
0 ⎝ Trs ⎠ 1 2 3 ⎝ Prs ⎠

Total entropy and enthalpy relative to ideal gas reference state


SL := DELSIG+ DELSL SV := DELSIG+ DELSV HL := DELHIG+ DELHL HV := DELHIG+ DELHV
SUMMARY OF RESULTS
th
Solutions to Chemical and Engineering Thermodynamics, 4 ed. Chapter 7

T = 100 K Vapor pressure, bar P = 2.52595


LIQUID VAPOR
−3
Compressibility Z( T , P) = 8.97042× 10 Z( T , P) = 0.94188
0 2
4 3
Enthalpy, J/mol HL = −1.45382× 10 HV = −7.94408× 10

Entropy, J/mol K SL = −111.53504 SV = −45.59335

Fugacity coefficient phil = 0.94495 phiv = 0.94495

Fugacity, bar fugl = 2.38691 fugv = 2.38691


th
Solutions to Chemical and Engineering Thermodynamics, 4 ed. Chapter 7

7.52

Pure component properties calculation using the SRK equation of state

Read in properties for oxygen Tc := 154.6 Pc := 50.46 om := 0.021

Heat capacity constants


−2 −5 −9
Cp := 25.460 Cp := 1.519⋅ 10 Cp := −0.715⋅ 10 Cp := 1.311⋅ 10
0 1 2 3
Reference state and kappa calculation

Trs := 298.15 Prs := 1.0 kap := 0.480 + 1.574⋅ om − 0.176⋅ om⋅ om


2 2
R ⋅ Tc R ⋅ Tc
S-R-K Constants: R := 0.00008314 b := 0.08664⋅ ac := 0.42748⋅
Pc Pc
Note that these are being defined as a
2
function of temperature for convenience. ⎡ ⎛
alf( T) := 1⋅ ⎢ 1 + kap⋅ ⎜ 1 −
⎞⎤
T
⎥ a( T) := ac ⋅ alf( T)
⎣ ⎝ Tc ⎠ ⎦
a ( T) ⋅ P P⋅ b
CA( T , P) := CB( T , P) := d
2 R⋅ T Da( T) := a ( T)
( R ⋅ T) dT
Z( T , P) := A ← CA( T , P)
B ← CB( T , P)
⎛ −A ⋅ B ⎞
⎜ 2 Vector of coefficients in the SRK equation
V← ⎜
A − B − B⎟ in the form
⎜ −1 ⎟ 0=-A*B+(A-B^2-B)*Z-*Z^2+Z^3

⎝ 1 ⎠
ZZ ← polyroots ( V) Solution to the cubic
for i ∈ 0 .. 2

(ZZi ← 0) if (Im(ZZi) ≠ 0) Set any imaginary roots to zero


ZZ ← sort ( ZZ) Sort the roots

ZZ ← ZZ ⎛ ZZ < 10− 5 ⎞
if
0 ⎝ 0
2 ⎠ Set the value of any imaginary roots
to value of the real root
ZZ ← ZZ if ⎛ ZZ < 10 ⎞
− 5
2 0 ⎝ 2 ⎠
ZZ
Enter temperature T, and pressure P.

T := −125
C T := 273.15 + T
K P := 100
th
Solutions to Chemical and Engineering Thermodynamics, 4 ed. Chapter 7

Fugacity expressions [actually ln(f/P)] for the liquid fl and vapor fv


CA( T , P) ⎛⎜
Z( T , P) + CB( T , P) ⎞
( ) ( )
0
fl( T , P) := Z( T , P) − 1 − ln Z( T , P) − CB( T , P) − ⋅ ln
0 0 CB( T , P) ⎜ Z( T , P)
⎝ 0 ⎠

CA( T , P) ⎛ Z( T , P) 2 + CB( T , P) ⎞
( ) (
fv( T , P) := Z( T , P) − 1 − ln Z( T , P) − CB( T , P) −
2 2 ) CB( T , P)
⋅ ln⎜
⎜ Z( T , P)
⎝ 2 ⎠
Fugacity Fugacity coefficient
fugl
fugl := P⋅ exp( fl( T , P) ) fugl = 34.64672 fl( T , P) = −1.05997 phil := phil = 0.34647
P
fugv
fugv := P⋅ exp( fv( T , P) ) fugv = 34.64672 fv( T , P) = −1.05997 phiv := phiv = 0.34647
P
Residual entropy for liquid (DELSL) and vapor (DELSV) phases

⎛ ⎛ Z( T , P) 0 + CB( T , P) ⎞ ⎞
(
DELSL:= ⎜ R⋅ ln Z( T , P) − CB( T , P) + ) ⋅ ln⎜
Da( T) 5
⋅ 10
⎜ 0 b ⎜ Z( T , P)
⎝ ⎝ 0 ⎠⎠
⎛ ⎛ Z( T , P) 2 + CB( T , P) ⎞ ⎞
(
DELSV:= ⎜ R⋅ ln Z( T , P) − CB( T , P) + ) ⋅ ln⎜
Da( T) 5
⋅ 10
⎜ 2 b ⎜ Z( T , P)
⎝ ⎝ 2 ⎠⎠

Residual enthalpy for liquid (DELHL) and vapor (DELHV) phases

⎡ T⋅ Da( T) − a( T) ⎛ Z( T , P) 0 + CB( T , P) ⎞⎤⎥


(
DELHL:= ⎢R⋅ T⋅ Z( T , P) − 1 + ) ⋅ ln⎜
5
⋅ 10
⎢ 0 b ⎜ Z( T , P) ⎥
⎣ ⎝ 0 ⎠⎦
⎡ T⋅ Da( T) − a( T) ⎛ Z( T , P) 2 + CB( T , P) ⎞⎤ 5
⎢ (
DELHV:= ⎢R⋅ T⋅ Z( T , P) − 1 +
2 ) b
⋅ ln⎜
⎜ Z( T , P)
⎥ ⋅ 10

⎣ ⎝ 2 ⎠⎦

Ideal gas properties changes relative to the reference state

(
Cp ⋅ T − Trs
1
2 2 ) Cp ⋅ T − Trs
2
( 3 3 ) (
Cp ⋅ T − Trs
3
4 4 )
DELHIG:= Cp ⋅ ( T − Trs ) + + +
0 2 3 4

Cp ⋅ T − Trs( 2 2 ) Cp ⋅ T − Trs( 3 3 )
DELSIG:= Cp ⋅ ln⎛⎜
T ⎞
− R⋅ 10 ⋅ ln⎛⎜ ⎞
2 3 5 P
+ Cp ⋅ ( T − Trs ) + +
0
⎝ Trs ⎠ 1 2 3 ⎝ Prs ⎠

Total entropy and enthalpy relative to ideal gas reference state


SL := DELSIG+ DELSL SV := DELSIG+ DELSV HL := DELHIG+ DELHL HV := DELHIG+ DELHV
th
Solutions to Chemical and Engineering Thermodynamics, 4 ed. Chapter 7

−2 T
V := Z( T , P) ⋅ 8.314⋅ 10 ⋅ V = 0.04081 −2 T
0 0 P 0 V := Z( T , P) ⋅ 8.314⋅ 10 ⋅
2 2 P V = 0.04081
2

SUMMARY OF RESULTS

T = 148.15
K Vapor pressure, bar P = 100
LIQUID VAPOR
Compressibility Z( T , P) = 0.3313 Z( T , P) = 0.3313
0 2
Volume, m^3/kmol V = 0.04081 V = 0.04081
0 2

3 3
Enthalpy, J/mol HL = −9.29109× 10 HV = −9.29109× 10

Entropy, J/mol K SL = −83.19194 SV = −83.19194

Fugacity coefficient phil = 0.34647 phiv = 0.34647

Fugacity, bar fugl = 34.64672 fugv = 34.64672

Some representative results are shown below.

T (C) -125 -150 -175 -200


P=1 bar
Z 0.9923 0.9872 0.9766 0.9505
V 12.2227 10.1072 7.9693 5.7804
H -4301.41 -4994.48 -5684.02 -6375.9
S -19.97 -25.1 -31.35 -35.49

P=10 bar
Z 0.9193 0.8565 0.03572 0.04292
V 1.1323 0.877 0.02914 0.0261
H -4561.07 -5357.77 -12395.6 -13706.2
S -40.28 -46.18 -106.11 -121.5

P=50 bar
Z 0.1946 0.1647 0.17634 0.21349
V 0.04795 0.03373 0.02878 0.02597
H -8938.78 -10919.2 -12338.7 -13628.9
S -79.34 -93.79 -106.71 -121.86

P=100 bar
Z 0.3313 0.318 0.34788 0.42446
V 0.04081 0.03256 0.02839 0.02581
H -9291.09 -10896.8 -12261.8 -13530.9
S -83.19 -95.02 -107.39 -122.29
.
th
Solutions to Chemical and Engineering Thermodynamics, 4 ed. Chapter 7

7.53 Pure component properties calculation using the SRK equation of state

Read in properties for Water Tc := 647.3 Pc := 220.48 om := 0.344

Heat capacity constants


−2 −5 −9
Cp := 32.218 Cp := 0.192⋅ 10 Cp := 1.055⋅ 10 Cp := −3.593⋅ 10
0 1 2 3
Reference state and kappa calculation

Trs := 373.15 Prs := 1.013 kap := 0.480 + 1.574⋅ om − 0.176⋅ om⋅ om


2 2
R ⋅ Tc R ⋅ Tc
S-R-K Constants: R := 0.00008314 b := 0.08664⋅ ac := 0.42748⋅
Pc Pc
Note that these are being defined as a
2
function of temperature for convenience. ⎡ ⎛ ⎞⎤
T
alf( T) := 1⋅ ⎢ 1 + kap⋅ ⎜ 1 − ⎥ a( T) := ac ⋅ alf( T)
⎣ ⎝ Tc ⎠ ⎦
a ( T) ⋅ P P⋅ b
CA( T , P) := CB( T , P) := d
2 R⋅ T Da( T) := a ( T)
( R ⋅ T) dT
Z( T , P) := A ← CA( T , P)
B ← CB( T , P)
⎛ −A ⋅ B ⎞
⎜ 2 Vector of coefficients in the SRK equation
V← ⎜
A − B − B⎟ in the form
⎜ −1 ⎟ 0=-A*B+(A-B^2-B)*Z-*Z^2+Z^3

⎝ 1 ⎠
ZZ ← polyroots ( V) Solution to the cubic
for i ∈ 0 .. 2

(ZZi ← 0) if (Im(ZZi) ≠ 0) Set any imaginary roots to zero


ZZ ← sort ( ZZ) Sort the roots

ZZ ← ZZ ⎛ ZZ < 10− 5 ⎞
if
0 ⎝ 0 2 ⎠ Set the value of any imaginary roots
to value of the real root
ZZ ← ZZ if ⎛ ZZ < 10 ⎞
− 5
2 0 ⎝ 2 ⎠
ZZ
Enter temperature T, and pressure P.

T := 50
C T := 273.15 + T
K P := 0.15

Fugacity expressions [actually ln(f/P)] for the liquid fl and vapor fv

CA( T , P) ⎛ Z( T , P) 0 + CB( T , P) ⎞
( ) (
fl( T , P) := Z( T , P) − 1 − ln Z( T , P) − CB( T , P) − ) ⋅ ln⎜
0 0 CB( T , P) ⎜ Z( T , P)
⎝ 0 ⎠

CA( T , P) ⎛ Z( T , P) 2 + CB( T , P) ⎞
( ) (
fv( T , P) := Z( T , P) − 1 − ln Z( T , P) − CB( T , P) − ) ⋅ ln⎜
2 2 CB( T , P) ⎜ Z( T , P)
⎝ 2 ⎠
th
Solutions to Chemical and Engineering Thermodynamics, 4 ed. Chapter 7

Fugacity Fugacity coefficient

fugl
fugl := P⋅ exp( fl( T , P) ) fugl = 0.09983 fl( T , P) = −0.40721 phil := phil = 0.6655
P

−3 fugv
fugv := P⋅ exp( fv( T , P) ) fugv = 0.14972 fv( T , P) = −1.83629× 10
phiv := phiv = 0.99817
P
Residual entropy for liquid (DELSL) and vapor (DELSV) phases

⎛ ⎛ Z( T , P) 0 + CB( T , P) ⎞ ⎞
(
DELSL:= ⎜ R⋅ ln Z( T , P) − CB( T , P) + ) ⋅ ln⎜
Da( T) 5
⋅ 10
⎜ 0 b ⎜ Z( T , P)
⎝ ⎝ 0 ⎠⎠

⎛ ⎛ Z( T , P) 2 + CB( T , P) ⎞ ⎞
(
DELSV:= ⎜ R⋅ ln Z( T , P) − CB( T , P) + ) ⋅ ln⎜
Da( T) 5
⋅ 10
⎜ 2 b ⎜ Z( T , P)
⎝ ⎝ 2 ⎠⎠

Residual enthalpy for liquid (DELHL) and vapor (DELHV) phases

⎡ T⋅ Da( T) − a( T) ⎛ Z( T , P) 0 + CB( T , P) ⎞⎤⎥


⎢ (
DELHL:= ⎢R⋅ T⋅ Z( T , P) − 1 +
0 ) b
⋅ ln⎜
⎜ Z( T , P) ⎥
⋅ 10
5

⎣ ⎝ 0 ⎠⎦

⎡ T⋅ Da( T) − a( T) ⎛ Z( T , P) 2 + CB( T , P) ⎞⎤⎥


⎢ (
DELHV:= ⎢R⋅ T⋅ Z( T , P) − 1 +
2 ) b
⋅ ln⎜
⎜ Z( T , P) ⎥
⋅ 10
5

⎣ ⎝ 2 ⎠⎦

Ideal gas properties changes relative to the reference state

(
Cp ⋅ T − Trs
1
2 )
2
Cp ⋅ T − Trs
2
( 3 3 ) Cp ⋅ T − Trs
3
( 4 4 )
DELHIG:= Cp ⋅ ( T − Trs ) + + +
0 2 3 4
(
Cp ⋅ T − Trs
2 2 ) Cp ⋅ T − Trs( 3 3 )
DELSIG:= Cp ⋅ ln⎛⎜
T ⎞
− R⋅ 10 ⋅ ln⎛⎜
2 3 5 P ⎞
+ Cp ⋅ ( T − Trs ) + +
0 ⎝ Trs ⎠ 1 2 3 ⎝ Prs ⎠

Total entropy and enthalpy relative to ideal gas reference state


SL := DELSIG+ DELSL SV := DELSIG+ DELSV HL := DELHIG+ DELHL HV := DELHIG+ DELHV

SUMMARY OF RESULTS

T = 323.15
K Vapor pressure, bar P = 0.15
LIQUID VAPOR
−4
Compressibility Z( T , P) = 1.35706× 10 Z( T , P) = 0.99816
0 2
4 3
Enthalpy, J/mol HL = −4.74037× 10 HV = −1.71382× 10

Entropy, J/mol K SL = −127.05678 SV = 10.96203

Fugacity coefficient phil = 0.6655 phiv = 0.99817

Fugacity, bar fugl = 0.09983 fugv = 0.14972


th
Solutions to Chemical and Engineering Thermodynamics, 4 ed. Chapter 7

7.54 ISENTHALPIC PENG-ROBINSON EQUATION OF STATE CALCULATION

Tc := 154.6 Pc := 50.46 om := 0.021 kap := 0.37464 + 1.54226om


⋅ − 0.26992om
⋅ ⋅ om
−2 −5 −9
Cp := 25.46 Cp := 1.591⋅ 10 Cp := −0.7151⋅ 10 Cp := 1.311⋅ 10
0 1 2 3
2 2
R ⋅ Tc R ⋅ Tc
Peng-Robinson Constants: R := 0.00008314 b := 0.07780⋅ ac := 0.45724⋅
Pc Pc

Input initial temperature and pressure of calculation Ti := 120. K, Pi := 30 bar


Input final pressure Pf := 3.0 bar
Initial state calculations T := Ti P := Pi
2
⎡ ⎛ T ⎞⎤ a ( T) ⋅ P P⋅ b
alf( T) := 1⋅ ⎢ 1 + kap⋅ ⎜ 1 − ⎥ a( T) := ac ⋅ alf( T) CA( T , P) := CB( T , P) :=
⎣ ⎝ Tc ⎠ ⎦
( R ⋅ T)
2 R⋅ T
d
Da( T) := a( T)
dT
Z( T , P) := A ← CA( T , P)
B ← CB( T , P)

(
⎡⎢ − A ⋅ B − B2 − B3 ) ⎥⎤ Vector of coefficients in the PR equation
⎢ 2 ⎥ in the form
V ← ⎢ A − 3⋅ B − 2⋅ B ⎥ 0=-(A*B-B^2-B^3)+(A-3*B^2-2*B)*Z-(1-B)*Z^2+Z^3
⎢ −( 1 − B) ⎥
⎢ ⎥
⎣ 1 ⎦
ZZ ← polyroots ( V) Solution to the cubic

for i ∈ 0 .. 2

(ZZi ← 0) if (Im(ZZi) ≠ 0) Set any imaginary roots to zero


Sort the roots
ZZ ← sort ( ZZ)

ZZ ← ZZ ⎛ ZZ < 10− 5 ⎞
if Set the value of any imaginary roots
0 ⎝ 0 2 ⎠ to value of the real root

ZZ ← ZZ if ⎛ ZZ < 10 ⎞
− 5
2 0 ⎝ 2 ⎠
ZZ
Calculate inital properties Zf ( T , P) := Z( T , P)

⎛ 0.0888⎞
Calculate initial molar volume Z( T , P) ⋅ R ⋅ T
0 3 Z( T , P) = ⎜ 0
and enthalpy and entropy VL := ⋅ 10 ⎜
departure
P ⎝ 0.0888⎠

⎡ T⋅ Da( T) − a( T) ⎡ Z( T , P) 0 + ( 1 + 2) ⋅ CB( T , P) ⎤ ⎤ 5
⎢ (
DELHin:= ⎢ R ⋅ T⋅ Z( T , P) − 1 +
0 ) 2⋅ 2⋅ b
⋅ ln⎢ ⎥ ⎥ ⋅ 10
⎢ Z( T , P) + ( 1 − 2) ⋅ CB( T , P) ⎥ ⎥
⎣ ⎣ 0 ⎦⎦

⎡ ⎡ Z( T , P) 0 + ( 1 + 2) ⋅ CB( T , P) ⎤ ⎤ 5
(
DELSin := ⎢ R⋅ ln Z( T , P) − CB( T , P) + ) ⋅ ln⎢ ⎥ ⎥ ⋅ 10
Da( T)
⎢ 0 2⋅ 2⋅ b ⎢ Z( T , P) + ( 1 − 2) ⋅ CB( T , P) ⎥ ⎥
⎣ ⎣ 0 ⎦⎦
th
Solutions to Chemical and Engineering Thermodynamics, 4 ed. Chapter 7

3
DELHin = −5.9875× 10 DELSin = −40.1647

Guess for final state T := 0.8⋅ Ti P := Pf

Fugacity expressions [actually ln(f/P)] for the liquid fl and vapor fv

CA( T , P) ⎡ Z( T , P) 0 + ( 1 + 2) ⋅ CB( T , P) ⎤
( ) (
fl( T , P) := Z( T , P) − 1 − ln Z( T , P) − CB( T , P) −
0 0 ) 2⋅ 2⋅ CB( T , P)
⋅ ln⎢
⎢ Z( T , P) + ( 1 − 2) ⋅ CB( T , P) ⎥

⎣ 0 ⎦

CA( T , P) ⎡ Z( T , P) 2 + ( 1 + 2) ⋅ CB( T , P) ⎤
( ) (
fv( T , P) := Z( T , P) − 1 − ln Z( T , P) − CB( T , P) −
2 2 ) 2⋅ 2⋅ CB( T , P)
⋅ ln⎢
⎢ Z( T , P) + ( 1 − 2) ⋅ CB( T , P) ⎥

⎣ 2 ⎦

Given fl( T , P) − fv( T , P) 0 T := Find( T) T = 101.906

Residual entropy for liquid (DELSL) and vapor (DELSV) phases

⎡ ⎡ Z( T , P) 0 + ( 1 + 2) ⋅ CB( T , P) ⎤ ⎤ 5
(
DELSL( T , P) := ⎢ R⋅ ln Z( T , P) − CB( T , P) + ) ⋅ ln⎢ ⎥ ⎥ ⋅ 10
Da( T)
⎢ 0 2⋅ 2⋅ b ⎢ Z( T , P) + ( 1 − 2) ⋅ CB( T , P) ⎥ ⎥
⎣ ⎣ 0 ⎦⎦

⎡ ⎡ Z( T , P) 2 + ( 1 + 2) ⋅ CB( T , P) ⎤ ⎤ 5
(
( , P) := ⎢ R⋅ ln Z( T , P) − CB( T , P) + ) ⋅ ln⎢ ⎥ ⎥ ⋅ 10
Da( T)
DELSVT
⎢ 2 2⋅ 2⋅ b ⎢ Z( T , P) + ( 1 − 2) ⋅ CB( T , P) ⎥ ⎥
⎣ ⎣ 2 ⎦⎦

Residual enthalpy for liquid (DELHL) and vapor (DELHV) phases

⎡ T⋅ Da( T) − a( T) ⎡ Z( T , P) 0 + ( 1 + 2) ⋅ CB( T , P) ⎤ ⎤ 5
⎢ (
DELHL( T , P) := ⎢ R⋅ T⋅ Z( T , P) − 1 +
0 ) 2⋅ 2⋅ b
⋅ ln⎢ ⎥ ⎥ ⋅ 10
⎢ Z( T , P) + ( 1 − 2) ⋅ CB( T , P) ⎥ ⎥
⎣ ⎣ 0 ⎦⎦

⎡ T⋅ Da( T) − a( T) ⎡ Z( T , P) 2 + ( 1 + 2) ⋅ CB( T , P) ⎤ ⎤ 5
( , P) := ⎢ R⋅ T⋅ Z( T , P) − 1 +
DELHVT
⎢ ( 2 ) 2⋅ 2⋅ b
⋅ ln⎢ ⎥ ⎥ ⋅ 10
⎢ Z( T , P) + ( 1 − 2) ⋅ CB( T , P) ⎥ ⎥
⎣ ⎣ 2 ⎦⎦
Ideal gas properties changes relative to the initial state

(
Cp ⋅ T − Ti
1
2 2 ) (
Cp ⋅ T − Ti
2
3 3 ) Cp ⋅ T − Ti
3
( 4 4 )
( , P) := Cp ⋅ ( T − Ti) +
DELHIGT + +
0 2 3 4

Cp ⋅ T − Ti ( 2 2 ) Cp ⋅ T − Ti ( 3 3 )
( , P) := Cp ⋅ ln⎛⎜ ⎞ + Cp ⋅ ( T − Ti) + − R⋅ 10 ⋅ ln⎛⎜
T 2 3 5 P⎞
DELSIGT +
0
⎝ Ti ⎠ 1 2 3 ⎝ Pi ⎠
Find vapor-liquid split x := 0.5

Given

x⋅ DELHVT
( , P) + ( 1 − x) ⋅ DELHL( T , P) + DELHIGT
( , P) DELHin

x := Find( x) x = 0.1618 Fraction vapor


th
Solutions to Chemical and Engineering Thermodynamics, 4 ed. Chapter 7

HV := DELHVT
( , P) + DELHIGT
( , P) SV := DELSVT
( , P) + DELSIGT
( , P)

HL := DELHL( T , P) + DELHIGT
( , P) SL := DELSL( T , P) + DELSIGT
( , P)

δH := x⋅ HV + ( 1 − x) ⋅ HL − DELHin δS := x⋅ SV + ( 1 − x) ⋅ SL − DELSin

SUMMARY OF RESULTS

FEED LIQUID VAPOR


Temperature, K Ti = 120 T = 101.906 T = 101.906
Pressure, bar Pi = 30 P=3 P=3

Vapor-liquid split x = 0.1618

−3
Compressibility Z( Ti , Pi) = 0.0888 Z( T , P) = 9.3464× 10 Z( T , P) = 0.9309
0 0 2
Enthalpy, J/mol 3
(relative to feed) 0 HL = −7.0203× 10 HV = −635.2466

Entropy, J/mol K 0 SL = −48.8038 SV = 13.8527


(relative to feed)

Enthalpy change
J/mol δH = 0

Entropy change
J/mol K δS = 1.4957
th
Solutions to Chemical and Engineering Thermodynamics, 4 ed. Chapter 7

7.55
ISENTROPIC PENG-ROBINSON EQUATION OF STATE CALCULATION

Tc := 154.6 Pc := 50.46 om := 0.021 kap := 0.37464 + 1.54226om


⋅ − 0.26992om
⋅ ⋅ om
−2 −5 −9
Cp := 25.46 Cp := 1.591⋅ 10 Cp := −0.7151⋅ 10 Cp := 1.311⋅ 10
0 1 2 3
2 2
R⋅ Tc R ⋅ Tc
Peng-Robinson Constants: R := 0.00008314 b := 0.07780⋅ ac := 0.45724⋅
Pc Pc

Input initial temperature and pressure of calculation Ti := 120. K, Pi := 30 bar


Input final pressure Pf := 3.0 bar
Initial state calculations T := Ti P := Pi
2
⎡ ⎛ T ⎞⎤ a ( T) ⋅ P P⋅ b
alf( T) := 1⋅ ⎢ 1 + kap⋅ ⎜ 1 − ⎥ a( T) := ac ⋅ alf( T) CA( T , P) := CB( T , P) :=
⎣ ⎝ Tc ⎠ ⎦
( R ⋅ T)
2 R⋅ T
d
Da( T) := a( T)
dT
Z( T , P) := A ← CA( T , P)
B ← CB( T , P)

(
⎡⎢ − A ⋅ B − B2 − B3 ) ⎥⎤
Vector of coefficients in the PR equation
⎢ 2 ⎥ in the form
V ← ⎢ A − 3⋅ B − 2⋅ B ⎥ 0=-(A*B-B^2-B^3)+(A-3*B^2-2*B)*Z-(1-B)*Z^2+Z^3
⎢ −( 1 − B) ⎥
⎢ ⎥
⎣ 1 ⎦
ZZ ← polyroots ( V) Solution to the cubic

for i ∈ 0 .. 2

(ZZi ← 0) if (Im(ZZi) ≠ 0) Set any imaginary roots to zero


Sort the roots
ZZ ← sort ( ZZ)

ZZ ← ZZ ⎛ ZZ < 10− 5 ⎞
if Set the value of any imaginary roots
0 ⎝ 0 2 ⎠ to value of the real root
ZZ ← ZZ if ⎛ ZZ < 10
− 5⎞
2 0 ⎝ 2 ⎠
ZZ
Calculate inital properties Zf ( T , P) := Z( T , P)

⎛ 0.0888⎞
Z( T , P) = ⎜
Calculate initial molar volume Z( T , P) ⋅ R⋅ T
0 3 0
and enthalpy and entropy VL := ⋅ 10 ⎜
departure
P ⎝ 0.0888⎠

⎡ T⋅ Da( T) − a( T) ⎡ Z( T , P) 0 + ( 1 + 2) ⋅ CB( T , P) ⎤ ⎤ 5
⎢ (
DELHin:= ⎢ R⋅ T⋅ Z( T , P) − 1 +
0 ) 2⋅ 2⋅ b
⋅ ln⎢ ⎥ ⎥ ⋅ 10
⎢ Z( T , P) + ( 1 − 2) ⋅ CB( T , P) ⎥ ⎥
⎣ ⎣ 0 ⎦⎦

⎡ ⎡ Z( T , P) 0 + ( 1 + 2) ⋅ CB( T , P) ⎤ ⎤ 5
(
DELSin:= ⎢ R⋅ ln Z( T , P) − CB( T , P) + ) ⋅ ln⎢ ⎥ ⎥ ⋅ 10
Da( T)
⎢ 0 2⋅ 2⋅ b ⎢ Z( T , P) + ( 1 − 2) ⋅ CB( T , P) ⎥ ⎥
⎣ ⎣ 0 ⎦⎦
3
DELHin = −5.9875× 10 DELSin = −40.1647
th
Solutions to Chemical and Engineering Thermodynamics, 4 ed. Chapter 7

Guess for final state T := 0.8⋅ Ti P := Pf

Fugacity expressions [actually ln(f/P)] for the liquid fl and vapor fv

CA( T , P) ⎡ Z( T , P) 0 + ( 1 + 2) ⋅ CB( T , P) ⎤
( ) (
fl( T , P) := Z( T , P) − 1 − ln Z( T , P) − CB( T , P) −
0 0 ) 2⋅ 2⋅ CB( T , P)
⋅ ln⎢
⎢ Z( T , P) + ( 1 − 2) ⋅ CB( T , P) ⎥

⎣ 0 ⎦

CA( T , P) ⎡ Z( T , P) 2 + ( 1 + 2) ⋅ CB( T , P) ⎤
( ) (
fv( T , P) := Z( T , P) − 1 − ln Z( T , P) − CB( T , P) −
2 2 ) 2⋅ 2⋅ CB( T , P)
⋅ ln⎢
⎢ Z( T , P) + ( 1 − 2) ⋅ CB( T , P) ⎥

⎣ 2 ⎦

Given fl( T , P) − fv( T , P) 0 T := Find( T) T = 101.906

Residual entropy for liquid (DELSL) and vapor (DELSV) phases

⎡ ⎡ Z( T , P) 0 + ( 1 + 2) ⋅ CB( T , P) ⎤ ⎤ 5
(
DELSL( T , P) := ⎢ R⋅ ln Z( T , P) − CB( T , P) + ) ⋅ ln⎢ ⎥ ⎥ ⋅ 10
Da( T)
⎢ 0 2⋅ 2⋅ b ⎢ Z( T , P) + ( 1 − 2) ⋅ CB( T , P) ⎥ ⎥
⎣ ⎣ 0 ⎦⎦

⎡ ⎡ Z( T , P) 2 + ( 1 + 2) ⋅ CB( T , P) ⎤ ⎤ 5
(
( , P) := ⎢ R⋅ ln Z( T , P) − CB( T , P) + ) ⋅ ln⎢ ⎥ ⎥ ⋅ 10
Da( T)
DELSVT
⎢ 2 2⋅ 2⋅ b ⎢ Z( T , P) + ( 1 − 2) ⋅ CB( T , P) ⎥ ⎥
⎣ ⎣ 2 ⎦⎦

Residual enthalpy for liquid (DELHL) and vapor (DELHV) phases

⎡ T⋅ Da( T) − a( T) ⎡ Z( T , P) 0 + ( 1 + 2) ⋅ CB( T , P) ⎤ ⎤ 5
⎢ (
DELHL( T , P) := ⎢ R⋅ T⋅ Z( T , P) − 1 +
0 ) 2⋅ 2⋅ b
⋅ ln⎢ ⎥ ⎥ ⋅ 10
⎢ Z( T , P) + ( 1 − 2) ⋅ CB( T , P) ⎥ ⎥
⎣ ⎣ 0 ⎦⎦

⎡ T⋅ Da( T) − a( T) ⎡ Z( T , P) 2 + (1 + 2) ⋅ CB( T , P) ⎤ ⎤ 5
( , P) := ⎢ R⋅ T⋅ Z( T , P) − 1 +
DELHVT
⎢ ( 2 ) 2⋅ 2⋅ b
⋅ ln⎢ ⎥ ⎥ ⋅ 10
⎢ Z( T , P) + (1 − 2) ⋅ CB( T , P) ⎥ ⎥
⎣ ⎣ 2 ⎦⎦
Ideal gas properties changes relative to the initial state

(
Cp ⋅ T − Ti
1
2 2) Cp ⋅ T − Ti
2
( 3 3 ) Cp ⋅ T − Ti
3
( 4 4 )
( , P) := Cp ⋅ ( T − Ti) +
DELHIGT + +
0 2 3 4

(
Cp ⋅ T − Ti
2 )
2
Cp ⋅ T − Ti ( 3 )
3
( , P) := Cp ⋅ ln⎛⎜ ⎞ + Cp ⋅ ( T − Ti) + − R⋅ 10 ⋅ ln⎛⎜
T 2 3 5 P⎞
DELSIGT +
0 ⎝ Ti ⎠ 1 2 3 ⎝ Pi ⎠
Find vapor-liquid split x := 0.5

Given

x⋅ DELSVT
( , P) + ( 1 − x) ⋅ DELSL( T , P) + DELSIGT
( , P) DELSin

x := Find( x) x = 0.1379 Fraction vapor


HV := DELHVT
( , P) + DELHIGT
( , P) SV := DELSVT
( , P) + DELSIGT
( , P)

HL := DELHL( T , P) + DELHIGT
( , P) SL := DELSL( T , P) + DELSIGT
( , P)

δH := x⋅ HV + ( 1 − x) ⋅ HL − DELHin δS := x⋅ SV + ( 1 − x) ⋅ SL − DELSin
th
Solutions to Chemical and Engineering Thermodynamics, 4 ed. Chapter 7

SUMMARY OF RESULTS

FEED LIQUID VAPOR


Temperature, K Ti = 120 T = 101.906 T = 101.906
Pressure, bar Pi = 30 P=3 P=3

Vapor-liquid split x = 0.1379

−3
Compressibility Z( Ti , Pi) = 0.0888 Z( T , P) = 9.3464× 10 Z( T , P) = 0.9309
0 0 2
Enthalpy, J/mol 3
(relative to feed) 0 HL = −7.0203× 10 HV = −635.2466

Entropy, J/mol K 0 SL = −48.8038 SV = 13.8527


(relative to feed)

Enthalpy change
J/mol δH = −152.4165

Entropy change
J/mol K δS = 0
th
Solutions to Chemical and Engineering Thermodynamics, 4 ed. Chapter 7

7.56
7.56 ISENTHALPIC S-R-K EQUATION OF STATE CALCULATION

Tc := 154.6 Pc := 50.46 om := 0.021 kap := 0.480 + 1.574⋅ om − 0.176⋅ om⋅ om


−2 −5 −9
Cp := 25.46 Cp := 1.591⋅ 10 Cp := −0.7151⋅ 10 Cp := 1.311⋅ 10
0 1 2 3
2 2
R⋅ Tc R ⋅ Tc
S-R-K Constants: R := 0.00008314 b := 0.08664⋅ ac := 0.42748⋅
Pc Pc

Input initial temperature and pressure of calculation Ti := 120. K, Pi := 30 bar


Input final pressure Pf := 3.0 bar
Initial state calculations T := Ti P := Pi
2
⎡ ⎛ T ⎞⎤ a ( T) ⋅ P P⋅ b
alf( T) := 1⋅ ⎢ 1 + kap⋅ ⎜ 1 − ⎥ a( T) := ac ⋅ alf( T) CA( T , P) := CB( T , P) :=
⎣ ⎝ Tc ⎠ ⎦
( R ⋅ T)
2 R⋅ T
d
Da( T) := a( T)
dT
Z( T , P) := A ← CA( T , P)
B ← CB( T , P)
⎛ −A ⋅ B ⎞
⎜ 2 Vector of coefficients in the S-R-K equation
V← ⎜
A − B − B⎟ in the form
⎜ −1 ⎟ 0=-A*B+(A-B^2-B)*Z-Z^2+Z^3

⎝ 1 ⎠
ZZ ← polyroots ( V) Solution to the cub ic
for i ∈ 0 .. 2

(ZZi ← 0) if (Im(ZZi) ≠ 0) Set any imaginary roots to zero


ZZ ← sort ( ZZ) Sort the roots

ZZ ← ZZ ⎛ ZZ < 10− 5 ⎞
if
0 ⎝ 0 2 ⎠ Set the value of any imaginary roots
to value of the real root
ZZ ← ZZ if ⎛ ZZ < 10
− 5⎞
2 0 ⎝ 2 ⎠
ZZ
Calculate inital properties Zf ( T , P) := Z( T , P)

⎛ 0.1004⎞
Z( T , P) = ⎜
Calculate initial molar volume Z( T , P) ⋅ R⋅ T
0 3 0
and enthalpy and entropy VL := ⋅ 10 ⎜
departure
P ⎝ 0.1004⎠

⎡ T⋅ Da( T) − a( T) ⎛ Z( T , P) 0 + CB( T , P) ⎞⎥⎤ 5


⎢ (
DELHin:= ⎢R⋅ T⋅ Z( T , P) − 1 +
0 ) b
⋅ ln⎜
⎜ Z( T , P) ⎥
⋅ 10
⎣ ⎝ 0 ⎠⎦

⎛ ⎛ Z( T , P) 0 + CB( T , P) ⎞ ⎞
(
DELSin:= ⎜ R⋅ ln Z( T , P) − CB( T , P) + ) ⋅ ln⎜
Da( T) 5
⋅ 10
⎜ 0 b ⎜ Z( T , P)
⎝ ⎝ 0 ⎠⎠
3
DELHin = −6.0618× 10 DELSin = −40.9502
th
Solutions to Chemical and Engineering Thermodynamics, 4 ed. Chapter 7

Guess for final state T := 0.8⋅ Ti P := Pf

Fugacity expressions [actually ln(f/P)] for the liquid fl and vapor fv

CA( T , P) ⎛ Z( T , P) 0 + CB( T , P) ⎞
( ) (
fl( T , P) := Z( T , P) − 1 − ln Z( T , P) − CB( T , P) −
0 0 ) CB( T , P)
⋅ ln⎜
⎜ Z( T , P)
⎝ 0 ⎠

CA( T , P) ⎛ Z( T , P) 2 + CB( T , P) ⎞
( ) (
fv( T , P) := Z( T , P) − 1 − ln Z( T , P) − CB( T , P) −
2 2 ) CB( T , P)
⋅ ln⎜
⎜ Z( T , P)
⎝ 2 ⎠

Given fl( T , P) − fv( T , P) 0 T := Find( T) T = 102.0671

Residual entropy for liquid (DELSL) and vapor (DELSV) phases

⎛ ⎛ Z( T , P) 0 + CB( T , P) ⎞ ⎞
(
DELSL( T , P) := ⎜ R⋅ ln Z( T , P) − CB( T , P) + ) ⋅ ln⎜
Da( T) 5
⋅ 10
⎜ 0 b ⎜ Z( T , P)
⎝ ⎝ 0 ⎠⎠

⎛ ⎛ Z( T , P) 2 + CB( T , P) ⎞ ⎞ 5
(
( , P) := ⎜ R⋅ ln Z( T , P) − CB( T , P) + ) ⋅ ln⎜
Da( T)
DELSVT ⋅ 10
⎜ 2 b ⎜ Z( T , P)
⎝ ⎝ 2 ⎠⎠

Residual enthalpy for liquid (DELHL) and vapor (DELHV) phases

⎡ T⋅ Da( T) − a( T) ⎛ Z( T , P) 0 + CB( T , P) ⎞⎥⎤


⎢ (
DELHL( T , P) := ⎢R⋅ T⋅ Z( T , P) − 1 +
0 ) b
⋅ ln⎜
⎜ Z( T , P) ⎥
⋅ 10
5

⎣ ⎝ 0 ⎠⎦

⎡ T⋅ Da( T) − a( T) ⎛ Z( T , P) 2 + CB( T , P) ⎞⎤ 5
DELHVT
⎢ (
( , P) := ⎢R⋅ T⋅ Z( T , P) − 1 +
2 ) b
⋅ ln⎜
⎜ Z( T , P)
⎥ ⋅ 10

⎣ ⎝ 2 ⎠⎦
Ideal gas properties changes relative to the initial state

Cp ⋅ T − Ti
1
( 2 2 ) (
Cp ⋅ T − Ti
2
3 3 ) Cp ⋅ T − Ti
3
( 4 4 )
( , P) := Cp ⋅ ( T − Ti) +
DELHIGT + +
0 2 3 4

Cp ⋅ T − Ti( 2 2 ) Cp ⋅ T − Ti ( 3 3 )
( , P) := Cp ⋅ ln⎛⎜ ⎞ + Cp ⋅ ( T − Ti) + − R⋅ 10 ⋅ ln⎛⎜ ⎞
T 2 3 5 P
DELSIGT +
0
⎝ ⎠
Ti 1 2 3 ⎝ Pi ⎠
Find vapor-liquid split x := 0.5

Given

x⋅ DELHVT
( , P) + ( 1 − x) ⋅ DELHL( T , P) + DELHIGT
( , P) DELHin

x := Find( x) x = 0.1661 Fraction vapor


th
Solutions to Chemical and Engineering Thermodynamics, 4 ed. Chapter 7

HV := DELHVT
( , P) + DELHIGT
( , P) SV := DELSVT
( , P) + DELSIGT
( , P)

HL := DELHL( T , P) + DELHIGT
( , P) SL := DELSL( T , P) + DELSIGT
( , P)

δH := x⋅ HV + ( 1 − x) ⋅ HL − DELHin δS := x⋅ SV + ( 1 − x) ⋅ SL − DELSin

SUMMARY OF RESULTS

FEED LIQUID VAPOR


Temperature, K Ti = 120 T = 102.0671 T = 102.0671
Pressure, bar Pi = 30 P=3 P=3

Vapor-liquid split x = 0.1661

Compressibility Z( Ti , Pi) = 0.1004 Z( T , P) = 0.0106 Z( T , P) = 0.934


0 0 2
Enthalpy, J/mol 3
(relative to feed) 0 HL = −7.1435× 10 HV = −630.1699

Entropy, J/mol K 0 SL = −49.936 SV = 13.8781


(relative to feed)

Enthalpy change
J/mol δH = 0

Entropy change
J/mol K δS = 1.6121
th
Solutions to Chemical and Engineering Thermodynamics, 4 ed. Chapter 7

7.57 ISENTROPIC S-R-K EQUATION OF STATE CALCULATION

Tc := 154.6 Pc := 50.46 om := 0.021 kap := 0.480 + 1.574⋅ om − 0.176⋅ om⋅ om


−2 −5 −9
Cp := 25.46 Cp := 1.591⋅ 10 Cp := −0.7151⋅ 10 Cp := 1.311⋅ 10
0 1 2 3
2 2
R⋅ Tc R ⋅ Tc
S-R-K Constants: R := 0.00008314 b := 0.08664⋅ ac := 0.42748⋅
Pc Pc

Input initial temperature and pressure of calculation Ti := 120. K, Pi := 30 bar


Input final pressure Pf := 3.0 bar
Initial state calculations T := Ti P := Pi
2
⎡ ⎛ T ⎞⎤ a ( T) ⋅ P P⋅ b
alf( T) := 1⋅ ⎢ 1 + kap⋅ ⎜ 1 − ⎥ a( T) := ac ⋅ alf( T) CA( T , P) := CB( T , P) :=
⎣ ⎝ Tc ⎠ ⎦
( R ⋅ T)
2 R⋅ T
d
Da( T) := a( T)
dT
Z( T , P) := A ← CA( T , P)
B ← CB( T , P)
⎛ −A ⋅ B ⎞
⎜ 2 Vector of coefficients in the S-R-K equation
V← ⎜
A − B − B⎟ in the form
⎜ −1 ⎟ 0=-A*B+(A-B^2-B)*Z-Z^2+Z^3

⎝ 1 ⎠
ZZ ← polyroots ( V) Solution to the cub ic
for i ∈ 0 .. 2

(ZZi ← 0) if (Im(ZZi) ≠ 0) Set any imaginary roots to zero


ZZ ← sort ( ZZ) Sort the roots

ZZ ← ZZ ⎛ ZZ < 10− 5 ⎞
if
0 ⎝ 0 2 ⎠ Set the value of any imaginary roots
to value of the real root
ZZ ← ZZ if ⎛ ZZ < 10
− 5⎞
2 0 ⎝ 2 ⎠
ZZ
Calculate inital properties Zf ( T , P) := Z( T , P)

⎛ 0.1004⎞
Z( T , P) = ⎜
Calculate initial molar volume Z( T , P) ⋅ R⋅ T
0 3 0
and enthalpy and entropy VL := ⋅ 10 ⎜
departure
P ⎝ 0.1004⎠

⎡ T⋅ Da( T) − a( T) ⎛ Z( T , P) 0 + CB( T , P) ⎞⎤ 5
⎢ (
DELHin:= ⎢R⋅ T⋅ Z( T , P) − 1 +
0 ) b
⋅ ln⎜
⎜ Z( T , P)
⎥ ⋅ 10

⎣ ⎝ 0 ⎠⎦

⎛ ⎛ Z( T , P) 0 + CB( T , P) ⎞ ⎞ 5
(
DELSin:= ⎜ R⋅ ln Z( T , P) − CB( T , P) + ) ⋅ ln⎜
Da( T)
⋅ 10
⎜ 0 b ⎜ Z( T , P)
⎝ ⎝ 0 ⎠⎠
3
DELHin = −6.0618× 10 DELSin = −40.9502

Guess for final state T := 0.8⋅ Ti P := Pf


th
Solutions to Chemical and Engineering Thermodynamics, 4 ed. Chapter 7

Fugacity expressions [actually ln(f/P)] for the liquid fl and vapor fv

CA( T , P) ⎛ Z( T , P) 0 + CB( T , P) ⎞
( ) (
fl( T , P) := Z( T , P) − 1 − ln Z( T , P) − CB( T , P) −
0 0 ) CB( T , P)
⋅ ln⎜
⎜ Z( T , P)
⎝ 0 ⎠

CA( T , P) ⎛ Z( T , P) 2 + CB( T , P) ⎞
( ) (
fv( T , P) := Z( T , P) − 1 − ln Z( T , P) − CB( T , P) −
2 2 ) CB( T , P)
⋅ ln⎜
⎜ Z( T , P)
⎝ 2 ⎠

Given fl( T , P) − fv( T , P) 0 T := Find( T) T = 102.0671

Residual entropy for liquid (DELSL) and vapor (DELSV) phases

⎛ ⎛ Z( T , P) 0 + CB( T , P) ⎞ ⎞
(
DELSL( T , P) := ⎜ R⋅ ln Z( T , P) − CB( T , P) + ) ⋅ ln⎜
Da( T) 5
⋅ 10
⎜ 0 b ⎜ Z( T , P)
⎝ ⎝ 0 ⎠⎠

⎛ ⎛ Z( T , P) 2 + CB( T , P) ⎞ ⎞
(
( , P) := ⎜ R⋅ ln Z( T , P) − CB( T , P) + ) ⋅ ln⎜
Da( T) 5
DELSVT ⋅ 10
⎜ 2 b ⎜ Z( T , P)
⎝ ⎝ 2 ⎠⎠

Residual enthalpy for liquid (DELHL) and vapor (DELHV) phases

⎡ T⋅ Da( T) − a( T) ⎛ Z( T , P) 0 + CB( T , P) ⎞⎤ 5
⎢ (
DELHL( T , P) := ⎢R⋅ T⋅ Z( T , P) − 1 +
0 ) b
⋅ ln⎜
⎜ Z( T , P)
⎥ ⋅ 10

⎣ ⎝ 0 ⎠⎦

⎡ T⋅ Da( T) − a( T) ⎛ Z( T , P) 2 + CB( T , P) ⎞⎤ 5
DELHVT
⎢ (
( , P) := ⎢R⋅ T⋅ Z( T , P) − 1 +
2 ) b
⋅ ln⎜
⎜ Z( T , P)
⎥ ⋅ 10

⎣ ⎝ 2 ⎠⎦
Ideal gas properties changes relative to the initial state

Cp ⋅ T − Ti
1
( 2 2 ) (
Cp ⋅ T − Ti
2
3 3 ) (
Cp ⋅ T − Ti
3
4 4 )
( , P) := Cp ⋅ ( T − Ti) +
DELHIGT + +
0 2 3 4

Cp ⋅ T − Ti( 2 2 ) Cp ⋅ T − Ti ( 3 3 )
( , P) := Cp ⋅ ln⎛⎜ ⎞ + Cp ⋅ ( T − Ti) + − R⋅ 10 ⋅ ln⎛⎜
T 2 3 5 P⎞
DELSIGT +
0
⎝ Ti ⎠ 1 2 3 ⎝ Pi ⎠
Find vapor-liquid split x := 0.5

Given

x⋅ DELSVT
( , P) + ( 1 − x) ⋅ DELSL( T , P) + DELSIGT
( , P) DELSin

x := Find( x) x = 0.1408 Fraction vapor


th
Solutions to Chemical and Engineering Thermodynamics, 4 ed. Chapter 7

HV := DELHVT
( , P) + DELHIGT
( , P) SV := DELSVT
( , P) + DELSIGT
( , P)

HL := DELHL( T , P) + DELHIGT
( , P) SL := DELSL( T , P) + DELSIGT
( , P)

δH := x⋅ HV + ( 1 − x) ⋅ HL − DELHin δS := x⋅ SV + ( 1 − x) ⋅ SL − DELSin

SUMMARY OF RESULTS

FEED LIQUID VAPOR


Temperature, K Ti = 120 T = 102.0671 T = 102.0671
Pressure, bar Pi = 30 P=3 P=3

Vapor-liquid split x = 0.1408

Compressibility Z( Ti , Pi) = 0.1004 Z( T , P) = 0.0106 Z( T , P) = 0.934


0 0 2
Enthalpy, J/mol 3
(relative to feed) 0 HL = −7.1435× 10 HV = −630.1699

Entropy, J/mol K 0 SL = −49.936 SV = 13.8781


(relative to feed)

Enthalpy change
J/mol δH = −164.5454

Entropy change − 15
J/mol K δS = 7.1054× 10
th
Solutions to Chemical and Engineering Thermodynamics, 4 ed. Chapter 7

7.58 This problem was solved using the attached Mathcad worksheet. The results are

T(oC) Pvap with α(T) Pvap with α =1 (P in kPa)

273.15 0.3137 166.57


283.15 0.5529 221.329
293.15 1.697 288.55
303.15 3.208 369.83
323.25 9.994 580.97
343.15 26.681 867.65
373.15 92.355 1467.0
393.15 186.67 1997.1
423.25 463.23 3016.5
448.15 886.08 4094.2
474.15 1599.4 5456.5
523.15 4065.2 8759.0
623.15 16744 18865
643.15 21060

As can be seen, the S-R-K equation is of comparable accuracy to the P-R


equation. In both cases if the α parameter is set to one, the results are not very
good, indeed quite bad at low temperatures.

The Mathcad worksheet used in solving this problem is given below.


th
Solutions to Chemical and Engineering Thermodynamics, 4 ed. Chapter 7

VAPOR PRESSURE CALCULATION USING SRK EQUATION

Read in properties for Water Tc := 647.3 Pc := 220.48 om := 0.344

kappa calculation kap := 0.480 + 1.574⋅ om − 0.176⋅ om⋅ om


2 2
R⋅ Tc R ⋅ Tc
S-R-K Constants: R := 0.00008314 b := 0.08664⋅ ac := 0.42748⋅
Pc Pc
Note that these are being defined as a
2
function of temperature since we will ⎡ ⎛ T⎞⎤
need to interate on temperature. alf( T) := 1⋅ ⎢ 1 + kap⋅ ⎜ 1 − ⎥ a( T) := ac ⋅ alf( T)
⎣ ⎝ Tc ⎠ ⎦
a ( T) ⋅ P P⋅ b
CA( T , P) := CB( T , P) :=
2 R⋅ T
( R ⋅ T)
Z( T , P) := A ← CA( T , P)
B ← CB( T , P)
⎛ −A ⋅ B ⎞
⎜ 2 Vector of coefficients in the SRK equation
V← ⎜
A − B − B⎟ in the form
⎜ −1 ⎟ 0=-A*B+(A-B^2-B)*Z-*Z^2+Z^3

⎝ 1 ⎠
ZZ ← polyroots ( V) Solution to the cubic
for i ∈ 0 .. 2

(ZZi ← 0) if (Im(ZZi) ≠ 0) Set any imaginary roots to zero


ZZ ← sort ( ZZ) Sort the roots

ZZ ← ZZ ⎛ ZZ < 10− 5 ⎞
if
0 ⎝2 0 ⎠ Set the value of any imaginary roots
to value of the real root
ZZ ← ZZ if ⎛ ZZ < 10
− 5⎞
2 0 ⎝ 2 ⎠
ZZ
Enter temperature T, and pressure P.
Depending on what is specified in the Given and Find statements
below, either T or P is specified and the other is an initial guess
which may have to be adjusted as the other variable is changed,
especially as the critical point is approached.This worksheet will
probably not converge to a solution when T or P are within 3 to 5% T := 643.15
of the critical values unless an extremely good initial guess is
provided. One way to obtain a good initial is to start well below the P := 210
critical region and step towards it using the result of previous calculations.
th
Solutions to Chemical and Engineering Thermodynamics, 4 ed. Chapter 7

Fugacity expressions [actually ln(f/P)] for the liquid fl and vapor fv

CA( T , P) ⎛ Z( T , P) 0 + CB( T , P) ⎞
( ) (
fl( T , P) := Z( T , P) − 1 − ln Z( T , P) − CB( T , P) −
0 0 ) CB( T , P)
⋅ ln⎜
⎜ Z( T , P)
⎝ 0 ⎠

CA( T , P) ⎛ Z( T , P) 2 + CB( T , P) ⎞
( ) (
fv( T , P) := Z( T , P) − 1 − ln Z( T , P) − CB( T , P) −
2 2 ) CB( T , P)
⋅ ln⎜
⎜ Z( T , P)
⎝ 2 ⎠

Given fl( T , P) − fv( T , P) 0 Equate log of fugacity coefficients


Solve equality of fugacities
P := Find( P) P = 210.59561

Fugacity Fugacity coefficient

fugl
fugl := P⋅ exp( fl( T , P) ) fugl = 141.78565 fl( T , P) = −0.39562 phil :=
P

fugv := P⋅ exp( fv( T , P) ) fugv = 141.78565 fv( T , P) = −0.39562

SUMMARY OF RESULTS

T = 643.15
K Vapor pressure, bar P = 210.59561
LIQUID VAPOR
Compressibility Z( T , P) = 0.25283 Z( T , P) = 0.42294
0 2
Fugacity, bar
fugl = 141.78565 fugv = 141.78565
th
Solutions to Chemical and Engineering Thermodynamics, 4 ed. Chapter 7

7.59 (also available as a Mathcad worksheet)


The solution is that the final temperature is 131.34 K, and the final pressure is
37.036 bar.
7.59 Using SRK EOS with the approximate two-constant heat capacity expression

Property Data Tc := 126.2 Pc := 33.94 om := 0.04 Cp1 := 27.2 Cp2 := 0.0042


(T in K, P in bar):
R := 0.00008314 kap := 0.480 + 1.574⋅ om − 0.176⋅ om⋅ om
Initial Conditions (Vt=total volume, m^3): Ti := 170 Pi := 100 Vt := 0.15
2 2
R⋅ Tc R ⋅ Tc
Peng-Robinson Constants: b := 0.08664⋅ ac := 0.42748⋅
Pc Pc
Initial temperature T := Ti
Note that these are being defined as a 2
⎡ ⎛ T ⎞⎤
function of temperature since we will need to alf( T) := 1⋅ ⎢ 1 + kap⋅ ⎜ 1 − ⎥
interate on temperature later to obtain the final ⎣ ⎝ Tc ⎠ ⎦ a( T) := ac ⋅ alf( T)
state of the system d
Da( T) := a ( T)
dT
Find initial molar volume and number of moles R⋅ Ti
Start with initial guess for volume, m^3/mol V :=
Pi

R⋅ T a ( T)
Solve P-R EOS for initial volume Given Pi − Vi := Find( V)
V− b [ V⋅ ( V + b ) ]

−4 Vt 3
Initial molar volume and Vi = 1.02 × 10 Ni := Ni = 1.471 × 10
number of moles Vi

DELSi:= ⎡⎢R⋅ ln⎡⎢( Vi − b ) ⋅ ⎤ + Da( T) ⋅ ln⎛ Vi + b ⎞⎤ ⋅ 105


Pi
Entropy departure at the ⎥ ⎜ ⎥
initial conditions ⎣ ⎣ R⋅ T⎦ b ⎝ Vi ⎠⎦

0.15
Now consider final state Nf := Ni − 10⋅ 50 Vf := V := Vf
Nf
Type out final number of −4
moles and specific volume Nf = 971.269 Vf = 1.544 × 10

Final pressure, will change in course R⋅ T a ( T)


of solving for the final temperature Pf ( T) := −
V− b V⋅ ( V + b )

V + b ⎞⎤
DELS( T) := ⎡⎢R⋅ ln⎡⎢( V − b ) ⋅
Pf ( T) ⎤
⋅ ln⎛⎜
Entropy departure Da( T) 5
at final conditions ⎥+ ⎥ ⋅ 10
⎣ ⎣ R⋅ T ⎦ b ⎝ V ⎠⎦

27.2⋅ ln⎛⎜ ⎞ + 0.0042⋅ ( T − Ti) − R⋅ 105⋅ ln⎛ Pf( T) ⎞ + DELS( T) − DELS


Solve for final Given T
temperature using 0 ⎜ Pi
⎝ Ti ⎠ ⎝ ⎠
S(final)-S(initial)=0
T := Find( T)
Type out solution T = 131.34 Pf ( T) = 37.076
th
Solutions to Chemical and Engineering Thermodynamics, 4 ed. Chapter 7

7.60 a) At a given temperature, the stability limit of a fluid is determined by the


following criterion (Note that this leads to the spinodal curve)
FG IJ
∂P
=0
H K
∂V T

For the given EOS, the stability limit of a fluid undergoing a pressure change at
constant temperature is
FG IJ
∂P RT BRT
= − 2 −2 3 −3 4 = 0
CRT
H K
∂V T V V V
or V 2 + 2 BV + 3C = 0

In order to have a phase transition, there must be two distinct stability limits,
i.e., the above quadratic equation must have two different roots of V. Therefore,
(2 B) 2 − 4 × 1 × (3C) > 0
or B 2 > 3C

b) According to Illustration 5.2-1


∂P LM F I OP
NH K
dU = CVdT + T − P dV
∂T V Q
LM F ∂P I OP
N H ∂T K
But for the given EOS T −P =0
V Q
Therefore,
z z
∆U = dU = CV (V , T )dT

FG ∂C IJ =T
FG ∂ P IJ
2
= 0 (Because B and C are not functions of T)
H ∂V K H ∂T K
V
Since 2
T V

Therefore Cv = Cv (T ) = CV* = a + bT
and

z
T2
b 2
∆U = (a + bT )dT = a(T2 − T1) + (T2 − T12 )
T1
2
The internal energy change is the same for an ideal gas.

c) According to Eqn 6.2-19


C F I
∂P
d S = V dT +
T H K
∂T V
dV so that

TF I
∂P FG RT + BRT + CRT IJ
H K
FG ∂T IJ = − ∂T = P = H V V V K V
2 3

H ∂V K S C a + bT V a + bT

For an ideal gas,


RT FG IJ
FG ∂T IJ =
P
=
V
=
H K
RT
H ∂V K S a + bT a + bT V (a + bT )
th
Solutions to Chemical and Engineering Thermodynamics, 4 ed. Chapter 7

7.62 a) Since
∆ F→ D V > 0 and ∆ F→ D H > 0, and that for an equibrium transition (i.e., G F = G D )
∆ F→ D H ⎛ ∂G ⎞
it follows that ∆ F→D S = > 0, so that SD > SF . Further, since ⎜ ⎟ =S
T ⎝ ∂T ⎠P
the slope of the Gibbs energy versus temperature must be greater for the
denatured of unfolded protein than for the folded protein. This is shown below:

G F

T*

If T < T* then GF > GD, so that the folded state or natural state is the stable
equilibrium state. If T > T* then GF < GD, so that the unfolded state is then the
stable equilibrium state. So T* is the “melting temperature” of the protein.

⎛ dP ⎞ ∆ F→D H
b) Since, by the Clapeyron equation ⎜ ⎟= > 0 , it follows that as the
⎝ dT ⎠ T∆ F→ D V
pressure on the protein is increases, T* will increase. That is, the protein will
unfolded or denature at a higher temperature as the pressure is increased.

7.63 The VISUAL Basic program Peng-Robinson Equation of State is used for the
Peng-Robinson predictions, the correlation in the Visual Basic Property program will be
used for the ‘literature data”, and the MATHCAD worksheet PRVAPP.MCD is used for
the a(T) = 1 calculation, since it is the easiest to modify. The results are as follows.

T( K) P (bar) literature, P(bar), P-R a(T) P(bar), P-R a(T)=1


115 1.297 1.343 3.452
120 1.884 1.943 4.467
130 3.636 3.723 7.085
140 6.377 6.496 10.59
150 10.39 10.54 15.11
160 15.94 16.14 20.76
170 23.36 23.61 27.65
180 33.01 33.27 35.89
190 45.42 45.48 45.59

and, of course at Tc = 190.4, both P-R calculations give P = 46.0 bar = Pc, since the E.O.S.
parameters have been fitted to give the correct critical point conditions (since α(TC ) = 1. )

Name : methane
Formula: CH4
Molecular weight (g/mol) = 16.043
Normal boiling point (K) = 111.6
th
Solutions to Chemical and Engineering Thermodynamics, 4 ed. Chapter 7

Critical temperature (K) = 190.4


Critical pressure (bar) = 46.0
Critical volume (cm3/mol) = 99.2
Critical compressibility factor = 0.288
Pitzer's acentric factor = 0.011
Isobaric heat capacity of the ideal gas (J/mol.K)
Cp = A + B*T + C*T^2 + D*T^3
A = 1.925e+01
B = 5.213e-02
C = 1.197e-05
D = -1.132e-08
------------------------------------------------------------------
** Vapor-Liquid Equilibrium **

Temperature (K) = 111.6


Vapor pressure (bar) = 1.0249

Vapor Liquid
Compressibility factor 0.96673 0.00373
Specific volume (m^3/mol) 8.75155e-03 3.37236e-
05
Density (g/cm^3) 1.83316e-03 4.75720e-
01
Fugacity (bar) 0.99183 0.99186
Fugacity coeff 0.96771 0.96774
Enthalpy departure (J/mol) -73.98 -8266.44
Enthalpy ideal (J/mol) -5661.75 -5661.75
Enthalpy (J/mol) -5735.73 -13928.18
Entropy departure (J/mol K) -0.39 -73.80
Entropy ideal (J/mol K) -29.21 -29.21
Entropy (J/mol K) -29.60 -103.01
** Vapor-Liquid Equilibrium **

Molar volume (m^3/mol) Density (g/cm^3)


T (K) P (bar) Vapor Liquid Vapor Liquid
115.0 1.343 6.82605e-03 3.41685e-05 2.35026e-03 4.69525e-01
120.0 1.943 4.85847e-03 3.48771e-05 3.30207e-03 4.59986e-01
125.0 2.726 3.54925e-03 3.56597e-05 4.52011e-03 4.49891e-01
130.0 3.723 2.65044e-03 3.65291e-05 6.05295e-03 4.39184e-01
135.0 4.969 2.01609e-03 3.75015e-05 7.95749e-03 4.27797e-01
140.0 6.496 1.55770e-03 3.85977e-05 1.02992e-02 4.15647e-01
145.0 8.342 1.21889e-03 3.98450e-05 1.31620e-02 4.02636e-01
150.0 10.539 9.63678e-04 4.12800e-05 1.66477e-02 3.88639e-01
155.0 13.128 7.67798e-04 4.29535e-05 2.08948e-02 3.73497e-01
160.0 16.143 6.15040e-04 4.49387e-05 2.60845e-02 3.56998e-01
165.0 19.625 4.93846e-04 4.73457e-05 3.24858e-02 3.38848e-01
170.0 23.612 3.96190e-04 5.03529e-05 4.04932e-02 3.18611e-01
175.0 28.146 3.15943e-04 5.42758e-05 5.07781e-02 2.95583e-01
180.0 33.270 2.48265e-04 5.97618e-05 6.46204e-02 2.68449e-01
185.0 39.030 1.88258e-04 6.85514e-05 8.52179e-02 2.34029e-01
190.0 45.478 1.18996e-04 9.18577e-05 1.34820e-01 1.74651e-01
th
Solutions to Chemical and Engineering Thermodynamics, 4 ed. Chapter 7

Volume-temperature coexistence curve of methane


Vapor Pressure of methane
190
50

180
45

170 40

160 35

150 30

25
140
20
130
15
120
10

110
5
0.00001 0.00010 0.00100 0.01000

V (m^3/mol) 0
110 120 130 140 150 160 170 180 190

Vapor Liquid Temperature (K)

Modified MATHCAD worksheet for Peng-Robinson vapor pressure calculation with a(T) = 1.
th
Solutions to Chemical and Engineering Thermodynamics, 4 ed. Chapter 7

Tc := 190.4 Pc := 46.0 om := 0.011 kap := 0.37464 + 1.54226om


⋅ − 0.26992om
⋅ ⋅ om
2 2
R ⋅ Tc R ⋅ Tc
Peng-Robinson Constants: R := 0.00008314 b := 0.07780⋅ ac := 0.45724⋅
Pc Pc
a( T) := ac a ( T) ⋅ P P⋅ b d
CA( T , P) := CB( T , P) := Da( T) := a( T)
2 R⋅ T dT
( R ⋅ T)
Z( T , P) := A ← CA( T , P)
B ← CB( T , P)

(
⎡⎢ − A ⋅ B − B2 − B3 ) ⎥⎤ Vector of coefficients in the PR equation
⎢ 2 ⎥ in the form
V ← ⎢ A − 3⋅ B − 2⋅ B ⎥ 0=-(A*B-B^2-B^3)+(A-3*B^2-2*B)*Z-(1-B)*Z^2+Z^3
⎢ −( 1 − B) ⎥
⎢ ⎥
⎣ 1 ⎦
ZZ ← polyroots ( V) Solution to the cubic

for i ∈ 0 .. 2
(ZZi ← 0) if (Im(ZZi) ≠ 0) Set any imaginary roots to zero
Sort the roots
ZZ ← sort ( ZZ)

ZZ ← ZZ ⎛ ZZ < 10− 5 ⎞
if Set the value of any imaginary roots
0 ⎝ 0 2 ⎠ to value of the real root
ZZ ← ZZ if ⎛ ZZ < 10
− 5⎞
2 0 ⎝ 2 ⎠
ZZ

T := 170 K P := 20 bar

Fugacity expressions [actually ln(f/P)] for the liquid fl and vapor fv

CA( T , P) ⎡ Z( T , P) 0 + ( 1 + 2) ⋅ CB( T , P) ⎤
( ) (
fl( T , P) := Z( T , P) − 1 − ln Z( T , P) − CB( T , P) −
0 0 ) 2⋅ 2⋅ CB( T , P)
⋅ ln⎢
⎢ Z( T , P) + ( 1 − 2) ⋅ CB( T , P) ⎥

⎣ 0 ⎦

CA( T , P) ⎡ Z( T , P) 2 + ( 1 + 2) ⋅ CB( T , P) ⎤
( ) (
fv( T , P) := Z( T , P) − 1 − ln Z( T , P) − CB( T , P) −
2 2 ) 2⋅ 2⋅ CB( T , P)
⋅ ln⎢
⎢ Z( T , P) + ( 1 − 2) ⋅ CB( T , P) ⎥

⎣ 2 ⎦

Given fl( T , P) − fv( T , P) 0 Equate log of fugacity coefficients


Solve equality of fugacities
P := Find( P) P = 27.65147

Fugacity Fugacity coefficient


fugl
fugl := P⋅ exp( fl( T , P) ) fugl = 19.90655 fl( T , P) = −0.32863 phil := phil = 0.71991
P
fugv
fugv := P⋅ exp( fv( T , P) ) fugv = 19.90655 fv( T , P) = −0.32863 phiv := phiv = 0.71991
P
SUMMARY OF RESULTS
T = 170 K Vapor pressure, bar P = 27.65147
LIQUID VAPOR
Compressibility Z( T , P) = 0.10692 Z( T , P) = 0.60688
0 2
Fugacity coefficient phil = 0.71991 phiv = 0.71991

Fugacity, bar fugl = 19.90655 fugv = 19.90655


th
Solutions to Chemical and Engineering Thermodynamics, 4 ed. Chapter 7

7.64 The VISUAL Basic program Peng-Robinson Equation of State is used for the
Peng-Robinson predictions, the correlation in the Visual Basic Property program will be
used for the ‘literature data”, and the MATHCAD worksheet PRVAPP.MCD is used for
the a(T) = 1 calculation, since it is the easiest to modify. The results are as follows.

T( K) P (bar) literature, P(bar), P-R a(T) P(bar), P-R a(T)=1


270 0.918 0.918 3.466
290 1.879 1.874 5.817
310 3.472 3.462 8.366
330 5.921 5.909 11.57
350 9.472 9.470 15.52
370 14.40 14.43 20.25
390 21.00 21.09 25.85
410 29.67 29.79 32.38

and, of course at Tc = 425.2, both P-R calculations give P = 38.0 bar = Pc, since the E.O.S.
parameters have been fitted to give the correct critical point conditions (since α(TC ) = 1. )

7.65 The VISUAL Basic program Peng-Robinson Equation of State is used for the
Peng-Robinson predictions, the correlation in the Visual Basic Property program will be
used for the ‘literature data”, and the MATHCAD worksheet PRVAPP.MCD is used for
the a(T) = 1 calculation, since it is the easiest to modify. The results are as follows.

T( K) P (bar) literature, P(bar), P-R a(T) P(bar), P-R a(T)=1


380 0.125 0.129 1.780
405 0.302 0.307 2.595
430 0.642 0.648 3.635
455 1.233 1.243 4.298
480 2.185 2.204 6.499
505 3.625 3.664 8.372
530 5.701 5.777 10.57
555 8.591 8.723 13.12
580 12.52 12.70 16.04
605 17.82 17.95 19.36

and, of course at Tc = 617.7, both P-R calculations give P = 21.2 bar = Pc, since the E.O.S.
parameters have been fitted to give the correct critical point conditions (since α(TC ) = 1. )
th
Solutions to Chemical and Engineering Thermodynamics, 4 ed. Chapter 7

7.66 Will use the PR EOS

a) find that f = 15.296 bar = 1.5296 MPa


b) to correct for increase in pressure
i) we can use PR EOS and find f = 21.463 bar
or ii) use Poynting Correction

⎡ V L (136.1 − 19.1) bar ⎤


f (13.61 MPa) = f (1.91 MPa) exp ⎢ ⎥
⎣ R ⋅T ⎦
but
⎡ 78.114 g 117 bar × 10−6 mcc
3

exp ⎢ mol
× ⎥ = exp [ 0.35382]
⎢⎣ 0.63 cc 8.314 × 10−5 bar
3

mol K × 493.15 K ⎥
g m

f(13.61 MPa) = 15.296 exp (0.35382) = 15.296 × 1.4245 = 21.789


So the two results are close, but not exactly the same. Perhaps because of
inaccuracy in liquid density, or that the liquid density changes with pressure.

7.66 Clausius-Coapeyron Eq. applied to sublimation

a)
Psub (T2 ) ∆ H⎛ 1 1⎞
ln sub
= − sub ⎜ − ⎟
P (T1 ) R ⎝ T2 T1 ⎠

101.3 ∆ H⎛ 1 1 ⎞ ∆ H
ln = − sub ⎜ − ⎟ = 0.83344 = − sub (−2.6402 × 10)
44.02 R ⎝ 194.5 185 ⎠ R

∆ sub H = 0.31568 × 10 4 R = 26, 246 J / mol

b) Estimate Sublimation pressure at 190 K

Psub (190K) ⎛ 1 1 ⎞
ln = −3156.8 ⎜ − ⎟ = 0.44905
44.02 ⎝ 190 185 ⎠
Psub = 44.02 × 1.5668 = 88.97 kPa
Since the pressure is below atmospheric, we can assume that
fsolid (190 K, 88.97 kPa) = 88.97 kPa = 0.8897 bar

Now need to correct to 200 bar


fsolid (190 K, 200 bar) = 0.8897 bar Exp
⎡ ( 200 − 0.8897 ) bar × 2.8 × 10−5 m3 ⎤
⎢ mol

⎢ 8.314 × 10 −5 bar m3
i190 K ⎥
⎣ mol k ⎦
= 0.8897 ×1.4232 = 1.2662 bar
th
Solutions to Chemical and Engineering Thermodynamics, 4 ed. Chapter 7

7.67 a) A plot of the logarithm of the sublimation pressure is shown below. We see that the plot is very close to
∆ H
linear. Since ln Psub = − sub + C , we find that
R ⋅T
∆ sub H
= 3151.2 K or that ∆sub H = 26.2 kJ / mol
R

ln Psub versus 1/T

10

5
ln( P i)
0

5
0.004 0.005 0.006 0.007 0.008
⎛1⎞

⎝ Ti ⎠
b) Estimate Sublimation pressure at 190 K

Psub (190K) ⎛ 1 1 ⎞
ln = −3156.8 ⎜ − ⎟ = 0.44905
44.02 ⎝ 190 185 ⎠
Psub = 44.02 × 1.5668 = 88.97 kPa
Since the pressure is below atmospheric, we can assume that
fsolid (190 K, 88.97 kPa) = 88.97 kPa = 0.8897 bar

Now need to correct to 200 bar


⎡ ( 200 − 0.8897 ) bar × 2.8 × 10−5 m3 ⎤
f solid
(190 K, 200 bar) = 0.8897 bar exp ⎢ mol

⎢⎣ 8.314 × 10 −5 bar m3
mol K i190 K ⎥⎦
= 0.8897 ×1.4232 = 1.2662 bar

7.68 From the condition of mechanical stability, the condition for a stable equilibrium
⎛ ∂P ⎞
state is ⎜ ⎟ < 0. If one follows the isotherm T3,
⎝ ∂V ⎠T
⎛ ∆P ⎞ ⎛ ∂P ⎞
∆V = V2 − V1 > 0, while ∆P=P2 − P1 > 0. Therefore ⎜ ⎟ =⎜ ⎟ > 0.
⎝ ∆V ⎠T ⎝ ∂V ⎠T
Consequently, the pressure-enthalpy diagram for this system is not consistent
with thermodynamic stability.
th
Solutions to Chemical and Engineering Thermodynamics, 4 ed. Chapter 7

R ⋅T a
7.69 a) Starting from P = − we obtain
V−b T ⋅ V2
⎛ ∂P ⎞ R ⋅ TC 2a ⎛ ∂2P ⎞ 2 ⋅ R ⋅ TC 2a
⎜ ⎟ =− + =0 and ⎜⎜ 2 ⎟⎟ = + =0
⎝ ∂V ⎠TC (V − b) 2
TC ⋅ V 3
⎝ ∂V ⎠TC ( V − b )
3
TC ⋅ V 4
Solving these two equations simultaneously we get VC = 3b and
9
a = R ⋅ TC3/ 2 VC . Substituting this expression into the equation of state at the
8
3 R ⋅ TC
critical point, we obtain PC = , so that the critical compressibility is 3/8,
8 VC
as for the van der Waals equation. Using this last relation to eliminate VC, we
obtain
27 R 2TC5 / 2 R ⋅ TC
a= and b=
64 PC 8 ⋅ PC
b) Starting from
V,T
⎛ ∂2P ⎞
CV ( V,T ) = C*V ( T ) + T ∫ ⎜⎜ 2 ⎟⎟ dV and using that
V =∞ ⎝
∂T ⎠ V

⎛ ∂P ⎞ R 1 a ⎛ ∂2P ⎞ 2 a
⎜ ⎟ = + , and ⎜⎜ 2 ⎟⎟ = − so that
⎝ ∂T ⎠V V − b 2 3/ 2 2
T V ⎝ ∂T ⎠ V 4 T 5/ 2 V 2
V,T
3⋅ a

T 3a
CV ( V,T ) = C*V (T) − dV = C*V ( T ) +
3
4⋅V 2
4 T ⋅V
T 2 V=∞
This expression des not diverge at the critical point, so the heat capacity is finite
at the critical point.

7.70 Start with the triple product rule (for simplicity in typing, neglecting the
underbars that indicate molar properties)
⎛ ∂S ⎞
⎜ ⎟
⎛ ∂V ⎞ ⎛ ∂S ⎞ ⎛ ∂T ⎞ ⎛ ∂V ⎞ ⎝ ∂T ⎠ V
⎜ ⎟ ⎜ ⎟ ⎜ ⎟ = −1 ⇒ ⎜ ⎟ = − ∂S
⎝ ∂T ⎠S ⎝ ∂V ⎠T ⎝ ∂S ⎠ V ⎝ ∂T ⎠S ⎛ ⎞
⎜ ⎟
⎝ ∂V ⎠T
C ⎛ ∂P ⎞ ⎛ ∂S ⎞ CV ⎛ ∂S ⎞ ⎛ ∂P ⎞
Starting from dS = V dT + ⎜ ⎟ dV gives ⎜ ⎟ = and ⎜ ⎟ =⎜ ⎟
T ⎝ ∂T ⎠ V ⎝ ∂T ⎠ V T ⎝ ∂V ⎠T ⎝ ∂T ⎠ V
⎛ ∂S ⎞
⎜ ⎟
⎛ ∂V ⎞ ⎝ ∂T ⎠ V C ⎛ ∂T ⎞
so that ⎜ ⎟ = − ∂S =− V⎜ ⎟
⎝ ∂T ⎠S ⎛ ⎞ T ⎝ ∂P ⎠V
⎜ ⎟
⎝ ∂V ⎠T

⎛ ∂V ⎞ ⎛ ∂P ⎞ ⎛ ∂T ⎞ ⎛ ∂V ⎞ 1
⎜ ⎟ ⎜ ⎟ ⎜ ⎟ = −1 ⇒ ⎜ ⎟ = − ∂P
⎝ ∂T ⎠P ⎝ ∂V ⎠T ⎝ ∂P ⎠V ⎝ ∂T ⎠P ⎛ ⎞ ⎛ ∂T ⎞
⎜ ⎟ ⎜ ⎟
⎝ ∂V ⎠T ⎝ ∂P ⎠V
Therefore
th
Solutions to Chemical and Engineering Thermodynamics, 4 ed. Chapter 7

⎛ ∂V ⎞ ⎛ ∂V ⎞
⎜ ⎟ 2 ⎜ ⎟ 2
⎝ ∂T ⎠S CV ⎛ ∂P ⎞ ⎛ ∂T ⎞ 1 ⎛ ∂V ⎞ ⎝ ∂P ⎠S C ⎛ ∂T ⎞
= ⎜ ⎟ ⎜ ⎟ ; also κT = − ⎜ ⎟ ; so that =− V ⎜ ⎟
⎛ ∂V ⎞ T ⎝ ∂V ⎠T ⎝ ∂P ⎠ V V ⎝ ∂P ⎠T ⎛ ∂V ⎞ TVκT ⎝ ∂P ⎠ V
⎜ ⎟ ⎜ ⎟
⎝ ∂T ⎠ P ⎝ ∂P ⎠ P

b) Clearly
2
⎛ ∂T ⎞
⎜ ⎟ > 0, T>0, and V > 0. Also, we have shown that for a stable system, CV ≥ 0 and κT ≥ 0.
⎝ ∂P ⎠ V
⎛ ∂V ⎞
⎜ ⎟
⎝ ∂T ⎠S ⎛ ∂V ⎞ ⎛ ∂V ⎞
Therefore < 0 or ⎜ ⎟ and ⎜ ⎟ must have opposite signs.
⎛ ∂V ⎞ ⎝ ∂T ⎠S ⎝ ∂T ⎠ P
⎜ ⎟
⎝ ∂T ⎠ P

c) The entropy balance for the first measurement is

dS = 0 since there is no heat flow and the process is reversible. So that can be determined from the
⎛ ∂V ⎞
measurement is ⎜ ⎟ . In that measurement, T has increased when V is decreased (gas is compressed)
⎝ ∂T ⎠S
⎛ ∂V ⎞
so that ⎜ ⎟ < 0. In the second measurement, as pressure is constant, the relation between T and V is
⎝ ∂T ⎠S
⎛ ∂V ⎞ ⎛ ∂V ⎞ ⎛ ∂V ⎞
⎜ ⎟ . Since we have shown in part b that ⎜ ⎟ has opposite sign from ⎜ ⎟ which is negative,
⎝ ∂T ⎠P ⎝ ∂T ⎠P ⎝ ∂T ⎠S
⎛ ∂V ⎞
it follows that ⎜ ⎟ > 0, so that the temperature increases as the volume increases.
⎝ ∂T ⎠ P
th
Solutions to Chemical and Engineering Thermodynamics, 4 ed. Chapter 7

7.71
Glycine

⎛⎜ 453 0.0587⎞

data := ⎜
457 0.0859⎟
R := 8.314
⎜ 466 0.159 ⎟
⎜ 471
⎝ 0.243 ⎠

〈0〉 〈1〉
X := data Y := data Number of data points:
1 Y := ln( Y)
X := n := rows ( data ) n := 4
X
Regression Statistics

Intercept b := intercept ( X , Y) b = 33.349


0 0

4
Slope b := slope ( X , Y) b = −1.638 × 10
1 1

⎛ b
1⎞
P( T) := exp⎜ b +
⎝ 0 T ⎠ P( 453) = 0.06 reasonably good fit

P( 466) = 0.165

DfusH := −R⋅ b 5
1 DfusH = 1.362 × 10
th
Solutions to Chemical and Engineering Thermodynamics, 4 ed. Chapter 7

Alanine

⎛⎜ 453 0.0759⎞

data := ⎜
460 0.122 ⎟
R := 8.314
⎜ 465 0.203 ⎟
⎜ 469
⎝ 0.258 ⎠

〈0〉 〈1〉
X := data Y := data Number of data points:
1 Y := ln( Y)
X := n := rows ( data ) n := 4
X
Regression Statistics
Intercept b := intercept ( X , Y) b = 34.284
0 0

4
Slope b := slope ( X , Y) b = −1.671 × 10
1 1

⎛ b
1⎞
P( T) := exp⎜ b +
⎝ 0 T ⎠ P( 453) = 0.074 reasonably good fit

P( 469) = 0.261

DfusH := −R⋅ b 5
1 DfusH = 1.389 × 10
7
7.65 The VISUAL Basic program Peng-Robinson Equation of State is used for the
Peng-Robinson predictions, the correlation in the Visual Basic Property program will be
used for the ‘literature data”, and the MATHCAD worksheet PRVAPP.MCD is used for
the a(T) = 1 calculation, since it is the easiest to modify. The results are as follows.

T( K) P (bar) literature, P(bar), P-R a(T) P(bar), P-R a(T)=1


380 0.125 0.129 1.780
405 0.302 0.307 2.595
430 0.642 0.648 3.635
455 1.233 1.243 4.298
480 2.185 2.204 6.499
505 3.625 3.664 8.372
530 5.701 5.777 10.57
555 8.591 8.723 13.12
580 12.52 12.70 16.04
605 17.82 17.95 19.36

and, of course at Tc = 617.7, both P-R calculations give P = 21.2 bar = Pc, since the E.O.S.
parameters have been fitted to give the correct critical point conditions (since α(TC ) = 1. )
7
7.66 Will use the PR EOS

a) find that f = 15.296 bar = 1.5296 MPa


b) to correct for increase in pressure
i) we can use PR EOS and find f = 21.463 bar
or ii) use Poynting Correction

⎡ V L (136.1 − 19.1) bar ⎤


f (13.61 MPa) = f (1.91 MPa) exp ⎢ ⎥
⎣ R ⋅T ⎦
but
⎡ 78.114 g 117 bar × 10−6 mcc
3

exp ⎢ mol
× ⎥ = exp [ 0.35382]
⎢⎣ 0.63 cc 8.314 × 10−5 bar
3

mol K × 493.15 K ⎥
g m

f(13.61 MPa) = 15.296 exp (0.35382) = 15.296 × 1.4245 = 21.789


So the two results are close, but not exactly the same. Perhaps because of
inaccuracy in liquid density, or that the liquid density changes with pressure.

7.66 Clausius-Coapeyron Eq. applied to sublimation

a)
Psub (T2 ) ∆ H ⎛ 1 1⎞
ln sub
= − sub ⎜ − ⎟
P (T1 ) R ⎝ T2 T1 ⎠
101.3 ∆ H ⎛ 1 1 ⎞ ∆ sub H
ln = − sub ⎜ 194.5 − 185 ⎟ = 0.83344 = − R (−2.6402 × 10)
44.02 R ⎝ ⎠
∆ sub H = 0.31568 × 10 4 R = 26, 246 J / mol

b) Estimate Sublimation pressure at 190 K

Psub (190K) ⎛ 1 1 ⎞
ln = −3156.8 ⎜ − ⎟ = 0.44905
44.02 ⎝ 190 185 ⎠
Psub = 44.02 × 1.5668 = 88.97 kPa
Since the pressure is below atmospheric, we can assume that
fsolid (190 K, 88.97 kPa) = 88.97 kPa = 0.8897 bar

Now need to correct to 200 bar


⎡ 200 − 0.8897 ) bar × 2.8 × 10−5
( m3 ⎤
fsolid (190 K, 200 bar) = 0.8897 bar Exp ⎢ mol

⎢ 8.314 × 10−5 bar m3
i190 K ⎥
⎣ mol k ⎦
= 0.8897 ×1.4232 = 1.2662 bar
7
7.67 a) A plot of the logarithm of the sublimation pressure is shown below. We see that the plot is very close to
∆ sub H
linear. Since ln Psub = − + C , we find that
R ⋅T
∆ sub H
= 3151.2 K or that ∆sub H = 26.2 kJ / mol
R

ln Psub versus 1/T

10

5
ln( Pi)
0

5
0.004 0.005 0.006 0.007 0.008
⎛1⎞

⎝ Ti ⎠

b) Estimate Sublimation pressure at 190 K

Psub (190K) ⎛ 1 1 ⎞
ln = −3156.8 ⎜ − ⎟ = 0.44905
44.02 ⎝ 190 185 ⎠
Psub = 44.02 × 1.5668 = 88.97 kPa
Since the pressure is below atmospheric, we can assume that
fsolid (190 K, 88.97 kPa) = 88.97 kPa = 0.8897 bar

Now need to correct to 200 bar


⎡ ( 200 − 0.8897 ) bar × 2.8 × 10−5 m3 ⎤
fsolid (190 K, 200 bar) = 0.8897 bar exp ⎢ −5 bar m3
mol

⎢⎣ 8.314 × 10 mol K i190 K ⎥⎦
= 0.8897 ×1.4232 = 1.2662 bar
7
7.68 From the condition of mechanical stability, the condition for a stable equilibrium state is
⎛ ∂P ⎞
⎜ ⎟ < 0. If one follows the isotherm T3, ∆V = V2 − V1 > 0, while ∆P=P2 − P1 > 0. Therefore
⎝ ∂V ⎠T
⎛ ∆P ⎞ ⎛ ∂P ⎞
⎜ ⎟ =⎜ ⎟ > 0. Consequently, the pressure-enthalpy diagram for this system is not
⎝ ∆V ⎠T ⎝ ∂V ⎠T
consistent with thermodynamic stability.
7
R ⋅T a
7.69 a) Starting from P = − we obtain
V−b T ⋅ V2
⎛ ∂P ⎞ R ⋅ TC 2a ⎛ ∂2P ⎞ 2 ⋅ R ⋅ TC 2a
⎜ ⎟ =− + =0 and ⎜
⎜ 2 ⎟
⎟ = + =0
⎝ ∂V ⎠TC ( V − b )2 TC ⋅ V 3
⎝ ∂V ⎠TC ( V − b )
3
TC ⋅ V 4
9
Solving these two equations simultaneously we get VC = 3b and a = R ⋅ TC3/ 2 VC .
8
Substituting this expression into the equation of state at the critical point, we
3 R ⋅ TC
obtain PC = , so that the critical compressibility is 3/8, as for the van der
8 VC
Waals equation. Using this last relation to eliminate VC, we obtain
27 R 2TC5 / 2 R ⋅ TC
a= and b=
64 PC 8 ⋅ PC
b) Starting from
V,T
⎛ ∂2P ⎞
CV ( V,T ) = C*V ( T ) + T ∫ ⎜⎜ 2 ⎟⎟ dV
V =∞ ⎝
∂T ⎠ V
and using that

⎛ ∂P ⎞ R 1 a ⎛ ∂2P ⎞ 2 a
⎜ ⎟ = + , and ⎜
⎜ 2⎟⎟ =− so that
⎝ ∂T ⎠ V V − b 2 T V
3/ 2 2
⎝ ∂T ⎠ V 4 T 5/ 2 V 2
V,T
3⋅ a

T 3a
CV ( V,T ) = C*V ( T ) − dV = C*V ( T ) +
3
4 ⋅ V2 4 T ⋅V
T 2 V=∞
This expression des not diverge at the critical point, so the heat capacity is finite at the critical
point.
7
7.70 Start with the triple product rule (for simplicity in typing, neglecting the
underbars that indicate molar properties)
⎛ ∂S ⎞
⎜ ⎟
⎛ ∂V ⎞ ⎛ ∂S ⎞ ⎛ ∂T ⎞ ⎛ ∂V ⎞ ⎝ ∂T ⎠ V
⎜ ⎟ ⎜ ⎟ ⎜ ⎟ = − 1 ⇒ ⎜ ⎟ = −
⎝ ∂T ⎠S ⎝ ∂V ⎠T ⎝ ∂S ⎠ V ⎝ ∂T ⎠S ⎛ ∂S ⎞
⎜ ⎟
⎝ ∂V ⎠T
C ⎛ ∂P ⎞ ⎛ ∂S ⎞ CV ⎛ ∂S ⎞ ⎛ ∂P ⎞
Starting from dS = V dT + ⎜ ⎟ dV gives ⎜ ⎟ = and ⎜ ⎟ =⎜ ⎟
T ⎝ ∂T ⎠ V ⎝ ∂T ⎠ V T ⎝ ∂V ⎠T ⎝ ∂T ⎠ V
⎛ ∂S ⎞
⎜ ⎟
⎛ ∂V ⎞ ⎝ ∂T ⎠ V C ⎛ ∂T ⎞
so that ⎜ ⎟ = − =− V⎜ ⎟
⎝ ∂T ⎠S ⎛ ∂S ⎞ T ⎝ ∂P ⎠V
⎜ ⎟
⎝ ∂V ⎠T
⎛ ∂V ⎞ ⎛ ∂P ⎞ ⎛ ∂T ⎞ ⎛ ∂V ⎞ 1
⎜ ⎟ ⎜ ⎟ ⎜ ⎟ = −1 ⇒ ⎜ ⎟ = − ∂P
⎝ ∂T ⎠ P ⎝ ∂V ⎠T ⎝ ∂P ⎠ V ⎝ ∂T ⎠ P ⎛ ⎞ ⎛ ∂T ⎞
⎜ ⎟ ⎜ ⎟
⎝ ∂V ⎠T ⎝ ∂P ⎠V
Therefore

⎛ ∂V ⎞ ⎛ ∂V ⎞
⎜ ⎟ 2 ⎜ ⎟ 2
⎝ ∂T ⎠S CV ⎛ ∂P ⎞ ⎛ ∂T ⎞ 1 ⎛ ∂V ⎞ ⎝ ∂P ⎠S C ⎛ ∂T ⎞
= ⎜ ⎟ ⎜ ⎟ ; also κT = − ⎜ ⎟ ; so that =− V ⎜ ⎟
⎛ ∂V ⎞ T ⎝ ∂V ⎠T ⎝ ∂P ⎠ V V ⎝ ∂P ⎠T ⎛ ∂V ⎞ TVκT ⎝ ∂P ⎠ V
⎜ ⎟ ⎜ ⎟
⎝ ∂T ⎠ P ⎝ ∂P ⎠ P
b) Clearly
2
⎛ ∂T ⎞
⎜ ⎟ > 0, T>0, and V > 0. Also, we have shown that for a stable system, CV ≥ 0 and κT ≥ 0.
⎝ ∂P ⎠ V
⎛ ∂V ⎞
⎜ ⎟
⎝ ∂T ⎠S ⎛ ∂V ⎞ ⎛ ∂V ⎞
Therefore < 0 or ⎜ ⎟ and ⎜ ⎟ must have opposite signs.
⎛ ∂V ⎞ ⎝ ∂T ⎠S ⎝ ∂T ⎠ P
⎜ ⎟
⎝ ∂T ⎠ P
c) The entropy balance for the first measurement is
dS = 0 since there is no heat flow and the process is reversible. So that can be determined from the
⎛ ∂V ⎞
measurement is ⎜ ⎟ . In that measurement, T has increased when V is decreased (gas is compressed)
⎝ ∂T ⎠S
⎛ ∂V ⎞
so that ⎜ ⎟ < 0. In the second measurement, as pressure is constant, the relation between T and V is
⎝ ∂T ⎠S
⎛ ∂V ⎞ ⎛ ∂V ⎞ ⎛ ∂V ⎞
⎜ ⎟ . Since we have shown in part b that ⎜ ⎟ has opposite sign from ⎜ ⎟ which is negative,
⎝ ∂T ⎠ P ⎝ ∂T ⎠P ⎝ ∂T ⎠S
⎛ ∂V ⎞
it follows that ⎜ ⎟ > 0, so that the temperature increases as the volume increases.
⎝ ∂T ⎠ P
th
Solutions to Chemical and Engineering Thermodynamics, 4 ed. Chapter 7

7.71
Glycine

⎛⎜ 453 0.0587⎞

data := ⎜
457 0.0859⎟
R := 8.314
⎜ 466 0.159 ⎟
⎜ 471
⎝ 0.243 ⎠

〈0〉 〈1〉
X := data Y := data Number of data points:
1 Y := ln( Y)
X := n := rows ( data ) n := 4
X
Regression Statistics

Intercept b := intercept ( X , Y) b = 33.349


0 0

4
Slope b := slope ( X , Y) b = −1.638 × 10
1 1

⎛ b
1⎞
P( T) := exp⎜ b +
⎝ 0 T ⎠ P( 453) = 0.06 reasonably good fit

P( 466) = 0.165

DfusH := −R⋅ b 5
1 DfusH = 1.362 × 10
th
Solutions to Chemical and Engineering Thermodynamics, 4 ed. Chapter 7

Alanine

⎛⎜ 453 0.0759⎞

data := ⎜
460 0.122 ⎟
R := 8.314
⎜ 465 0.203 ⎟
⎜ 469
⎝ 0.258 ⎠

〈0〉 〈1〉
X := data Y := data Number of data points:
1 Y := ln( Y)
X := n := rows ( data ) n := 4
X
Regression Statistics
Intercept b := intercept ( X , Y) b = 34.284
0 0

4
Slope b := slope ( X , Y) b = −1.671 × 10
1 1

⎛ b
1⎞
P( T) := exp⎜ b +
⎝ 0 T ⎠ P( 453) = 0.074 reasonably good fit

P( 469) = 0.261

DfusH := −R⋅ b 5
1 DfusH = 1.389 × 10
8

8.1 (a) By Eqn. (8.2-3)

FG ∂ H IJ = Gi ;
H∂ N K i P , S , N j ≠i

but Gi = Hi − TSi . Thus

FG ∂ H IJ = Hi − TSi
H∂ N K i P , S , N j ≠i

a
(b) Since U = U S ,V , Nf
F ∂ U IJ
dU = G
FG ∂ U IJ dV + ∑ FG ∂ U IJ
dS +
H∂SK V,N
H ∂V K H∂ N K S,N i i S ,V , N j ≠i
dNi

= TdS − PdV + ∑ G
F ∂ U IJ dN
H∂ N K i i S ,V , N j ≠i
i (1)

However, we also have U = H − PV ; dU = dH − PdV − VdP , and, by Eqn. (8.2-3)

dU = VdP + TdS + ∑ Gi dNi − PdV − VdP = TdS − PdV + ∑ Gi dNi (2)

Equating (1) and (2) shows that Gi =


FG ∂ U IJ
H∂ N K i S ,V , N j ≠i
. Next we start from

A = A(T ,V , N )

⇒ dA =
FG ∂ A IJ dT +
FG ∂ A IJ dV + ∑
FG ∂ A IJ
H∂TK V ,N
H ∂V K T,N i H∂ N K i T ,V , N j ≠i
dNi

or

dA = − SdT − PdV + ∑
FG ∂ A IJ
i H∂ N K i T ,V , N j ≠i
dNi (3)

However, we also have that A = U − TS ;

dA = dU − TdS − SdT = TdS − PdV + ∑ Gi dNi − TdS − SdT

or
dA = − SdT − PdV + ∑ Gi dNi (4)

Comparing (3) and (4) yields

Gi =
FG ∂ A IJ
H∂ N K
i T ,V , N j ≠i
8

8.2 (a) General: θ = ∑ Niθ i where θ i = ∂ θ ∂ Ni a f


T , P , N j ≠i and

dθ = ∑ θ i dNi + ∑ Ni dθ i (1)

However, we also have that

dθ =
FG ∂ θ IJ dT +
FG ∂ θ IJ dV + ∑
FG ∂ θ IJ
H∂ TK V ,N
H∂V K T,N
H∂ N K i T ,V , N j ≠i
dNi (2)

Subtracting (2) from (1) yields

FG ∂ θ IJ FG ∂ θ IJ LM F ∂ θ I OPdN + N dθ
0=−
H∂ TK V ,N
dT −
H∂V K T,N
dV + ∑ θ i −
MN GH ∂ N JK i T ,V , N j ≠i PQ ∑ i i i

At constant T and V

LM F ∂ θ I OPdN + N dθ
MN GH ∂ N JK
0 = ∑ θi −
i T ,V , N PQ ∑ i i i

(general equation)

θ = A, θ i = Ai
FG ∂ θ IJ =
FG ∂ A IJ = Gi .
For and
H∂ N K i T ,V , N j ≠i H∂ N K i T ,V , N j ≠i
Thus,

θi −
FG ∂ θ IJ = Ai − Gi = − PVi and
H∂ N K
i T ,V , N j ≠i

∑ Ni dAi T , V = P∑ Vi dNi T , V specific equation for θ = A

(b) Following the analysis above, we also get

FG ∂ θ IJ FG ∂ θ IJ LM F ∂ θ I OPdN + N dθ
0=−
H∂UK V ,N
dU −
H∂V K U ,N
dV + ∑ θ i −
MN GH ∂ N JK i U ,V , N j ≠i PQ ∑ i i i

and, at constant U and V

L F ∂θ I OPdN + N dθ
0= ∑ MMθ − GH ∂ N JK PQ ∑
N
i i i i
i U ,V , N j ≠i
Now, choosing θ = S , and using that
FG ∂ S IJ =−
Gi
H∂ N K i U ,V , N j ≠i T
, which is easily derived, yields

−T ∑ Ni dSi U ,V = ∑ Hi dNi U ,V

(c) Following a similar analysis to those above, we obtain

FG ∂ θ IJ FG ∂ θ IJ LM F ∂ θ I OPdN + N dθ
0=−
H∂ SK V ,N
dS −
H∂V K S,N
dV + ∑ θ i −
MN GH ∂ N JK i S ,V , N j ≠i PQ ∑
i i i

which, at constant V and S, reduces to

LM F ∂ θ I OPdN + N dθ
MN GH ∂ N JK
0 = ∑ θi −
i S ,V , N j ≠i PQ ∑
i i i

Finally, using θ = U , and a∂ U ∂ N f i S ,V , N j ≠i = Gi yields

∑ Ni dUi S ,V = ∑ l− PVi + TSi qdNi S ,V


8
8.3 (a) At constant U and V, S = maximum at equilibrium

C C
S = S I + S II = ∑ NiI SiI + ∑ NiII SiII
i =1 i =1

but

dS = 0 =
FG ∂ S IJ I
dU I +
FG ∂ S IJ I
dV I + ∑
FG ∂ S IJ I
dNiI
H∂U K I
V ,N
H∂V K I
U ,N
H∂ N K I
i U ,V , N j ≠i

F ∂ S IJ
+G
II
dU II +G
F ∂ S IJ II
dV II + ∑G
F ∂ S IJ II
dNiII
H∂U K II
V ,N
H∂V K II
U ,N
H∂ N K II
i U ,V , N j ≠i

Since U = U I + U II = constant, dU II = −dU I


Since V = V I + V II = constant, dV II = −dV I
and since Ni = NiI + NiII = constant, dNiII = −dNiI
Also,

FG ∂ S IJ =
1 ∂S FG IJ =
P ∂S FG IJ =−
Gi
H∂U K V ,N T
;
∂V H K U ,N T
and
∂ Ni H K U ,V , N j ≠i T
(see previous problem)

Thus

F 1 − 1 I dU + FG P I
P II IJ G1I G1II
∑ T I − T II dNiI
FG IJ
dS = 0 =
HT T K HT − −
I I
I II I
T II
dV
K i H K
⇒ T I = T II ; P I = P II ; and Gi I = Gi II

for equilibrium in a closed system at constant U and V.


(b) For a closed system at constant S and V, U has an extremum. Thus

dU = 0 =
FG ∂ U IJ I
dS I +
FG ∂ U IJ I
dV I + ∑
FG ∂ U IJ I
dNiI
H∂S K I
V ,N
H ∂V K I
S, N iH∂ N K I
i S ,V , N j ≠i

F ∂ U IJ
+G
II
dS II +G
F ∂ U IJ II
dV II + ∑G
F ∂ U IJ II
dNiII
H∂S K II
V ,N
H ∂V K II
S,N
H∂ N K
i
II
i U ,V , N j ≠i

but S, V and N j , j = 1, " , C are constant. Thus


c h c h c h
dU = 0 = T I − T II dS I + P I − P II dV I + ∑ GiI − GiII dNiI
i

⇒ T I = T II , P I = P II and Gi I = Gi II

for equilibrium in a closed system at constant S and V.


8

8.4 (a) For a closed system at constant T and V, A is a minimum at equilibrium; thus dA V , T = 0 . From
Eqn. (8.2-5)

dA = − PdV − SdT + ∑ Gi dNi or dA V ,T = ∑ Gi dNi

But, Ni = Ni , 0 + ν i X . Thus dNi = ν i dX and

dA V ,T = b∑ ν G gdX = 0 or FGH ∂∂ XA IJK


i i = ∑ ν i Gi = 0 .
i
V ,T
(b) For a closed system at constant U and V, S = maximum, or dS U ,V = 0 . From Eqn. (8.2-4)
1 P 1
dS = dU + dV − ∑ Gi dNi ; thus
T T T

dS U ,V = −
1
T
1
∑ Gi dNi or dS U ,V = − T b∑ G ν gdX
i i

and

∂S 1
∂X
=− ∑ ν i Gi = 0
T i
U ,V
8

8.5 Let mi = molecular weight of species i. Multiplying Eqn. (8.3-2a) by mi and summing over all
species i yields, for a closed system

∑ mi Ni = total mass in system = ∑ mi Ni ,0 + X ∑ ν i mi


total mass in
system initially

However, since the total mass is a conserved quantity,

∑ mi Ni = ∑ mi Ni,0 ⇒ X ∑ ν i mi = 0 , where X can take on any value.


Consequently, if this equation is to be satisfied for all values of X, then ∑ ν i mi = 0 !
M
Similarly, in the multi-reaction case, starting from Ni = Ni ,0 + ∑ ν ij X j , we get
j =1
C C C M C M M C
∑ mi Ni = ∑ mi Ni,o + ∑ mi ∑ ν ij X j ⇒ ∑ mi ∑ ν ij X j = 0 = ∑ X j ∑ ν ij mi
i =1 i =1 i =1 j =1 i =1 j =1 j =1 i =1

Since the X j ’s are not, in general, equal to zero, we have

C
∑ ν ij mi = 0
i =1
a f a f
In particular, for the reaction H 2O = H 2 + 1 2 O2 , or H 2 + 1 2 O2 − H 2O = 0 , we have

∑ ν ij mi = (+1)(2) + FH 2 IK (32) + (−1)(18) = 0 .


1
i
8
8.6 From Eqns. (8.6-4) we have

V1 = V 1 + ∆V mix + x2
a
∂ ∆V mix f (1)
∂ x1 T,P

and

V2 = V 2 + ∆V mix + x1
a
∂ ∆V mix f (2)
∂ x1 T,P

Now since T, P and X, are the independent variables, we have that

0 since pure component volume is a function of


T and P only
dV1 T , = dV 1 T , a
+ d ∆V mix T , P + d x2f∂ ∆V mix LMN a f OP
Q
P P
∂ x1 T, P

=
a
∂ ∆V mix f +
a
∂ ∆V mix f ∂ x2
dx1 + x2
∂ 2 ∆V mix a f dx1
∂ x1 T, P ∂ x1 T, P ∂ x1 ∂ x12 T, P

= x2
a
∂ 2 ∆V mix f dx1 since
∂ x2
= −1
∂ x12 T, P
∂ x1

Similarly

dV2 T , P = − x1
a
∂ 2 ∆V mix f dx1
∂ x12 T,P

Thus

∑ xiαVi T , P = x1x2
a
∂ 2 ∆V mix f dx1 − x2 x1
a
∂ 2 ∆V mix f dx1 ≡ 0
∂ x12 T,P
∂ x12 T,P

Thus, V1 and V2 given by equations (1) and (2) identically satisfy the Gibbs-Duhem equation
∑ xi dθ i T , P = 0 .
A similar argument applies for the partial molar enthalpies of Eqn. (8.6-9).
8
8.7 (also available as a Mathcad worksheet)

The students can solve this problem by drawing tangent lines to the ∆V mix curves. Polak and Lu
smoothed their data using the Redhich-Kister equation (see Eqn. (8.6-5a)). That is, they fitted their
data to

a f a f∑ C (1 − 2 x)
n
∆V mix = x1x2 ∑ C j x2 − x1 j −1
= x1 1 − x1 j
j −1

j =1
Now
a
∂ ∆V mix f a
= 1 − x1f∑ C a1 − 2 x f j −1
∂ x1 j 1

− x ∑ C a1 − 2 x f − 2 x a1 − x f∑ C ( j − 1)a1 − 2 x f
1 j 1
j −1
1 1 j 1
j −2

∂ a∆V f
Thus V − V = a∆V f − x = a1 − x f k A − 2 x Bp mix 2
(1)
1 1 mix
∂x 2
1
1 1

and

a
V2 − V 2 = ∆V mix − x1 f a
∂ ∆V mix f
= x12 A + 2 x2 B k p (2)
∂ x1
where

∑ C j a1 − 2 x1f j −1 ∑ C j ( j − 1)a1 − 2 x1f j − 2


n n
A= and B =
j =1 j =1
Taking species 1 to be methyl formate, Polak and Lu found

C1 C2 C3 C4
methyl formate - Methanol −0.33259 −010154
. −0.0516 0.0264
methyl formate - Ethanol 081374
. −0.00786 0.0846 0.0448
[units are cc/mol; multiply by 10−3 to get m3 kmol ]

I have used the equations above and the constants given to find V1 − V 1 and V2 − V2 , since this leads
to more accurate results than the graphical method.
The results are tabulated and plotted below.
Methyl formate - Methanol

xMF 0 0.1 0.2 0.3 0.4 0.5


∆V mix acc molf 0 –0.039 –0.065 –0.080 –0.085 –0.083
V1 − V 1 –0.459 –0.329 –0.225 –0.148 –0.093 –0.058
V2 − V 2 0 –0.007 –0.025 –0.051 –0.080 –0.109

xMF 0.6 0.7 0.8 0.9 1.0


∆V mix acc molf –0.075 –0.063 –0.047 –0.027 0
V1 − V 1 –0.035 –0.021 –0.011 –0.004 0
V2 − V 2 –0.136 –0.162 –0.192 –0.236 –0.309

b g
Thus VMF = 62.78 + V1 − V 1 cc/mol or 10−3 m3 kmol .
b
VM = 40.73 + V2 − V 2 . g
Methyl formate - Ethanol
xMF 0 0.1 0.2 0.3 0.4 0.5
∆V mix a
cc mol 0f 0.080 0.136 0.174 0.196 0.203
V1 − V 1 0.935 0.682 0.507 0.381 0.285 0.205
V2 − V 2 0 0.013 0.043 0.085 0.137 0.201

xMF 0.6 0.7 0.8 0.9 1.0


∆V mix acc molf 0.196 0.174 0.134 0.077 0
V1 − V 1 0.138 0.081 0.037 0.010 0
V2 − V 2 0.284 0.390 0.522 0.680 0.861

b g
Thus VMF = 62.78 + V1 − V 1 cc/mol. Multiply by 10−3 for m3 kmol .
b
VE = 58.68 + V2 − V 2 g
8
8.8 This problem is similar to the last one, and will be treated in a similar fashion. Fenby and
Ruenkrairergasa give their data in the form

a f a f∑ C a1 − 2x f
n
j −1
∆H mix J mol = x2 1 − x2 j 2 (1)
j =1
where component 2 is the fluorobenzene. The constants given in the aforementioned reference and
Fenby and Scott J. Phys. Chem 71, 4103 (1967) are given below
System C1 C2 C3 C4
C6 H 6 − C6 F5Cl –2683 929 970 0
C6 H 6 − C6 F5Br –3087 356 696 0
C6 H 6 − C6 F5I –4322 –161 324 0
C6 H 6 − C6 F6 –1984 +1483 +1169 0
C6 H 6 − C6F5H 230 +578 +409 +168

If we replace x2 with 1 − x1 in Eqn. (1), we regain the equation of the previous illustration, except
for a factor of (−1) j −1 in the sum and the corresponding places in the other equations.

xC 6 H 6 ∆H mix bH − Hg C6 H 6
bH − Hg C 6 F5Cl
xC 6 F5Cl
0 0 –2642 0 1.0
0.1 –252 –2171 –39.2 0.9
0.2 –463 –1790 –130 0.8
0.3 –609 –1466 –242 0.7
0.4 –679 –1175 –349 0.6
0.5 –671 –903 –439 0.5
0.6 –590 –646 –506 0.4
0.7 –453 –409 –555 0.3
0.8 –284 –205 –601 0.2
0.9 –119 –57.8 –666 0.1
1.0 0 0 –784 0

[Note: J/mol]

C6 H 6 − C6 F5Br C6 H 6 − C6 F5I
xC 6 H 6 ∆H mix bH − Hg bH − Hg ∆H mix bH − Hg bH − Hg xC 6 F5 x
C6 H 6 C6F5Br C6 H 6 C6F5I
0 0 –2747 0 0 –3837 0 1.0
0.1 –263 –2248 –42.9 –359 –3119 –52.1 0.9
0.2 –488 –1829 –153 –657 –2489 –200 0.8
0.3 –654 –1469 –306 –883 –1937 –431 0.7
0.4 –751 –1149 –486 –1026 –1456 –740 0.6
0.5 –772 –861 –683 –1081 –1040 –1121 0.5
0.6 –717 –600 –893 –1042 –689 –1572 0.4
0.7 –595 –370 –1120 –910 –402 –2095 0.3
0.8 –420 –181 –1374 –688 –187 –2695 0.2
0.9 –212 –50.0 –1671 –382 –48.9 –3379 0.1
1.0 0 0 –2035 0 0 –4159 0

C6 H 6 − C6 F6 C6 H 6 − C6F5H
xC 6 H 6 ∆H mix bH − Hg bH − Hg ∆H mix bH − Hg bH − Hg xC 6 F5 x
C6 H 6 C6F6 C6 H 6 C6 F5H
0 0 –2298 0 0 61.0 0 1.0
0.1 –218 –1899 –31.2 –2.2 36.2 –1.1 0.9
0.2 –392 –1590 –93.0 –3.9 –2.8 +6.8 0.8
0.3 –502 –1332 –146 13.5 –42.3 +37.4 0.7
0.4 –536 –1097 –162 31.4 –72.3 +100 0.6
0.5 –496 –867 –125 57.5 –87.0 +202 0.5
0.6 –394 –637 –28.9 86.9 –84.5 344 0.4
0.7 –253 –413 +121 110 –66.7 524 0.3
0.8 –108 –212 +308 116 –39.4 +737 0.2
0.9 –4.5 –60.9 +503 85.9 –12.6 +973 0.1
1.0 0 0 +688 0 0 1217 0
↑ ↑ ↑
Note: Changes in sign in column
8
8.9 (a) Gibbs Phase Rule: F = C − M − P + 2
P = 2 , C = 2 , M = 0 ⇒ F = 2 − 0 − 2 + 2 = 2 degrees of freedom
Thus can fix two variables, usually from among T, P, x and y.
(b) P = 1 , C = 3 and M = 1 ⇒ F = 3 − 1 − 1 + 2 = 3 degrees of freedom
Thus, we can fix 3 variables, for example, T, P and xH 2 .
(c) Formation reactions
C + 2O → CO2
C + O → CO
2H → H 2
C + 4H → CH 4
2H + O → H 2O
1
Use O = CO − C and H = H 2 to eliminate O and H from the set so that
2

C + 2(CO − C) → CO2 2CO → CO2 + C


a f
C + 4 1 2 H 2 → CH 4 ⇒ C + 2H 2 → CH 4
a f
2 1 2 H 2 + (CO − C) → H 2O H 2 + CO → H 2O + C

Thus we have found a set (there is no unique set) of three independent reactions among the six
species. Consequently, C = 6 , M = 3 , P = 2 (solid carbon + gas phase).
F = C − M − P + 2 = 6 − 3 − 2 + 2 = 3 degrees of freedom. As a check:

c
# of unknowns = 8 T S , PS , T V , P V , xCO 2 , xCO , xH 2 , xCH 4 h
Note: xH 2 O = 1 − xCO 2 − xCO − xH 2 − xCH 4

Relations among the unknowns T S = T V , PS = P V , no phase equilibrium relations, but 3


chemical equilibrium relations of the form ∑ ν ij Gi = 0 .

8 unknowns − 5 eqns. = 3 unspecified unknowns or


3 degrees of freedom
8

a
8.10 (a) In general, for a binary, two-phase mixture C = 2, M = 0, P = 2 f
F = C − M − P + 2 = 2 − 0 − 2 + 2 = 2 degrees of freedom.
However, for an azeotrope there is the additional restriction x1 = y1 , which eliminates one
degree of freedom. Thus, there is only 1 degree of freedom for a binary, azeotropic system.
(b) In osmotic equilibrium P I ≠ P II , since the membrane is capable of supporting a pressure
difference, and G2I ≠ G2II , where 2 is the species which does not pass through the membrane.
Therefore, the independent unknowns are T I , P I , x1I , T II , P II and x1II . [Note, x2I and x2II
are not independent unknowns since x2I = 1 − x1I and x2II = 1 − x1II ]. There are two equilibrium
relations between these six unknowns: viz. T I = T II and G1I = G1II . Consequently, there are
four degrees of freedom … that is, as we shall see in Sec. 8.7, if T, P I , P II and x1I are
specified, x1II will be fixed.
(c) Case I: M = 0, C = 2, P = 2 ⇒ F = 2 − 0 − 2 + 2 = 2
Case II: M = 0, C = 2, P = 3 ⇒ F = 2 − 0 − 3 + 2 = 1
8

8.11 (a) Gibbs Phase Rule: F = C − M − P + 2


C = 2 , M = 0 ⇒ F = 2 − 0 − P + 2 = 4 − P degrees of freedom.
Therefore, a maximum of 4 phases can exist at equilibrium (for example a solid, two liquids and
a vapor, or two solids, a liquid and a vapor, etc.)

(b) Gibbs Phase Rule: F = C − M − P + 2


C = 2 , M = 1 ⇒ F = 2 − 1 − P + 2 = 3 − P degrees of freedom.
Therefore, a maximum of e phases can exist at equilibrium (for example a two liquids and a
vapor, or a solid, a liquid and a vapor, etc.)
8

dNi
8.12 (a) = N i + N i , rxn
dt
dU 0 dV
= ∑ N i Hi + Q − Ws − P
dt dt
dS Q
= ∑ N i Si + + Sgen
dt T
dS
T − T ∑ N i Si − TSgen = Q
dt
dU dS dV
= ∑ N i Hi + T − T ∑ N i Si − TSgen − P
dt dt dt
dU
dt
+P
dV
dt
−T
dS
dt
b g
= ∑ N i Hi − TSi − TSgen
dU dV dS
= ∑ N i µ i − TSgen = ∑F dNi I
dX
µ i − TSgen
dt
+P
dt
−T
dt H dt
− νi
K
dt
General expression
Now
System is only permeable to species 1
dU dV dS F dN1 dXI µ1 = −TSgen ≤ 0
dt
+P
dt
−T
dt H−
dt
− ν1
K
dt
When T and P constant
d
dt
(U + PV − TS ) −
d
dt
a f
N1 − ν 1 X µ1 ≤ 0

d
dt
a f
G − N1 − ν 1 X µ1 ≤ 0

a f
⇒ G − N1 − ν 1 X µ1 = minimum at equilibrium
(b) When T and V are constant
d
dt
(U − TS ) −
d
dt
a f
N1 − ν 1 X µ1 ≤ 0

a f
⇒ A − N1 − ν 1 X µ1 = minimum at equilibrium
8

8.13 (a) 2N → N 2
2O → O2
1
2N + O → N 2O N2 + O2 → N 2 O
2
2N + 2O → 2NO N 2 + O2 → 2NO
2N + 4O → N 2O4 N 2 + 2O2 → N 2O4
2N + 4O → 2NO2 N 2 + 2O2 → 2NO2
5
2N + 5O → N 2O5 N 2 + O2 → N 2O5
2
⇒ 5 independent reactions
(b) F = C − M − P + 2 = 7 − 5 − 1 + 2 = 9 − 6 = 3
F = 3 degrees of freedom
(c) 1 degree of freedom used in O2:N 2 ratio ⇒ 2 degres of freedom
8

8.14 Mass balance: M1 + M 2 = M f Molecular weight H 2 O = 18.02 g mol


Energy balance: M1U1 + M 2U 2 = M f U f
In each case the system is M1 kg of solution 1 + M 2 kg of solution 2.
Since Q = 0 , Ws = 0 (adiabatic mixing)
For liquids U ≡ H . Thus we have
M H + M 2 H 2
H f = 1 1
M1 + M 2
1
c
when M1 = M 2 ; H f = H 1 + H 2 .
2
h
(a) Read from Figure 8.1-1
H 1 = 6.9 × 103 J kg
H = −61
2 . × 103 J kg
1
c
Thus H f = 5410
2
. h
× 104 = 2.705 × 104 J kg
To find the composition, so a sulfuric acid balance
1
a f
ρ1 M1 + ρ 2 M2 = ρ f M f ⇒ ρ f = ρ1 + ρ 2 since M1 = M2
2
where ρ i = weight percent of ith flow stream.
1
Thus ρ f = (10 + 90) = 50 wt % sulfuric acid. From Figure 6.1-1
2

50 wt % H 2SO4
⇒ Tf ~ 110° C
H = U = 2.705 × 104 J kg

(b) Here H 1 = 6.9 × 103 J kg ,


1
H 2 = −3186
. × 105 J kg ⇒ H f = (6.9 − 318.6) × 103 = −156
. × 105 J kg and ρ1 = 10 wt % ,
2
ρ 2 = 60 wt % ⇒ ρ f = 35 wt % . Using
Figure 8.1-1, Tf ~ 22° C .
Notice that there is a balance between the energy released in mixing, ∆H mix , and the energy
absorbed in heating the mixture, CP ∆T . In case (a), ∆H mix is very large, and Tf > T1 or T2 ,
while in case (b) ∆H mix is smaller, so that Tf ~ T1 .
8

8.15 (a) MW H 2O = 18.02 g mol ; MW H 2SO4 = 98.08 g mol

100 g H 2O = 555
. mol
100 g H 2SO4 = 102
. mol

Note: When these are mixed, a solution containing


5.44 mol H 2O /mol acid is formed. ∆H s for such a solution is –58,390 J/mol acid. Thus,

total heat released = 102 a


. mol acid × −58,390 J mol acid = −59,558 Jf
(Negative sign means that heat is released!)
(b) Adding another 100 grams of water produces a solution which contains 10.88 mol H 2O /mol
acid. From the graph ∆H s = −64,850 J mol acid . However, –58,390 J/mol of acid were
released in preparing the first solution, so that only –6,460 J/mol acid, or 6,590 J, are released
on this further dilution.
40 18.02
(c) 60 wt % H 2SO4 ⇒ = 3629
. moles H 2O moles acid for which
60 98.08
∆ H s = −52,300 J mol acid , and
60 mol acid
∆Hs = −52,300 J mol acid × = −31,990 J
98.08
Note: Enthalpy of 60 WT% solution is –31,990 J relative to pure components at the same
temperature. Similarly 25 wt % H 2SO4 ⇒ 16.27 mol H 2O mol acid ,
∆H s ~ −68,830 J mol acid and
0.25 × 75
∆Hs = −68,830 J mol acid × = −13160
, J
98.08
Final solution = 175 grams ; 78.75 grams acid = 0803
. mol,
96.25 grams water = 5347
. mol ⇒ 6.66 mol H 2O mol acid . So that
∆ H s = −60,670 J mol acid
∆Hs = −48,720 J
Thus, enthalpy change on mixing, ∆Hmix is
∆Hmix = −48,720 − (−31,990 − 13160
, ) = −3570 J
Thus, 3570 J = 357 kJ must be removed to keep solution isothermal!

a f N FG IJ
(d) For 1 mole of solute: 1 + N 2 H mix = H 1 + N 2 H 2 + 1⋅ ∆ H s 2 (argument of ∆ H s ) and for
N1 H K
N1 moles of solute and N 2 moles of solvent.

a N + N fH = N1 H 1 + N 2 H 2 + N1∆ H s
FG N IJ = H
HN K
2
1 2 mix mix
1
Now

H1 =
FG ∂ H IJ = H1 + ∆ H s
FG N IJ + N ∂ a∆ H f ⋅
a
∂ N 2 N1 f
H ∂N K H N K ∂aN N f
mix 2 s
1 T,P 1
1
2 1 T,P ∂ N1 T,P
or
FG N IJ − N LM ∂ ∆ H a N N f OP since ∂ a N N f = − N
H1 − H 1 = ∆ H s
H N K N N ∂aN N f Q
2 2 s 2 1 2 1 2

1 1 ∂N2 N1 T,P 1
2
1

Similarly, starting from H = G


F ∂ H IJ we obtain
H ∂N K
mix
2
2 T,P

∂ ∆H aN N f
H −H = 2 1
∂a N N f
s
2 2
2 1 T,P
50 18.02
(e) 50 wt % acid ⇒ = 5443
. mol H 2O mol acid
50 98.08
∆ H s (5443
. ) = −58,370 J mol and, from the accompanying graph

a
∂ ∆ H s N 2 N1 f =
(−91,630) − (−46,030)
= −2,280 J mol
a
∂ N 2 N1 f at N 2 N1 = 5.443 20

so that H2 − H 2 = −2,280 J mol and


H1 − H 1 = (−58,370) − 544
. (−2,280) = −45,967 J mol .
8

8.16 To get partial molar properties it is easiest to first convert all data in problem to mole fractions and
properties per mole.

wt % CCl4 15384
.
xCCl 4 =
awt % CCl 4 f a
. + 100 − wt % CCl4 7811
15384 . f
where MWCCl 4 = 15384
. ; MWC 6 H 6 = 7811
. .
CP (mole mixture) = CP (grams mixture) × ( MW of mixture)
= CP × xCCl 4 × 15384 c
. + 1 − xCCl 4 × 7811
. h
also, ∑ xi CP,i , where CP,i = heat capacity
compute of pure species i and
∆CP, mix = CP (mixture) − ∑ xi CP,i . Results are given below:

Wt % CCl4 xCCl 4 CP J mol K ∑ xi CP,i ∆CP,mix J mol K a f


0 0 137.90 137.90 0
10 0.0534 133.91 137.17 –3.26
20 0.1126 129.55 136.35 –6.80
30 0.1787 124.45 135.44 –10.99
40 0.2529 118.85 134.42 –15.67
50 0.3368 113.98 133.72 –19.74
60 0.4323 111.29 131.96 –20.67
70 0.5423 110.48 130.44 –19.96
80 0.6701 110.59 128.69 –18.10
90 0.8205 114.44 126.62 –12.18
100 1 124.15 124.15 0

Using these data, and the graphical procedure introduced in Sec. 6.6, we obtain the following
results.

xCCl 4 0 0.1 0.2 0.3 0.4 0.5


bC P − CP g
CCl 4
–71 –60.7 –58.0 –44.5 –27.5 –17.5

CP,CCl 4 53.15 63.5 66.2 79.7 96.7 106.7

bC P − CP g
C6 H 6
0 –0.5 –1.3 –6.7 –16.0 –24.0

CP,C 6 H 6 137.9 137.4 136.6 131.2 121.9 113.9

xCCl 4 0.6 0.7 0.8 0.9 1.0


bC P − CP g
CCl 4
–11.8 –8.7 –4.1 –1.2 0

CP,CCl 4 112.4 115.45 120.1 123.0 124.15

bC P − CP g
C6 H 6
–30.8 –36.7 –49.8 –67.5 –80.5

CP,C 6 H 6 107.1 101.2 88.1 70.4 57.4


An alternate solution to this problem follows.

Alternate Solution to Problem 8.16


Instead of using Equations (8.6-10a and b) and ∆CP, mix data, Equations (8.6-11a and b) and the
heat capacity data for the mixture can be used. Since Equations (8.6-11a and b) are very similar
to Equations (8.6-10a and b) [of which Equations (8.6-4a and b) and (8.6-9a and b) are special
cases], it follows that the graphical construction discussed in Sec. 8.6 can be used. The
difference, however, is that the tangents to the CP , mix vs. mole fraction curve will give CP,CCl4
b
and CP ,C6H6 directly, rather than CP − CP g
CCl4
b
and CP − CP g
C6H 6
as before. An illustrative
graph, and the numerical results obtained using a much larger graph are given below:

xCCl 4 0 0.1 0.2 0.3 0.4 0.5


CP,CCl4 3.0 63.0 63.0 82.0 97.0 106.2
CP ,C6H6 137.9 137.9 137.9 130.7 122.9 114.7

xCCl 4 0.6 0.7 0.8 0.9 1.0


CP,CCl4 113.0 117.0 120.3 123.2 124.15
CP ,C6H6 105.7 97.6 87.3 70.8 55.9
Note that these results differ from previous results by small amounts. Previous results are probably more
accurate since the curvature of ∆CP, mix vs. xCCl 4 is greater than that of CP , mix vs. xCCl 4 , so tangents are
found with greater accuracy.
8

8.17 Let
UV
x = lbs. of 20 WT% solution used to make 1 lb.
y = lbs. of pure acid W
of 60 WT% solution

(a) Total mass balance: x + y = 1


Species mass balance on acid: 0.2 x + y = (0.6)(1)
⇒ 0.2 x + (1 − x ) = 0.6 or x = 05
. kg 20 WT% solution, y = 05
. kg pure acid.
(b) From Figure 6.1-1

H (20 wt%, 5° C) = −122


. × 105 J kg
H (100 wt%, 50° C) = 710
. × 104 J kg
H (60 wt%, 70° C) = −159
. × 105 J kg
H (60 wt%, boiling point ) = H (60 wt%, 143° C) ~ 0 J kg

Using the change over a time interval form of the energy balance equation, considering the
initial state to be two 0.5 lbs. of separated 20 wt % and pure acid solutions, and the final state to
be 1 lb. of mixed solution, and neglecting the difference between H and U for these liquids,
yields

c
H 60 wt%, Tf − 05 h
. H (20 wt%, 5° C) − 05
. H (100 wt%, 50° C) = Q

at Tf = 70° C

Q = −159
. × 105 − 05
. × −122 b
. × 105 − 05g
. × 71 b
. × 104 g
= −6.25 × 10 J kg final solution
4

at Tf = boiling point = 143° C

Q = 0 − 05 b
. −122
. × 105 − 05 g b g
. × 104 = −2.55 × 104 J kg final solution
. 71
8

8.18 Suppose there was enough information available on ∆θ mix , where θ is any extensive
thermodynamic property of a mixture, as a function of the three mole numbers N1 , N 2 , and N 3 ,
that the data could be fitted to a polynomial expression in x1 , x2 and x3 or, equivalently, in N1 ,
N 2 and N 3 where N = ∑ Ni . The partial molar properties could then be obtained by
i
differentiation of the polynomial expression for ∆θ mix . That is since

a f
3
θ = N θ = ∑ Ni θ i + ∆θ mix N1, N 2 , N 3
i =1

θi =
∂θ
= θi +
a
∂ ∆θ mix f
∂ Ni T , P , N j ≠i ∂ Ni T , P , N j ≠i

so that

θi − θ i =
a
∂ ∆θ mix f
∂ Ni T , P , N j ≠i

Alternately, graphical methods could be developed for finding θ i − θ i along paths where Ni is
varied, and other mole numbers are fixed (i.e., xi is varied, while the mole ratios of the other species
in the mixture are fixed.)
Since it is unlikely that enough information will be available for any mixing property to obtain
∆θ mix as an explicit function of mole fractions or species mole numbers for ternary, quaternary, etc.
mixtures, it is not surprising that there is little information on partial molar properties in such
systems.
8

8.19 (a) The Gibbs-Duhem equation is (Equation 8.2-19b)

∂ H1 ∂ H2
x1 +x =0
∂ x1 T , P 2 ∂ x1 T,P

∂ H1 ∂x ∂ H2
Now = 2 x2b1 2 = −2b1x2 , and = +2b2 x1 so that
∂ x1 T , P ∂ x1 ∂ x1 T,P

∂ H1 ∂ H2
x1
∂ x1 T , P
+ x2
∂ x1
a f
= 2 b2 − b1 x1x2 = 0 for all x1 ⇒ b1 = b2 = b
T,P

(b) lim θ i = θ i ⇒ a1 = H 1 and a2 = H 2 where H 1 and H 2 are the pure component molar
xi →1
enthalpies. Thus

H1 = H 1 + bx22 ; H2 = H 2 + bx12

and

∆ mix H = x1 ( H1 − H 1 ) + x2 ( H 2 − H 2 ) = x1bx22 + x2bx12


∆ mix H = ( x1 + x2 ) bx1 x2 = bx1 x2
8

8.20 Note: Sorry about 1 set of data being given in alcohol wt% and other in water mole %, but this is the
way the data appeared in the International Critical Tables.
(a) First will convert the data to mole fractions.

kg A × 100 kg A MWA
wt% A = ; xA =
kg A + kg W kg A MWA + kg W MWW
wt% A
⇒ xA =
wt% A + (100 − wt% A ) MWA MWW

Also, V mix = MW ρ mix where ρ mix = mixture density and MW is the mole fraction averaged
molecular weight of mixture (i.e., MW = ∑ xi MWi )
Also, V A = MWA ρ(100 wt% alcohol) and
V W = MWW ρ(0% alcohol) .
wt% xA MW V mix − ∑ xiV i ∆ mix V (cc/mol)
alcohol
[multiply by 10−3 for
m3 kmol ]
0 0 18 18.083–18.033 0
5 0.0202 18.566 18.765–18.846 –0.081
10 0.0417 19.168 19.521–19.711 –0.190
15 0.0464 19.809 20.315–20.633 –0.318
20 0.0891 20.495 21.159–21.690 –0.531
25 0.1151 21.231 22.077–22.678 –0.601
30 0.1436 22.021 23.088–23.813 –0.725
35 0.1740 22.872 24.091–25.036 –0.945
40 0.2069 22.793 25.442–26.360 –0.918
45 0.2425 24.790 26.809–27.793 –0.984
50 0.2813 25.876 28.317–29.355 –1.038
55 0.3235 27.058 29.978–31.053 –1.075
60 0.3699 28.357 31.823–32.920 –1.097
65 0.4209 29.785 33.865–34.973 –1.108
70 0.4773 31.364 36.147–37.243 –1.096
75 0.5440 33.120 38.710–39.766 –1.056
80 0.6102 35.038 41.600–42.592 –0.992
85 0.6892 37.298 44.883–45.771 –0.888
90 0.7788 39.806 48.663–49.377 –0.714
95 0.8814 42.679 53.070–53.507 –0.437
100 1.0 46. 58.280–52.280 0

b g
The ∆V mix data are plotted, and the graphical procedure of Sec. 8.6 used to find VA − V A and
bVW g
− V W . Results are given in the following table.

Next note that


⎛ per mole ⎞ Heat evolved Mole fraction
∆ mix H ⎜ ⎟= × × ( −1)
⎝ mixture ⎠ per mole ethanol of ethanol Since heat is
evolved, ∆ mix H
is negative.

Once ∆ mix H is computed, graphical procedure is used to get HA − H A b g b g


and HW − H W .
b g b g b
Table below gives VA − V A , VW − V W , HA − H A g and b H W g
− H W as a function of the
water mole fraction.

xW VW − V W VA − V A ∆ mix H HW − H W HA − H A
cc/mol kJ/mol
0 –4.5 0 0. –0.85 0
0.05 –0.0400 –0.099 +0.015
0.1 –5.0 –0.05 –0.0828 –1.15 +0.039
0.15 –0.142
0.2 –3.43 –0.42 –0.201 –1.13 +0.038
0.25 –0.251
0.3 –2.5 –0.78 –0.296 –0.85 –0.055
0.35 –0.337
0.4 –1.22 –1.04 –0.382 –0.88 –0.03
0.45 –0.416
0.5 –0.82 –1.37 –0.473 –1.02 +0.087
0.55 –0.541
0.6 –0.58 –1.67 –0.603 –1.13 +0.183
0.65 –0.674
0.7 –0.42 –2.0 –0.743 –1.175 +0.388
0.75 –0.805
0.8 –0.17 –2.86 –0.854 –1.02 –0.26
0.85 –0.873 –0.79 –1.36
0.9 –0.025 –3.50 –0.780 –0.30 –5.0
0.95 –0.491
1.0 0 –3.88 0. 0. ?
↑ ↑
8
8.21
c h
We want to evaluate HAV T , P, y − HAL (T , P, x) , where xA and yA denote the liquid and vapor
alcohol mole fractions, respectively, and the superscripts V and L designate the vapor and liquid
phases. To an excellent approximation, at the temperatures here, HAV T , P, y = H AV (T , P) . To c h
proceed further we use
H AV (T , P) − HAL (T , P, x) = H AV (T , P) − H AL (T , P) + H AL (T , P) − HAL (T , P, x )

where
A (T , P ) − H A (T , P ) = ∆ vap H
HV = 44,770 J mol (See Problem 7.13a)
L
pure
ethanol

and HAL (T , P, x) − H AL (T , P) was computed in Problem 8.20. Thus,

∆ vap H A 75 mole = 44,770 J mol − ⎡ H AL (T , P, xA = 0.75 ) − H A


L
(T , P )⎤⎦
%A

= 44,770 − ( ~ 0 ) = 44,770 J mol
∆ vap H A 50 mole = 44,770 − ( 87 ) = 44,683 J mol
%A

∆ vap H A 25 mole = 44,770 − ( ~ 0 ) = 44,770 J mol


%A

To evaluate ∆ vap H A →0 mole , care must be taken since


%A
dH L
A − H AL i becomes very large (and

negative) in this limit. To avoid serious errors, we will fit the low alcohol (high water) mole fraction
b
data with a polynomial in mole fraction, and evaluate HA − H A analytically. I used g
∆ mix H = xA xW ( A + B ( xA − xW ) ) See Eqn. (8.6-5a)
Then

HA − H A =
FG ∂ N∆ H IJ c
= xW AxW + B 3xW − 4 xW h
H ∂N K
mix 2

A T , P, N W

and
xW →1
b g
lim HA − H A = A − B . Fitting ∆ mix H data at xW = 085
. , 0.9 and 0.95, I find that
xA → 0

A = +5368
. kJ mol and +17.45 kJ/mol. Thus

HA − H A xA →0
= +5368
. − 17.45 = −12.08 kJ mol

Thus
∆H Avap = 44.77 − (−12.08) = 5685
. kJ mol
xA → 0

Clearly at moderate and high ethanol concentrations, an ethanol molecule must interact with the
water-ethanol mixture to almost the same extent as an ethanol molecule interacts with pure ethanol.
Thus its heat of vaporization from solution is about the same as from pure ethanol. However, at very
low alcohol concentrations there is a dramatic change. Presumably, water (now almost pure) forms
a more ordered structure (probably as a result of hydrogen-bonding). Thus, it seems reasonable that
at very low ethanol concentrations, each ethanol molecule is more involved in hydrogen bonding
(with water molecules) than it is in pure ethanol (with ethanol molecules).
Consequently, the heat of vaporization of ethanol from dilute alcohol solutions is greater (that is,
more energy is needed) than for the pure substance.
wt% xA MW V mix − ∑ xiV i ∆ mix V (cc/mol)
alcohol
[multiply by 10−3 for
m3 kmol ]
0 0 18 18.083–18.033 0
5 0.0202 18.566 18.765–18.846 –0.081
10 0.0417 19.168 19.521–19.711 –0.190
15 0.0464 19.809 20.315–20.633 –0.318
20 0.0891 20.495 21.159–21.690 –0.531
25 0.1151 21.231 22.077–22.678 –0.601
30 0.1436 22.021 23.088–23.813 –0.725
35 0.1740 22.872 24.091–25.036 –0.945
40 0.2069 22.793 25.442–26.360 –0.918
45 0.2425 24.790 26.809–27.793 –0.984
50 0.2813 25.876 28.317–29.355 –1.038
55 0.3235 27.058 29.978–31.053 –1.075
60 0.3699 28.357 31.823–32.920 –1.097
65 0.4209 29.785 33.865–34.973 –1.108
70 0.4773 31.364 36.147–37.243 –1.096
75 0.5440 33.120 38.710–39.766 –1.056
80 0.6102 35.038 41.600–42.592 –0.992
85 0.6892 37.298 44.883–45.771 –0.888
90 0.7788 39.806 48.663–49.377 –0.714
95 0.8814 42.679 53.070–53.507 –0.437
100 1.0 46. 58.280–52.280 0

b g
The ∆V mix data are plotted, and the graphical procedure of Sec. 8.6 used to find VA − V A and
bVW g
− V W . Results are given in the following table.

Next note that


⎛ per mole ⎞ Heat evolved Mole fraction
∆ mix H ⎜ ⎟= × × ( −1)
⎝ mixture ⎠ per mole ethanol of ethanol Since heat is
evolved, ∆ mix H
is negative.

Once ∆ mix H is computed, graphical procedure is used to get HA − H A b g b g


and HW − H W .
b g b g b
Table below gives VA − V A , VW − V W , HA − H A g and b H W g
− H W as a function of the
water mole fraction.

xW VW − V W VA − V A ∆ mix H HW − H W HA − H A
cc/mol kJ/mol
0 –4.5 0 0. –0.85 0
0.05 –0.0400 –0.099 +0.015
0.1 –5.0 –0.05 –0.0828 –1.15 +0.039
0.15 –0.142
0.2 –3.43 –0.42 –0.201 –1.13 +0.038
0.25 –0.251
0.3 –2.5 –0.78 –0.296 –0.85 –0.055
0.35 –0.337
0.4 –1.22 –1.04 –0.382 –0.88 –0.03
0.45 –0.416
0.5 –0.82 –1.37 –0.473 –1.02 +0.087
0.55 –0.541
0.6 –0.58 –1.67 –0.603 –1.13 +0.183
0.65 –0.674
0.7 –0.42 –2.0 –0.743 –1.175 +0.388
0.75 –0.805
0.8 –0.17 –2.86 –0.854 –1.02 –0.26
0.85 –0.873 –0.79 –1.36
0.9 –0.025 –3.50 –0.780 –0.30 –5.0
0.95 –0.491
1.0 0 –3.88 0. 0. ?
↑ ↑
8

a f
n
8.22 V (T , P, x) = x1b1 + x2b2 + x1x2 ∑ ai x1 − x2 i

i =0
(a) lim V (T , P, x) = V 1(T , P) = b1 ; lim V T , P, xi = V 2 (T , P) = b2
x1 →1 x2 →1
a f

(b) V1 = ( NV )T , P, N
∂ N1 2

=
LM Nx V + Nx V + a Nx fa Nx f ∑ a a Nx − Nx f OP
∂ 1 2
n
1 2
i

∂N N 1
1 1
N N
2
Q 2
i =0
i i

∂ L a N − N f OP kNx = N p n i
=
∂N N
M N V + N V + N N ∑a
1
1 1
aN + N f Q 2 2 1 2
i =0
i
1
1

2
2
i +1 i i

a aN − N fn
a ia N − N f i n i −1
=V + N ∑
aN + N f + N N ∑ aN + N f
i 1 2 i 1 2
1 2 i +1 1 2 i +1
i 1 2 i =0 1 2

− N N ∑ a (i + 1)
aN − N f
n
1 2
i
1 2
aN + N f
i =0
i
1 2
i +2

= V + x ∑ a a x − x f + x x ∑ a ia x − x f
n
i i −1
1 2 i 1 2 1 2 i 1 2
i =0

− x x ∑ a (i + 1)a x − x f
1 2 i 1 2
i

= V + x ∑ a a x − x f x − x + ix − (i + 1) x a x − x f
n
i −1
1 2 i 1 2 1 2 1 1 1 2
i =0

a f
n
V1 = V 1 + x22 ∑ ai x1 − x2 i −1
2(i + 1) x1 − 1
i =0
Similarly, by taking derivative with respect to N 2 , we obtain
V2 = V 2 − x12 ∑ ai x1 − x2 a f
i −1
2(i + 1) x2 − 1
Therefore

a f
n
V1ex = V1 − V 1 = x22 ∑ ai x1 − x2 i −1
2(i + 1) x1 − 1
i =0
and
V2ex = V2 − V 2 = − x12 ∑ ai x1 − x2 a f i −1
2(i + 1) x2 − 1

a f
n n
(c) V1ex T , P, x1 → 0 = V1ex, ∞ = ∑ ai (−1)i −1(−1) = ∑ ai (−1)i
i =0 i =0
= a0 − a1 + a2 − a3 + "

a f
n n
V2ex T , P, x2 → 0 = V2ex, ∞ = − ∑ ai (1)i −1(−1) = ∑ ai
i =0 i =0
= a0 + a1 + a2 + a3 + "
8
8.23 These derivations are the same as in Sec. 7.1. For example, starting with the closed system
equations

dN dU dV dS Q 
=0 = Q + Ws − P and = + Sgen
dt dt dt dt T

with Sgen ≥ 0 , U = ∑ NiUi and S = ∑ Ni Si . We have first N (or M) = constant.


Now for a constant volume, adiabatic system with no shaft work we have

dU
= 0 ⇒ constant ; also V = constant
dt
and
dS 
= Sgen ≥ 0 ⇒ S can only increase at constant N, V and U
dt
⇒ S = maximum at equilibrium at constant N, V, U.

Similarly with Ws = 0 for an isothermal system (constant N, T, and V) at constant volume we have

dU dS Q  dS
= Q and = + Sgen or Q = T − TSgen
dt dt T dt

so

dU dS d (U − TS ) dA
=T − TSgen ⇒ = = −TSgen
dt dt dt dt
⇒ A = minimum at equilibrium at constant N , T , and V .

Similarly, one can show that G = minimum at constant N, T, and P.


8

8.24 a
M = M x1, x2 ,..., x N f
Mi =
∂( N M )
= M+N
FG ∂ M IJ
dNi T , P , N j ≠i
H∂ N K i T , P , N j ≠i

but

FG ∂ M IJ F∂ MI FG ∂ x IJ
H∂ N K i T , P , N j ≠i
=∑
j
GH ∂ x JK j T , P, N j≠k
H∂ N K
j

i T , P , N j ≠i

Now

R N
N I | −
xj
xj =
Nj ∂
=
F ∂
G J=S N j
j
2
=−
N
; if j ≠ i
∂N H N K |1 − N
; xj
N ∂ Ni 1 − xi
= ; if j = i
TN Ni i
2
N

Therefore

FG ∂ M IJ F∂ MI x F∂ MI
−G J F 1− x I
H∂ N K i T , P , N j ≠i
= −∑ GH ∂ x JK
j ≠i N Hj∂x K H NK
T , P, N k ≠ j
j

i T , P , N k ≠i
i

1 F∂ MI
= G J − ∑G
F∂ MI x
NH∂x K i H∂x KJ N
T , P , x j ≠i j ≠i j T , P, xk ≠ j
j

⇒ M = M +G
F ∂ M IJ − ∑ x FG ∂ M IJ
i
H∂x K H∂x K i T , P , x j ≠i
j
j T , P, xk ≠ j
8
8.25 (See Section 11.2)
Since the stable equilibrium state of the mixture for some range of compositions is as two liquid
phases, rather than a single homogeneous phase, the Gibbs free energy of the homogeneous mixture
must be greater than a linear combination of the Gibbs free energies of mixtures of compositions on
either side of the actual mixture composition. That is, the Gibbs free energy of the mixture must be
as indicated in Figure 1 below.

Now, the Gibbs free energy of the mixture is


G = N1IG1I + N 2I G2I + N1II G1II + N 2II G2II
with the restrictions that
N1I + N1II = constant = N1o
and N 2I + N 2II = constant = N 2o . Thus, we have that
c
G = N1IG1I + N 2I G2I + N1o − N1I G1II + N 2o − N 2II G2IIh c h
At equilibrium
FG ∂ G IJ = 0 ⇒ G1I = G1II
H∂ N K I
1 T , P , N 2I

(where, in evaluating each of these derivations, we have used the Gibbs-Duhem equation) and

FG ∂ G IJ = 0 ⇒ G2I = G2II
H∂ N K I
2 T , P , N1I

a
However, G1I = ∂ G ∂ N1 f is a tangent to the G versus N1 curve. Thus, the equilibrium
requirement that =
G1I G1II
requires that the compositions of the two equilibrium phases are at the
intersections of each lobe of the G versus N1 curve with the single tangent line to both curves. See
Fig. 1. [Note: See Problem 7.28]
The limit of stability of a single phase is found from the condition that d 2G = 0 . Here that implies
FG∂ 2G IJ = 0 Now, G = N1G1 + N 2G2
H K
∂ N12 T , P, N
2

FG ∂ G IJ = G1 + N1
FG ∂ G IJ + N2
FG ∂ G IJ = G1
H∂ N K H∂ N K H∂ N K
1 2

1 T , P, N 2 1 T , P, N 2 1 T , P, N 2
Since the last two terms sum to zero by the Gibbs-Duhem equation. Thus
∂ 2G FG
=
∂ G1 IJ FG IJ
∂ N1 T , P, N
2
H K
∂ N1 T , P, N
2
H K 2

Thus, the limits of stability of a single phase are the inflection points on the G vs N1 (or G vs x1 )
curve and the local maximum and minimum on the G1 vs. N1 (or G1 vs. x1 ) curves. (See Figure
2.) The region between each coexisting phase composition and the adjacent inflection point
represents a metastable region. This is illustrated in Figure 2.
8

8.26 (a) f a
H = xB H B + 1 − xB H EA
+ x a1 − x f 1418 − 482.4a1 − 2 x f + 187.4a1 − 2 x f
B B B B
3

but ∆ mix H = H − xB H B − (1 − xB ) H EA
∆ mix H = xB (1 − xB ) ⎡1418 − 482.4 (1 − 2 xB ) + 187.4 (1 − 2 xB ) ⎤
3
⎣ ⎦
From Equation (8.6-9b)
∂ ( ∆ mix H )
∆ mix H − xB = H EA − H EA ; also
∂ xB
∂ ( ∆ mix H )
∆ mix H + (1 − xB ) = HB − H B
∂ xB
Now
∂ ( ∆ mix H )
= (1 − 2 xB ) ⎡1418 − 482.4 (1 − 2 xB ) + 187.4 (1 − 2 xB ) ⎤
3
∂ xB ⎣ ⎦

+2 xB (1 − xB ) ⎡ 482.4 − 562.2 (1 − 2 xB ) ⎤
2
⎣ ⎦
Therefore,
bH EA g a
− H EA = x B2 1418 − 482.4 1 − 2 x B + 187.4 1 − 2 x B f a f
3

a f
+2 xB2 1 − xB 482.4 − 562.2 1 − 2 xB a f
2

and
bH B g a
− H B = 1 − xB f 2
a f
1418 − 482.4 1 − 2 xB + 187.4 1 − 2 xB a f
3

a f
+2 xB 1 − xB 482.4 − 562.2 1 − 2 xB a f
2

(b) At xB = 05
.

bH g FH 21IK 1418 − 2FH 21 IK FH 21 IK (482.4) = 2339. J mol


2 2
EA − H EA =

bH g = FH 21 IK 1418 − 2FH 21IK FH 21 IK (482.4) = 4751. J mol


2 2
B − HB

(c) Mixing process

System = 1 mole of each initial mixture (2 moles total)


Difference form of mass balance: N f = N1, i + N 2, i = 1 + 1 = 2 mol
Balance on benzene (species balance):
xB, f N f = xB,1i N1i + xB,2i N 2i = 0.25 + 0.75
0.25 + 0.75
xB, f = . (obviously!)
= 05
2

Energy balance: N f U f = N1,i U 1,i + N 2,i U 2,i + Q


For liquid solutions, U ≈ H . Consequently,

a
Q = 2 H f − H 1, i − H 2, i = 2 H xB = 05 f a f a
. − H x B = 0.25 − H x B = 0.75 f
but

∆ mix H = H mix − ⎡⎣ xB H B + (1 − xB ) H EA ⎤⎦
⇒ H mix = ∆ mix H + xB H B + (1 − xB ) H EA
polynomial given ↓ ↓
in problem statement pure component
enthalpies

Q = 2 ⎡⎣ ∆ mix H ( xB = 0.5 ) + 0.5 H B + 0.5 H EA ⎤⎦


− ⎣⎡ ∆ mix H ( xB = 0.25 ) + 0.25H B + 0.75 H EA ⎦⎤
− ⎡⎣ ∆ mix H ( xB = 0.75 ) + 0.75H B + 0.25 H EA ⎤⎦
= 2∆ mix H ( xB = 0.5 ) − ∆ mix H ( xB = 0.25 ) − ∆ mix H ( xB = 0.75 )
= 2 × 354.5 − 225 − 306.7 = 177.3 J
8

RS kJ UV × mole fraction = QRS kJ UV


8.27 Q
T mol n - propanol W of n - propanol T mol of solution W
Thus, we obtain the following

Mole % H 2O 0 5 10 15 20
Q kJ/mol propanol 0 +0.042 0.084 0.121 0.159
∆ mix H kJ/mol solution 0 0.040 0.076 0.103 0.127

Mole % H 2O 25 30 35 40 45
Q kJ/mol propanol 0.197 0.230 0.243 0.243 0.209
∆ mix H kJ/mol solution 0.148 0.161 0.158 0.146 0.115

Mole % H 2O 50 55 60 65 70 75
Q 0.167 0.084 –0.038 –0.201 –0.431 –0.778
∆ mix H 0.084 0.038 –0.015 –0.070 –0.129 –0.195

Mole % H 2O 80 85 90 95 100
Q –1.335 –2.264 –4.110 –7.985 0
∆ mix H –0.267 –0.340 –0.411 –0.399 0

∆ mix H is plotted below, and H1 − H 1 and H2 − H 2 are evaluated using the graphical procedure of
Sec. 8.6.
Dashed lines are sample tangent lines. [Note: Results in table and figure below gotten using a much bigger
graph.]
xH 2 O HnP − H nP HH 2 O − H H 2 O
kJ/mol
0.1 0.015 0.642
0.2 0.042 0.470
0.3 0.118 0.260
0.4 0.335 –0.130
0.5 0.510 –0.340
0.6 0.610 –0.425
0.7 0.732 –0.500
0.8 0.843 –0.540
0.9 0.940 –0.565
8

8.28 (a) ∆ mix H = x1 x2 ⎡⎣ A + B ( x1 − x2 ) ⎤⎦ = x1 (1 − x1 ) ⎡⎣ A + B ( 2 x1 − 1) ⎤⎦

c h a
= x1 − x12 A + B 2 x1 − 1 f
⎛ ∂ mix ∆ H ⎞
⎜ ⎟ = ( x2 − x1 ) ⎡⎣ A + B ( x1 − x2 ) ⎤⎦ + 2 Bx1 x2
⎝ ∂ x1 ⎠

⎛ ∂ ∆ mix H ⎞
∆ mix H − x1 ⎜ ⎟ = x1 x2 ⎡⎣ A + B ( x1 − x2 ) ⎤⎦
⎝ ∂ x1 ⎠
− x1 ( x2 − x1 ) A − x1 ( x2 − x1 )( x1 − x2 ) B − 2 Bx12 x2
= x12 ⎡⎣ A + B ( 4 x1 − 3) ⎤⎦ = H 2 − H 2

1 1
at x1 = 05
. , H2 − H 2 = A + B(2 − 3) = A − B
4 4

⎛ ∂ ∆ mix H ⎞ ⎛ ∂ ∆ mix H ⎞
∆ mix H + x2 ⎜ ⎟ = ∆ mix H − x2 ⎜ ⎟
⎝ ∂ x1 ⎠ ⎝ ∂ x2 ⎠
= x1 x2 ⎡⎣ A + B ( x1 − x2 ) ⎤⎦
+ x2 ( x2 − x1 ) ⎡⎣ A + B ( x1 − x2 ) ⎤⎦ + 2 Bx1 x22
= x22 ⎣⎡ A + B ( 3 − 4 x2 ) ⎦⎤ = H1 − H 1

1 LM F 1 I OP = 1
. , H1 − H 1 =
at x1 = 05
4 H
A + B 3− 4
N 2 KQ 4 A+ B

at 300 K, A = −12,974 + 51505


. × 300 = 2477.5
B = 8782.8 − 34129
. × 300 = −14559
.

1
⇒ H2 − H 2 = 2477.5 + 14559
. = 9834
. J mol
4
1
H1 − H 1 = 2477.5 − 14559 . = 2554
. J mol
4
∂ ∂A ∂B
(b) CP ,i − CP,i =
∂T
b
Hi − H i ; g
∂T
= 51505
. ;
∂T
= −34129
.
⇒ CP,2 − CP,2 0.5 =

∂T
b
H2 − H 2 = g FGH
1 ∂A ∂B

4 ∂T ∂T
1
= (51505
4
. . )
+ 34129
IJ
K
= 21409
. J mol K
1
CP,1 − CP,1 0.5 = (51505
. . ) = 4.344 J mol K
− 34129
4

(c) Overall mass balance: 0 = N 1 + N 2 + N 3


n-octanol mass balance: 0 = x1 N 1 + x2 N 2 + x3 N 3
Also (problem information): N = 2 N 1 2
Basis of calculation: N 2 = 1 ⇒ N 3 = −3; N 1 = 2

0.2 × 2 + 0.9 × 1
x3 = = 0.4333
3

Energy balance: 0 = N 1( H )1 + N 2 ( H )2 + N 3( H )3 + Q

∆ mix H = H − x1 H − x2 H 2

( H )1 = ∆ mix H − x1 H 1 − x2 H 2

⇒ −Q = N 1 ( ∆ mix H )1 + N 2 ( ∆ mix H )2 − ( N 1 + N 2 ) ( ∆ mix H )3


= 2 ( ∆ mix H )1 + 1( ∆ mix H )2 − 3 ( ∆ mix H )3

∆ mix H = x1 x2 ⎡⎣ 2477.5 − 1455.9 ( x1 − x2 ) ⎤⎦


( ∆ mix H )1 = 0.2 × 0.8 ⎣⎡ 2477.5 − 1455.9 ( 0.2 − 0.8)⎦⎤ = 536.17 J mol
( ∆ mix H )2 = 0.9 × 0.1 ⎡⎣2477.5 − 1455.9 ( 0.9 − 0.1)⎤⎦ = 118.15 J mol
( ∆ mix H )3 = 0.433 × 0.567 ⎡⎣ 2477.5 − 1455.9 ( 0.433 − 0.567 )⎤⎦ = 655.79 J mol

−Q = 2 × 53617
. + 1 × 11815
. − 3 × 65579
. = −77689
.
⇒ Q = 77689
. J 3 mol solution
⇒ Heat must be added

Q = 258.96 J mol solution


8

8.29 The mass balance of the acetic acid-pyridine streams at steady-state is


kmol
0 = N P + N A + N mix ⇒ − N mix = N P + N A ⇒ N mix = -2
min

The energy balance is


0 = N P H P + N A H A + N mix H mix + Q
a f
= N P H P + N A H A + N mix xP H P + xA H A + ∆ H mix + Q
a f a f
= 1⋅ H P − H P + 1⋅ H A − H A − 2 ⋅ ∆ H mix xP = 05 a
. +Q f
so
a
Q = 2 ⋅ ∆ H mix xP = 05
. f
Now from the table

a f
∆ H mix xP = 0.4786 = −4833 J / mol and
∆ H mix ax P = 05029
. f = −4765 J / mol
By interpolation
a
∆ H mix xP = 05 f
. ≈ −4773 J / mol

kmol J 1 kJ 1000 mol


Q=2 ⋅ (−4773) ⋅ ⋅
min mol 1000 J kmol
kJ
= −9546
min

Negative sign means that heat must be removed (or cooling supplied) to keep the process at a
constant temperature. Since ethylene glycol has a value of CP = 2.8 kJ/kg K. From an energy
balance we have that
kJ kJ
2.8 × 20 K × M = 9546
kg ⋅ K min
Therefore
kJ
9546 kg ethylene glycol
M= min = 1705
.
kJ min
2.8 × 20 K
kg ⋅ K
Solutions to Chemical and Engineering Thermodynamics, 3e

8.29 The mass balance of the acetic acid-pyridine streams at steady-state is


kmol
0 = N P + N A + N mix ⇒ − N mix = N P + N A ⇒ N mix = -2
min

The energy balance is


0 = N P H P + N A H A + N mix H mix + Q
a
= N P H P + N A H A + N mix xP H P + xA H A + ∆ H mix + Q f
a f a f
= 1⋅ H P − H P + 1⋅ H A − H A − 2 ⋅ ∆ H mix xP = 05
. +Q a f
so
Q = 2 ⋅ ∆ H mix xP = 05
. a f
Now from the table

a f
∆ H mix xP = 0.4786 = −4833 J / mol and
∆ H mix ax P = 05029
. f = −4765 J / mol
By interpolation
a
∆ H mix xP = 05 f
. ≈ −4773 J / mol

kmol J 1 kJ 1000 mol


Q=2 ⋅ (−4773) ⋅ ⋅
min mol 1000 J kmol
kJ
= −9546
min

Negative sign means that heat must be removed (or cooling supplied) to keep
the process at a constant temperature. Since ethylene glycol has a value of CP =
2.8 kJ/kg K. From an energy balance we have that
kJ kJ
2.8 × 20 K × M = 9546
kg ⋅ K min
Therefore
kJ
9546 kg ethylene glycol
M= min = 1705
.
kJ min
2.8 × 20 K
kg ⋅ K

8.30 (also available as an Mathcad worksheet)

x 0 H0 0 x1 0.0371 H1 1006 x2 0.0716 H2 1851


0

x3 0.1032 H3 2516 x4 0.1340 H4 3035 x5 0.1625 H5 3427

x6 0.1896 H6 3765 x7 0.2190 H7 4043 x8 0.2494 H8 4271


x9 0.2760 H9 4440 x10 0.3006 H10 4571 x11 0.3234 H11 4676
x12 0.3461 H12 4760 x13 0.3671 H13 4819 x14 0.3874 H14 4863
x15 0.3991 H15 4832 x16 0.4076 H16 4880 x17 0.4235 H17 4857
x18 0.4500 H18 4855 x19 0.4786 H19 4833 x20 0.5029 H20 4765
Solutions to Chemical and Engineering Thermodynamics, 3e

18 18 19 19 20 20
x21 0.5307 H21 4669 x22 0.5621 H22 4496 x23 0.5968 H23 4253
x24 0.6372 H24 3920 x25 0.6747 H25 3547 x26 0.7138 H26 3160
x27 0.7578 H27 2702 x28 0.8083 H28 2152 x29 0.8654 H29 1524

x30 0.9277 H30 806 x31 1.0 H31 0

i 0 , 1 .. 31

2000
H
i
4000

6000
0 0.5 1
x
i
One-constant Margules fit

x. ( 1
4
f( x) x) S linfit( x, H , f ) S = 1.961 10

Two-constant Margules fit

x. ( 1 x) 4
f( x) SS linfit( x, H , f ) 1.893 10
x. ( 1 x) ( 2 . x 1 )
. SS =
3
8.068 10

Three-constant Margules fit

x. ( 1 x) 4
1.88 10
f( x) x. ( 1 x) . ( 2 . x 1 ) SS linfit( x, H , f )
SS = 7.983 10
3
x. ( 1 x) . ( 2 . x 1 )
2
3
1.143 10

1.88 . 10 . ( x. ( 1 7.983 . 10 . ( x. ( 1 x) . ( 2 . x 1 ) ) 1.143 . 10 . x. ( 1 x) . ( 2 . x 1 )


4 3 3 2
HH( x) x) )

d
dHH( x) HH( x)
dx

∆ H1( x) HH( x) (1 x) . dHH( x) ∆ H2( x) HH( x) x. dHH( x)


Solutions to Chemical and Engineering Thermodynamics, 3e
Solutions to Chemical and Engineering Thermodynamics, 3e
Solutions to Chemical and Engineering Thermodynamics, 3e

HH xi ∆ H1 xi ∆ H2 xi
0 2.793 . 10 0
4
970.622 64.168
2.45 . 10
4
1.76 . 10 230.38
3
2.16 . 10
4
462.512
2.393 . 10
3
1.917 . 10
4
753.807
2.931 . 10
3
1.7 . 10 1.074 . 10
4 3

3.363 . 10
3
1.516 . 10 1.418 . 10
4 3

3.718 . 10
3
1.355 . 10 1.828 . 10
4 3

4.045 . 10
3
1.195 . 10 2.288 . 10
4 3

4.322 . 10
3
1.044 . 10 2.716 . 10
4 3

4.517 . 10
3
9.243 . 10 3.127 . 10
3 3

4.66 . 10
3
8.227 . 10 3.52 . 10
3 3

4.762 . 10
3
7.36 . 10 3.92 . 10
3 3

4.835 . 10
3
6.565 . 10 4.296 . 10
3 3

4.88 . 10
3
5.886 . 10 4.663 . 10
3 3

4.902 . 10
3
5.281 . 10 4.875 . 10
3 3

4.906 . 10
3
4.953 . 10 5.03 . 10
3 3

4.905 . 10
3
4.724 . 10 5.319 . 10
3 3

4.895 . 10
3
4.317 . 10 5.8 . 10
3 3

4.853 . 10
3
3.696 . 10 6.315 . 10
3 3

4.777 . 10
3
3.102 . 10 6.747 . 10
3 3

4.688 . 10
3
2.653 . 10 7.231 . 10
3 3

4.561 . 10
3
2.2 . 10 7.763 . 10
3 3

4.388 . 10
3
1.759 . 10 8.327 . 10
3 3

4.162 . 10
3
1.349 . 10 8.946 . 10
3 3

3.86 . 10
3
9.48 . 10
963.67 3

3.545 . 10
3
683.156
9.989 . 10
3

3.186 . 10 458.239
3
1.05 . 10
4
274.249
2.751 . 10
3
1.1 . 10
4
135.442
2.218 . 10
3
48.028 1.144 . 10
4

1.581 . 10
3 8.49
1.177 . 10
4
859.046 0
1.196 . 10
4
0
8

8.31 Starting by writing the equation for the formation of each of the six compounds
present from their elements

C + 4H = CH4 (1)
2O = O2 (2)
C + 2O = CO2 (3)
C + O = CO (4)
2H = H2 (5)
2H + O = H2O (6)

Now using eqn. (2) to eliminate the oxygen atom, and eqn. (5) to eliminate the hydrogen atom. We
obtain
C + 2H2 = CH4
C + O2 = CO2
C + 1/2O2 = CO
H2 + 1/2O2 = H2O

Thus from the Denbigh method, we find there are four independent reactions. One such set is
listed above.
8

8.32 (a) N 2 ( g ) + H 2 ( g ) = 2NH 3( g ) ∆ rxn H = 2 × ( −46.1) = −92.2 kJ mol


∆ rxn G = 2 × ( −16.5 ) = −33.0 kJ mol
(b) C3H8 ( g ) = C2 H 4 ( g ) + CH 4 ( g )
∆ rxn H = 52.5 − 74.5 − (−104.7) = 82.7 kJ mol
∆ rxn G = 68.5 − 50.5 − (−24.3) = 42.3 kJ mol
(c) CaCO3( s) = CaO(s) + CO2 ( g )
∆ rxn H = −635.1 − 393.5 − (−1206.9) = 178.3 kJ mol
∆ rxn G = −604.0 − 394.4 − (−1128.8) = 130.4 kJ mol
(d) 4CO( g ) + 8H 2 ( g ) = 3CH 4 ( g ) + CO2 ( g ) + 2H 2O( g )
∆ rxn H = 3 × ( −74.5 ) + ( −393.5 ) + 2 × ( −241.8 ) − 4 × ( −110.5 ) = −658.6 kJ mol
∆ rxn G = 3 × ( −50.5 ) + ( −394.4 ) + 2 × ( −228.6 ) − 4 × ( −137.2 ) = −454.3 kJ mol
8

8.33 Buckmasterfullerene C60(BF) + 60 O2 = 60 CO2 for which


∆combH = 26,033 kJ/mol= 26,033 kJ/60 mols C

Graphite 60C + 60 O2 = 60CO2 for which ∆combH = 60×393.513= 23,611 kJ/60


mols C

For these reactions sice only carbon, carbon dioxide and oxygen are involved,
∆fH = -∆combH

Subtracting the first chemical reaction above from the second yields
60C > C60(BF) => -26033 – (-23611) = -2422 kJ/mol C60(BF)
Solutions to Chemical and Engineering Thermodynamics, 3e

8.31 Starting by writing the equation for the formation of each of the six compounds
present from their elements

C + 4H = CH4 (1)
2O = O2 (2)
C + 2O = CO2 (3)
C + O = CO (4)
2H = H2 (5)
2H + O = H2O (6)

Now using eqn. (2) to eliminate the oxygen atom, and eqn. (5) to eliminate the
hydrogen atom. We obtain
C + 2H2 = CH4
C + O2 = CO2
C + 1/2O2 = CO
H2 + 1/2O2 = H2O

Thus from the Denbigh method, we find there are four independent reactions.
One such set is listed above.

8.32 (a) N 2 ( g ) + H 2 ( g ) = 2NH 3( g ) ∆ rxn H = 2 × ( −46.1) = −92.2 kJ mol


∆ rxn G = 2 × ( −16.5 ) = −33.0 kJ mol
(b) C3H8 ( g ) = C2 H 4 ( g ) + CH 4 ( g )
∆ rxn H = 52.5 − 74.5 − (−104.7) = 82.7 kJ mol
∆ rxn G = 68.5 − 50.5 − (−24.3) = 42.3 kJ mol
(c) CaCO3( s) = CaO(s) + CO2 ( g )
∆ rxn H = −635.1 − 393.5 − (−1206.9) = 178.3 kJ mol
∆ rxn G = −604.0 − 394.4 − (−1128.8) = 130.4 kJ mol
(d) 4CO( g ) + 8H 2 ( g ) = 3CH 4 ( g ) + CO2 ( g ) + 2H 2O( g )
∆ rxn H = 3 × ( −74.5 ) + ( −393.5 ) + 2 × ( −241.8 ) − 4 × ( −110.5 ) = −658.6 kJ mol
∆ rxn G = 3 × ( −50.5 ) + ( −394.4 ) + 2 × ( −228.6 ) − 4 × ( −137.2 ) = −454.3 kJ mol

8.33 Buckmasterfullerene C60(BF) + 60 O2 = 60 CO2 for which


∆combH = 26,033 kJ/mol= 26,033 kJ/60 mols C

Graphite 60C + 60 O2 = 60CO2 for which ∆combH = 60×393.513= 23,611 kJ/60


mols C

For these reactions sice only carbon, carbon dioxide and oxygen are involved,
∆fH = -∆combH

Subtracting the first chemical reaction above from the second yields
60C > C60(BF) => -26033 – (-23611) = -2422 kJ/mol C60(BF)

8.34 (also available as an Mathcad worksheet)

x0 0 H0 0 x1 0.0120 H1 68.8 x2 0.0183 H2 101.3

x3 0.0340 H3 179.1 x4 0.0482 H4 244.4 x5 0.0736 H5 344.6

x6 0.1075 H6 451.1 x7 0.1709 H7 565.3 x8 0.1919 H8 581.0


x9 0.2301 H9 585.0 x10 0.2636 H10 566.1 x11 0.2681 H11 561.9
x12 0.2721 H12 557.8 x13 0.3073 H13 519.6 x14 0.3221 H14 508.0
Solutions to Chemical and Engineering Thermodynamics, 3e

12 12 13 13 14 14
x15 0.3486 H15 468.5 x16 0.3720 H16 424.4 x17 0.3983 H17 369.1
x18 0.4604 H18 197.1 x19 0.4854 H19 135.4 x20 0.5137 H20 66.1
x21 0.5391 H21 1.9 x22 0.5858 H22 117.1 x23 0.6172 H23 186.5
x24 0.6547 H24 266.9 x25 0.7041 H25 360.3 x26 0.7519 H26 436.6
x27 0.7772 H27 470.5 x28 0.7995 H28 495.9 x29 0.8239 H29 510.0
x30 0.8520 H30 515.8 x31 0.8784 H31 505.3 x32 0.8963 H32 486.0
x33 0.9279 H33 420.5 x34 0.9532 H34 329.2 x35 0.9778 H35 184.7

x36 0.9860 H36 123.3 x37 0.9971 H37 25.1 x38 1.0 H38 0.0

i 0 , 1 .. 38

1000

500

H 0
i

500

1000
0 0.5 1
x
i
One-constant Margules fit
f( x) x. ( 1 x) SS linfit( x, H , f )
SS = 528.45491
HH( x) SS0 . x. ( 1 x)

d
dHH( x) HH( x)
dx

PH1( x) HH( x) (1 x) . dHH( x) PH2( x) HH( x) x. dHH( x)


Solutions to Chemical and Engineering Thermodynamics, 3e

Two-constant Margules fit

x. ( 1 x) 337.24041
f ( x) SS linfit( x, H , f ) SS =
x. ( 1 x) . ( 2 . x 1 ) 5707.44046

HH( x) x. ( 1 x) . SS0 SS1 . ( 2 . x 1 ) d


dHH( x) HH( x)
dx

PH1( x) HH( x) (1 x) . dHH( x) PH2( x) HH( x) x. dHH( x)


Solutions to Chemical and Engineering Thermodynamics, 3e

Three-constant Margules fit


x. ( 1 x)
488.57112
f ( x) x. ( 1 x) . ( 2 . x 1 ) SS linfit( x, H , f )
SS = 5672.45617
x. ( 1 x) . ( 2 . x 1 )
2
970.2807

SS0 . ( x. ( 1 SS1 . ( x. ( 1 x) . ( 2 . x 1 ) ) SS2 . x. ( 1 x) . ( 2 x 1 )


2
HH( x) x) )

d
dHH( x) HH( x)
dx PH1( x) HH( x) (1 x) . dHH( x) PH2( x) HH( x) x. dHH( x)
Solutions to Chemical and Engineering Thermodynamics, 3e

Four-constant Margules fit


x. ( 1 x)
x. ( 1 x) . ( 2 . x 1 )
f ( x)
x. ( 1 x) . ( 2 . x 1 )
2 SS linfit( x, H , f )

x. ( 1 x) . ( 2 . x 1 )
3

x. ( 1 x) . SS0 SS1 . ( 2 . x 1 ) SS2 . ( 2 . x 1 ) SS3 . ( 2 . x 1 )


2 3
HH( x)

d
dHH( x) HH( x)
dx PH1( x) HH( x) (1 x) . dHH( x) PH2( x) HH( x) x. dHH( x)
Solutions to Chemical and Engineering Thermodynamics, 3e
Solutions to Chemical and Engineering Thermodynamics, 3e

HH x PH1 xi PH2 xi
i
6620.61619 0
0
6037.1938 3.48837
75.89284
5746.57438 7.95429
112.97104
5066.79383 26.13746
197.51978
4503.47003 50.23256
264.87861
3607.09619 108.09291
365.61955
2606.80402 207.10545
465.07305
1219.73582 428.17388
563.45181
871.2077 505.3063
575.52278
347.75299 644.75104
576.41179
14.29521 763.09798
558.17713
57.06648 778.58558
554.54726
94.0214 792.26028
551.10303
379.84612 908.52549
512.60889
481.19324 955.04467
492.43244
638.98059 1034.5447
451.15379
756.15737 1100.47128
409.80542
866.44984 1169.46453
358.55983
1052.07298 1307.91205
221.37494
1101.2544 1351.95197
161.16559
1140.97708 1391.46839
90.55115
1162.73326 1415.53852
25.5922
1169.06642 1422.80738
95.51229
1148.81031 1391.88005
176.23404
1098.66681 1303.49667
269.19976
990.04823 1070.73515
380.26243
841.94903 670.81348
466.63265
748.83482 367.73066
500.06403
660.0669 36.47376
520.4105
557.74661 405.60256
530.95404
436.6149 1033.96809
525.02317
323.9931 1759.05242
498.49631
250.96992 2335.16915
467.10138
135.3294 3542.17012
380.96262
62.11408 4706.43722
279.46841
15.16587 6030.57744
148.70801
6.1957 6517.96343
97.36045
0.27562 7216.85219
21.20369
0 7407.10877
0
8
8.35 Assume reactor operates in steady state. Then the mass balance is

0 = Ni ,in − Ni ,out + ν i X
or
Ni ,in = + Ni ,out − ν i X
and the energy balance is
0= ∑i
Ni ,in H i ,in − ∑
Ni ,out H i ,out +Q
i

or
Q=− ∑ N H +∑ N H
i
i , in i , in
i
i , out i , out

= −∑ ( N − ν X ) H +∑ N H
i , out i i , in i , out i , out
i i

= ∑ N bH − H g + ∆ H (T ) X
i , out i , out i ,in rxn in
i

Using a Mathcad worksheet, the heats of formation and heat capacities in the
appendices of the textbook, we find that 59.93 kJ must be supplied per mole of N2
entering reactor. See Mathcad worksheet for this problem.
Solutions to Chemical and Engineering Thermodynamics, 3e

8.35 Assume reactor operates in steady state. Then the mass balance is

0 = Ni ,in − Ni ,out + ν i X
or
Ni ,in = + Ni ,out − ν i X
and the energy balance is
0= ∑
Ni ,in H i ,in −
i
Ni ,out H i ,out +Q ∑
i

or
Q=− ∑ N H +∑ N H
i
i , in i , in
i
i , out i , out

= −∑ ( N − ν X ) H +∑ N H
i , out i i , in i , out i , out
i i

= ∑ N bH − H g + ∆ H (T ) X
i , out i , out i ,in rxn in
i

Using a Mathcad worksheet, the heats of formation and heat capacities in the
appendices of the textbook, we find that 59.93 kJ must be supplied per mole of N2
entering reactor. See Mathcad worksheet for this problem.

8.36 (also available as an Mathcad worksheet)

x25 0 H250 0 x500 0 H500 0 Nomenclature


0
H25 = enthalpy at 25C
x251 0.027 H251 223.16 x501 0.031 H501 76.20
H50 = enthalpy at 50C
x252 0.034 H252 290.15 x502 0.043 H502 121.84 HH25 and HH50 are
correlated enthalpies.
x253 0.054 H253 329.50 x503 0.082 H503 97.55
∆ H125 = difference
x254 0.094 H254 384.25 x504 0.098 H504 52.75 between partial molar
and pure component
x255 0.153 H255 275.07 x505 0.206 H505 125.60
enthalpies of species
x256 0.262 H256 103.41 x506 0.369 H506 370.53 1 at 25C,
etc.
x257 0.295 H257 81.22 x507 0.466 H507 435.43
x258 0.349 H258 11.35 x508 0.587 H508 473.11
x259 0.533 H259 133.98 x509 0.707 H509 460.55

x2510 0.602 H2510 168.31 x5010 0.872 H5010 238.23

x2511 0.739 H2511 177.94 x5011 0.9999 H5011 0.0

x2512 1.0 H2512 0.0 x5012 1.0 H5012 0.0


i 0 .. 12
Solutions to Chemical and Engineering Thermodynamics, 3e

Three-constant Margules fit x. ( 1 x)


f( x) x. ( 1 x) . ( 2 . x 1 )

x. ( 1 x) . ( 2 . x 1 )
2

851.268
s25 linfit( x25 , H25 , f )
3
s25 = 1.543 10
3
5.027 10
3
1.95 10
s50 linfit( x50 , H50 , f )
s50 = 1.443 10
3

3
2.099 10

s250 . x. ( 1 s251 . x. ( 1 x) . ( 2 . x 1 ) s252 . x. ( 1 x) . ( 2 . x 1 )


2
HH25( x) x)

s500 . x. ( 1 s501 . x. ( 1 x) . ( 2 . x 1 ) s502 . x. ( 1 x) . ( 2 . x 1 )


2
HH50( x) x)
Solutions to Chemical and Engineering Thermodynamics, 3e

d d
dHH25 ( x ) HH25 ( x ) dHH50 ( x ) HH50 ( x )
dx dx

∆ H150( x50 ) HH50( x50 ) (1 x50 ) . dHH50( x50 )

∆ H125( x25 ) HH25( x25 ) (1 x25 ) . dHH25( x25 )

∆ H250( x50 ) HH50( x50 ) x50 . dHH50( x50 )

∆ H225( x25 ) HH25( x25 ) x25 . dHH25( x25 )


Solutions to Chemical and Engineering Thermodynamics, 3e

8.37 (also available as an Mathcad worksheet)

x0 0.2108 H0 738 x1 0.2834 H1 900 x2 0.3023 H2 933

x3 0.4285 H3 1083 x4 0.4498 H4 1097 x5 0.5504 H5 1095

x6 0.5562 H6 1096 x7 0.6001 H7 1061 x8 0.6739 H8 976


x9 0.7725 H9 780 x10 0.8309 H10 622

i 0 .. 10
Hi
Hxxi
xi . 1 xi

600 4420

800 4430
H Hxx
i i
1000 4440

1200 4450
0.2 0.4 0.6 0.8 1 0.2 0.4 0.6 0.8 1
x x
i i

4430 . 1 4430 . xi
2 2
∆ H1i xi ∆ H2i
Hxxi
∆ H1i ∆ H2i
4.436 . 10
3
2.759 . 10 196.854
3

4.432 . 10
355.798 3
2.275 . 10
3

4.424 . 10
404.837 3
2.156 . 10
3
813.402
4.422 . 10
3
1.447 . 10
3 896.278
4.433 . 10
3
1.341 . 10 1.342 . 10
3 3

4.425 . 10
3
1.37 . 10
895.481 3

4.44 . 10
3
872.526
1.595 . 10
3

4.421 . 10
708.446 3
2.012 . 10
3
471.092
4.441 . 10
3
2.644 . 10
3
229.28
4.438 . 10
3
126.675 3.058 . 10
3

4.427 . 10
3
Solutions to Chemical and Engineering Thermodynamics, 3e

8.37 (also available as an Mathcad worksheet)

x0 0.2108 H0 738 x1 0.2834 H1 900 x2 0.3023 H2 933

x3 0.4285 H3 1083 x4 0.4498 H4 1097 x5 0.5504 H5 1095

x6 0.5562 H6 1096 x7 0.6001 H7 1061 x8 0.6739 H8 976


x9 0.7725 H9 780 x10 0.8309 H10 622

i 0 .. 10
Hi
Hxxi
xi . 1 xi

600 4420

800 4430
H Hxx
i i
1000 4440

1200 4450
0.2 0.4 0.6 0.8 1 0.2 0.4 0.6 0.8 1
x x
i i

4430 . 1 4430 . xi
2 2
∆ H1i xi ∆ H2i
Hxxi
∆ H1i ∆ H2i
4.436 . 10
3
2.759 . 10 196.854
3

4.432 . 10
355.798 3
2.275 . 10
3

4.424 . 10
404.837 3
2.156 . 10
3
813.402
4.422 . 10
3
1.447 . 10
3 896.278
4.433 . 10
3
1.341 . 10 1.342 . 10
3 3

4.425 . 10
3
1.37 . 10
895.481 3

4.44 . 10
3
872.526
1.595 . 10
3

4.421 . 10
708.446 3
2.012 . 10
3
471.092
4.441 . 10
3
2.644 . 10
3
229.28
4.438 . 10
3
126.675 3.058 . 10
3

4.427 . 10
3
8
1
8.38 (a) C8H18 + 12 O2 + 47.02N 2 → 8CO2 + 9H 2O + 47.02N 2
2
∆ rxn H = 8∆ f H CO2 + 9∆ f H H 2O − ∆ f H C8 H18
= 8 ( −393.5 ) + 9 ( −241.8 ) − ( −208.4 ) = −5115.8 kJ
298.15
∆ rxnU = ∆ rxn H − ∆NRT = −5115.8 kJ (17 − 13.5 ) × 8.314 × kJ
1000
= −5115.8 − 8.7 kJ = −5124.5 kJ
CP of mixture = 8 × 5125
. + 9 × 39.75 + 47.02 × 32.43 = 2292.61 J mol K
CV of mixture = CP − NR = 2292.61 − 64.02 × 8.314 = 1760.35 J mol K
A
# of
moles

∆ rxnU 5124.5 × 103


Tfinal = Tin + = 298.15 + = 298.15 + 2911.07 = 3209.2 K
CV 1760.35
by ideal gas law
NT N f Tf N f Tf
PV = NRT ⇒ i i = ; Pf = Pi
Pi Pf Ni Ti
64.02 3209.2
Pf = 1 bar × = 11386
. bar
6052
. 29815
.
(b) Adiabatic expansion
2292.61 2292.61
CP (per mole) = = = 3581
.
8 + 9 + 47.02 64.02
FG P IJ R CP
F 1 I 8.314 35.81
T2 = T1
HPK
2

1
= 3209.2 ×
H 11386
. K
T2 = 3209.2(0.087827)0.23217 = 1824.5 K
dU dS 0
= W ; = 0 +Sgen
dt dt
W = CV ∆T = −2.438 × 106 J mol of octane

(c) This is like Carnot cycle with a varying upper T


dU dT dS Q 0 C dT
= Q + W = CV ; = +Sgen = P ; T2 = 150° C
dt dt dt TL T dt
dU dS 
= TL +W or
dt dt
∆U − TL ∆S = W
TL
W = CV ∆T − TL ⋅ CP ln
TH
.
42315
W = 1760.35(42315
. − 1824.5) − 42315
. × 2292.61ln
1824.5
= −1049 . × 106 J mol of octane
dT C dT  dT C dT
CV = TL P + W ⇒ W = CV − TL P
dt T dt dt T dt
a
WS = CV TL − TH − TLCP ln f TL
TH
dU
= ∑ N i ,in Hi ,in − ∑ N i ,out Hi ,out + Q + W = 0
dt
dS Q 0
= ∑ N i ,in Si ,in − ∑ N i ,out Si ,out + +Sgen =0
dt Tamb
(for maximum work)
Q = −Tamb ∑ N i ,in Si ,in − ∑ N i ,out Si ,out
b g
0 = ∑ N i ,in Hi ,in − Tamb Si ,in − ∑ N i ,out Hi ,out − Tamb Si ,out + W b g
b g
−W = ∑ N i ,in Hi ,in − Tamb Si ,in − ∑ N i ,out Hi ,out − Tamb Si ,out b g
b g b
= ∑ N i ,in Hi ,in − Tamb Si ,in − ∑ N i ,in + Xν i Hi ,out − Tamb Si ,out gb g
Absolute maximum work Tin = Tout = Tamb
Ideal gas Hi = H i ; Gi ,in = Gi ,out ; Si = S i − R ln xi
−W = ∑Ni
i , in G i ,in − RTamb ∑N i
i , in ∑N G +
ln xi ,in −
i
i , in i , in

RTamb ∑N i , in ln xi ,out + X ∑ ν bG − RT ln x g
i i amb i , out
i i

∑N ∑ν G − RT ∑ ln xν
xi ,in
−W = RTamb i ,in ln +X i i amb
i
i ,out
xi ,out

∑N ∑ ln xν
xi ,in
= RTamb i ,in ln + X ∆ rxn G − RTamb i
i ,out
xi ,out
8

8.39
C is the number of components, and M is the number of phases.
Then the unknowns are
NiK (number of moles of species i in phase K)
= C × P unknowns
P K (pressure in phase K) = P unknowns
T K (temperature in phase K) = P unknowns.
Total number of unknowns is C × P + P + P = P × (C + 2)

Then restrictions are that


T is the same in all phases, i.e., T I = T II = T III =.....
P -1 restrictions
P is the same in all phases, i.e., P I = P II = P III =.....
P -1 restrictions
K
Gi must be the same for species i in all phases
= C × ( P -1) restrictions
In addition we have the stoichiometric relation for each species that
K M
Ni = ∑ Nik = Ni ,o + ∑ ν ij X j which provides an addition C restrictions.
k=1 j =1

Therefore the number of degrees of freedom F are


F = (C + 2) ⋅ P - (C + 2) ⋅ ( P -1) - C = C + 2 - C = 2
independent of the number of components, phases or independent chemical reactions.
Therefore Duhem' s theorem is valid.
9

9.1 a
PV T , P, N1, N 2… = ∑ Ni RT f ⇒ Vi (T , P, x) = V i (T , P)
a f
U T , P, N1, N 2… = ∑ Ni U i (T , P) ⇒ Ui (T , P, x) = U i (T , P)
Also Si (T , P, x ) = S i (T , P) − R ln xi
∆ mix U = ∑ x ⎡⎣U (T , P, x ) − U (T , P )⎤⎦ = ∑ x [0] = 0
i
i i i
i
i

∆ mix V = ∑ x ⎡⎣V (T , P, x ) − V (T , P ) ⎤⎦ = ∑ x [ 0] = 0
i i i i
i i
∆ mix H = ∆ mix U + ∆ mix V = 0
∆ mix S = ∑ x ⎡⎣ S (T , P, x ) − S (T , P )⎤⎦ = ∑ x [− R ln x ] = − R∑ x ln x
i i i i i i i

∆ mix G = ∑ x ⎡⎣G (T , P, x ) − G (T , P ) ⎤⎦ = ∑ x ⎡⎣( H − TS ) − ( H − T S ) ⎤⎦


i i i i i i i i

= ∑ x (H − H ) −T ∑ x (S − S )
i i i i i i

= ∆ H − T ∆ S = RT ∑ x ln x
mix mix i i
Similarly
∆ mix A = ∆ mix U − T ∆ mix S = RT ∑ x ln x
i i
9

9.2 The picture of the process here is as follows


1 T ,V , P1 Mixture ∑ Ni Moles of gas at T and V !
B
2 T ,V , P2 ⎯ ⎯⎯⎯
⎯→ T ,V , Pf
Mix keeping
T and V fixed
#
C T ,V , PC

(a) Let
Pi = initial pressure of species i (pressure in unmixed state)
P = final pressure of mixed gas
Pi = xi P = partial pressure of species i in final state
want to show that Pi = Pi
P = NRT V = b∑ N gRT V
i
Pi = Ni RT V
and
↓ initial pressure of pure i
Pi = xi P = xi NRT V = Ni RT V = Pi
↑ partial pressure of species i Q.E.D.
(b) Since the internal energy of an ideal gas is independent of total pressure, it follows that
Ui IGM (T , x ) = U iIG (T ) for mixing at constant total pressure or constant partial pressure. Thus,
∆ mix U IGM = 0 .
Next,

Vi IGM (T , P, x) =
∂VRS ∑ N RT UV = RT = RTx = RTx j i i

T P W P Px P
∂ Ni T,P i
i

V a P , Tf
= x V aT , P f
IG
RT
=x =x
i i
i
i
IG
i i
Pi Ni

Thus Vi IGM (T , P, x) = xiV iIG T , Pi a f


Can now define two ∆ mix θ IGM

∆ mix θ IGM =
1
∑ x {θ (T , P, x ) − θ
i i i (T , P )} and ∆ mix θ IGM
2
= ∑ x {θ (T , P, x ) − θ (T , P )}
i i i

∆ mix θ IGM was computed in Section 9.1 and will not be considered here. We will be
1

concerned with ∆ mix θ IGM !


2

∆ mix V IGM = ∑ x {Vi i


IGM
(T , P, x ) − V i (T , Pi )} = ∑ xi { xi V i (T , Pi ) − V i (T , Pi )}
C

∑ x ( x − 1) N = ∑ x ( x − 1) x N = N ∑ ( x − 1)
V V V
= i i i i i
i i i =1

V V (1 − C )
= (1 − C ) = C
N
Ni ∑
i =1

C
Note: ∑1 = C
i =1

where C = number of components.


For enthalpy we have
HiIGM (T , P, x) = UiIGM (T , P, x) + PVi IGM (T , P, x) = U iIG (T , P) + PxiV iIG T , Pi a f
= U iIG (T , P) + PVi (T , P) =
IG
H iIG (T , P)
Thus
∆ mix H IGM =
1
∑ x (H i i
IGM
(T , P, x ) − H iIG (T , P ) ) = 0

and
∑ x ( H ( T , P, x ) − H (T , P ) )
C
∆ mix H IGM = i i
IGM IG
1 i
2
i =1

= ∑ x {U (T , P ) + PV (T , P ) − U (T , P ) − PV (T , P )}
i
IG
i i
IG
i i i i

= ∑ x {0 + PV (T , P ) − PV (T , P )} = ∑ x { RT − RT } = 0
i i i i i i
i

To compute ∆ mix S IGM we use the same sort of argument as in Section 9.1, but noting here that
the volume occupied by each gas in the initial and final states are the same. Therefore
a f
SiIGM (T , P, x) = S iIG T , Pi , since T and V of each species is unchanged (see eqn. (4.4-2)).
Therefore

∆ mix S IGM
2
= ∑ x {S
i
i i
IGM
(T , P, x ) − S iIG (T , Pi )} = 0

For the Helmholtz free energy we note that

A = U − TS ⇒ AiIGM (T , P, x ) = UiIGM (T , P, x ) − TSiIGM (T , P, x )


a f
= U iIG (T ) − T S iIG T , Pi = AiIG T , Pi a f
Thus, ∆ mix AIGM = 0 . Finally G = A + PV , ⇒
2

GiIGM (T , P, x) = AiIGM (T , P, x) + PVi IGM (T , P, x) = AiIG T , Pi + PxiV iIG T , Pia f a f


a f
= AiIG T , Pi + PV
i i
IG
a f
T , Pi = GiIG T , Pi a f
So that

∆ mix G IGM =0
2
9

9.3 Generally mixing at constant T and P and mixing at constant T and V are quite
different. However, for the ideal gas we have
PVi = Ni RT (pure fluids) and PV = ∑ Ni RT (mixtures)
Thus for the pure fluids (same T and P)
N1RT N RT
V1 = and V2 = 2
P P
a f
⇒ V1 + V2 =
N1RT N 2 RT
P
+
P
a
= N1 + N 2 f
RT
P
=V

So for the ideal gas the mixing process described in problem statement is also a
mixing process at constant T and P and Table 9.1-1 applies here also.
9
9.4 We have the following properties for a mixture for mixing at constant T and P:
U (T , P, x ) = ∑ Ni U i (T , P)
V (T , P, x) = ∑ NiV i (T , P)
S (T , P, x ) = ∑ Ni S i (T , P) − R∑ Ni ln xi
Ui Vi
and S i = S i0 + CV,i ln + R ln
U i0 V i0
S i0 , U i0 , V i0 are at some reference state.
(a) Find Vi , U i , S i and Gi in terms of S i0 , U i0 , V i0 , CV,i , R, T, and P . Need U i , V i .
∂S 1 ∂S P U V
We know dU = Td S − PdV → = ; = and S i = S i0 + CV,i ln 0i + R ln 0i for
∂U V T ∂V U T Ui Vi
pure component i.
∂S 1 1
= = CV,i → U i = CV,i T
∂U V T Ui
∂S 1 ∂ S ∂U ∂V
=R and ⋅ ⋅ = −1
∂V U Vi ∂V U ∂ S V ∂U S

∂U ∂V 1 ∂S −1 ∂S R P
=T =− ⇒ = T⋅ = −1 → = =
∂S V ∂U S P ∂V U P ∂V U Vi T
RT
⇒Vi =
P
∂U ∂
So U i =
∂ Ni
=
∂ Ni
∑ Ni CV,i T = CV,i T = U i
T , P , N j ≠i

U i = CV,i T
∂V ∂ RT
Vi =
∂ Ni
=
∂ Ni
∑ NiV i = P
= Vi = V i
T , P , N j ≠i
9
9.5 (a) Start with eqn. 9.2-13

LM RT − N F dP I OPdV − ln Z
ln
fi
xi P
= ln φ i =
1
RT z
V =∞
V = ZRT P

MN V GH ∂N JK PQ i V

P=
RT a
− 2 =
NRT N 2a
− 2 =
NRT

∑ ∑ Ni N j aij
V −b V V − Nb V V − ∑ Nibi V2
∂P
=
RT

NRT
a−b f − 2∑VN a j ij
∂Ni V V − ∑ Nibi V − ∑ Nibi b g 2 i 2

RT NRTbi 2∑ N j aij
= + −
V − Nb (V − b)2 V2
∂P RT RTbi 2∑ x j aij
N = + −
∂Ni V V (V − b)2
V2
LM RT RT RTb 2∑ x j aij OP
z
ZRT P
f 1
ln φ i = ln i =
xi P RT MM V − V − b − (V − b) i
2
+
j
V2 PPdV − ln Z
V =∞
N Q
2∑ x j aij
ZRT P

=
1 LM
V OP ZRT P
+
RTbi

ZRT P
j
− ln Z
RT
RT ln
N
V −b QV =∞ (V − b) V =∞ V2
V =∞
2∑ x j aij
Z Bi F where B = Pb I
H RT K
j
= ln + − − ln Z i
Z − B ( Z − B)
i
RTV
2∑ x j aij
fi Bi
⇒ ln φ i = ln = − ln(Z − B) −
j
xi P Z − B RTV

(b) For a pure van der Waals fluid (Eqn. 7.4-13)

ln
fi
P
a
= ( Z − 1) − ln Z − Bi − i
RTV
a f
and, by definition of the activity coefficient
f i = xi f iγ i ⇒
R| 2∑ x A U
|V j ij

f = x P expS
B
− ln(Z − B) −
i j
i i
|T Z − B Z |W
mixture

f = P expRS( Z − 1) − lna Z − B f − UV
A
T ZW
ii
i i
pure fluid i

so

RS LMF 2∑ x A I OPUV
NGH JK
exp Bi (Z − B) − ln( Z − B) −
fi
=
|T j
j ij Z
Q|W =γi
expk( Z − 1) − lna Z − B f − a A f Z p
mixture
xi f i i ii pure fluid

Note that the compressibilities in pure fluid and mixture will generally be different at the same T
and P.
9
9.6 As a preliminary note that, from Eqns. (6.4-27 and 28)

LMT F ∂ P I − POPdV
z
V = ZRT P

MN GH ∂ T JK PQ
H (T , P) − H IG (T , P) = RT ( Z − 1) +
V =∞ V
and
LMF ∂ P I − R OPdV
z
V = ZRT P

MNGH ∂ T JK V PQ
S (T , P) − S IG (T , P) = R ln Z +
V =∞ V

RT a
vdw E.O.S. P = − so
V −b V2

FG ∂ P IJ = R ; T FG ∂ P IJ −P=
RT

RT a a
+ 2 = 2
H ∂ T K V −b H ∂ T K
V V V − b (V − b) V V
FG ∂ P IJ − R = R − R ;
H ∂ T K V (V − b) V
V

z
V = ZRT P
a a RTA
⇒ H (T , P) − H IG (T , P) = RT ( Z − 1) + 2
dV = RT (Z − 1) − = RT ( Z − 1) −
V =∞ V V Z

and

S (T , P) − S IG (T , P) = R ln Z + z
V = ZRT P
LM R − R OPdV
V =∞ NV − b V Q
ZRT P
(V − b)
= R ln Z + R ln = R ln( Z − B)
V V =∞

Now on to solution of problem.


(a) V ex = V mix − ∑
xi V i =
RT
P
(
Z mix − ∑ )
xi Z i = ∆ mix V

Zmix = compressibility of mixture at T and P


Zi = compressibility of pure fluid i at T and P
Will leave answer to this part in this form since the analytic expression for Zi and Zmix
(solution to cubic) is messy. Though it can be analytically and symbolically with a computer
algebra program such as Mathcad, Mathematica, Maple, etc.)

(b) H ex = H mix − ∑xi H i =


RT

RTAmix

L
∑ MN a
xi RT Zi − 1 −
RTAi
f OP
Zmix − 1 Zmix i
Zmix Q
b
= RT Zmix − ∑ xi Zi + ∑ RT
FG x A IJ − RTA g
HZK Z
i i mix

i mix

F x A − A IJ
= RT bZ − ∑ x Z g + RT G ∑
H Z Z K
i i mix
mix i i
i mix

U ex = a H − PV f − ∑ x a H − PV f
mix mix i i i
= b H − ∑ x H g − PbV
mix −∑xV gi i mix i i

F x A − A IJ − RTbZ
= RT bZ − ∑ x Z g + RT G ∑ − ∑ xi Zi g
H Z Z K
i i mix
mix i i mix
i mix

F x A − A IJ
= + RT G ∑
H Z Z K
i i mix

i mix

(c) S mix − ∑ xi Si − R∑ xi ln xi
= S ex
a f a f
= P ln Zmix − Bmix − R X i ln Zi − Bi − R ∑ xi ln xi
= R lnaZ mix − Bmix f − R∑ x lnaZ − B f + ln x
i i i i
= R lnaZ mix − Bmix f − R x ln x aZ − B f
i i i i
Zmix − Bmix
= R ln
a
Π Zi − Bi
i
f xi

(d) G ex = H ex − TS ex = RT Zmix − ∑ xi Zi + RT b g FG ∑ x A − A IJ
H Z Z K
i i mix

i mix
Zmix − Bmix
− RT ln
a
Π Zi − Bi
i
f xi

Aex = U ex − T S ex = + RT
FG ∑ x A − A IJ − RT ln Z − B
H Z Z K
i i mix mix mix

i ΠaZ − B f mix i i
xi
i
9
9.7 (a) Start from eqn. (9.2-13)

LM RT − N F ∂P I OPdV − ln Z
ln φ i = ln
fi
=
1
yi P RT z
V =∞
V = ZRT P

MN V GH ∂N JK i T ,V , N j ≠i PQ
but

PV B
= 1 + mix = 1 +
∑ ∑ yi y j Bij
RT V V

P=
RT Bmix RT
+ =
∑ Ni RT + RT ∑ ∑ Ni N j Bij
V V2 V V2

∂P RT
2 ∑ N B RT
j
j ij

= +
∂Ni V V2

∂P NRT
2N ∑ N B RT
j
j ij

RT 2 x j Bij
N = + = + RT
∂Ni V V 2
V V2

f V = ZRT P ⎢ RT
⎡ 2 RT ∑x B j ij



1 RT j
ln i = ⎢ − − ⎥dV − ln Z
yi P RT V =∞ ⎢V V V2 ⎥
⎢⎣ ⎥⎦

∑x B
2
= j ij − ln Z ( eqn. 9.4-6 )
V

Note also that

PV B PV 2 PV 2
= 1+ or =V + B −V − B = 0
RT V RT RT

1 ± 1 + 4 PB RT
⇒V =
2 P RT
or
PV
RT
1
= Z = 1 ± 1 + 4 PB RT
2
d i
as P → 0, Z → 1 (ideal gas limit) so only + sign allowed

Z=
1
2
d
1 + 1 + 4 PB RT i
Note that at low pressures we can obtain a simpler expression.
At low pressures

PV B B P
= 1 + mix ≈ 1 + mix
RT V RT

Then

V=
RT
P
+ Bmix =
RT
P
m
+ y12 B11 + 2 y1 y2 B12 + y22 B22 r
RT RT
V − V IG = + Bmix − = ∑ ∑ yi y j Bij = V ex
P P

Also

a
V = NV = N1 + N 2 f RTP + N +1 N mN B 1 11 +
2
N 22 B12 + 2 N1 N 2 B22 r
1 2
2∑ N j Bij
∂ RT 1
V1 = V= − ∑ ∑ Ni N j Bij +
j
∂ N1 P N1 + N 2 i N1 + N 2
RT
= − ∑ ∑ yi y j Bij + 2∑ y j Bij
P
RT
= + 2 y1B11 + 2 y2 B12 − y12 B11 − 2 y1 y2 B12 − y22 B22
P
=
RT
P
a f a
+ y1 2 − y1 B11 + 2 y2 1 − y1 B12 − y22 B22 f
+ y a2 − y f B
RT
= 1 1 11 + 2 y2 B12
2
− y22 B22
P
+ y a2 − y f B
RT RT
V1 − V1IGM = 1 1 11 + 2 y2 B12
2
− y22 B22 −
P P

zc a f
P
ln
f1
=
1
y1 P RT
h
V1 − V1IGM dP = y1 2 − y1 B11 + 2 y22 B12 − y22 B22
P
RT
0


f1 RS a f
= exp y1 2 − y1 B11 + 2 y22 B12 − y22 B22
P UV
y1 P T RT W
This is an alternate, some approximate expression that we will use in what follows. Also, for
the pure component we have

f1 B P
= exp 11 LM OP
P RT N Q
Note that these expressions are slightly easier to use then the full expressions since we don’t
have to solve for V (or Z) first
f1
=γ1 =
l a f
exp y1 2 − y1 B11 + 2 y22 B12 − y22 B22 P RT a fq
y1 f1 exp B11 P RT k p
RS b g
= exp − 1 − 2 y1 + y12 B11 + 2 y22 B12 − y22 B22
P UV
T RT W
LM a
= exp y22 2 B12 − B11 − B22
P
f OQP
=γ1
N RT

δ 12 y22 P
or ln γ 1 = where δ 12 = 2 B12 − B11 − B22
RT
(b) Repeating the argument for a ternary mixture

ln γ 1 =
P 2
RT
a f
y2 δ 12 + y2 y3 δ 12 + δ 13 − δ 22 + y32δ 13

By simple generalization

ln γ 1 =
P
c
∑ ∑ yi y j δ 1i + δ ij − δ ij
2 RT i j
h Note: δ ii = 0
i ≠1 j ≠1
i≠ j
9
9.8 k
(a) G ex = x1x2 A + B x1 − x2 fp a
NN R F N − N IJ UV
G ex = NG = ex
N +N T
S A + BG
1H N + N KW
1 2

2
1

1
2

=G
F ∂ G IJ = N RS A + BFG N − N IJ UV
ex
G1ex
H ∂ N K N + N T H N + N KW
2 1 2

1 T,P 1 2 1 2

R aN − N fU
a N + N f ST A + B N + N VW
N N
− 1 2
2
1 2

1 2 1 2

N N R B Ba N − N f U
+ S
1
N + N TN + N
1
2

2 a N + N f VW
1 2 1
1

2
2
2

= x k A + Ba x − x fp − x x k A + Ba x − x fp + x x Ba1 − x + x f
2 1 2 1 2 1 2 1 2 1 2

= Ax2 + Bx2 − 2 Bx2 + 2 Bx2 − 2 Bx2


2 2 3 2 3

Thus G1ex = RT ln γ 1 = ( A + 3B) x22 − 4 Bx23 .


Now by repeating the calculation, or by using the symmetry of G ex and replacing B by −B and
interchanging the subscripts 1 and 2 we obtain

G2ex = RT ln γ 2 = ( A − 3B) x12 + 4 Bx13

2 RTa12 x1q1x2q2 2 RTa12q1q2 N1 N 2


(b) G ex = ⇒ G ex = N G ex =
x1q1 + x2q2 N1q1 + N 2q2
After some algebra

G1ex =
d
∂ N G ex i =
2 RTa12q1
∂ N1
T , P, N 2
a
1 + x1q1 x2q2 f 2

G1ex 2a12q1 α
⇒ ln γ 1 = = =
RT 1 + x1q1 x2q2 a f 2
a
1 + αx1 βx2 f 2

β
Similarly ln γ 2 =
a
1 + βx2 αx1 f 2
.

G ex xi qi
(c) = 2a12 z1z2 + 2a13z1z3 + 2a23z2 z3 ; where zi = and
RT ∑ x jq j
j

FG N q IJ −1

G ex = N G ex = RT
H ∑ K ∑ ∑ a N N q q ; thus
k
k k
k j
kj k j k j
−qi ∑ akj qk q j xk x j 2∑ aij qi q j x j
G ex ∂
ln γ i = i
j, k j
G ex = +
RT ∂ Ni T , P, N FG ∑ x q IJ 2
∑ xk q k
H K
j ≠i
k k k
k
Now setting
q1β 12
α 12 = 2q1a12 , β 12 = 2q2a21 = 2q2a12 = α 21 ⇒ q2 = ;
α 12
q1β 13
similarly, q3 = , etc. For the case i = 1
α 13

ln γ 1 =
nx α a β
2
2 12 12 f a
α 12 2 + x32α 13 β 13 α 13 f s + x x aβ α faβ α f α
2
2 3 12 12 13 13 12 a
+ α 13 − α 23 α 12 β 12 f
kx + x aβ α f + x aβ α fp
1 2 12 12 3 13 13
2

Interchanging indices 1 and 2

ln γ 2 =
nx β aα
2
1 12 12 f a
β 12 2 + x32α 23 β 23 α 23 f s + x x aα β faβ α f β
2
1 3 12 12 23 23 12 a
+ α 13 − α 23 β 12 α 12 f
kx + x aα β f + x aβ α fp
2 1 12 12 3 23 23
2

Finally, interchanging indices 1 and 3 in the original equation yields

ln γ 3 =
nx β aα
2
2 23 23 f a
β 23 2 + x12 β 13 α 13 β 13 f s + x x aα β faα β f β
2
1 2 23 23 13 13 23 a
+ β 13 − β 12 β 23 α 23 f
kx + x aα β f + x aα β fp
3 2 23 23 1 13 13
2
9
9.9 (also available as a Mathcad worksheet)

Using Eqns. (9.6-9) yields: α =


b1 LM a
1

a2 OP 2
and β =
b2 LM a
1

a2 OP 2
. From Section 6.6,
RT Nb1 b2 Q RT Nb1 b2 Q
we have three different expressions relating the a and b parameters to the critical properties:
27 R 2TC2 RT
1) a = and b = C . Eqn. (6.6-4a)
64 PC 8 PC
VC
2) a = 3PCV C2 and b =. Eqn. (6.6-4b)
3
9V C RTC V
3) a = and b = C . Eqn. (6.6-3a)
8 3
Since ZC is not equal to 3/8 for the fluids under consideration, each set of relations will give a
different pair of values for a and b. Generally, set 1 is used, since V C is known with less accuracy
than PC and TC . All three sets of parameters will be considered here.

Benzene 2,2,4-trumethyl pentane


VC = MW ρ C 0.2595 m3 kg 0.4776 m3 kg
Set 1 a
1875
. c h 2
× 106 m3 Pa kmol2 3609
. c h
× 106 m3 Pa kmol2
2

b
0119
. cm h 3 2
kmol 0.232 c m h 3 2
kmol
α 0.480
β 0.937

Set 2 a
c h 2
9.945 × 105 m3 Pa kmol2 1698
. c h
× 106 m3 Pa kmol2
2

b 0.0865 m3 kmol 0159


. m3 kmol
α 0.353
β 0.658

Set 3 a 13655
. b g 2
× 106 m3 Pa kmol2 c h
2.476 × 106 m3 Pa kmol2
2

b 0.0865 m3 kmol 0159


. m3 kmol
α 0.433
β 0.807

Set 4
Example 9.5-1 α = 0.415
Fitting the van Laar equation β = 0.706

Set 5
Example 9.6-1 α=
V1
RT
a f
δ 1 − δ 2 2 = 0.703
V
Regular Solution Theory β = 2 α = 1304
.
V1

The 5 sets of results are plotted below.

Numbers in circles denote parameter sets used.


Parameter set 4 , fitted to the experimental data, should be the most accurate. Parameter set 3 ,
obtained using V C and TC data should be reasonably good, also.

B
9
9.10 i) One-constant Margules equation
RT ln γ 1 = Ax22 RT ln γ 1 x1 = 0 = A a f
Thus
RT ln γ 1* = RT ln
γ 1 x1 a f
= Ax22 − A = − A 1 − x22 c h
a
γ 1 x1 = 0 f
or

γ 1* = exp
LM − Ac1 − x h OP 2
2
and γ1 =
x1 ⋅ 1000
exp
− A 1 − x22 LM c h OP
MN RT PQ ms M1 RT MN PQ
ii) Two-constant Margules equation
a f
RT ln γ 1 x1 = α 1x22 + β 1x23
RT ln γ a x = 0f = α + β 1 1 1 1
γ ax f
RT ln γ 1* = RT ln = −α c1 − x h − β c1 − x h 2 3
γ a x = 0f
1 1
1 2 1 2
1 1
Thus

γ = exp M
*
L −α c1 − x h − β c1 − x h OP 1
2
2 1
3
2
1
MN RT PQ
and

γ1 =
x ⋅ 1000
1
L −α c1 − x h − β c1 − x h OP
expM
1
2
2 1
3
2
mM s MN 1 RT PQ
iii) van Laar equation

α
a f
ln γ 1 x1 =
a f ; ln γ ax = 0f = α
1 + αx1 βx2 2 1 1

α
ln γ a x f = ln γ a x f − ln γ a x = 0f =
*
−α
1 + aα x β x f
1 1 1 1 1 1 2
1 2
or, upon rearrangement,
LM −α x a2β x + α x f OP
γ 1* = exp
2
1 2 1

N aβ x + α x f Q 2 1
2

and

γ1 =
x ⋅ 1000 1 L −α x a2β x + α x f OP
expM
2
1 2 1
mM N aβ x + α x f Q
s 1 2 1
2

iv) Regular Solution Theory

a f
RT ln γ 1 x1 = V 1φ 22 δ 1 − δ 2a f 2
and RT ln γ 1 x1 = 0 = V 1 δ 1 − δ 2a f a f
2

x2V 2
(since φ 2 = ⇒ 1 as x1 → 0 )
x1V 1 + x2V 2
Thus
a f a f a f c ha
RT ln γ 1* x1 = RT ln γ 1 x1 − RT ln γ 1 x1 = 0 = V 1 φ 22 − 1 δ 1 − δ 2 f
2

γ = exp M
*
LV cφ − 1haδ − δ f OP and
1
2
2 1 2
2

1
MN RT PQ
γ1 =
x ⋅ 1000
1
LV cφ − 1haδ − δ f OP
expM
1
2
2 1 2
2

mM s MN 1 RT PQ
v) UNIQUAC Model

φi θ φ
LM θ jτ ij
OP
ln γ i = ln +
z
qi ln i + li − i
φi
c h
∑ x j l j − qi 1 − ln ∑ θ jτ ij − ∑
MM ∑ θ kτ kj PP
xi 2 xi
N j j
k Q
ln γ a x → 0f = lim aln γ f
i i i
xi → 0
Now consider xi → 0 , then θ j → 1 and θ i → 0
φi
=
xi ri 1
⋅ =
ri φ r
, i = i;
a f θ i xi qi x1q1 + x2q2
=
q rj
= i
xi x1r1 + x2r2 xi x1r1 + x2r2 xi rj a f φi xi ri x1r1 + x2r2 ri q j

FrI z Fq r I r L
ln γ = lnG J + q lnG ⋅ J + l − l − q M1 − lncτ h −
i i
τ O
j
P i ij

Hr K 2 Hq r K r N
i
j
i
j τ Q
i
i
j
j i ij
jj

γ F φ r IJ + z q lnFG θ ⋅ q r IJ − φ ∑ x l + r l
= ln γ = lnG
i j j i
H x r K 2 Hφ q r K x
*
γ a x → 0f
1 i i i
ln 1 i j j i
1 1 i i r i i j i j

LM θ τ
O
τ P
−q M− ln ∑ θ τ − ∑
τ P
+ ln τ +
j ij ij

MN ∑ θ τ
QP
i j ij ij
j j k kj jj
k

x1 ⋅ 1000 *
γ1 = γ 1 where γ 1* is as given above.
ms M1
9
9.11 An ideal gas constrained to remain at constant volume and T, is also a system at constant internal
energy and volume, since U is only a function of temperature for the ideal gas. Consequently, at
equilibrium, the entropy should be a maximum.
Suppose there were N1 molecules and N lattice sites. For any distribution of the N1 molecules
among the N lattice sites there will be N1 lattice sites with molecules, and N 2 = N − N1 empty
lattice sites. Thus we can consider the “lattice gas” to be a mixture of N1 molecules and N 2
holes, and the entropy of various configurations of this binary system can be computed.
Following the analysis of Appendix 7.1, it is clear that the random mixture, or uniform
distribution of gas molecules, is the state of maximum entropy. A completely ordered state (for
example, the first N1 lattice sites filled, and the next N 2 = N − N1 lattice sites empty) is an
especially low entropy configuration.
9
9.12 The principle of corresponding states, and the pseudo-critical constant concept will be used first,
then the Peng-Robinson equation of state (program PR1)
290 800
(a) O2 : TC = 154.6 K ; PC = 50.46 bar ; Tr = = 1876
. ; Pr = = 1585
. .
154.6 50.46
f f F I
From Figure 7.4-1:
P
= 1025
. ⇒ f =P
P H K
= 820 bar .

Using the P-R E.O.S.. and the program PR1 f O 2 = 7351 . bar .
290 800 f
(b) N 2 : TC = 126.2 K ; PC = 3394
. bar ; Tr = = 2.298 ; Pr = = 2357
. ; ≅ 136
. ;
126.2 3394
. P
f N 2 = 1088 bar .
Using the P-R e.o.s. f N 2 = 1043 bar .
(c) Lewis-Randall Rule

Corresponding states P-R e.o.s.


f O2 0.3 × 820 = 246.0 bar 0.3 × 7351
. = 2205 . bar
f N2 0.7 × 1088 = 7616
. bar 0.7 × 1043 = 7301
. bar
(d) Using the program PR1we find f O 2 = 224.9 bar and f N 2 = 732.3 bar .

SUMMARY
f N2 f O2
Lewis-Randall 761.6 bar 246.0 bar
with corresponding states
with Peng-Robinson eos 730.1 220.5
Peng-Robinson e.o.s. directly 732.3 224.9
(program PR1)
9
9.13
This problem was solved using the original UNIFAC model. The results may be slightly different in
the VISUAL BASIC UNIFAC program is used as that uses the updated, modified UNIFAC model.
To present the extent of nonideality, two measures will be used. One is the infinite dilution activity
coefficients, and the other is G ex (max) , that is, the maximum value of the excess Gibbs free energy.
The results appear below for the case of T = 50° C

1 2 γ 1∞ γ 2∞ G ex (J/mol)
water ethanol 2.7469 7.2861 829.6
benzene 304.0 1867.7 3507.9
toluene 446.5 8776.2 3765.4
ethanol benzene 8.8774 4.5590 1162.0
toluene 8.1422 5.4686 1177.7
benzene toluene 0.9650 0.9582 –26.1

These results were obtained treating toluene as 5 ACH groups + 1 ACCH 3 group.
An alternative is to consider toluene to be 5 ACH groups, 1 AC group and 1 CH 3 group. We do this
just to demonstrate that there can be a number of possible group assignments, each of which will
result in somewhat different activity coefficients.

1 2 γ 1∞ γ 2∞ G ex (J/mol)
water toluene 340.1 6162.0 3685.0
ethanol toluene 9.928 5.966 1269.2
benzene toluene 1.0058 1.0080 4.5

We see, from the results (independent of which group assignment is used for toluene) that the
benzene-toluene mixture, which contains chemically similar species, is virtually an ideal solution.
The water-toluene and water-benzene mixtures consist of very dissimilar species and, therefore, the
mixtures are very nonideal. Ethanol contains a hydrocarbon end and a polar -OH end.
Consequently, it is almost equally compatible (or incompatible) with both water and hydrocarbon
solvents and forms only moderately nonideal mixtures with both this behavior is predicted above.
9
9.14 Regular solution theory should not be used with hydrogen-bonded solvents such as water and
ethanol. However, merely for demonstration, we will use regular solution theory for these
compounds. The “Handbook of Chemistry and Physics” reports δ EtOH = 10.0 and δ H 2 O = 9.9 .
Also, V H 2 O = 18 cc mol and V EtOH = 58.4 cc mol .
In regular solution theory

a
RT ln γ i = V iφ 2j δ 1 − δ 2 f
2
so γ i∞ = exp
LMV aδ − δ f OP
i 1 2
2

N RT Q
so
1 2 γ 1∞ γ 2∞
Water ethanol 1.000 1.001 δ EtOH = 10.0
benzene 1.014 1.070 δ benz = 9.2
toluene 1.028 1.181 δ tol = 8.9
Ethanol benzene 1.060 1.093
toluene 1.116 1.223
Benzene toluene 1.013 1.015
Since the solubility parameters of all the components are similar, regular solution theory predicts
essentially ideal solution behavior, even though, for example, the water-aromatic hydrocarbon
mixtures are highly nonideal. This is an example of how bad the regular solution theory predictions
can be when used for mixtures for which it is not appropriate.

This example should serve as a warning about the improper use of thermodynamic models.
9
9.15 The activity coefficient is defined as
f i ( T, P, x )
f i ( T, P, x ) = x i γ i ( T, P, x ) fi ( T, P ) or γ i ( T, P, x ) =
x i γ i fi ( T, P )
To get the activity coefficient, use Eq. (9.4-11) for f i ( T, P, x ) and Eq. (7.4-14) for fi ( T, P ) .
However, remember that the Peng-Robinson equation of state parameters are different for the pure
component and for the mixture. Also, therefore the liquid volume (really compressibility Z) will
be different for the pure component and for the mixture.
9
9.16
T, P
x1 moles pure 1
T, P
Process x1, x2
T, P
x2 moles pure 2

System: 1 mole of initial mixture


system is closed, isothermal and isobaric
Mass balance: x1 + x2 = 1
dU dV
Energy balance: = Q − P + Ws
dt dt
dS Q 
Entropy balance: = + Sgen
dt T
Thus
dU  dV
Ws = −Q+ P
dt dT
dS
Q = T − TSgen
dt
dU dS dV
⇒ Ws = −T + TSgen + P
dt dt dt

a) Since both P and T are constant, we can write

dU d d
Ws = + ( PV ) − (TS ) + TSgen
dt dt dt
d dG
= (U + PV − TS ) + TSgen = + TSgen
dt dt

dG
Clearly, for Ws to be a minimum, Sgen = 0 , and Wsmin = .
dt

FG per mole of IJ = G − G i = x1G1 + x2 G 2 − x1G1 − x2G2


Wsmin
H initial mixtureK f

b g b
= x1 G1 − G1 + x2 G 2 − G2 g
f1(T , P) f (T , P )
= x1 RT ln + x2 RT ln 2
f 1(T , P, x ) f 2 (T , P, x )
f i (T , P, x)
b) Now for either ideal mixtures or Lewis-Randall mixtures, = xi . Therefore,
f i (T , P)

Wsmin = RT − x1 ln x1 − x2 ln x2 ≥ 0 , so work must be added!

dS dS
[Note: Q = T − TSgen ; thus Q max = T , which occurs when Ws = Wsmin . Following same
dt dt
analysis as above leads to

l
Q max = T x1 S 1(T , P) − S1(T , P, x) + x2 S 2 (T , P) − S2 (T , P, x ) q
c) Using the ideal gas or ideal mixture assumption, since isomers can be expected to form ideal
mixtures, and the result above

J
Wsmin = RT − x1 ln x1 − x2 ln x2 = 8.314 × 300 K × −05 . − 05
. ln 05 . ln 05
.
mol ⋅ K
J J
= −8.314 × 300 K × ln 05
. = −17288 .
mol ⋅ K mol of feed
9

NG =
N1 N 2
A+
LM
C N1 − N 2 2 a f OP
9.17
N1 + N 2 N
N1 + N 2 2 a f Q
∂ ( N G)
G1 =
∂ N1 T , P , N 2
= x2 A + C x1 − x2 a f 2
− x1 x2 A + C x1 − x2 a f 2

LM a N − N f − 2Ca N − N f OP
+ x1 N 2 2C
2

N aN + N f aN + N f Q
1 2 1 2
2 3
1 2 1 2

= x a1 − x f A + Ca x − x f + x x 2Ca x − x f − 2Ca x − x f
2 2
2 1 1 2 1 2 1 2 1 2

= x A + Ca x − x f + 2Cx x a x − x f 1 − x + x
2 2
2 1 2 1 2 1 2 1 2

= x A + Ca x − x f + 4Cx x a x − x f = RT ln γ
2 2 2
2 1 2 1 2 1 2 1

∂ ( N G)
G2 =
∂ N 2 T , P , N1
= x1 A + C x1 − x2 a f
2
− x1 x2 A + C x1 − x2 a f
2

a fLMN −a2Na N+ −N Nf f − 2Ca Na N+−NNf f OPQ


2
+ x1 x2 N1 + N 2 1 2
2
1 2
3
1 2 1 2

= x a1 − x f A + Ca x − x f − 2 x x Ca x − x f 1 + x − x
2
1 2 1 2 1 2 1 2 1 2

= x A + Ca x − x f − 4Cx x a x − x f = RT ln γ
2 2 2
1 1 2 2 1 1 2 2

or
a f + 4Cx a x − x f
RT ln γ 1 = x22 A + C x1 − x2 2
1 1 2

RT ln γ 2 = x12 A + Ca x − x f − 4Cx a x − x f
1 2
2
2 1 2
9

9.18 (a) i) One constant Margules equation.

∂ ln γ 1
a
RT ln γ 1 = A 1 − x1 2 ; RT f ∂ x1
= −2 A 1 − x1 a f
∂ ln γ 1 −2 A 1 − x1
=
a f
∂ x1 RT
or
lim
∂ ln γ 1
= lim
2 A 1 − x1
=0
a f
x1 →1 ∂ x1 x1 →1 RT

ii) Two constant Margules equations

a f a f
RT ln γ 1 = α 1 1 − x1 2 + β 1 1 − x1 3

∂ ln γ 1
∂ x1
=
1
RT
n−2α a1 − x f − 3β a1 − x f s → 0 as x → 1
1 1 1 1
2
1

iii) van Laar Equation

α
ln γ 1 =
a
1 + αx1 βx2 f 2

Thus

∂ ln γ 1
=
2α 2 β 2 1 − x1 a
→ 0 as x1 → 1
f
∂ x1 a
β 1 − x1 + αx1 3 f
iv) Regular Solution Theory expression
R.S.T. has the same form as the van Laar Equation, so that proof follows from (iii) above.
(b) Starting from the Gibbs-Duhem Equation, Eqn. (9.3-16)

C F ∂ ln γ I
0 = ∑ xi
i =1
GH ∂ x JK j
i

T,P
we obtain

0 = x1
FG ∂ ln γ IJ + x2
FG ∂ ln γ IJ
H ∂x K H ∂x K
1 2

1 T,P 1 T,P

Alternatively, since dx2 = −dx1 , we have


FG ∂ ln γ IJ = x2
FG ∂ ln γ IJ
H ∂x K H ∂x K
1 2
x1 (*)
1 T,P 2 T,P

Now lim x1 = 1 and lim


FG ∂ ln γ IJ =0
H ∂x K
1
x1 →1 x1 →1 1 T,P

⇒ lim x1
FG ∂ ln γ IJ =0 ⇒
FG ∂ ln γ IJ =0
H ∂x K H ∂x K
1 2
lim x2
x1 →1 1 x1 →1 2
T,P T,P

FG ∂ ln γ IJ = 0 or, more generally


H ∂x K
2
which also implies that lim x2
x2 → 0 2 T,P

FG ∂ ln γ IJ =0
H ∂x K
i
lim xi
xi → 0 i T,P
Thus we have
ln γ 1 departs, with
FG ∂ ln γ IJ = 0 ⇒ zero slope, from its
H ∂x K
i
lim
xi →1
i T,P
value of 0 at xi = 1

and

ln γ 1 rises less rapidly


F ∂ ln γ IJ
lim x G = 0 ⇒ than
1
as xi → 0. Thus
H ∂x K
i
i
xi → 0 i xi
T,P
xi ln γ i is bounded!!!
9

9.19 Let M = molality of salt in solution.


i) For KCl: z+ = 1 , z− = 1 , M K = M , M Cl = M ;
1 1
I = ∑ zi2 Mi = (1 × M + 1 × M ) = M
2 2
z+ = 3 z− = −1UV 1
m
I = 32 × M + 1 × 3 M = 6 Mr
ii) For CrCl3 :
M+ = M M− = 3 M W 2

a f z+ = 3 z− = −2UV 1
m
I = 32 × 2 M + 4 × 3 M = 15 Mr
iii) For Cr2 SO4 3 :
M+ = 2 M M− = 3 M W 2
Now, the Debye-Hückel expression is ln γ ± = −α z+ z− I
and Equation (7.11-18)
−α z+ z− I F I
mol 12
ln γ ± =
1+ I
+ 01
. z+ z− I ; α = 1178
.
H K
lit
at 25°C

i) KCl
M ln γ ± Debye-Hückel Eqn. (7.11-18)
experiment ln γ ± = −1178
. M −1178
. M
ln γ ± = + 01
. M
1+ M
0.1 0.770 0.689 0.761
0.2 0.718 0.590 0.709
0.3 0.688 0.525 0.679
0.5 0.649 0.435 0.645
0.6 0.637 0.402 0.635
0.8 0.618 0.349 0.621
1.0 0.604 0.308 0.613

You might also like